You are on page 1of 246

Lai et al, Introduction to Continuum Mechanics

CHAPTER 2, PART A

2.1 Given
1 0 2 1
Sij = 0 1 2 and [ ai ] = 2


3 0 3 3
Evaluate (a) Sii , (b) Sij Sij , (c) S ji S ji , (d) S jk Skj (e) am am , (f) Smn am an , (g) Snm am an
-------------------------------------------------------------------------------
Ans. (a) Sii = S11 + S22 + S33 = 1 + 1 + 3 = 5 .
2 2 2 2 2 2 2 2 2
(b) Sij Sij = S11 + S12 + S13 + S21 + S22 + S23 + S31 + S32 + S33 =
1 + 0 + 4 + 0 + 1 + 4 + 9 + 0 + 9 = 28 .
(c) S ji S ji = Sij Sij =28.
(d) S jk S kj = S1k S k1 + S2 k S k 2 + S3k S k 3
= S11S11 + S12 S21 + S13 S31 + S21S12 + S22 S22 + S23 S32 + S31S13 + S32 S23 + S33 S33
= (1)(1) + ( 0 )( 0 ) + ( 2 )( 3) + ( 0 )( 0 ) + (1)(1) + ( 2 )( 0 ) + ( 3)( 2 ) + ( 0 )( 2 ) + ( 3)( 3) = 23 .
(e) am am = a12 + a22 + a32 = 1 + 4 + 9 = 14 .
(f) Smn am an = S1n a1an + S2 n a2 an + S3n a3 an =
S11a1a1 + S12 a1a2 + S13 a1a3 + S21a2 a1 + S22 a2 a2 + S23a2 a3 + S31a3a1 + S32 a3a2 + S33a3a3
= (1)(1)(1) + ( 0 )(1)( 2 ) + ( 2 )(1)( 3) + ( 0 )( 2 )(1) + (1)( 2 )( 2 ) + ( 2 )( 2 )( 3) + ( 3)( 3)(1)
+ ( 0 )( 3)( 2 ) + ( 3)( 3)( 3) = 1 + 0 + 6 + 0 + 4 + 12 + 9 + 0 + 27 = 59.
(g) Snm am an = Smn am an =59.
__________________________________________________________________
2.2 Determine which of these equations have an identical meaning with ai = Qij a 'j .
(a) a p = Q pm am' , (b) a p = Qqp aq' , (c) am = an' Qm n .
-------------------------------------------------------------------------------
Ans. (a) and (c)
__________________________________________________________________
2.3 Given the following matrices
1 2 3 0
[ ai ] = 0 , Bij = 0 5 1

2 0 2 1
Demonstrate the equivalence of the subscripted equations and corresponding matrix equations in
the following two problems.
(a) bi = Bij a j and [b ] = [ B ][ a ] , (b) s = Bij ai a j and s = [ a ] [ B ][ a ]
T

-------------------------------------------------------------------------------
Ans. (a)
bi = Bij a j b1 = B1 j a j = B11a1 + B12 a2 + B13a3 = ( 2 )(1) + ( 3)( 0 ) + ( 0 )( 2 ) = 2
b2 = B2 j a j = B21a1 + B22 a2 + B23a3 = 2, b3 = B3 j a j = B31a1 + B32 a2 + B33 a3 = 2 .

Copyright 2010, Elsevier Inc


2-1
Lai et al, Introduction to Continuum Mechanics

2 3 0 1 2
[b] = [ B ][ a ] = 0 5 1 0 = 2 . Thus, bi = Bij a j gives the same results as [b] = [ B ][ a ]
0 2 1 2 2
(b)
s = Bij ai a j = B11a1a1 + B12 a1a2 + B13 a1a3 + + B21a2 a1 + B22 a2 a2 + B23 a2 a3
+ B31a3 a1 + B32 a3a2 + B33a3a3 = ( 2 ) (1)(1) + ( 3) (1)(0) + ( 0 ) (1)(2) + ( 0 ) (0)(1)
+ ( 5 ) (0)(0) + (1) (0)(2) + ( 0 ) (2)(1) + ( 2 ) (2)(0) + (1) (2)(2) = 2 + 4 = 6.
2 3 0 1 2
and s = [ a ] [ B ][ a ] = [1 0 2] 0 5 1 0 = [1 0 2] 2 = 2 + 4 = 6 .

T

0 2 1 2 2
__________________________________________________________________
Write in indicial notation the matrix equation (a) [ A] = [ B ][C ] , (b) [ D ] = [ B ] [C ] and (c)
T
2.4
[ E ] = [ B ]T [C ][ F ] .
-------------------------------------------------------------------------------
Ans. (a) [ A] = [ B ][C ] Aij = Bi m Cm j , (b) [ D ] = [ B ] [C ] Aij = Bm i Cmj .
T

(c) [ E ] = [ B ] [C ][ F ] Eij = Bmi Cmk Fkj .


T

__________________________________________________________________
2 2 2
2.5 Write in indicial notation the equation (a) s = A12 + A22 + A32 and (b) 2 + 2 + 2 = 0 .
x1 x2 x3
-------------------------------------------------------------------------------
2 2 2 2
Ans. (a) s = A12 + A22 + A32 = Ai Ai . (b) 2 + 2 + 2 = 0 =0.
x1 x2 x3 xi xi
__________________________________________________________________
2.6 Given that Si j =ai a j and Sij =aiaj , where ai =Qmi am and aj =Qn j an , and Qik Q jk = ij .
Show that Sii =Sii .
-------------------------------------------------------------------------------
Ans. Sij =Qmi am Qn j an =Qmi Qn j am an Sii =Qmi Qni am an = mn am an =am am = S mm = Sii .
__________________________________________________________________
v v
2.7 Write ai = i + v j i in long form.
t x j
-------------------------------------------------------------------------------
Ans.
v v v v v v
i = 1 a1 = 1 + v j 1 = 1 + v1 1 + v2 1 + v3 1 .
t x j t x1 x2 x3
v2 v v v v v
i = 2 a2 = + v j 2 = 2 + v1 2 + v2 2 + v3 2 .
t x j t x1 x2 x3
v3 v v v v v
i = 3 a3 = + v j 3 = 3 + v1 3 + v2 3 + v3 3 .
t x j t x1 x2 x3
__________________________________________________________________
Copyright 2010, Elsevier Inc
2-2
Lai et al, Introduction to Continuum Mechanics

2.8 Given that Tij = 2 Eij + Ekk ij , show that


(a) Tij Eij = 2 Eij Eij + ( Ekk ) and (b) TijTij = 4 2 Eij Eij + ( Ekk ) (4 + 3 2 )
2 2

-------------------------------------------------------------------------------
Ans. (a)
Tij Eij = (2 Eij + Ekk ij ) Eij = 2 Eij Eij + Ekk ij Eij = 2 Eij Eij + Ekk Eii = 2 Eij Eij + ( Ekk )2
(b)
TijTij = (2 Eij + Ekk ij )(2 Eij + Ekk ij ) = 4 2 Eij Eij + 2 Eij Ekk ij + 2 Ekk ij Eij

+ 2 ( Ekk ) ij ij = 4 2 Eij Eij + 2 Eii Ekk + 2 Ekk Eii + 2 ( Ekk ) ii


2 2

= 4 2 Eij Eij + ( Ekk ) (4 + 3 2 ).


2

__________________________________________________________________
2.9 .
Given that ai =Tij b j , and ai =Tijbj , where ai =Qim am and Tij =Qim Q jnTmn
bn = Qim Q jnTmn
(a) Show that QimTmn b j and (b) if Qik Qim = km , then Tkn
(bn Q jn b j ) = 0 .
-------------------------------------------------------------------------------
Ans. (a) Since ai =Qim am and Tij =Qim Q jnTmn , therefore, ai =Tij b j .
Qim am = Qim Q jnTmn
b j (1), Now, ai =Tijbj am =Tmj
bj = Tmn
bn , therefore, Eq. (1) becomes
Qi mTm n bn = Qi m Q j nTm n b j . (2)
(b) To remove Qim from Eq. (2), we make use of Qik Qim = km by multiplying the above equation,
Eq.(2) with Qik . That is,
Qik QimTm nbn = Qik QimQ jnTm nb j kmTm nbn = kmQ jnTm nb j Tknbn = Q jnTknb j
Tkn (bn Q jnb j ) = 0 .
__________________________________________________________________

1 0
2.10 Given [ ai ] = 2 and [bi ] = 2 Evaluate [di ] , if d k = ijk ai b j and show that this result is

0 3
the same as d k = ( a b ) e k .
-------------------------------------------------------------------------------

Ans. d k = ijk ai b j
d1 = ij1ai b j = 231a2b3 + 321a3b2 = a2b3 a3b2 = (2)(3) (0)(2) = 6
d 2 = ij 2 ai b j = 312 a3b1 + 132 a1b3 = a3b1 a1b3 = (0)(0) (1)(3) = 3
d3 = ij 3 ai b j = 123 a1b2 + 213a2b1 = a1b2 a2b1 = (1)(2) (2)(0) = 2
Next, ( a b ) = ( e1 + 2e2 ) ( 2e2 + 3e3 ) = 6e1 3e2 + 2e3 .
d1 = ( a b ) e1 = 6, d 2 = ( a b ) e 2 = 3, d3 = ( a b ) e3 = 2 .
__________________________________________________________________
2.11 (a) If ijk Tij = 0 , show that Tij = T ji , and (b) show that ij ijk =0
-------------------------------------------------------------------------------
Copyright 2010, Elsevier Inc
2-3
Lai et al, Introduction to Continuum Mechanics

Ans. (a) for k = 1, ij1Tij = 0 231T23 + 321T32 = 0 T23 T32 T23 = T32 .
for k = 2, ij 2Tij = 0 312T31 + 132T13 = 0 T31 T13 T31 = T13 .
for k = 3, ij 3Tij = 0 123T12 + 213T21 = 0 T12 T21 T12 = T21 .
(b) ij ijk = 1111k + 22 22k + 33 33k = (1)( 0 ) + (1)( 0 ) + (1)( 0 ) = 0 .
__________________________________________________________________
2.12 Verify the following equation: ijm klm = ik jl il jk .
(Hint): there are 6 cases to be considered (i) i = j , (2) i = k , (3) i = l , (4) j = k , (5) j = l , and (6)
k =l .
-------------------------------------------------------------------------------

Ans. There are 4 free indices in the equation. Therefore, there are the following 6 cases to consider:
(i) i = j , (2) i = k , (3) i = l , (4) j = k , (5) j = l , and (6) k = l . We consider each case below
where we use LS for left side, RS for right side and repeated indices with parenthesis are not sum:
(1) For i = j , LS= (i )(i ) m klm = 0, RS = (i ) k (i )l (i )l (i ) k = 0.
(2) For i = k , LS= (i ) j1 (i )l1 + (i ) j 2 (i )l 2 + (i ) j 3 (i )l 3 , RS = (i )(i ) jl (i )l j (i )
0 if j l

LS=RS = 0 if j = l = i .
1 if j = l i

(3) For i = l , LS= (i ) jm k (i ) m , RS = (i ) k j (i ) (i )(i ) jk
0 if j k

LS=RS = 0 if j = k = i
1 if j = k i

(4) For j = k , LS= i ( j ) m ( j )lm , RS = i ( j ) ( j )l il ( j )( j )
0 if i l

LS=RS = 0 if i = l = j
1 if i = l j

(5) For j = l , LS= i ( j ) m k ( j ) m , RS = ik ( j )( j ) i ( j ) ( j ) k
0 if i k

LS=RS = 0 if i = k = j
1 if i = k j

(6) For k = l , LS= ijm ( k )( k ) m =0, RS = i ( k ) j ( k ) i ( k ) j ( k ) = 0
__________________________________________________________________
2.13 Use the identity ijm klm = ik jl il jk as a short cut to obtain the following results:
(a) ilm jlm = 2 i j and (b) ijk ijk = 6 .
-------------------------------------------------------------------------------
Ans. (a) ilm jlm = i j ll il lj = 3 i j i j = 2 i j .
(b) ijk ijk = ii jj i j ji = (3)(3) ii = 9 3 = 6 .
__________________________________________________________________
2.14 Use the identity ijm klm = ik jl il jk to show that a (b c) = (a c)b (a b)c .

Copyright 2010, Elsevier Inc


2-4
Lai et al, Introduction to Continuum Mechanics

-------------------------------------------------------------------------------
Ans. a (b c) = am e m ( ijk b j ck ei ) = ijk am b j ck (e m ei )
= ijk am b j ck ( nmi e n ) = ijk nmi amb j ck e n = jki nmi amb j ck e n
= ( jn km jm kn ) am b j ck e n = jn km am b j ck e n jm kn am b j ck e n
= ak bn ck e n a j b j cne n = (a c)b (a b)c .
__________________________________________________________________
2.15 (a) Show that if Tij = T ji , Tij ai a j = 0 and (b) if Tij = T ji , and Sij = S ji , then Tij Sij = 0
-------------------------------------------------------------------------------
Ans. Since Tij ai a j = T ji a j ai (switching the original dummy index i to j and the original index
j to i ), therefore Tij ai a j = T ji a j ai = Tij a j ai = Tij ai a j 2Tij ai a j = 0 Tij ai a j = 0 .
(b) Tij Sij = T ji S ji (switching the original dummy index i to j and the original index j to i ),
therefore, Tij Sij = T ji S ji = Tij S ji = Tij Sij 2Tij Sij = 0 Tij Sij = 0 .
__________________________________________________________________
2.16 ( ) ( )
Let Tij = Sij + S ji /2 and Rij = Sij S ji / 2 , show that Tij = T ji , Rij = R ji ,
and Sij = Tij + R ij .
-------------------------------------------------------------------------------
( ) (
Ans. Tij = Sij + S ji / 2 T ji = S ji + Sij / 2 = Tij . )
( ) ( ) (
Rij = Sij S ji / 2 R ji = S ji Sij / 2 = Sij S ji / 2 = Rij . )
( ) ( )
Tij +Rij = Sij + S ji /2+ Sij S ji / 2 = Sij .
__________________________________________________________________
2.17 Let f ( x1 , x2 , x3 ) be a function of x1 , x2 ,and x3 and vi ( x1 , x2 , x3 ) be three functions of
x1 , x2 ,and x3 . Express the total differential df and dvi in indicial notation.
-------------------------------------------------------------------------------
f f f f
Ans. df = dx1 + dx2 + dx3 = dxi .
x1 x2 x3 xi
v v v v
dvi = i dx1 + i dx2 + i dx3 = i dxm .
x1 x2 x3 xm
__________________________________________________________________
2.18 Let Aij denote that determinant of the matrix Aij . Show that Aij = ijk Ai1 A j 2 Ak 3
-------------------------------------------------------------------------------
Ans. ijk Ai1 A j 2 Ak 3 = 1 jk A11 A j 2 Ak 3 + 2 jk A21 A j 2 Ak 3 + 3 jk A31 A j 2 Ak 3
= 123 A11 A22 A33 + 132 A11 A32 A23 + 231 A21 A32 A13 + 213 A21 A12 A33 + 312 A31 A12 A23 + 321 A31 A22 A13
= A11 A22 A33 A11 A32 A23 + A21 A32 A13 A21 A12 A33 + A31 A12 A23 A31 A22 A13
A11 A12 A13
= A21 A22 A23
A31 A32 A33
__________________________________________________________________

Copyright 2010, Elsevier Inc


2-5
Lai et al, Introduction to Continuum Mechanics

CHAPTER 2, PART B

2.19 A transformation T operate on any vector a to give Ta = a / a , where a is the magnitude


of a . Show that T is not a linear transformation.
-------------------------------------------------------------------------------
a a+b a b
Ans. Since Ta = for any a , therefore T(a + b) = . Now Ta + Tb = +
a a+b a b
therefore T(a + b) Ta + Tb and T is not a linear transformation.
_________________________________________________________________
2.20 (a) A tensor T transforms every vector a into a vector Ta = m a where m is a specified
vector. Show that T is a linear transformation and (b) If m = e1 + e2 , find the matrix of the
tensor T .
-------------------------------------------------------------------------------
Ans. (a) T( a + b) = m ( a + b) = m a + m b = m a + m b = Ta + Tb. Thus,
the given T is a linear transformation.
(b) Te1 = m e1 = (e1 + e2 ) e1 = e3 , Te2 = m e2 = (e1 + e2 ) e2 = e3 ,
Te3 = m e3 = (e1 + e2 ) e3 = e2 + e1 . Thus,
0 0 1
[T] = 0 0 1 .
1 1 0
_________________________________________________________________
2.21 A tensor T transforms the base vectors e1 and e2 such that Te1 = e1 + e2 and Te2 = e1 e2 .
If a = 2e1 + 3e2 and b = 3e1 + 2e2 , use the linear property of T to find (a) Ta ,(b) Tb , and (c)
T(a + b) .
------------------------------------------------------------------------------
Ans.
(a)Ta = T(2e1 + 3e2 ) = 2Te1 + 3Te2 = 2 ( e1 + e2 ) + 3 ( e1 e2 ) = 5e1 e2 .
(b)Tb = T(3e1 + 2e2 ) = 3Te1 + 2Te2 =3 ( e1 + e2 ) + 2 ( e1 e2 ) =5e1 + e2 .
(c)T(a + b) = Ta + Tb = ( 5e1 e2 ) + ( 5e1 + e2 ) = 10e1 .
_________________________________________________________________
2.22 Obtain the matrix for the tensor T which transforms the base vectors as follows:
Te1 = 2e1 + e3 , Te2 = e2 + 3e3 , Te3 = e1 + 3e2 .
------------------------------------------------------------------------------
2 0 1
Ans. [ T] = 0 1 3 .
1 3 0
_________________________________________________________________
2.23 Find the matrix of the tensor T which transforms any vector a into a vector b = m(a n)
where m = ( )
2 / 2 ( e1 + e 2 ) and n = ( )
2 / 2 ( e1 + e3 ) .
------------------------------------------------------------------------------
( )( )
Ans. Te1 = m ( e1 n ) = n1m = 2 / 2 2 / 2 ( e1 + e 2 ) = ( e1 + e2 ) / 2 .

Copyright 2010, Elsevier Inc
2-6
Lai et al, Introduction to Continuum Mechanics

Te2 = m ( e 2 n ) = n2m = 0m = 0 .
Te3 = m ( e3 n ) = n3m = ( )( )
2 / 2 2 / 2 ( e1 + e 2 ) = ( e1 + e2 ) / 2 .

1 / 2 0 1 / 2
Thus, [ T] = 1 / 2 0 1 / 2 .
0 0 0
_________________________________________________________________
2.24 (a) A tensor T transforms every vector into its mirror image with respect to the plane whose
normal is e 2 . Find the matrix of T . (b) Do part (a) if the plane has a normal in the e3 direction.
------------------------------------------------------------------------------
1 0 0
Ans. (a) Te1 = e1 , Te 2 = e 2 , Te3 = e3 , thus, [ T] = 0 1 0 .
0 0 1
1 0 0
(b) Te1 = e1 , Te 2 = e 2 , Te3 = e3 , thus, [ T] = 0 1 0 .
0 0 1
_________________________________________________________________
2.25 (a) Let R correspond to a right-hand rotation of angle about the x1 -axis. Find the matrix
of R . (b) do part (a) if the rotation is about the x2 -axis. The coordinates are right-handed.
------------------------------------------------------------------------------
Ans.(a) Re1 = e1 , Re 2 = 0e1 + cos e 2 + sin e3 , Re3 = 0e1 sin e 2 + cos e3 . Thus,
1 0 0
[ R ] = 0 cos sin .

0 sin cos
(b) Re1 = sin e3 + cos e1 , Re2 = e2 , Re3 = cos e3 + sin e1 . Thus,
cos 0 sin
[ R ] = 0 1 0 .
sin 0 cos
_________________________________________________________________
2.26 Consider a plane of reflection which passes through the origin. Let n be a unit normal
vector to the plane and let r be the position vector for a point in space. (a) Show that the reflected
vector for r is given by Tr = r 2(r n)n , where T is the transformation that corresponds to the
reflection. (b) Let n = (e1 + e 2 + e3 ) / 3 , find the matrix of T . (c) Use this linear transformation to
find the mirror image of the vector a = e1 + 2e 2 + 3e3 .
------------------------------------------------------------------------------
Ans. (a) Let the vector r be decomposed into two vectors rn and rt , where rn is in the direction of
n and rt is in a direction perpendicular to n . That is, rn is normal to the plane of reflection and rt is
on the plane of reflection and r = rt + rn . In the reflection given by T , we have,
Trn = rn and Trt = rt , so that Tr = Trt + Trn = rt rn = ( r rn ) rn = r 2rn = r 2(r n)n .
(b) n = (e1 + e 2 + e3 ) / 3 e1 n = e2 n = e3 n = 1 / 3 .

Copyright 2010, Elsevier Inc


2-7
Lai et al, Introduction to Continuum Mechanics

( )
Te1 = e1 2(e1 n)n = e1 2 1 / 3 (e1 + e 2 + e3 ) / 3 = ( e1 2e2 2e3 ) / 3 .

( )
Te 2 = e 2 2(e 2 n)n = e 2 2 1 / 3 (e1 + e 2 + e3 ) / 3 = ( 2e1 + e 2 2e3 ) / 3 .

( )
Te3 = e3 2(e3 n)n = e3 2 1 / 3 (e1 + e 2 + e3 ) / 3 = ( 2e1 2e 2 + e3 ) / 3 .
1 2 2
1
[T] = 2 1 2 .
3
2 2 1
1 2 2 1 3
1
(c) [ T][a ] = 2 1 2 2 = 2 Ta = ( 3e1 + 2e 2 + e3 ) .
3
2 2 1 3 1
_________________________________________________________________
2.27 Knowing that the reflected vector for r is given by Tr = r 2(r n)n (see the previous
problem), where T is the transformation that corresponds to the reflection and n is the normal to the
mirror, show that in dyadic notation, the reflection tensor is given by T = I 2nn and find the
matrix of T if the normal of the mirror is given by n = (e1 + e 2 + e3 ) / 3 ,
------------------------------------------------------------------------------
Ans. From the definition of dyadic product, we have ,
Tr = r 2(r n)n = r 2(nn)r = (Ir 2(nn)r ) = (I 2nn)r T = I 2nn .
1 1 1 1
2 2
For n = (e1 + e 2 + e3 ) / 3 [2nn] = 1 [1 1 1] = 1 1 1 .
3 3
1 1 1 1
1 2 2
1
[T] = [I ] [2nn] = 2 1 2 .
3
2 2 1
_________________________________________________________________
2.28 A rotation tensor R is defined by the relation Re1 = e 2 , Re 2 = e3 , Re3 = e1 (a) Find the
matrix of R and verify that R T R = I and det R = 1 and (b) find a unit vector in the direction of the
axis of rotation that could have been used to effect this particular rotation.
------------------------------------------------------------------------------
0 0 1 0 1 0 0 0 1 1 0 0 0 0 1

Ans. (a) [ R ] = 1 0 0 [ R ] [ R ] = 0 0 1 1 0 0 = 0 1 0 , det [ R ] = 1 0 0 = 1 .
T

0 1 0 1 0 0 0 1 0 0 0 1 0 1 0
(b) Let the axis of rotation be n = 1e1 + 2e2 + 3e3 , then
1 0 1 1 0
Rn = n [ R I ][n ] = [ 0] 1 1 0 2 = 0 1 + 3 = 0, 1 2 = 0, 2 3 =0 .
0 1 1 3 0
Thus, 1 = 2 = 3 , so that a unit vector in the direction of the axis of rotation is
n = (e1 + e 2 + e3 ) / 3 .
_________________________________________________________________

Copyright 2010, Elsevier Inc


2-8
Lai et al, Introduction to Continuum Mechanics

2.29 A rigid body undergoes a right hand rotation of angle about an axis which is in the
direction of the unit vector m . Let the origin of the coordinates be on the axis of rotation and r be
the position vector for a typical point in the body. (a) show that the rotated vector of r is given by:
Rr = (1 cos )( m r ) m + cos r + sin ( m r ) , where R is the rotation tensor. (b) Let
m = (e1 + e 2 + e3 ) / 3 , find the matrix for R .
------------------------------------------------------------------------------
Ans. (a) Let the vector r be decomposed into two vectors rm and r p , where rm is in the direction
of m and r p is in a direction perpendicular to m , that is, r = r p + rm . Let p rp / rp be the unit
vector in the direction of r p , and let q m p . Then, ( m , p, q) forms an orthonormal set of
vectors which rotates an angle of about the unit vector m . Thus,
Rrm = rm and Rrp = r p ( cos p + sin q ) . From r = rp + rm , we have,

{
Rr = Rrp + Rrm = r p ( cos p + sin q ) + rm = cos r p p + sin rp ( m p ) + rm }
{ (
= cos r p + sin m rp )} + rm = {cos ( r rm ) + sin ( m ( r rm ) )} + rm
= r cos + rm (1 cos ) + sin m ( r rm ) = r cos + rm (1 cos ) + sin m r
We note that rm = (r m)m , so that Rr = r cos + (r m)m (1 cos ) + sin m r .
(b) Use the result of (a), that is, Rr = r cos + (r m) (1 cos ) + sin m r , we have,
Re1 = e1 cos + (e1 m)m (1 cos ) + sin m e1 ,
Re2 = e 2 cos + (e 2 m)m (1 cos ) + sin m e 2 ,
Re3 = e3 cos + (e3 m )m (1 cos ) + sin m e3 .
Now, m = (e1 + e 2 + e3 ) / 3 , therefore, m e1 = m e 2 = m e3 = 1 / 3
( ) ( ) ( )
m e1 = 1 / 3 (e3 + e 2 ), m e 2 = 1 / 3 (e3 e1 ), m e3 = 1 / 3 ( e2 + e1 ) . Thus,
Re1 = e1 cos + (e1 m)m (1 cos ) + sin m e1

( )
= e1 cos + (1 / 3) (e1 + e2 + e3 ) (1 cos ) + sin 1 / 3 (e3 + e2 )

{ ( )} { (
= (1 / 3){1 + 2cos } e1 + e 2 (1 / 3)(1 cos ) + sin 1 / 3 + e3 (1 / 3)(1 cos ) sin 1 / 3 )}
Re2 = e 2 cos + (e 2 m)m (1 cos ) + sin m e 2

( )
= e 2 cos + (1 / 3) (e1 + e 2 + e3 ) (1 cos ) + sin 1 / 3 (e3 - e1 )

{ ( ) } { (
= (1 / 3)(1 cos ) 1 / 3 sin e1 + (1 / 3)(1 + 2cos ) e 2 + (1 / 3)(1 cos ) + sin 1 / 3 e3 )}
Re3 = e3 cos + (e3 m)m (1 cos ) + sin m e3

( )
= e3 cos + (1 / 3) (e1 + e2 + e3 ) (1 cos ) + sin 1 / 3 (-e2 + e1 )

{ ( ) } { ( )}
= (1 / 3)(1 cos ) + 1 / 3 sin e1 + (1 / 3)(1 cos ) sin 1 / 3 e2 + (1 / 3)(1 + 2cos ) e3
Thus,

Copyright 2010, Elsevier Inc


2-9
Lai et al, Introduction to Continuum Mechanics

1 + 2cos (1 cos ) 3 sin (1 cos ) + 3 sin


1
[T] = (1 cos ) + 3 sin (1 + 2cos ) (1 cos ) 3 sin .
3
(1 cos ) 3 sin (1 cos ) + 3 sin (1 + 2cos )
_________________________________________________________________
2.30 For the rotation about an arbitrary axis m by an angle , (a) show that the rotation tensor is
given by R = (1 cos )(mm ) + cos I + sin E , where mm denotes that dyadic product of m and
E is the antisymmetric tensor whose dual vector (or axial vector) is m , (b) find the R A , the
antisymmetric part of R and (c) show that the dual vector for R A is given by (sin )m . Hint,
Rr = (1 cos )( m r ) m + cos r + sin ( m r ) (see previous problem).
------------------------------------------------------------------------------
Ans. (a) We have, from the previous problem, Rr = (1 cos )( m r ) m + cos r + sin ( m r ) .
Now, by the definition of dyadic product, we have ( m r ) m = (mm)r , and by the definition of dual
vector we have, m r = Er , thus Rr = (1 cos ) (mm)r + cos r + sin Er
= {(1 cos ) (mm) + cos I + sin E} r , from which, R = (1 cos ) (mm) + cos I + sin E .
(b) R A = ( R R T ) / 2
{
2R A = {(1 cos ) (mm) + cos I + sin E} (1 cos ) (mm)T + cos I + sin ET . Now }
[mm ] = mi m j = m j mi = [mm ]
T
, and the tensor E , being antisymmetric, E = ET , therefore,
2R A = 2sin E , that is, R A = sin E .
(c) dual vector of R A = (sin )(dual vector of E) = sin m .
_________________________________________________________________
2.31 (a) Given a mirror whose normal is in the direction of e 2 . Find the matrix of the tensor S
which first transforms every vector into its mirror image and then transforms them by a 45o right-
hand rotation about the e1 -axis. (b) Find the matrix of the tensor T which first transforms every
vector by a 45o right-hand rotation about the e1 -axis, and then transforms them by a reflection with
respect to the mirror (whose normal is e2 ). (c) Consider the vector a = (e1 + 2e2 + 3e3 ) , find the
transformed vector by using the transformation S .
(d) For the same vector a = (e1 + 2e2 + 3e3 ) , find the transformed vector by using the
transformation T .
------------------------------------------------------------------------------
Ans. Let T1 and T2 correspond to the reflection and the rotation respectively. We have
1 0 0
T1e1 = e1 , T1e2 = e2 , T1e3 = e3 [ T1 ] = 0 1 0 .
0 0 1
1 0 0
1 1
T2e1 = e1 , T2e 2 = ( e2 + e3 ) , T2e3 = ( e2 + e3 ) [T2 ] = 0 1 / 2 1 / 2 .
2 2 0 1 / 2
1 / 2

Copyright 2010, Elsevier Inc


2-10
Lai et al, Introduction to Continuum Mechanics

1 0 1 0 0 1
0 0 0

(a) [S ] = [ T2 ][ T1 ] = 0 1 / 2 1 / 2 0 1 0 = 0 1 / 2 1 / 2 .
0 1 / 2 1 / 2 0 0 1 0 1 / 2 1 / 2

1 0 0 1 0 0 1 0 0

(b) [ T] = [ T1 ][ T2 ] = 0 1 0 0 1 / 2 1 / 2 = 0 1 / 2 1 / 2 .
0 0 1 0 1 / 2 1 / 2 0 1 / 2 1 / 2

1 0 0 1 1

(c) [b ] = [S ][a ] = 0 1 / 2 1 / 2 2 = 5 / 2 .
0 1 / 2 1 / 2 3 1 / 2

1 0 0 1 1

(d) [c ] = [ T ][a ] = 0 1 / 2 1 / 2 2 = 1 / 2 .
0 1 / 2 1 / 2 3 5 / 2

_________________________________________________________________
2.32 Let R correspond to a right-hand rotation of angle about the x3 -axis (a) find the matrix
of R 2 . (b) Show that R 2 corresponds to a rotation of angle 2 about the same axis (c) Find the
matrix of R n for any integer n .
-------------------------------------------------------------------------------
cos sin 0
Ans. (a) [ R ] = sin cos 0 .
0 0 1

0 cos sin 0
2 2
cos sin 0 cos sin 2sin cos

R = sin
2
cos 0 sin cos 0 = 2sin cos cos 2 sin 2 0 .

0 0 1 0 0 1 0 0 1

(b)
cos 2 sin 2 2sin cos 0 cos 2 sin 2 0

R 2 = 2sin cos 2
cos sin 2
0 = sin 2 cos 2 0 .

0 0 1 0 0 1

Thus, R 2 corresponds to a rotation of angle 2 about the same axis
cos n sin n 0
(c) R = sin n cos n 0 .
n

0 0 1
_________________________________________________________________
2.33 Rigid body rotations that are small can be described by an orthogonal
transformation R = I + R* where 0 as the rotation angle approaches zero. Consider two
successive small rotations R1 and R 2 , show that the final result does not depend on the order of
rotations.
------------------------------------------------------------------------------

Copyright 2010, Elsevier Inc


2-11
Lai et al, Introduction to Continuum Mechanics

( )( ) ( )
Ans. R 2 R1 = I + R*2 I + R1* = I + R*2 + R1* + 2 R*2 R1* = I + R*2 + R1* + 2 R*2 R1* .

(
As 0 , R 2 R1 I + R*2 + R1* = R1R 2 . )
_________________________________________________________________
2.34 Let T and S be any two tensors. Show that (a) TT is a tensor, (b) TT + S T = (T + S)T and (c)
(TS)T = S T TT .
-------------------------------------------------------------------------------
Ans. Let a , b , c be three arbitrary vectors and be any two scalars, then
(a) a TT ( b + c) = ( b + c) Ta = b Ta + c Ta = a TT b + a TT c
( ) ( )
= a TT b + TT c TT ( b + c) = TT b + TT c . Thus, TT is a linear transformation, i.e.,
tensor.
(b) a (T + S)T b = b (T + S)a = b Ta + b Sa = a TT b + a S T b
= a (TT + S T )b (T + S)T = TT + S T .
(c) a (TS)T b = b (TS)a = b T(Sa) = (Sa) TT b = a S T TT b (TS)T = S T TT .
_________________________________________________________________
2.35 For arbitrary tensors T and S , without relying on the component form, prove that (a)
(T1 )T = (TT )1 and (b) (TS)1 = S 1T1
-------------------------------------------------------------------------------
Ans. (a) TT1 = I (TT1 )T = I (T1 )T TT = I (T1 )T = (TT )1 .
(b) (TS)(S 1T1 ) = T(SS 1 )T1 = TT1 = I , thus, (TS)1 = S 1T1 .
_________________________________________________________________
2.36 Let {ei } and {ei } be two Rectangular Cartesian base vectors. (a) Show that if ei = Qmi em ,
then ei = Qim em and (b) verify Qmi Qmj = ij = Qim Q jm .
-------------------------------------------------------------------------------
Ans. (a) ei = Qmi e m ei e j = Qmi e m e j = Qmi mj = Q ji e j = Q jm em ei = Qim em .
(b) We have, ei ej = ij = ei e j , thus,
ij = ei ej = Qmi e m Qnj e n = Qmi Qnj em en = Qmi Qnj mn = Qmi Qmj . And
ij = ei e j = Qimem Q jn en = QimQ jn em en = QimQ jn mn = QimQ jm .
_________________________________________________________________
2.37 The basis {ei } is obtained by a 30o counterclockwise rotation of the {ei } basis about the e3
axis. (a) Find the transformation matrix [Q ] relating the two sets of basis, (b) by using the vector
transformation law, find the components of a = 3e1 + e 2 in the primed basis, i.e., find ai and (c)
do part (b) geometrically.
------------------------------------------------------------------------------
Ans. (a) e1 = cos30o e1 + sin 30o e2 , e2 = sin 30o e1 + cos30o e2 , e3 = e3 . Thus,
cos30o sin 30o 0

[Q ]ei = sin 30o cos30o 0 .

0 0 1

Copyright 2010, Elsevier Inc
2-12
Lai et al, Introduction to Continuum Mechanics

a1' 3 / 2 1/ 20 3 2

(b) [a ]e' = [Q ] [a ]e 3 / 2 0 1 = 0 a = 2e1
T
a2' = 1 / 2

1 0 0
i i

a3' 0 0

(c) Clearly a = 3e1 + e 2 is a vector in the same direction as e1 and has a length of 2. See figure
below

_________________________________________________________________
2.38 Do the previous problem with the {ei } basis obtained by a 30o clockwise rotation of the
{ei } basis about the e3 axis.
-------------------------------------------------------------------------------
Ans.
(a) e1 = cos30o e1 sin 30o e 2 , e2 = sin 30o e1 + cos30o e 2 , e3 = e3 . Thus,
cos30o sin 30o 0

[Q ]ei = sin 30o cos30o 0 .

0 0 1

a1' 3 / 2 1 / 2 0 3 1

(b) [a ]e' = [Q ] [a ]e a2' = 1 / 2
T
3 / 2 0 1 = 3 a = e1 + 3e2

1 0 0
i i

a3' 0 0

(c) See figure below

_________________________________________________________________
2.39 The matrix of a tensor T with respect to the basis {ei } is
Copyright 2010, Elsevier Inc
2-13
Lai et al, Introduction to Continuum Mechanics

1 5 5
[T] = 5 0 0
5 0 1
with respect to a right-handed basis {ei } where e1 is in the direction of
Find T11 , T12 and T31
e 2 + 2e3 and e2 is in the direction of e1 .
------------------------------------------------------------------------------
Ans. The basis {ei } is given by:
e1 = (e 2 + 2e3 ) / 5, e2 = e1 , e3 = e1 e2 = (2e2 + e3 ) / 5 .
1 5 5 0

T11 = e1 Te1 = 0 1 / 5 2 / 5 5 0 0 1 / 5 = 4 / 5 .
5 0 1 2 / 5

1 5 5 1
T12 = e1 Te2 = 0 1 / 5 2 / 5 5 0 0 0 = 15 / 5 .
5 0 1 0

1 5 5 0

= e3 Te1 = 0 2 / 5 1 / 5 5 0 0 1 / 5 = 2 / 5 .
T31
5 0 1 2 / 5

_________________________________________________________________
2.40 (a) For the tensor of the previous problem, find Tij , i.e., [ T]e' if {ei } is obtained by a
i
o
90 right hand rotation about the e3 axis and (b) obtain Tii and the determinant Tij and compare
them with Tii and Tij .
------------------------------------------------------------------------------
0 1 0
Ans. (a) e1 = e 2 , e2 = e1 , e3 = e3 [Q ] = 1 0 0 .
0 0 1
0 1 0 1 5 5 0 1 0 0 5 0
Tij = [ T] = [Q ] [ T][Q ] = 1 0 0 5 0 0 1 0 0 = 5 1 5
T

0 0 1 5 0 1 0 0 1 0 5 1
(b) Tii = T11 + T22 + T33 = 0 + 1 + 1 = 2, Tij = 25 .
Tii = T11 + T22 + T33 = 1 + 0 + 1 = 2, Tij = 25 .
_________________________________________________________________
2.41 The dot product of two vectors a = ai ei and b = bi ei is equal to ai bi . Show that the dot
product is a scalar invariant with respect to orthogonal transformations of coordinates.
-------------------------------------------------------------------------------
Ans. From ai = Qmi am and bi' = Qmi bm , we have,
aibi' = Qmi am Qni bn = Qmi Qni am bn = mn am bn = am bm = ai bi .
__________________________________________________________________

Copyright 2010, Elsevier Inc


2-14
Lai et al, Introduction to Continuum Mechanics

2.42 If Tij are the components of a tensor (a) show that TijTij is a scalar invariant with respect to
orthogonal transformations of coordinates, (b) evaluate TijTij with respect to the basis {ei } for
1 0 0 0 0 1
[T] = 1 2 5 , (c) find [T] , if ei = Qei , where [Q ] = 1 0 0 and

1 2 3 e 0 1 0 e
i i

(d) verify for the above [ T] and [ T] that TijTij = TijTij .


------------------------------------------------------------------------------
Ans. (a) Since Tij are the components of a tensor, Tij = Qmi QnjTmn . Thus,
TijTij = Qmi Qnj Tmn (Q pi QqjT pq ) = (Qmi Q pi )(Qnj Qqj )TmnT pq = mp nqTmnTpq = TmnTmn
(b) TijTij = T112 + T122 + T132 + T21
2 2
+ T22 2
+ T23 + T312 + T322 + T332 = 1 + 1 + 4 + 25 + 1 + 4 + 9 = 45 .
0 1 0 1 0 0 0 0 1 0 1 0 0 0 1 2 5 1
(c) [ T] = [Q ] [ T][Q ] = 0 0 1 1 2 5 1 0 0 = 0 0 1 2 5 1 = 2 3 1
T

1 0 0 1 2 3 0 1 0 1 0 0 2 3 1 0 0 1
(d) TijTij = 4 + 25 + 1 + 4 + 9 + 1 + 1 = 45 .
_________________________________________________________________
2.43 Let [ T] and [ T] be two matrices of the same tensor T , show that det [ T] =det [ T] .
------------------------------------------------------------------------------
Ans. T = Q T Q det T = det Q det Q det T = (1)(1) det T = det T .
T T

_________________________________________________________________
2.44 (a) If the components of a third order tensor are Rijk , show that Riik are components of a
vector, (b) if the components of a fourth order tensor are Rijkl , show that Riikl are components of a
second order tensor and (c) what are components of Riik ... , if Rijk ... are components of a tensor of
nth order?
-------------------------------------------------------------------------------
Ans. (a) Since Rijk are components of a third order tensor, therefore,
= Qmi Qnj Q pk Rmnp Riik
Rijk = Qmi Qni Q pk Rmnp = mn Q pk Rmnp = Q pk Rnnp , therefore, Riik are
components of a vector.
(b) Consider a 4th order tensor Rijkl , we have,
= Qmi Qnj Q pk Qql Rmnpq Riikl
Rijkl = Qmi Qni Q pk Qql Rmnpq = m n Q pk Qql Rm npq = Q pk Qql Rnnpq ,
therefore, Riikl are components of a second order tensor.
(c) Riik ... are components of a tensor of the (n 2)th order.
_________________________________________________________________
2.45 The components of an arbitrary vector a and an arbitrary second tensor T are related by a
triply subscripted quantity Rijk in the manner ai = Rijk T jk for any rectangular Cartesian basis {ei } .
Prove that Rijk are the components of a third-order tensor.
------------------------------------------------------------------------------

Copyright 2010, Elsevier Inc


2-15
Lai et al, Introduction to Continuum Mechanics

Ans. Since ai = Rijk T jk is true for any basis, therefore, ai = Rijk


T jk ; Since a is a vector, therefore,
ai = Qmi am and since T is a second order tensor, therefore, Tij = Qmi QnjTmn . Thus,
ai = Qmi am Rijk
T jk = Qmi ( Rmjk T jk ) . Multiply the last equation with Qsi and noting that
Qsi Qmi = sm , we have,
' T ' = Q Q ( R T ) Q R' T ' = R T Q R' T ' = R T
Qsi Rijk jk si mi mjk jk si ijk jk sm mjk jk si ijk jk sjk jk

Qsi Rijk mj nk mn = Rsjk T jk Qsi Rijk Qmj Qnk Tmn = RsmnTmn . Thus,
' Q Q T '

(R smn
'
)
Qsi Qmj Qnk Rijk Tmn = 0 . Since this last equation is to be true for all Tmn , therefore,
, which is the transformation law for components of a third order tensor.
Rsmn = Qsi Qmj Qnk Rijk
_________________________________________________________________
2.46 For any vector a and any tensor T , show that (a) a TA a = 0 and (b) a Ta = a TSa ,
where TA and TS are antisymmetric and symmetric part of T respectively.
------------------------------------------------------------------------------
Ans. (a) TA is antisymmetric, therefore, (TA )T = TA , thus,
a TA a = a (TA )T a = a TA a 2a TA a = 0 a T A a = 0 .
(b) Since T = TS + TA , therefore, a Ta = a (TS + TA )a = a TSa + a TA a = a TSa .
__________________________________________________________________
2.47 Any tensor can be decomposed into a symmetric part and an antisymmetric part, that is
T = TS + TA . Prove that the decomposition is unique. (Hint, assume that it is not true and show
contradiction).
-------------------------------------------------------------------------------
Ans. Suppose that the decomposition is not unique, then ,we have,
T = TS + TA = SS + S A (TS SS ) + (TA S A ) = 0 . Let a be any arbitrary vector, we have,
a (TS SS )a + a (TA S A )a = 0 a TSa a SSa + a TA a a S A a = 0 .
But a T A a = a S A a = 0 (see the previous problem). Therefore,
a TSa a SSa = 0 a (TS SS )a = 0 TS SS = 0 TS = SS . It also follows from
(TS SS ) + (TA S A ) = 0 that TA = S A . Thus , the decomposition is unique.
_________________________________________________________________
1 2 3
2.48 Given that a tensor T has the matrix [ T] = 4 5 6 , (a) find the symmetric part and the
7 8 9
anti-symmetric part of T and (b) find the dual vector (or axial vector) of the anti-symmetric part of
T.
-------------------------------------------------------------------------------
1 2 3 1 4 7 2 6 10 1 3 5
S
2
1
{ }
T 1
2
1
Ans. (a) T = [ T] + [ T] = 4 5 6 + 2 5 8 = 6 10 14 = 3 5 7 .

2 10 14 18 5 7 9
7 8 9 3 6 9

Copyright 2010, Elsevier Inc


2-16
Lai et al, Introduction to Continuum Mechanics

1 2 3 1 4 7 0 2 4 0 1 2
2 { }
TA = 1 [ T] [ T]T = 1 4 5 6 2 5 8 = 1 2 0 2 = 1 0 1 .
2
2 4 2 18 2 1 0

7 8 9
3 6 9

(b) t A = (T23 A
e1 + T31A e2 + T12A e3 ) = (1e1 + 2e 2 1e3 ) = e1 2e2 + e3 .
_________________________________________________________________
2.49 Prove that the only possible real eigenvalues of an orthogonal tensor Q are = 1 . Explain
the direction of the eigenvectors corresponding to them for a proper orthogonal(rotation) tensor and
for an improper orthogonal (reflection) tensor.
------------------------------------------------------------------------------
Ans. Since Q is orthogonal, therefore, for any vector n , we have, Qn Qn = n n . Let n be an
eigenvector, then Qn = n , so that Qn Qn = n n
2 (n n) = (n n) ( 2 1)(n n) = 0 2 1 = 0 = 1 .
The eigenvalue = 1 ( Qn = n ) corresponds to an eigenvector parallel to the axis of rotation for a
proper orthogonal tensor (rotation tensor); Or, it corresponds to an eigenvector parallel to the plane
of reflection for an improper orthogonal tensor (reflection tensor). The eigenvalue = 1 ,
( Qn = n ) corresponds to an eigenvector perpendicular to the axis of rotation for an 180o rotation;
or, it corresponds to an eigenvector perpendicular to the plane of reflection.
_________________________________________________________________
1 2 2
1
2.50 Given the improper orthogonal tensor [Q ] = 2 1 2 . (a) Verify that det [Q ] = 1 .
3
2 2 1
(b) Verify that the eigenvalues are = 1 and 1 (c) Find the normal to the plane of reflection (i.e.,
eigenvectors corresponding to = 1 ) and (d) find the eigenvectors corresponding = 1 (vectors
parallel to the plane of reflection).
-------------------------------------------------------------------------------
Ans. (a) det [Q ] = (1 / 3) (1 8 8 4 4 4) = (27) / 27 = 1 .
3

(b) I1 = 3 / 3 = 1, I 2 = (1 / 3) {(1 4) + (1 4) + (1 4)} = 1,


2
I 3 = 1
3 2 + 1 = 0 ( 1)( 2 1) = 0 = 1,1, 1
(c) For = 1 ,
1 2 2 2 1 2 2 2 1
+ 1 1 2 3 = 0, 1 + + 1 2 3 = 0, 1 2 + + 1 3 = 0 . That
3 3 3 3 3 3 3 3 3
is, 21 2 3 = 0, 1 + 2 2 3 = 0, 1 2 + 2 3 = 0 , thus, 1 = 2 = 3 , therefore,
n = (e1 + e 2 + e3 ) / 3 , this is the normal to the plane of reflection.
(d) For = 1 ,
1 2 2 2 1 2 2 2 1
1 1 2 3 = 0, 1 + 1 2 3 = 0, 1 2 + 1 3 = 0
3 3 3 3 3 3 3 3 3
All three equations lead to 1 + 2 + 3 = 0 3 = 1 2 . Thus,
1 1
n= [1e1 + 2e 2 (1 + 2 )e3 ] , e.g., n = (e1 + e2 2e3 ) etc. these vectors are all
12 + 22 + 32 6
perpendicular to n = (e1 + e2 + e3 ) / 3 and thus parallel to the plane of reflection.
_________________________________________________________________
Copyright 2010, Elsevier Inc
2-17
Lai et al, Introduction to Continuum Mechanics

2.51 Given that tensors R and S have the same eigenvector n and corresponding eigenvalue
r1 and s1 respectively. Find an eigenvalue and the corresponding eigenvector for the tensor T = RS .
------------------------------------------------------------------------------
Ans. We have, Rn = r1n and Sn = s1n , thus, Tn = RSn = Rs1n = s1Rn = r1s1n . Thus, an eigenvalue
for T = RS is r1s1 with eigenvector n .
_________________________________________________________________
2.52 Show that if n is a real eigenvector of an antisymmetric tensor T , then the corresponding
eigenvalue vanishes.
------------------------------------------------------------------------------
Ans. Tn = n n Tn = (n n) . Now, from the definition of transpose, we have n Tn = n TT n .
But, since T is antisymmetric, i.e., TT = T , therefore, n TT n = n Tn . Thus,
n Tn = n Tn 2n Tn = 0 n Tn = 0 . Thus, (n n) = 0 = 0 .
_________________________________________________________________
2.53 (a) Show that a is an eigenvector for the dyadic product ab of vectors a and b with
eigenvalue a b , (b) find the first principal scalar invariant of the dyadic product ab and (c) show
that the second and the third principal scalar invariants of the dyadic product ab vanish, and that
zero is a double eigenvalue of ab .
------------------------------------------------------------------------------
Ans. (a) From the definition of dyadic product, we have, (ab )a = a(b a) , thus a is an eigenvector
for the dyadic product ab with eigenvalue a b .
(b) Let T ab , then Tij = ai b j and the first scalar invariant of ab is Tii = ai bi = a b .
a1b1 a1b2 a2b2 a2b3 a1b1 a1b3
(c) I 2 = + + =0+0+0=0.
a2b1 a2b2 a3b2 a3b3 a3b1 a3b3
a1b1 a1b2 a1b3 b1 b2 b3
I 3 = a2b1 a2b2 a2b3 = a1a2 a3 b1 b2 b3 = 0 .
a3b1 a3b2 a3b3 b1 b2 b3
Thus, the characteristic equation is
3 I1 2 = 0 ( I1 ) 2 = 0 1 = I1 , 2 = 3 = 0 .
_________________________________________________________________
2.54 For any rotation tensor, a set of basis {ei } may be chosen with e3 along the axis of rotation
so that Re1 = cos e1 + sin e2 , Re2 = sin e1 + cos e2 , Re3 = e3 , where is the angle of right
hand rotation. (a) Find the antisymmetric part of R with respect to the basis {ei } , i.e., find [R A ]ei .
(b) Show that the dual vector of R A is given by t A = sin e3 and (c) show that the first scalar
invariant of R is given by 1 + 2cos . That is, for any given rotation tensor R , its axis of rotation
and the angle of rotation can be obtained from the dual vector of R A and the first scalar invariant of
R.
------------------------------------------------------------------------------
Ans. (a) From Re1 = cos e1 + sin e2 , Re2 = sin e1 + cos e2 , Re3 = e3 , we have,
cos sin 0 0 sin 0
[ R ]e'i = sin cos 0 R = sin
A
0 0
e'i
0 0 1 e' 0 0 0 e'
i i
A
(b) the dual vector (or axial vector) of R is given by
Copyright 2010, Elsevier Inc
2-18
Lai et al, Introduction to Continuum Mechanics

t A = (T23 e1 + T31
e2 + T12 e3 ) = (0e1 + 0e2 sin e3 ) = sin e3 .
(c) The first scalar invariant of R is I1 = cos + cos + 1 = 1 + 2cos .
__________________________________________________________________
2.55 The rotation of a rigid body is described by Re1 = e2 , Re2 = e3 , Re3 = e1 . Find the axis
of rotation and the angle of rotation. Use the result of the previous problem.
------------------------------------------------------------------------------
Ans From the result of the previous problem, we have, the dual vector of R A is given by
t A = sin e3' , where e3' is in the direction of axis of rotation and is the angle of rotation. Thus, we
can obtain the direction of axis of rotation and the angle of rotation by obtaining the dual vector
of R A . From Re1 = e2 , Re2 = e3 , Re3 = e1 , we have,
0 0 1 0 1 1
A 1 1
[ R ] = 1 0 0 R = 1 0 1 t A = ( e1 + e2 + e3 ) . Thus ,

2 2
0 1 0 1 1 0
3 ( e1 + e2 + e3 ) 3 ' 1
tA = = e3 , where e3' = ( e1 + e2 + e3 ) is in the direction of the axis of
2 3 2 3
rotation and the angle of rotation is given by sin = 3 / 2 , which gives = 60o or 120o . On the
other hand, the first scalar invariant of R is 0. Thus, from the result in (c) of the previous problem,
we have, I1 = 1 + 2cos = 0 , so that cos = 1 / 2 which gives = 120o or 240o . We therefore
conclude that = 120o .
_________________________________________________________________
1 0 0
2.56 Given the tensor [Q ] = 0 1 0 . (a) Show that the given tensor is a rotation tensor. (b)
0 0 1
Verify that the eigenvalues are = 1 and 1 . (c) Find the direction for the axis of rotation (i.e.,
eigenvectors corresponding to = 1 ). (d) Find the eigenvectors corresponding = 1 and (e) obtain
the angle of rotation using the formula I1 = 1 + 2cos (see Prob. 2.54), where I1 is the first scalar
invariant of the rotation tensor.
-------------------------------------------------------------------------------
Ans. (a) det [Q ] = +1 , and [Q ][Q ] = [ I ] therefore it is a rotation tensor.
I

(b) The principal scalar invariants are: I1 = 1, I 2 = 1, I 3 = 1 characteristic equation is


( )
3 + 2 1 = ( + 1) 2 1 = 0 the eigenvalues are: = 1,1,1 .
(c) For = 1 , clearly, the eigenvector are: n = e3 , which gives the axis of rotation.
(d) For = 1 , with eigenvector n = 1e1 + 2e2 + 3e3 , we have
01 = 0, 0 2 = 0, 2 3 = 0 . Thus, 1 = arbitrary, 2 = arbitrary, 3 = 0 . The eigenvectors are:
n = 1e1 + 2e2 , 12 + 22 = 1 . That is, all vectors perpendicular to the axis of rotation are
eigenvectors.
(e) The first scalar invariant of Q is I1 = 1 . Thus, 1 + 2cos = 1 cos = 1 = . ( We
note that for this problem, the antisymmetric part of Q = 0 , so that t A = 0 = sin n , of which
= is a solution).
_________________________________________________________________
Copyright 2010, Elsevier Inc
2-19
Lai et al, Introduction to Continuum Mechanics

2.57 Let F be an arbitrary tensor. (a) Show that F T F and FF T are both symmetric tensors. (b) If
F = QU = VQ , where Q is orthogonal and U and V are symmetric, show that U 2 = F T F and
V 2 = FF T (c) If and n are eigenvalue and the corresponding eigenvector for U , find the
eigenvalue and eigenvector for V . [note corrections for text]
-------------------------------------------------------------------------------
Ans. (a) (F T F )T = F T (F T )T = F T F , thus F T F is symmetric. Also (FF T )T = (F T )T F T = FF T ,
therefore, FF T is also symmetric.
(b) F = QU F T = U T QT F T F = U T QT QU = U T U F T F = U 2 .
F = VQ F T = QT V T FF T = VQQT V T = VV T FF T = V 2 .
(c) Since F = QU = VQ , and Un = n , therefore, VQn = QUn = Q( n) V (Qn) = (Qn) ,
therefore, Qn is an eigenvector for V with the eigenvalue .
_________________________________________________________________
2.58 Verify that the second principal scalar invariant of a tensor T can be written:
( )
I 2 = TiiT jj TijT ji / 2 .
------------------------------------------------------------------------------
Ans. TiiT jj = (T11 + T22 + T33 )2 = T112 + T222
+ T332 + 2T11T22 + 2T22T33 + 2T33T11 .
TijT ji = T1 j T j1 + T2 jT j 2 + T3 jT j 3 = T112 + T12T21 + T13T31 + T21T12 + T22
2
+ T23T32 + T31T13 + T32T23 + T332 .
Thus, TiiT jj Tij T ji = (T112 + T22
2
+ T332 + 2T11T22 + 2T22T33 + 2T33T11 )
(T112 + T222
+ T332 + 2T12T21 + 2T13T31 + 2T23T32 ) = 2(T11T22 T12T21 + T22T33 T23T32 + T33T11 T13T31 ) .
Thus,
( )
TiiT jj TijT ji / 2 = (T11T22 T12T21 + T22T33 T23T32 + T33T11 T13T31 )
T11 T12 T22 T23 T11 T13
= + + = I2 .
T21 T22 T32 T33 T31 T33
_________________________________________________________________
2.59 A tensor has a matrix [ T] given below. (a) Write the characteristic equation and find the
principal values and their corresponding principal directions. (b) Find the principal scalar
invariants. (c) If n1 , n 2 , n3 are the principal directions, write [ T]n . (d) Could the following matrix
i

[S ] represent the same tensor T with respect to some basis.


5 4 0 7 2 0
[T] = 4 1 0 , [S ] = 2 1 0 .

0 0 3 0 0 1
------------------------------------------------------------------------------
Ans.
(a) The characteristic equation is:
5 4 0
4 1 0 = 0 (3 ) [ (5 )(1 ) 16] = (3 )( 2 4 21) = (3 )( + 3)( 7) = 0
0 0 3
Thus, 1 = 3, 2 = 3, 3 = 7 .
For 1 = 3, clearly, n1 = e3 .
Copyright 2010, Elsevier Inc
2-20
Lai et al, Introduction to Continuum Mechanics

For 2 = 3
(5 + 3)1 + 4 2 = 0, 41 + (1 + 3) 2 = 0, (3 + 3) 3 = 0, 12 + 22 + 32 = 1
81 + 4 2 = 0, 41 + 2 2 = 0, 6 3 = 0 2 = 21 , 3 = 0. n 2 = (e1 2e 2 ) / 5.
For 3 = 7
(5 7)1 + 4 2 = 0, 41 + (1 7) 2 = 0, (3 7) 3 = 0, 12 + 22 + 32 = 1
21 + 4 2 = 0, 41 8 2 = 0, 4 3 = 0 1 = 2 2 , 3 = 0. n3 = (2e1 + e 2 ) / 5.
(b) The principal scalar invariants are:
I1 = 5 1 + 3 = 7, I 2 = (5 16) + (3 0) + (15 0) = 9, I 3 = 15 48 = 63 . We note that
3 7 2 9 + 63 = 0 ( 7 ) 2 9 ( 7 ) = 0 ( 7 ) ( 2 9) = 0 , same as obtained in (a)
3 0 0 7 2 0

(c) [ T]n = 0 3 0 . (d) det 2 1 0 = 3 63 , therefore, the answer is NO. Or,
i
0 0 7 n ,n ,n 0 0 1
{ 1 2 3}
clearly one of the eigenvalue for [S ] is 1 , which is not an eigenvalue for [ T] , therefore the
answer is NO.
_________________________________________________________________
2.60 Do the previous problem for the following matrix:
3 0 0
[T] = 0 0 4
0 4 0
------------------------------------------------------------------------------
Ans. (a) The characteristic equation is:
3 0 0
0 0 4 = 0 (3 )( 2 16) = (3 )( 4)( + 4) = 0
0 4 0
Thus, 1 = 3, 2 = 4, 3 = 4 .
For 1 = 3, clearly, n1 = e1 , because Te1 = 3e1 .
For 2 = 4
(3 4)1 = 0, (0 4) 2 + 4 3 = 0, 4 2 + (0 4) 3 = 0, 12 + 22 + 32 = 1
1 = 0, 4 2 + 4 3 = 0, 4 2 4 3 = 0 1 = 0, 2 = 3 , n 2 = (e 2 + e3 ) / 2.
For 3 = 4
(3 + 4)1 = 0, (0 + 4) 2 + 4 3 = 0, 4 2 + (0 + 4) 3 = 0, 12 + 22 + 32 = 1
71 = 0, 4 2 + 4 3 = 0, 4 2 + 4 3 = 0 1 = 0, 2 = 3 , n3 = (e 2 e3 ) / 2
(b)
I1 = 3, I 2 = (0 0) + (0 16) + (0 0) = 16, I 3 = 48 .
3 3 2 16 + 48 = 0 ( 3) 2 16 ( 3) = 0 ( 3) ( 2 16) = 0 , same as in (a) .
3 0 0
(c) [ T]n = 0 4 0
i
0 0 4 n ,n ,n
{ 1 2 3}

Copyright 2010, Elsevier Inc


2-21
Lai et al, Introduction to Continuum Mechanics

7 2 0
(d) det [S ] = det 2 1 0 = (7 4) = 3 48 , therefore, the answer is NO.
0 0 1
Or, clearly one of the eigenvalue for [S ] is 1 , which is not an eigenvalue for [ T] , therefore the
answer is NO.
_________________________________________________________________
2.61 A tensor T has a matrix given below. Find the principal values and three mutually
1 1 0
perpendicular principal directions: [ T] = 1 1 0
0 0 2
------------------------------------------------------------------------------
Ans. The characteristic equation is:
1 1 0
0 = 0 ( 2 ) (1 )2 1 = ( 2 ) (2 + 2 ) = ( 2 ) = 0 .
2
1 1

0 0 2
Thus, 1 = 0, 2 = 3 = 2 . That is, there is a double root 2 = 3 = 2 .
For 1 = 0 ,
(1 0)1 + 2 = 0, 1 + (1 0) 2 = 0, (2 0) 3 = 0, 12 + 22 + 32 = 1
1 + 2 = 0, 2 3 = 0 1 = 2 , 3 = 0, n1 = (e1 e 2 ) / 2.
For 2 = 3 = 2 , one eigenvector is clearly n3 . There are infinitely many others all lie on the plane
whose normal is n1 = (e1 e 2 ) / 2 . In fact, we have,
(1 2)1 + 2 = 0, 1 + (1 2) 2 = 0, (2 2) 3 = 0, 12 + 22 + 32 = 1
1 + 2 = 0, 0 3 = 0 1 = 2 = , 3 = 1 2 2 n = ( e1 + e 2 + 3e3 ),
which include the case where = 0, 3 = 1 n = e3 .
_________________________________________________________________

CHAPTER 2, PART C
d dT dS
2.62 Prove the identity ( T + S ) = + , using the definition of derivative of a tensor.
dt dt dt
------------------------------------------------------------------------------
Ans.
d {T(t + t ) + S(t + t )} {T(t ) + S(t )} {T(t + t ) T(t )} + {S(t + t ) S(t )}
( T + S ) = lim = lim
dt t 0 t t 0 t
_
{T(t + t ) T(t )} {{S(t + t ) S(t )} dT dS
lim + lim = + .
t 0 t t 0 t dt dt
_________________________________________________________________
d dS dT
2.63 Prove the identity ( TS ) = T + S using the definition of derivative of a tensor.
dt dt dt
------------------------------------------------------------------------------

Copyright 2010, Elsevier Inc


2-22
Lai et al, Introduction to Continuum Mechanics

d T(t + t )S(t + t ) T(t )S(t )


Ans. ( TS ) = lim
dt t 0 t
T(t + t )S(t + t ) T(t + t )S(t ) + T(t + t )S(t ) T(t )S(t )
= lim
t 0 t
T(t + t ){S(t + t ) S(t )} {T(t + t ) T(t )}S(t )
= lim + lim
t 0 t t 0 t
{S(t + t ) S(t )} {T(t + t ) T(t )} dS dT
= T(t ) lim + lim S(t ) = T + S.
t 0 t t 0 t dt dt
_________________________________________________________________
T
dT T dT T
2.64 Prove that = by differentiating the definition a Tb = b T a , where
dt dt
a and b are constant arbitrary vectors.
------------------------------------------------------------------------------
Ans. a Tb = b TT a a ( dT / dt )b = b (dTT / dt )a . Now, the definition of transpose also gives
a (dT / dt )b = b ( dT / dt ) a . Thus, b ( dT / dt ) a = b (dTT / dt )a .
T T

T
dT dT T
Since a and b arbitrary vectors, therefore, = .
dt dt
_________________________________________________________________
2.65 Consider the scalar field = x12 + 3x1 x2 + 2 x3 . (a) Find the unit vector normal to the surface
of constant at the origin (0,0,0) and at (1,0,1) . (b) what is the maximum value of the directional
derivative of at the origin? At (1,0,1)? (c) Evaluate d / dr at the origin if dr = ds (e1 + e3 ) .
------------------------------------------------------------------------------
Ans. (a) = (2 x1 + 3 x2 )e1 + 3 x1e 2 + 2e3 ,
at (0,0,0), = 2e3 n = e3 , at (1,0,1), = 2e1 + 3e2 + 2e3 , n = ( 2e1 + 3e2 + 2e3 ) / 17 .
(b) At (0,0,0), (d / dr ) max = = 2 in the direction of n =e3 .
At (1,0,1) , (d / dr ) max = = 17 .
(c) At (0,0,0), d / dr = ( )o dr / dr = 2e3 (e1 + e3 ) / 2 = 2 .
_________________________________________________________________
2.66 Consider the ellipsoidal surface defined by the equation x 2 / a 2 + y 2 / b 2 + z 2 / c 2 = 1 . Find
the unit vector normal to the surface at a given point ( x, y, z ) .
------------------------------------------------------------------------------
x2 y 2 z 2
Ans. Let f ( x, y , z ) = 2 + 2 + 2 1 , then
a b c
f 2 x f 2 y f 2 z 2x 2y 2z
= 2, = 2, = 2 f = 2 e1 + 2 e 2 + 2 e3 , thus,
x a y b z c a b c
1/2
f 2 x 2 y 2 z
2 2 2
2x 2y 2z
n= = 2 + 2 + 2 2 e1 + 2 e 2 + 2 e3 .
f a b c a b c
_________________________________________________________________

Copyright 2010, Elsevier Inc


2-23
Lai et al, Introduction to Continuum Mechanics

2.67 Consider the temperature field given by: = 3x1 x2 . (a) Find the heat flux at the point
A(1,1,1) , if q = k . (b) Find the heat flux at the same point if q = K , where
k 0 0
[K ] = 0 2k 0
0 0 3k
------------------------------------------------------------------------------
Ans. = 3x1 x2 = 3( x2e1 + x1e2 ) () A = 3(e1 + e2 ) .
(a) q = k = 3k (e1 + e2 ) .
k 0 0 3 3k
(b) [q ] = [ K ] = 0 2k 0 3 = 6k q = (3ke1 + 6ke 2 ) .

0 0 3k 0 0
_________________________________________________________________
2.68 Let ( x1 , x2 , x3 ) and ( x1 , x2 , x3 ) be scalar fields, and let v( x1 , x2 , x3 ) and w ( x1 , x2 , x3 ) be
vector fields. By writing the subscripted components form, verify the following identities.
( + )
(a) ( + ) = + , sample solution: [( + )]i = = + = + .
xi xi xi
(b) div( v + w ) = divv + divw , (c) div( v ) = ( ) v + (divv ) and (d) div(curlv ) = 0 .
------------------------------------------------------------------------------
(vi + wi ) vi wi
Ans. (b) div( v + w ) = = + = divv + divw .
xi xi xi
( vi ) v
(c) div( v ) = = i + vi = (divv ) + ( ) v .
xi xi xi
v j vk v vk
(d) curlv = ijk ei = ijk ei div(curlv) = ijk k = ijk .
xk x j xi x j xi x j
vk vk
By changing the dummy indices, ( i j , j i ) we have, ijk = jik . Thus,
xi x j x j xi

vk vk vk v
ijk = ijk 2 ijk =0 ijk k
= 0 . Thus, div(curlv ) = 0 .
xi x j x j xi
xi x j
xi x j
_________________________________________________________________
2.69 Consider the vector field v = x12e1 + x32e 2 + x22e3 . For the point (1,1,0) , find (a) v , (b)
(v) v , (c) div v and curl v and (d) the differential dv for dr = ds (e1 + e 2 + e3 ) / 3 .
------------------------------------------------------------------------------
2 x1 0 0 2 0 0

Ans.(a) [v ] = 0 0 2 x3 [v ](1,1,0 ) = 0 0 0 .

0 2 x2 0 0 2 0
2 0 0 1 2
(b) [ (v) v ] = 0 0 0 0 = 0 (v) v = 2e1 .
0 2 0 1 0
(c) div v = 2 x1 + 0 + 0 = 2 x1 at (1,1,0), div v = 2 .

Copyright 2010, Elsevier Inc


2-24
Lai et al, Introduction to Continuum Mechanics

v v v v v v
curlv = 3 2 e1 + 1 3 e 2 + 2 1 e3 = 2 ( x2 x3 ) e1 .
x2 x3 x3 x1 x1 x2
At (1,1,0) , curlv = 2 (1 0 ) e1 = 2e1 .
(d)
2 0 0 ds / 3 2ds / 3

At (1,1,0 ) , dv = ( v ) dr [ dv ] = [v ][ dr ] = 0 0 0 ds / 3 = 0 .
0 2 0 ds / 3 2ds / 3

dv = 2ds (e1 + e3 ) / 3
_________________________________________________________________

CHAPTER 2, PART D

2.70 Calculate div u for the following vector field in cylindrical coordinates:
(a) ur = u = 0, u z = A + Br 2 . (b) ur = sin / r , u = u z = 0 , and
(c) ur = r 2 sin / 2, u =r 2 cos / 2, u z = 0 .
------------------------------------------------------------------------------
u 1 u ur u z
Ans.(a) ur = u = 0, u z = A + Br 2 div u = r + + + =0+0+0+0=0.
r r r z
u 1 u ur u z
(b) ur = sin / r , u = u z = 0 div u = r + + + = sin / r 2 + 0 + sin / r 2 + 0 = 0
r r r z
(c) ur = r 2 sin / 2, u =r 2 cos / 2, u z = 0
u 1 u ur u z
div u = r + + + = r sin r sin / 2 + r sin / 2 + 0 = r sin .
r r r z
_________________________________________________________________
2.71 Calculate u for the following vector field in cylindrical coordinate:
ur = A / r , u = Br , u z = 0 .
------------------------------------------------------------------------------
ur 1 ur ur A
r r u z 2 B 0
r
u 1 u u
0 .
A
Ans. [u ] = + ur = B
r r z 2
r

u z 1 u z u z 0 0 0

r r z
_________________________________________________________________
2.72 Calculate div u for the following vector field in spherical coordinates
ur = Ar + B / r 2 , u = u = 0
------------------------------------------------------------------------------
1 ( r 2 ur ) 1 (u sin ) 1 u
Ans. div u = 2 + +
r r r sin r sin

Copyright 2010, Elsevier Inc


2-25
Lai et al, Introduction to Continuum Mechanics

=
r
1 2 B 1
r Ar + 2 = 2
2 r
r r r
Ar 3 + B = 3 A. ( )
_________________________________________________________________
2.73 Calculate u for the following vector field in spherical coordinates:
ur = Ar + B / r 2 , u = u = 0 .
------------------------------------------------------------------------------
ur 1 ur u 1 ur u
r r
r r sin r
u 1 u u cot
1 u ur
Ans. [u ] = r + r
r r sin r

u 1 u 1 u ur u cot
+ +

r
r r sin r r
ur / r 0 0 A 2B / r 3 0 0

= 0 ur / r 0 = 0 A + B / r3 0 .
0 0 ur / r 0 0 3
A+ B / r
_________________________________________________________________
2.74 From the definition of the Laplacian of a vector, 2 v = ( div v ) curlcurl v , derive the
following results in cylindrical coordinates:
2v 1 2 vr 2 vr 2 v 1 vr vr
( )
2 v = 2r + 2
r
+ + and
r 2 z 2 r 2 r r r 2
r

2 v 1 2 v 1 v 2 v2 v v
( 2 v )r
= 2
r
+ 2
z 2
+ + 2 r 2 .
r r r r
2
+

---------------------------------------------------------------------------
Ans. Let v ( r ) be a vector field. The Laplacian of v is 2 v = ( div v ) curlcurl v . Now,
vr 1 v vr vz
div v = + + + , so that
r r r z
v 1 v vr vz 1 vr 1 v vr vz
( div v ) = r + + + er + + + +
r r r r z r r r r z e

vr 1 v vr vz 2 vr 1 2 v 1 v 1 vr vr 2 vz
+ + + + = +
z 2 r r + 2+ e r
z r r
e 2
r z r r r r r r z
1 vr
2 2
1 v 1 vr 1 vz
2 vr 1 v 1 vr vz
2 2 2
+ + + + e + + + + 2 ez .
r r r 2 2 r 2 r z zr r z r z z

Next,
1 vz v v v v v 1 vr
curl v = e r + r z e + + ez , so that
r z z r r r r

Copyright 2010, Elsevier Inc


2-26
Lai et al, Introduction to Continuum Mechanics

1 v v 1 vr vr vz
( curlcurl v )r = + z z r
r r r r
1 2 v 1 v 1 2 vr 2 vr 2 vz
= + 2 2
r r r r 2 z 2 zr
,

1 v v v 1 v
( curlcurl v ) = z + r
v
z r z r r r r
v v 1 v v 1 1 vz v
( curlcurl v ) z = r z + r z
r z r r z r r r z
Thus,
2 v 1 2 v 1 v 1 vr vr 2 vz
( )
2 v = 2r +

2 + +
r r r r r r 2 r z

r
r
1 2 v 1 v 1 2 vr 2 vr 2 vz 2 vr 1 2 vr 2 vr 2 v 1 vr vr
+ 2 2 + = + + +
r r r r 2 z 2 zr r 2 r 2 2 z 2 r 2 r r r 2
.

1 vr
2 2
1 v 1 v 1 vz
2
( 2 v = )
+ 2 + 2 r +
r r z

r r r
2

1 vz v v v 1 vr 1 2 vr 1 2 v 1 vr 1 2 vz
+ + = + + +
z r z r r r r r r r 2 2 r 2 r z
1 2 vz 2 v 2 v 1 v v 1 2 vr 1 vr
+ + + + +
r z z 2 r 2 r r r 2 r r r 2

2 v 1 2 v
1 v 2 v v 2 v
= + + 2 r 2 .
+ +
r 2
r z r r r r
2 2 2

_________________________________________________________________
2.75 From the definition of the Laplacian of a vector, 2 v = ( div v ) curlcurl v , derive the
following result in spherical coordinates:
1 2 r 2 vr 2 r 2 vr 1 2 vr cot vr 2 vr 2 v sin 2 v
( 2 v ) = 2
r r 2
3 r
+ 2
r 2
+ 2
+ 2
1
2
r sin 2
2
r sin
2
r sin
r
r r
------------------------------------------------------------------------------
Ans.

From 2
2
1 r vr
+
( )
1 ( v sin )
+
1 v
, we have,
r r r sin r sin

1 r 2 vr 1 v sin 1 v 1 1 r vr
2
1 v sin 1 v
( divv ) = 2 + + er + 2 + + e
r r r r sin r sin r r r r sin r sin

1 1 r 2 vr
1 v sin 1 v
+ 2 ++ + e , that is
r sin r r r sin r sin

1 2 r 2 vr
( divv ) =
( ) 2
2 r vr
+
( )2
1 v sin

1 v sin
+
1 v
2


1 v
er
r 2 r 2 3
r r sin r 2
r sin r sin r r 2 sin
r

Copyright 2010, Elsevier Inc


2-27
Lai et al, Introduction to Continuum Mechanics

+
( )
1 2 r 2 vr
+
1 ( v sin )
2

+
1
2
v sin

1 cos v sin 1
+
1 v
e
r
3
r 2
r sin
2 2
r sin
2
r sin
2 2
r r sin


+ 3
1 (
r vr
2
)
+ 2 2
1 2 ( v sin )
+ 2 2
1 2 v
e . Also,
r sin r r sin r sin 2

1 v sin 1 v 1 vr 1 rv 1 rv 1 vr
curl v = er + e +
r
e
r sin r sin r sin r r r r
so that
1 1 rv 1 vr 1 vr 1 rv
curlcurl v =
r sin
sin r sin r r er
r r r
1 1 v sin 1 v 1 1 rv 1 vr
+ e
r sin r r r r
r
r sin r sin r
1 1 vr 1 rv 1 1 v sin 1 v
+ r e
r r r sin r r r r sin r sin
i.e.,
curlcurl v =
1 2 rv 2 vr cot 1 rv 1 vr 1 2 vr 1 rv
2

2 2
+ er
r r
r r r r r 2 sin 2 2 r 2 sin r

1 2 v sin 2 v 1 rv
vr 1 2 rv 1 rv 1 2 vr 1 vr
+ 2 2 + e
r sin 2 r 2 r r r 2 r 2 r r r r 2

1 vr 1 1 vr 1 rv 1 rv 1 1 vr rv
2
2 + 2 +
r sin r r sin r r 2 r r r 2 sin r

+ e
1 1 2 v 2 v cos v sin v
2 v sin + sin +
r sin 2 r 2 sin 2

Thus, 2 v = ( divv ) curlcurl v gives:

2 r 2v ( 2
)
2 r vr ( )
1 ( v sin )
2
1 ( v sin ) 1 v
2
1 v

( 2
v = 2
r
)
r
1
r 2
r
3
r r
+
r sin r
2
r sin
+
r sin r r 2 sin

1 rv vr cot 1 rv 1 vr 1 rv
2 2 2
1 2 vr
2 +

r r
2 r r r r r 2 sin 2 2 r 2 sin r

i.e.,
1 2 r 2 vr 2 r 2 vr 1 2 vr cot vr 2 vr 2 v sin 2 v
( v )
2
= 2
r r 2
3
r r
+ 2
r 2
+ 2
1
+ 2 2
r r sin 2
2
r sin
2
r sin
r

_________________________________________________________________

Copyright 2010, Elsevier Inc


2-28
Lai et al, Introduction to Continuum Mechanics

2.76 From the equation (divT) a = div(TT a) tr(TT a) [See Eq. 2.29.3)] verify that in polar
T 1 T Tr + T r
coordinates, the -component of the vector (divT) is: (divT) = r + + .
r r r
------------------------------------------------------------------------------
Ans. (divT) a = div(TT a) tr(TT a) (divT) e = div(TT e ) tr(TT e )
Now,
Te r = Trr e r + T r e , Te = Tr e r + T e e r TT e = e Te r = T r , e TT e = e Te = T
T 1 T T r
i.e., TT e = T r e r + T e div(TT e ) = r + + . Also
r r r
0 1 / r T T T r 0 1 / r 0 Trr / r
e = 0e r + 1e [e ] = T e = rr =
0 0
Tr T 0 0 0 Tr / r
Thus,
T r 1 T T r T 1 T T r + Tr
(divT) = div(TT e ) tr(TT e ) = + + (0 Tr / r ) = r + + .
r r r r r r
_________________________________________________________________
2.77 Calculate div T for the following tensor field in cylindrical coordinates;
B B
Trr = A + 2 , T = A 2 , Tzz = constant, Tr = T r = Trz = Tzr = T z = Tz = 0
r r
------------------------------------------------------------------------------
T 1 Tr Trr T Trz 2B 2B
Ans. (divT)r = rr + + + = 3 + 3 = 0.
r r r z r r
T 1 T Tr + T r T z
(divT) = r + + + = 0.
r r r z
T 1 Tz Tzz Tzr
(divT) z = zr + + + =0.
r r z r
_________________________________________________________________
2.78 Calculate divT for the following tensor field in cylindrical coordinates;
Az 3Br 2 z Az Az 3Bz 3 Ar 3Brz 2
Trr = 3 , T = , T = + , T = T = 3 +
R5 R3
zz R3 R5
rz zr
R5
R R
Tr = T r = T z = Tz = 0, R 2 = r 2 + z 2 .
------------------------------------------------------------------------------
Ans.
T 1 Tr Trr T Trz Az 3Br 2 z 3Brz Ar 3Brz 2
(divT) r = rr + + + = 3 3+
r r r z r R R5 R5 z R R 5
1 1 3Bz 2 3Brz 1 3Br 2 1
= Az 3Br 2 z 5 r 5 Ar + 5 z + 3Brz 2
r R 3 r R 5
R r R z R 3
R z z R5
3 Az R 15Br 2 z R 6 Brz 3Brz 3 Ar R 6 Bzr 15Brz 2 R
= 4 + 5 5 4 + 5
R r R 6 r R R R z R 6 z
R
3 Arz 15Br 3 z 6 Brz 3Brz 3 Arz 6 Bzr 15Brz 3
= 5 + 5 5 + 5 5 +
R R7 R R R R 7
R

Copyright 2010, Elsevier Inc


2-29
Lai et al, Introduction to Continuum Mechanics

15r 3 z 15rz 15rz 3


R
15rz
R
(
R R
)
15rz 15rz 15rz
= B 7 5 + 7 = B 7 r2 + z2 5 = 5 5 = 0 .
R
R R

T r 1 T Tr + T r T z
(divT) = + + + =0+0+0+0=0
r r r z
T 1 Tz Tzz Tzr A 3Brz 2 Az 3Bz 3 A 3Bz 2
(divT) z = zr + + + = 3 + + 5 3 + 5
r r z r r R R5 z R3 R R R
A 3 Ar 2 3Bz 2 15 Br 2 z 2 A 3 Az 2 9 Bz 2 15Bz 4 A 3Bz 2
= 3 5 + 5 5 + 5 + 5
R R 7 R 3 R 7 R 3 R
R R R R
3A 3A 15 Bz 2 15 Bz 2 2 2 3 A 3 A 15 Bz 2 15Bz 2
R (
= 3 + 5 r2 + z2 ) R5
+
R7
( r +) z ==
R3 R3
+
R5
+ =0.
R 5
R
_________________________________________________________________
2.79 Calculate div T for the following tensor field in spherical coordinates;
2B B
Trr = A 3 , T = T = A + 3 , Tr = T r = T = T = Tr = T r = 0
r r
------------------------------------------------------------------------------
1 ( r Trr ) 1 (Tr sin )
2
1 Tr T + T
Ans. (divT) r = 2 + + -
r r r sin r sin r
(
1 r Trr
2
)T +T
= 2
1 2 2B A B
= 2 Ar 2 +
r r r r r r r r4
1 2B A B 2 A 2B A B
= 2 2 Ar + 2 2 + 4 = + 4 2 + 4 = 0.
r r r r r r r r

1 ( r T r ) 1 (T sin )
3
1 T Tr T r T cot
(divT) = 3 + + +
r r r sin r sin r
T cot T cot
= + = 0.
r r

1 ( r T r ) 1 (T sin )
3
1 T Tr T r + T cot
(divT) = 3 + + + = 0.
r r r sin r sin r
_________________________________________________________________
2.80 From the equation (divT) a = div(TT a) tr(TT a) [See Eq. 2.29.3)] verify that in
spherical coordinates, the -component of the vector (divT) is:
1 (r 3T r ) 1 (T sin ) 1 T Tr T r T cot
(divT) = + ++ + .
r 3 r r sin r sin r
------------------------------------------------------------------------------
Ans. (divT) a = div(TT a) tr(TT a) (divT) e = div(TT e ) tr(TT e ) . Now,

Copyright 2010, Elsevier Inc


2-30
Lai et al, Introduction to Continuum Mechanics

1 (r 2T r ) 1 (T sin ) 1 T
TT e = T r e r + T e + T e div(TT e ) = + + . Also,
r 2 r r sin r sin
0 1 / r 0

e = 0e r + 1e + 0e [e ] = 0 0 0
0 0 cot / r
Trr T r T r 0 1 / r 0 0 Trr / r T r cot / r

TT e = Tr T T 0 0 0 = 0 Tr / r T cot / r

0 0 cot / r 0 Tr / r T cot / r
Tr T T
T T cot
tr TT e = r + . Thus,
r r
(divT) = div(TT e ) tr(TT e )
1 (r 2T r ) 1 (T sin ) 1 T Tr T cot
= + + +
r 2 r r sin r sin r r
1 (r 3T r ) T r 1 (T sin ) 1 T Tr T cot
= + + + .
r3 r r r sin r sin r r
_________________________________________________________________

Copyright 2010, Elsevier Inc


2-31
Lai et al, Introduction to Continuum Mechanics

CHAPTER 3

3.1 Consider the motion: x1 = (1 + kt ) X1 / (1 + kto ), x2 = X 2 , x3 = X 3 .


(a) Show that reference time is t = to . (b) Find the velocity field in spatial coordinates. (c) Show
that the velocity field is identical to that of the following motion:
x1 = (1 + kt ) X1 , x2 = X 2 , x3 = X 3 .
-----------------------------------------------------------------------------------
Ans. (a) At t = to , x1 = X1 , x2 = X 2 , x3 = X 3 . Thus, t = to is the reference time.
(b) In material description, v1 = kX1 / (1 + kto ), v2 = v3 = 0 . Now, from x1 = (1 + kt ) X1 / (1 + kto ) ,
X1 = (1 + kto ) x1 / (1 + kt ) , therefore, v1 = kX1 = kx1 / (1 + kt ), v2 = v3 = 0 .
(c) For x1 = (1 + kt ) X1 , x2 = X 2 , x3 = X 3 , v1 = kX1 , v2 = v3 = 0
v1 = kx1 / (1 + kt ), v2 = v3 = 0 , which are the same as the velocity components in (b).
_________________________________________________________________

3.2 Consider the motion: x1 = t + X1 , x2 = X 2 , x3 = X 3 , where the material coordinates X i


designate the position of a particle at t = 0 . (a) Determine the velocity and acceleration of a
particle in both a material and a spatial description. (b) If the temperature field in spatical
description is given by = Ax1 , what is its material description? Find the material derivative of
, using both descriptions of the temperature. (c) Do part (b) if the temperature field is = Bx2
-----------------------------------------------------------------------------------
Ans. (a) Material description: v1 = Dx1 / Dt = ( x1 / t ) X fixed = , v2 = v3 = 0 ,
i

a1 = Dv1 / Dt = ( v1 / t ) X = 0, a2 = a3 = 0 .
i fixed

Spatial description: The same as above v1 = , v2 = v3 = 0, a1 = a2 = a3 = 0 ..


(b) The material description of is = A ( t + X1 ) .
Using the material description: = A ( t + X1 ) D / Dt = ( / t ) A ( t + X1 ) = A .
Using the spatical description: = Ax1
D
= + v1 + v2 + v3 = 0 + ( A) + (0)(0) + (0)(0) = A .
Dt t x1 x2 x3
(c) Using the material description: = BX 2 D / Dt = ( / t )( BX 2 ) = 0 .
Using the spatical description: = Bx2
D
= + v1 + v2 + v3 = 0 + (0) + ( 0 )( B ) + ( 0 )( 0 ) = 0 .
Dt t x1 x2 x3
_______________________________________________________________________

3.3 Consider the motion


x1 = X1 , x2 = X12t 2 + X 2 , x3 = X 3 , where X i are the material coordinates. (a) at t = 0 , the
corners of a unit square are at A(0,0,0), B(0,1,0), C (1,1,0) and D(1,0,0) . Determine the position
of ABCD at t = 1 and sketch the new shape of the square. (b) Find the velocity v and the
acceleration in a material description and (c) Find the spatial velocity field.
-----------------------------------------------------------------------------------------
Copyright 2010, Elsevier Inc
3-1
Lai et al, Introduction to Continuum Mechanics

Ans. For the material line AB, ( X1 , X 2 , X 3 ) = ( 0, X 2 ,0 ) ; at t = 1 , ( x1 , x2 , x3 ) = ( 0, X 2 ,0 )

( )
For the material line BC , ( X1 , X 2 , X 3 ) = ( X1 ,1,0 ) ; at t = 1 , ( x1 , x2 , x3 ) = X1 , X12 + 1,0

For the material line AD, ( X1 , X 2 , X 3 ) = ( X ,0,0 ) ; at t = 1 , ( x , x , x ) = ( X , X ,0 )


1 1 2 3 1
2
1

For the material line CD, ( X1 , X 2 , X 3 ) = (1, X 2 ,0 ) ; at t = 1 , ( x1 , x2 , x3 ) = (1, + X 2 ,0 )


The shape of the material square at t = 1 is shown in the figure.

x v
(b) vi = i , ai = i v1 = v3 = 0, v2 = 2 X12t ; a1 = a3 = 0, a2 = 2 X12
t X i fixed t X i fixed
(c) Since x1 = X1 , in spatial descrip. v1 = v3 = 0, v2 = 2 x12t ; a1 = a3 = 0, a2 = 2 x12
__________________________________________________________________

3.4 Consider the motion: x1 = X 22t 2 + X1 , x2 = kX 2t + X 2 , x3 = X 3


(a) At t = 0 , the corners of a unit square are at A(0,0,0), B(0,1,0), C (1,1,0) and D(1,0,0) . Sketch
the deformed shape of the square at t = 2 . (b) Obtain the spatial description of the velocity field.
(c) Obtain the spatial description of the acceleration field.
---------------------------------------------------------------------------------------------
Ans. (a)
(
For material line AB, ( X1 , X 2 , X 3 ) = ( 0, X 2 ,0 ) ; at t = 2 , ( x1 , x2 , x3 ) = 4 X 22 , 2kX 2 + X 2 ,0 .)
For material line BC , ( X1 , X 2 , X 3 ) = ( X1 ,1,0 ) ; at t = 2 , ( x1 , x2 , x3 ) = ( 4 + X1 , 2k + 1,0 ) .
For material line AD, ( X1 , X 2 , X 3 ) = ( X1 ,0,0 ) ; at t = 2 , ( x1 , x2 , x3 ) = ( X1 ,0,0 ) .

(
For mat. line CD, ( X1 , X 2 , X 3 ) = (1, X 2 ,0 ) ; at t = 2 , ( x1 , x2 , x3 ) = 4 X 22 + 1, 2kX 2 + X 2 ,0 . )
The shape of the material square at t = 2 is shown in the figure.
x2
4
B C
2k
C
B

A x1
D

Copyright 2010, Elsevier Inc


3-2
Lai et al, Introduction to Continuum Mechanics

b) vi = i v
x 2 2
, ai = i , v1 = 2 X 2 t , v2 = kX 2 , v3 = 0; a1 = 2 X 2 , a2 = a3 = 0 .
t X fixed
i
t X fixed
i

2 x22t kx2 2 x22


(c) x2 = ( kt + 1) X 2 , v1 = , v2 = , v3 = 0; a1 = , a2 = a3 = 0 .
(1 + kt ) 2
(1 + kt ) (1 + kt )2
_________________________________________________________________

3.5 Consider the motion: x1 = k ( s + X1 ) t + X1 , x2 = X 2 , x3 = X 3 .


(a) For this motion, repeat part (a) of the previous problem. (b) Find the velocity and acceleration
as a function of time of a particle that is initially at the orgin. (c) Find the velocity and
acceleration as a function of time of the particles that are passing through the origin.
-----------------------------------------------------------------------------------
Ans. a) For material line AB, ( X1 , X 2 , X 3 ) = ( 0, X 2 ,0 ) ; at t = 2 , ( x1 , x2 , x3 ) = ( 2ks, X 2 ,0 ) .
For material line BC , ( X1 , X 2 , X 3 ) = ( X1 ,1,0 ) ; at t = 2 , ( x1 , x2 , x3 ) = ( 2ks + 2kX1 + X1 ,1,0 ) .
For material line AD, ( X1 , X 2 , X 3 ) = ( X1 ,0,0 ) ; at t = 2 , ( x1 , x2 , x3 ) = ( 2ks + 2kX1 + X1 ,0,0 ) .
For material line CD, ( X1 , X 2 , X 3 ) = (1, X 2 ,0 ) ; at t = 2 , ( x1 , x2 , x3 ) = ( 2ks + 2k + 1, X 2 ,0 ) .
The shape of the material square at t = 2 is shown in the figure.
x2
2ks
2k(s+1)
B B C
C
1

A D
x1
A D

(b) vi = i v
x
and ai = i , v1 = k ( s + X 1 ) , v2 = 0, v3 = 0; a1 = a2 = a3 = 0 .
t X fixed
i
t X fixed
i

Thus, for the particle ( X1 , X 2 , X 3 ) = ( 0,0,0 ) , v1 = ks, v2 = 0, v3 = 0 and a1 = 0, a2 = 0, a3 = 0


(c) x1 = k ( s + X 1 ) t + X 1 x1 = kst + ( kt + 1) X 1 X 1 = ( x1 kst ) / (1 + kt ) ,
thus, in spatial descriptions,
x kst k ( s + x1 )
v1 = k s + 1 = , v = 0, v3 = 0 and a1 = 0, a2 = 0, a3 = 0 .
(1 + kt ) (1 + kt ) 2
At the position ( x1 , x2 , x3 ) = ( 0,0,0 ) , v1 = ks / (1 + kt ), v2 = 0, v3 = 0 and a1 = 0, a2 = 0, a3 = 0 .
_________________________________________________________________

3.6 The position at time t of a particle initially at ( X 1 , X 2 , X 3 ) is given by

x1 = X 1 2 X 22t 2 , x2 = X 2 kX 3t , x3 = X 3 , where = 1 and k = 1 .


(a) Sketch the deformed shape, at time t = 1 of the material line OA which was a straight line at
t = 0 with the point O at ( 0,0,0 ) and the point A at ( 0,1,0 ) . (b) Find the velocity at t = 2 , of the
particle which was at (1,3,1) at t = 0 . (c) Find the velocity of the particle which is at (1,3,1) at
t=2.
Copyright 2010, Elsevier Inc
3-3
Lai et al, Introduction to Continuum Mechanics

---------------------------------------------------------------------------------------------
Ans. With = 1 and k = 1 , x1 = X 1 2 X 22t 2 , x2 = X 2 X 3t , x3 = X 3
For the material line OA : ( X 1 , X 2 , X 3 ) = (0, X 2 , 0) : at t = 1 , x1 = 2 X 22 , x2 = X 2 , x3 = 0 . Thus,
the deformed shape of the material line at t = 1 is a parabola given in the figure shown.

x2
A A

x1
O

(b) v1 = Dx1 / Dt = 4 X 22t , v2 = Dx2 / Dt = X 3 , v3 = Dx3 / Dt = 0


For the particle ( X 1 , X 2 , X 3 ) = (1,3,1) , at t = 2 , v1 = 4(3) 2 (2) = 72, v2 = 1, v3 = 0.
(c) The particle, which is at ( x1 , x2 , x3 ) = (1,3,1) at t = 2 , has the material coordinates given by the
following equations: 1 = X 1 8 X 22 , 3 = X 2 2 X 3 , 1 = X 3 X 1 = 201, X 2 = 5, X 3 = 1
v1 = 4 X 22 t = 4(5) 2 (2) = 200, v2 = X 3 = 1, v3 = 0.
_________________________________________________________________

3.7 The position at time t of a particle initially at ( X 1 , X 2 , X 3 ) is given by:


x1 = X 1 + k ( X 1 + X 2 ) t , x2 = X 2 + k ( X 1 + X 2 ) t , x3 = X 3 ,
(a) Find the velocity at t = 2 , of the particle which was at (1,1,0) at the reference time t = 0 .
(b) Find the velocity of the particle which is at (1,1,0) at t = 2 .
-----------------------------------------------------------------------------------------
Ans. (a) v1 = Dx1 / Dt = k ( X 1 + X 2 ) , v2 = Dx2 / Dt = k ( X 1 + X 2 ) , v3 = Dx3 / Dt = 0.
For the particle ( X 1 , X 2 , X 3 ) = (1,1, 0) , at t = 2 , v1 = k (1 + 1) = 2k , v2 = k (1 + 1) = 2k , v3 = 0
(b) The particle, which is at ( x1 , x2 , x3 ) = (1,1,0 ) at t = 2 , has the material coordinates given by
the following equations: 1 = X 1 + 2k ( X 1 + X 2 ) , 1 = X 2 + 2k ( X 1 + X 2 ) , 0 = X 3 .
1 1 2k
X1 = , X2 = , X3 = 0 , v1 = v2 = k ( X 1 + X 2 ) = , v3 = 0
1 + 4k 1 + 4k 1 + 4k
_________________________________________________________________

3.8 The position at time t of a particle initially at ( X1 , X 2 , X 3 ) is given by

x1 = X1 + X 22t 2 , x2 = X 2 + kX 2t , x3 = X 3 , where = 1 and k = 1 .


(a) for the particle which was initially at (1,1,0), what are its positions in the following instant of
time: t = 0, t = 1, t = 2 . (b) Find the initial position for a particle which is at (1,3,2) at t = 2 . (c)
Find the acceleration at t = 2 of the particle which was initially at (1,3,2) and (d) find the
acceleration of a particle which is at (1,3,2) at t = 2 .
--------------------------------------------------------------------------------------------
Ans. With = 1 and k = 1 , x1 = X1 + X 22t 2 , x2 = X 2 + X 2t , x3 = X 3

Copyright 2010, Elsevier Inc


3-4
Lai et al, Introduction to Continuum Mechanics

(a) t = 0 ( x1 , x2 , x3 ) = ( X1 , X 2 , X 3 ) = (1,1,0) ,
t = 1 ( x1 , x2 , x3 ) = ( X1 + X 22 , X 2 + X 2 , X 3 ) = (2, 2,0)
t = 2 ( x1 , x2 , x3 ) = ( X1 + 4 X 22 , X 2 + 2 X 2 , X 3 ) = (5,3,0)
(b) x1 = X1 + X 22t 2 x2 = X 2 + X 2t , x3 = X 3 , at t = 2 1 = X1 + 4 X 22 , 3 = 3 X 2 , 2 = X 3
X1 = 3, X 2 = 1, X 3 = 2 .
(c) x1 = X1 + X 22t 2 x2 = X 2 + X 2t , x3 = X 3 v1 = 2 X 22t , v2 = X 2 , v3 = 0 .
a1 = 2 X 22 , a2 = 0, a3 = 0 . For ( X1 , X 2 , X 3 ) = (1,3, 2 ) , a1 = 2 ( 3) = 18, a2 = a3 = 0 at any
2

time.
(d) The initial position of this particle was obtained in (b), i.e., X1 = 3, X 2 = 1, X 3 = 2 .
Thus, a1 = 2 X 22 = 2(1) 2 = 2, a2 = 0, a3 = 0 .
_________________________________________________________________

3.9 (a) Show that the velocity field vi = kxi / (1 + kt ) corresponds to the motion xi = X i (1 + kt )
and (b) find the acceleration of this motion in material description.
-----------------------------------------------------------------------------------------
Ans. (a) From xi = X i (1 + kt ) and X i = xi / (1 + kt ) vi = kX i =kxi / (1 + kt ) .
(b) vi = kX i ai = 0 , or
v v k 2 xi kx j k ij k 2 xi kxi k
ai = i + vj i = + = + =0.
t xi fixed x j (1 + kt ) (
2 1 + kt )(1 + kt ) ( 1 + kt ) (1 + kt ) ( kt )
1 +
_________________________________________________________________

3.10 Given the two dimensional velocity field: vx = 2 y, v y = 2 x . (a) Obtain the acceleration
field and (b) obtain the pathline equation.
-----------------------------------------------------------------------------------------
v v v
Ans. (a) ax = x + vx x + v y x = 0 + ( 2 y ) (0) + ( 2 x )( 2 ) = 4 x ,
t x y
v y v y v y
ay = + vx = 0 + ( 2 y ) (2) + ( 2 x )( 0 ) = 4 y , i.e., a = 4 xe x 4 ye y
+ vy
t x y
dx dy dy x
(b) = 2 y and = 2x = xdx + ydy = 0 , x 2 + y 2 = constant=X 2 + Y 2 ,
dt dt dx y
dx dy d2x dy d 2x
Or, = 2 y and = 2x = 2 = 2 ( 2 x ) + 4x = 0
dt dt dt dt dt
x = A sin 2t + B cos 2t and y = A cos 2t + B sin 2t , where A = Y , B = X .
_________________________________________________________________

3.11 Given the two dimensional velocity field: vx = kx, v y = ky . (a) Obtain the acceleration
field and (b) obtain the pathline equation.
-----------------------------------------------------------------------------------------
v v v
Ans. (a) ax = x + vx x + v y x = 0 + ( kx ) ( k ) + ( ky )( 0 ) = k 2 x
t x y
Copyright 2010, Elsevier Inc
3-5
Lai et al, Introduction to Continuum Mechanics

v y v y v y
ay =
t
+ vx
x
+ vy
y
(
= 0 + ( kx ) (0) + ( ky )( k ) = k 2 y , That is, a = k 2 xe x + ye y )
x t
dx dx x
(b) = kx = kdt ln x ln X = kt ln = kt x = Xe kt .
dt X
x 0 X
Similarly derivation gives y = Ye kt . Or, xy = XY where ( X , Y ) are material coordinates.
_________________________________________________________________

3.12 Given the two dimensional velocity field: vx = k ( x 2 y 2 ), v y = 2kxy . Obtain the
acceleration field.
-----------------------------------------------------------------------------------------
v
t
v
x
v
( )
Ans. ax = x + vx x + v y x = 0 + k x 2 y 2 (2kx) + ( 2kxy )( 2ky ) = 2 xk 2 ( x 2 + y 2 ) .
y
v y v y v y
ay =
t
+ vx
x
+ vy
y
( )
= 0 + k x 2 y 2 ( 2ky ) 2kxy ( 2kx ) = 2 yk 2 ( x 2 + y 2 ) .

(
That is, a = 2k 2 x 2 + y 2 ) ( xe x + ye y )
_________________________________________________________________

Dv v
3.13 In a spatial description, the equation to evaluate the acceleration = + ( v ) v is
Dt t
nonlinear. That is, if we consider two velocity fields v A and v B , then a A + a B a A+B , where
a A and a B denote respectively the acceleration fields corresponding to the velocity fields
v A and v B each existing alone, a A+B denotes the acceleration field corresponding to the combined
velocity field v A +v B . Verify this inequality for the velocity fields:
v A = 2 x2e1 + 2 x1e2 , v B = 2 x2e1 2 x1e2
--------------------------------------------------------------------------------------------
Dv v
Ans. From = + ( v ) v
Dt t
0 2 2x2 4 x1 0 0 2 2x2 4 x1
a A = +
0
= , a B = + =
0 2 0 2x1 4 x2 0 2 0 2x1 4 x2

a A = 4 x1e1 4 x2e 2 , a B = 4 x1e1 4 x2e 2
a A + a B = 8 x1e1 8 x2e 2 .
On the other hand, v A +v B =0, so that a A+B = 0 . Thus, a A + a B a A+B
_________________________________________________________________

3.14 Consider the motion: x1 = X1 , x2 = X 2 + ( sin t )( sin X1 ) , x3 = X 3


(a) At t = 0 , a material filament coincides with the straight line that extends from ( 0,0,0 ) to
(1,0,0 ) . Sketch the deformed shape of this filament at t = 1 / 2, t = 1 and t = 3 / 2 .
(b) Find the velocity and acceleration in a material and a spatial description.
Copyright 2010, Elsevier Inc
3-6
Lai et al, Introduction to Continuum Mechanics

-----------------------------------------------------------------------------------------
Ans. (a) Since x1 = X1 and x3 = X 3 , therefore there is no motion of the particles in the
x1 and x3 directions. Every particle moves only up and down in the x2 direction.
When t = 1 / 2 x2 = X 2 + sin X1 , t = 1 x2 = X 2 , t = 3 / 2 x2 = X 2 sin X1
The deformed shapes of the material at three different times are shown in the figure.
y

t=1/2

t=0, t=1
(1,0)
x

t=3/2

(b) v1 = 0, v2 = ( cos t )( sin X1 ) , v3 = 0 , a1 = 0, a2 = 2 ( sin t )( sin X1 ) , a3 = 0


Since x1 = X1 , the spatial descriptions are of the same form as above except that X1 is replaced
with x1 .
_________________________________________________________________

3.15 Consider the following velocity and temperature fields:


v = ( x1e1 + x2e 2 ) / ( x12 + x22 ), =k ( x12 + x22 )
(a) Write the above fields in polar coordinates and discuss the general nature of the given velocity
field and temperature field (e.g.,what do the flow and the isotherms look like?) (b) At the point
A (1,1,0 ) , determine the acceleration and the material derivative of the temperature field.
-----------------------------------------------------------------------------------------
Ans. (a) In polar coordinates, x1e1 + x2e2 = rer , where r 2 = x12 + x22 and er is the unit vector in

the r direction, so that v = er , =kr 2 . Thus, the given velocity field is that of a two
r
dimensional source flow from the origin, the flow is purely radial with radial velocity inversely
proportional to the radial distance from the origin. With =kr 2 , the isotherms are circles.

(b) From vr = and v = 0 , and Eq. (3.4.12)
r
v v v v v2 2
ar = r + vr r + r = 0 + 2 + 0 + 0 = 3 .
t r r r r r r
v v v v vr v
a = + vr + + =0.
t r r r
That is, a = 2 / r 3e r . At the point A(1,1,0), r = 2 , a = 2 / ( 2)3 e r = 2 2 / 4e r .
D v
= + vr + = 0 + ( 2kr ) = 2 k .
Dt t r r r
_________________________________________________________________

3.16 Do the previous problem for the following velocity and temperature fields:

Copyright 2010, Elsevier Inc


3-7
Lai et al, Introduction to Continuum Mechanics

( x2e1 + x1e2 )
v=
x12
+ x22
(
, =k x12 + x22 )
------------------------------------------------------------------------------------------
Ans. With x1 = r cos , x2 = r sin and x12 + x22 = r 2 , we have
( - x2e1 + x1e2 ) r ( sin e1 + cos e2 )
v= = = e and =kr 2
+ x12 x22 r 2 r
Particles move in concentric circles with their speed inversely proportional to r . Isotherms are
circles.

(b) With vr = 0, v = , we have, from Eq.(3.4.12).
r
2
vr v v v v2 1 2 v v v v vv
ar = + vr r + r = = 3 , a = + vr + + r = 0
t r r r r r r t r r r
i.e., a = 2 / r 3e r . At the point A, r = 2 , therefore, a = 2 2 / 4e r
D
= + v = 0 + e 2kre r = 0 .
Dt t r
_________________________________________________________________

3.17 Consider : x = X + X1ke1 . let dX( ) = dS1 / 2


1
( ) (e1 +e2 ) & dX( 2) = ( dS2 / 2 ) ( e1 +e2 ) be
differential material elements in the undeformed configuration. (a) Find the deformed elements
dx( ) and dx( ) . (b) Evaluate the stretches of these elements ds / dS and ds / dS and the change
1 2
1 1 2 2
2
in the angle between them. (c) Do part (b) for k = 1 and k = 10 and (d) compare the results of
part (c) to that predicted by the small strain tensor E .
-------------------------------------------------------------------------------------------
1 + k 0 0
Ans. (a) x1 = X1 + kX1 , x2 = X 2 , x3 = X 3 [ F ] = 0 1 0 , dx = FdX
0 0 1
1 + k 0 0 1
dS
( ) 1 0 1 dx( ) = 1 (1 + k ) e1 + e2 .
dx =
1 dS1 1
0
2 2
0 0 1 0
1 + k 0 0 1
dx( 2 ) = dS2 0 1 dx( 2 ) = dS2 1 + k e + e .

2 1 0 ( ) 1 2
0 2
0 1 0
ds1 ds2 1
(b) = = (1 + k )2 + 1 .
dS1 dS2 2
Let be the decrease in angle (from 90o ), then ( / 2 ) is the angle between the two
deformed differential elements. Thus,

Copyright 2010, Elsevier Inc


3-8
Lai et al, Introduction to Continuum Mechanics

(1) ( 2)
= (1 + k ) + 1
2
dx dx 1 dS1 dS2
cos = = (1 + k ) 2
+ 1
2 ds1ds2 ds1ds2 2 2 (1 + k )2 + 1

(1 + k ) + 1
2
sin = .
(1 + k )2 + 1
ds1 ds2 5 3
(c) For k = 1 , = = , sin = .
dS1 dS2 2 5
ds1 ds2 1 1
For k = 102 , = = (1 + k )2 + 1 2 + 2k = 1 + k = 1.01 = 1.005 .
dS1 dS2 2 2
(1 + k ) + 1
2
2k k 0.01
sin = = = = 0.0099 radian ( sign indicates increase in
(1 + k ) 2
+1 2 + 2k 1 + k 1.01
angle).

k 0 0 k 0 0
(d) u = x X = kX1e1 u1 = kX1 , u2 = u3 = 0 , [u ] = 0 0 0 [ E] = 0 0 0 ,

0 0 0 0 0 0
k 0 0 1 k
1 1 1
( e1 + e2 ) E11 = [1 1 0] 0 0 0 1 = [1 1 0] 0 = ,
' k

e1 =
2 2 2 2
0 0 0 0 0
ds dS k ds k
=
E11 = = 1 + = 1.005 , same as the result of part (c).
dS 2 dS 2
Also with
k 0 0 1 k
1 1 1
( -e1 + e2 ) E12 = [1 1 0] 0 0 0 1 = [1 1 0] 0 = 2 E12 = k
k
e2 =
2 2 2 2
0 0 0 0 0
Thus, the decrease in angle = k , or the increase in angle is 0.01 0.0099 .
_________________________________________________________________

3.18 Consider the motion: x = X + AX , where A is a small constant tensor (i.e., whose
components are small in magnitude and independent of X i ). Show that the infinitesimal strain
tensor is given by E = ( A + A T ) / 2 .
-----------------------------------------------------------------------------------------
Ans. u = x X = AX u = ( AX ) . Since A is a constant, therefore,
u = ( AX ) = A ( X ) . Now, [X] = X i / X j = ij = [ I ] u = A E = ( A + A T ) / 2
_______________________________________________________________________

3.19 At time t , the position of a particle, initially at ( X1 , X 2 , X 3 ) is defined by:


x1 = X1 + kX 3 , x2 = X 2 + kX 2 , x3 = X 3 , k = 105 . (a) Find the components of the strain tensor
and (b) find the unit elongation of an element initially in the direction of e1 + e2 .

Copyright 2010, Elsevier Inc


3-9
Lai et al, Introduction to Continuum Mechanics

-----------------------------------------------------------------------------------------
Ans. (a) u1 = x1 X1 = kX 3 , u2 = x2 X 2 = kX 2 , u3 = x3 X 3 = 0
0 0 k 0 0 k / 2

[ u ] = 0 k 0 [ E ] =
[ u ] + [ u ]
T

= 0 k 0
2
0 0 0 k / 2 0 0
0 0 k / 2 1 5
1 1 1 = k = 10
(b) Let e1 = ( 1 2 ) 11 1 1 11 [
e + e E = e Ee E = 1 1 0 ] 0 k 0 2
2 2 2
k / 2 0 0 0
_________________________________________________________________

3.20 Consider the displacements: u1 = k (2 X12 + X1 X 2 ), u2 = kX 22 , u3 = 0, k = 104 . (a) Find the


unit elongations and the change of angles for two material elements
dX( ) = dX e and dX( ) = dX e that emanate from a particle designated by X = e + e . (b)
1 2
1 1 2 2 1 2
Sketch deformed positions of these two elements.
-----------------------------------------------------------------------------------------
4kX1 + kX 2 kX1 0
Ans. (a) [u ] = 0 2kX 2 0 ,
0 0 0
5k k 0 5k k / 2 0
At ( X1 , X 2 , X 3 ) = (1,1,0 ) , [u ] = 0 2k 0 [ E] = k / 2 2k 0 .

0 0 0 0 0 0
Unit elong. for dX( ) = dX1e1 is E11 = 5k = 5 104 , unit elong. for dX( ) = dX 2e 2 is
1 2

E22 = 2k = 2 104 .
Decrease in angle between them is 2 E12 = k = 104 radian .
(b) For dX( ) = dX1e1 , dx( ) = dX( ) + ( u ) dX( ) = dX1e1 + 5kdX1e1 = (1 + 5k ) dX1e1 ,
1 1 1 1

For dX( ) = dX e ,
2
2 2

dx( ) = dX( ) + ( u ) dX( ) = dX 2e 2 + (kdX 2e1 + 2kdX 2e 2 ) = kdX 2e1 + (1 + 2k )dX 2e 2


2 2 2

The deformed positions of these two elements are shown below:

k dX 2
(1 + 2 k) dX 2

dX 2 P
u2=k

(1 + 5 k) dX1
P dX1
u1= 3k

Copyright 2010, Elsevier Inc


3-10
Lai et al, Introduction to Continuum Mechanics

_________________________________________________________________

3.21 Given displacement field: u1 = kX1 , u2 = u3 = 0, k = 104 . Determine the increase in


length for the diagonal element OA of the unit cube (see figure below) in the direction of
e1 + e2 + e3 (a) by using the strain tensor and (b) by geometry.
-----------------------------------------------------------------------------------------
k 0 0
1
Ans. (a) [u ] = 0 0 0 = [ E] . Let e1 = ( e1 + e2 + e3 ) , then the unit elongation in the e1' -
3
0 0 0
k 0 0 1
1 k 104
direction is E11 = e1 Ee1 = [1 1 1] 0 0 0 1 = = .
3 3 3
0 0 0 1
(b) From the given displacement field, we see that the unit cube becomes longer in the
x1 direction by an amount of k , while the other two sides remain the same. The diagonal
OA becomes OA ' , (see Figure), where OA = 3 and
OA ' = (1 + k ) 2 + 1 + 1 = 3 + 2k + k 2 = 3(1 + 2k / 3 + k 2 / 3)
OA ' OA = 3(1 + 2k / 3 + k 2 / 3)1/2 3 .

( )
1/2
Using binomial theorem, 1 + 2k / 3 + k 2 / 3 = 1 + (1 / 2)(2k / 3) + ... 1 + k / 3
Thus, OA ' OA = 3(1 + k / 3) 3 = 3k / 3 (OA ' OA) / OA = k / 3 , same as that obtained in
part (a).
_________________________________________________________________

3.22 With reference to a rectangular Cartesian coordinate system, the state of strain at a point is
5 3 0
given by the matrix [ E ] = 3 4 1 104 . (a) What is the unit elongation in the direction of
0 1 2
2e1 + 2e 2 + e3 ? (b) What is the change in angle between two perpendicular lines (in the
undeformed state) emanating from the point and in the directions of 2e1 + 2e2 + e3 and 3e1 6e3 ?
-----------------------------------------------------------------------------------------
Ans. Let e1 = (2e1 + 2e2 + e3 ) / 3 , the unit elongation in this direction is:

Copyright 2010, Elsevier Inc


3-11
Lai et al, Introduction to Continuum Mechanics

5 3 0 2
1 58
= e1 Ee1 = [ 2 2 1] 3 4 1 2 104 = 104 .
E11
9 9
0 1 2 1
1
Let e2 = ( 3e1 6e3 ) , then the decrease in angle between the two elements is:
45
5 3 0 3
2 32
= 2e1 Ee2 =
2 E12 [ 2 2 1] 3 4 1 0 104 = 104 rad .
3 45 45
0 1 2 6
_________________________________________________________________

3.23 For the strain tensor given in the previous problem, (a) find the unit elongation in the
direction of 3e1 4e2 and (b) find the change in angle between two elements in the dir. of
3e1 4e3 and 4e1 + 3e3 .
-----------------------------------------------------------------------------------------
1
Ans. (a) Let e1 = ( 3e1 4e2 ) , the unit elongation in this direction is:
5
2 5 3 0 3
1 37
= e1 Ee1 = [3 4 0] 3 4 1 4 104 =
E11 104 = 1.48 104
5
25
0 1 2 0
1 1
(b) Let e1'' = ( 3e1 4e3 ) and e''2 = ( 4e1 + 3e3 ) , then the decrease in angle between these two
5 5
elements is:
2 5 3 0 4
1 72
= 2e1'' Ee''2 = 2 [3 0 4] 3 4 1 0 104 = 104 = 2.88 104 rad .
'''
2 E12
5 25
0 1 2 3
_________________________________________________________________

3.24 (a) Determine the principal scalar invariants for the strain tensor given below at the left and
(b) show that the matrix given below at the right can not represent the same state of strain.
5 3 0 3 0 0
[E] = 3 4 1 10 , 0 6 0 104
4

0 1 2 0 0 2
-----------------------------------------------------------------------------------------
Ans. (a) I1 = ( 5 + 4 + 2 ) 104 = 11 104 ,
5 3 4 1 5 0
I2 = 108 + 108 + 108 = 28 108
3 4 1 2 0 2
5 3 0
I 3 = 3 4 1 1012 = 17 1012
0 1 2

Copyright 2010, Elsevier Inc


3-12
Lai et al, Introduction to Continuum Mechanics

3 0 0
(b) For 0 6 0 104 , I 3 = 36 1012 , which is different from the I 3 in (a), therefore, the
0 0 2
two matrices can not represent the same tensor.
_________________________________________________________________

3.25 Calculate the principal scalar invariants for the following two tensors. What can you say
about the results?
0 0 0 0
T(1) = 0 0 and T( 2 ) = 0 0 .

0 0 0 0 0 0
-----------------------------------------------------------------------------------------
0 0
Ans. For T = 0 0 , I1 = 0, I 2 = 2 , I 3 = 0 .
(1)

0 0 0
{ei }

0 0
2)
(
For T = 0 0 I1 = 0, I 2 = 2 , I 3 = 0

0 0 0
{ei }
We see that these two tensors have the same principal scalar invariants. This result demonstrates
that two different tensors can have the same three principal scalar invariants and therefore the
same eigenvalues (in fact, 1 = , 2 = , 3 = 0 ). However, corresponding to the same
eigenvalue , the eigenvector for T( ) is (e + e ) / 2 , whereas the eigenvector for T( ) is
1 2
1 2
(e1 e 2 ) / 2 . We see from this example that having the same principal scalar invariants is a
necessary but not sufficient condition for the two tensors to be the same.
_________________________________________________________________

( )
3.26 For the displacement field: u1 = kX12 , u2 = kX 2 X 3 , u3 = k 2 X1 X 3 + X12 , k = 106 , find
the maximum unit elongation for an element that is initially at (1,0,0 ) .
-----------------------------------------------------------------------------------------
2kX1 0 0

Ans. [u ] = 0 kX 3 kX 2 , thus, for ( X1 , X 2 , X 3 ) = (1,0,0 ) ,
k ( 2 X 3 + 2 X1 ) 0 2kX1

2 k 0 0 2 k 0 k
[u ] = 0 0 0 [E] = 0 0 0 , the characteristic equation for this tensor is:

2k 0 2k k 0 2k

2k 0 k
0 = 0 ( ) ( 2k ) k 2 = 0 1 = 0, 2 = 3k , 3 = k .
2
0 0

k 0 2k

Copyright 2010, Elsevier Inc


3-13
Lai et al, Introduction to Continuum Mechanics

Thus, the maximum unit elongation at (1,0,0 ) is 2 = 3k = 3 106 .


_________________________________________________________________

3.27 Given the matrix of an infinitesimal strain tensor as


k1 X 2 0 0
[ E] = 0 k2 X 2 0 .
0 0 k2 X 2
(a) Find the location of the particle that does not undergo any volume change.
(b) What should the relation between k1 and k2 be so that no element changes its volume?
-----------------------------------------------------------------------------------------
( dV )
Ans. (a) = E11 + E22 + E33 = ( k1 2k2 ) X 2 = 0 . Thus, the particles which were on the plane
dV
X 2 = 0 do not suffer any change of volume.
(b) If ( k1 2k2 ) = 0, ie., k1 = 2k2 , then no element changes its volume.
_________________________________________________________________

3.28 The displacement components for a body are:


u1 = k ( X12 + X 2 ), u2 = k (4 X 32 X1 ), u3 = 0, k =104 .
(a) Find the strain tensor. (b) Find the change of length per unit length for an element which was
at (1, 2,1) and in the direction of e1 + e2 . (c) What is the maximum unit elongation at the same
point (1, 2,1) ? (d) What is the change of volume for the unit cube with a corner at the origin and
with three of its edges along the positive coordinate axes?
-----------------------------------------------------------------------------------------
2kX1 k 0 2kX1 0 0

Ans. (a) [u ] = k 0 8kX 3 [ E] = 0 0 4kX 3
0 0 0 0 4kX 3 0
2k 0 0
(b) At (1, 2,1) , [ E] = 0 0 4k ,
0 4k 0
2k 0 0 1
1 1
for e1' = ( e1 + e2 ) , E11' = [1 1 0] 0
= e1' Ee1' 0 4k 1 = k
2 2
0 4k 0 0
2k 0 0
4k = 0 ( 2k ) 2 ( 4k ) = 0
2
(c) The characteristic equation is 0

0 4k
1 = 2k , 2 = 4k , 3 = 4k . The maximum elongation is 4k .
(d) Change of volume per unit volume = Eii = 2kX1 , which is a function of X1 . Thus,
1 1
V = 2kX1dV = 2 kX1 (1) dX1 = kX12 = k .
o
o
_________________________________________________________________
Copyright 2010, Elsevier Inc
3-14
Lai et al, Introduction to Continuum Mechanics

3.29 For any motion, the mass of a particle (material volume) remains a constant (conservation of
mass principle). Consider the mass to be the product of its volume and its mass density and show
that (a) for infinitesimal deformation (1 + Ekk ) = o where o denote the initial density and ,
the current density. (b) Use the smallness of Ekk to show that the current density is given by
= o (1 Ekk ) .
-----------------------------------------------------------------------------------------
dV dV + dV dV
Ans. (a) o dVo = dV o = = o = 1 + ,
dVo dVo dVo
dV
For small deformation, = Ekk o = (1 + Ekk ) .
dVo
1
(b) From bionomial theorem, for small Ekk , (1+Ekk ) 1 Ekk , thus,
1
= o (1 + Ekk ) = o (1 Ekk ) .
_________________________________________________________________

3.30 True or false: At any point in a body, there always exist two mutually perpendicular material
elements which do not suffer any change of angle in an arbitrary small deformation of the body.
Give reason(s).
-----------------------------------------------------------------------------------------
Ans. True. The strain tensor E is a real symmetric tensor, for which there always exists three
principal directions, with respect to which, the matrix of E is diagonal. That is, the non-diagonal
elements, which give one-half of the change of angle between the elements which were along the
principal directions, are zero.
_________________________________________________________________

3.31 Given the following strain components at a point in a continuum:


E11 = E12 = E22 = k , E33 = 3k , E13 = E23 = 0, k = 106
Does there exist a material element at the point which decreases in length under the deformation?
Explain your answer.
-----------------------------------------------------------------------------------------
Ans.
k k 0 k k 0

[E] = k k 0 k k 0 = 0, ( 3k ) ( k )2 k 2 = 0
0 0 3k 0 0 3k

( )
( 3k ) 2 k + 2 = 0 1 = 3k , 2 = 0, 3 = 2k .
Thus, the minimum unit elongation is 0 . Therefore, there does not exist any element at the point
which has a negative unit elongation (i.e., decreases in length).
_________________________________________________________________

3.32 The unit elongation at a certain point on the surface of a body are measured experimentally
by means of strain gages that are arranged 45o apart (called the 45o strain rosette) in the direction
1
of e1 , ( e1 + e2 ) and e2 . If these unit elongation are designated by a, b, c respectively, what are
2
the strain components E11 , E22 and E12 ?
Copyright 2010, Elsevier Inc
3-15
Lai et al, Introduction to Continuum Mechanics

-----------------------------------------------------------------------------------------
Ans.

1
With e1 = ( e1 + e2 ) , we have,
2
E11 E12 E13 1
1 1
= e1 Ee1 = [1 1 0] E21 E22 E23 1 = ( E11 + E12 + E21 + E22 ) , with E12 = E21 ,
E11
2 2
E31 E32 E33 0
1
= ( E11 + 2 E12 + E22 ) E12 = E11
( E + E22 )
11
E11 . Thus, the strain components are:
2 2
(a + c)
E11 = a, E22 = c, E12 = b .
2
_________________________________________________________________

3.33 (a) Do the previous problem, if the measured strains are 200 106 , 50 106 and
100 106 in the direction e1 , e1 and e 2 respectively. (b) Find the principal directions, assuming
E31 = E32 = E33 = 0 . (c) How will the result of part b be altered if E33 0 .
-----------------------------------------------------------------------------------------
Ans. (a) With E11 = 200 106 , E11 = 50 106 and E22 = 100 106 , we have, from the results
E11 + E22 200 + 100 6 6

of the previous problem, E12 = E11 = 50 10 = 100 10
2 2
E11 E12 0
(b) E12 E22 0 = 0 ( E11 )( E22 ) E12
2
=0

0 0

(
+ 2 ( E11 + E22 ) + E11E22 E12
2

=0, )
( E11 + E22 ) ( E11 E22 )2 + 4 E122
1,2 = , 3 = 0 ,
2
thus,

( 200 + 100 ) ( 200 100 )2 + 4 ( 100 )2 6 261.8 106
1,2 = 10 = , 3 = 0
2 38.2 106

The principal direction for 3 is e3 . The principal directions corresponding to the other two
eigenvalues lie on the plane of e1 and e2 . Let
n = 1e1 + 2e 2 cos e1 + sin e 2 , then ( E11 )1 + E12 2 = 0 ,

Copyright 2010, Elsevier Inc


3-16
Lai et al, Introduction to Continuum Mechanics

2 ( E11 )
= tan = ,
1 E12
1 E11 261.8 200 61.8
For 1 = 261.8 106 , tan = = = = 0.618 = 31.7o ,
E12 100 100
Or, n = 0.851e1 0.525e 2
2 E11 38.2 200
For 2 = 38.2 106 , tan = = = 1.618 = 58.3o
E12 100
Or, n = 0.525e1 + 0.851e 2 .
(c) If E33 0 , then the principal strain corresponding to the direction e3 is E33 instead of zero.
Nothing else changes.
_________________________________________________________________

= E22 = 1000 106 .


3.34 Repeat the previous problem with E11 = E11
-----------------------------------------------------------------------------------------
E + E22 2000
Ans. (a) From the results of Problem 3.32, E12 = E11 11 = 1000 106 = 0 ,
2 2
103 0 0

(b) and (c) [ E ] = 0 103 0 , the principal strains are 103 in any directions lying on the

0 0 E33

plane of e1 and e 2 and the principal strain E33 is in e3 direction.
_________________________________________________________________

3.35 The unit elongation at a certain point on the surface of a body are measured experimentally
by means of strain gages that are arranged 60o apart (called the 60o strain rosette) in the direction
1
(
of e1 , e1 + 3e 2 and
2
) 1
2
( )
e1 + 3e2 . If these unit elongation are designated by a, b, c

respectively, what are the strain components E11 , E22 and E12 ?
-----------------------------------------------------------------------------------------
Ans.

With e1' = (e1 + 3e 2 ) / 2, e1'' = ( e1 + 3e2 ) / 2 , we have,

Copyright 2010, Elsevier Inc


3-17
Lai et al, Introduction to Continuum Mechanics

E11 E12 E13 1


1
E11' = e1' Ee1'
1
= 1
4
3 0 E21 E22
4
(
E23 3 = E11 + 2 3E12 + 3E22 (i) )
E31 E32 E33 0

E11 E12 E13 1


1
"'
E11 = e1" Ee1"
1
= 1
4
3 0 E21 E22
4
(
E23 3 = E11 2 3E12 + 3E22 ) (ii)
E31 E32 E33 0

1 1 E ' E11
''
bc
(i) & (ii), E22 = 2 E11' + 2 E11
'''
E11 = [ 2b + 2c a ] , E12 = 11 = , E11 = a .
3 3 3 3
_________________________________________________________________

3.36 If the 60o strain rosette measurements give a = 2 106 , b = 1 106 , c = 1.5 106 , obtain
E11 , E12 and E22 . Use the formulas obtained in the previous problem.
-----------------------------------------------------------------------------------------
Ans. Using the formulas drived in the previous problem, we have,
1 1
E22 = [ 2b + 2c a ] = ( 2 )(1) + ( 2 )(1.5 ) 2 106 = 1 106 ,
3 3
bc 1
E12 = = 106 , E11 = 2 106 .
3 2 3
_________________________________________________________________

3.37 Repeat the previous problem for the case a = b= c = 2000 106 .
-----------------------------------------------------------------------------------------
1 1
Ans. E22 = [ 2b + 2c a ] = ( 2 )( 2000 ) + ( 2 )( 2000 ) 2000 106 = 2 103 ,
3 3
bc
E12 = = 0 , E11 = 2 103
3
_______________________________________________________________________

3.38 For the velocity field: v = kx22e1 , (a) find the rate of deformation and spin tensors. (b) Find
the rate of extension of a material element dx = dsn where n = ( e1 + e2 ) / 2 at x = 5e1 + 3e 2 .
-----------------------------------------------------------------------------------------
Ans. v1 = kx22 , v2 = v3 = 0 ,
0 2kx2 0 0 kx2 0 0 kx2 0
[v ] = 0 0 [ D] = [v ] = kx2
0 , [ W ] = [v ] = kx2 0
S A
0 0 0
0 0 0 0 0 0 0 0 0
(b) At the position x = 5e1 + 3e 2 ,
0 3k 0 0 3k 0
[ D] = 3k 0 0 , [ W ] = 3k 0 0
0 0 0 0 0 0

Copyright 2010, Elsevier Inc


3-18
Lai et al, Introduction to Continuum Mechanics

For the element dx = dsn with n = (e1 + e2 ) / 2 , the rate of extension is:
0 3k 0 1
1
D( n )( n ) = n Dn = [1 1 0] 3k 0 0 1 = 3k .
2
0 0 0 0
_________________________________________________________________

t+k
3.39 For the velocity field: v = e1 , find the rates of extension for the following material
1 + x1
1
1 1
2
( 2 )( 1
elements: dx( ) = ds e and dx( ) = ds / 2 e + e at the origin at time t = 1 .
2 )
-----------------------------------------------------------------------------------------
( t + k ) / (1 + x )2 0 0
1
t+k
Ans. v1 = , v2 = v3 = 0 [v ] = 0 0 0 = [ D] .
1 + x1 0 0 0

(1 + k ) 0 0

At t = 1 and at ( x1 , x2 , x3 ) = ( 0,0,0 ) , [ D] = 0 0 0 .
0 0 0

(
Rate of extension for dx( ) = ds1e1 is D11 = (1 + k ) ; for dx( ) = ds2 / 2 ( e1 + e 2 ) , it is:
1 2
)
(1 + k ) 0 0 1
1 1
= [1 1 0]
D11' 0 0 0 1 = (1 + k )
2 2
0 0 0 0

_________________________________________________________________

3.40 For the velocity field v = ( cos t )( sin x1 ) e 2 (a) find the rate of deformation and spin tensors,
and (b) find the rate of extension at t = 0 for the following elements at the origin:
1 1
2
2 2
3
(
dx( ) = ds e , dx( ) = ds e and dx( ) = ds / 2 e + e .
1
3 )( 1 2 )
-----------------------------------------------------------------------------------------
Ans. (a) With v1 = 0, v2 = ( cos t )( sin x1 ) , v3 = 0 ,
0 0 0 0 ( cos t cos x1 ) / 2 0

[v ] = cos t cos x1 0 0 [ D] = ( cos t cos x1 ) / 2 0 0 ,
0 0 0 0 0 0

0 ( cos t cos x1 ) / 2 0

[ W ] = ( cos t cos x1 ) / 2 0 0 .
0 0 0

0 / 2 0
(b) At t = 0 and ( x1 , x2 , x3 ) = ( 0,0,0 ) , [ D] = / 2 0 0 .
0 0 0

Copyright 2010, Elsevier Inc


3-19
Lai et al, Introduction to Continuum Mechanics

For dx( ) = ds1e1 , rate of extension is D11 =0, for dx( ) = ds2e 2 , D22 = 0 and
1 2

0 / 2 0 1

( 3)
( ) 1
for dx = ds3 / 2 ( e1 + e 2 ) , D11 = [1 1 0] / 2
'
2
0 0 1 =
2
0 0 0 0
_________________________________________________________________

3.41 Show that the following velocity components correspond to a rigid body motion.
v1 = x2 x3 , v2 = x1 + x3 , v3 = x1 x2
----------------------------------------------------------------------------------------
0 1 1 0 0 0
Ans. [v ] = 1 0 1 [ D] = 0 0 0

1 1 0 0 0 0
Therefore, the velocity field is a rigid body motion..
_________________________________________________________________

1
3.42 Given the velocity field v = e r , (a) find the rate of deformation tensor and the spin tensor
r
and (b) find the rate of extension of a radial material line element.
-----------------------------------------------------------------------------------------
1
Ans. With vr = , v = vz = 0 , we have, using Eq. (2.34.5)
r
vr 1 vr vr 1
v 2 0 0
r r z r
v 1 v v 1 = D , W = 0 .
[ ]
v = + vr =
0 0 [ ] [ ] [ ]
r r z 2
r

v 1 vz vz 0 0 0
z
r r z
1
(b) The rate of extension for a radial element is Drr =
.
r2
_________________________________________________________________

3.43 Given the two-dimensional velocity field in polar coordinates:


4
vr = 0, v = 2r +
r
(a) Find the acceleration at r = 2 and (b) find the rate of deformation tensor at r = 2 .
-----------------------------------------------------------------------------------------
vr
Ans. (a) Using Eq. (3.4.12), ar =
v v v (v ) 2
1
+ vr r + r v = = 2r + ,
4
2

t r r r r r
v v v v
a = + vr + + vr = 0 . At r = 2 , ar = (6) 2 / 2 = 18 , a = 0 .
t r r

Copyright 2010, Elsevier Inc


3-20
Lai et al, Introduction to Continuum Mechanics

vr 1 vr v 4
v 0 2 + 2
r
0
r r
(b) Eq. (2.34.5) [v ] = =
r
= .
v 1 v v 4
+ vr 2 r 2
0 0
r r r

4 / r 2
0 0 1
[ D] = [v ]S = , at r = 2 , [ D] = .
4 / r 2 0 1 0
_________________________________________________________________

3.44 Given the velocity field in spherical coordinates:


B
vr = 0, v = 0, v = Ar + 2 sin
r
(a) Determine the acceration field and (b) find the rate of deformation tensor.
-----------------------------------------------------------------------------------------
Ans. (a) From Eq. (3.4.16),
vr vr
v vr v vr v2 1 B
2
ar = + vr + v + v sin = = Ar + 2 sin 2
t r r r sin r r r
v v
v v v v v2 cos sin B
2
a = + vr + + vr + v cos = cot = Ar + 2
t r r r sin r r r
v v v v v v
a = + vr + + + vr sin + v cos = 0
t r r r sin
(b) Eq. (2.35.25)
vr 1 vr v
1 vr v
0
0
v
r r r r sin
r
r

v 1 v 1 v v cot
= 0 v cot
[v ] = r r + rr
v
0 , thus
r sin r r
v v

v 1
1 v 1 v vr v cot



, 0
+ +
r r
r r r sin r r
the nonzero components of rate of deformation tensor are:
1 v v 3B
Dr = + = 3 sin ,
2 r r 2r
1 v cot 1 v 1 B B
D = + = A + 3 + A + 3 cos = 0 .
2 r r 2 r r
_________________________________________________________________

3.45 A motion is said to be irrotational if the spin tensor vanishes. Show that the following
velocity field is irrotational:
x e + x e
v = 2 2 2 1 2 , r 2 = x12 + x22
r
-----------------------------------------------------------------------------------------

Copyright 2010, Elsevier Inc


3-21
Lai et al, Introduction to Continuum Mechanics

v1 v1 2 x2 r 1 2 x2 r
x +
x2 x1 2 x2 r 3 x1 r2
r 3 x2
Ans. v1 = 2 , v2 = 2 , r = x1 + x2 , [v ] = =
2 2 1
,
r r v2 v2 1 2 x1 r 2 x r
31
x1 x2 r 2 r 3 x1 r x2
r r x1 r x
r 2 = x12 + x22 2r = 2 x1 = , also, = 2,
x1 x1 r x2 r
2 x1 x2 x22 x12
4
r4
[v ] = 2 r 2
= [v ] [ W ] = 0.
S

x x 2x x
2 4 1 14 2
r r
_________________________________________________________________

3.46 Let dx( ) = ds1n and dx( ) = ds2m be two material elements that emanate from a particle
1 2

P which at present has a rate of deformation D . (a) Consdier D / Dt (dx( ) dx( ) ) to show that
( )
1 2

1 D ( ds1 ) 1 D ( ds2 ) D
+ cos sin = 2m Dn
ds1 Dt ds2 Dt Dt
where is the angle between m and n .
(b) Consider the case of dx( ) = dx( ) , what does the above formula reduce to?
1 2


, i.e., dx( ) and dx( ) are perpendicular to each other, where
1 2
(c) Consider the case where =
2
does the above formula reduces to?
-----------------------------------------------------------------------------------------
Ans.
(1) Ddx( 2 )
D
Dt
(
(1)
dx dx ( 2)
=) D (1)
Dt
( 2)
dx dx + dx


Dt
= ( v ) dx(1) dx( 2 ) + dx(1) ( v ) dx( 2 )

1 T
) 2 1
( ) 2 1
{( T
) ( )}
= dx( ) v dx( ) + dx( ) v dx( ) = dx( ) v + v dx( ) = 2dx( ) Ddx( ) .
( 2 1 2

With dx( ) = ds1n and dx( ) = ds2m , the above formula give,
1 2

D D
( ds1ds2n m ) = 2ds1ds2 ( n Dm ) ( ds1ds2 cos ) = 2ds1ds2 ( n Dm ) . Thus,
Dt Dt
Dds1 Dds2 D cos
( ds2 cos ) + ( ds1 cos ) + ( ds1ds2 ) = 2ds1ds2 ( n Dm ) ,
Dt Dt Dt
1 D ( ds1 ) 1 D ( ds2 ) D
+ cos sin = 2 ( n Dm ) = 2 ( m Dn ) .
1
ds Dt ds2 Dt Dt
1 D ( ds )
(b) For, dx( ) = dx( ) = dsn the above formula = ( n Dn ) = D( n )( n ) , no sum on n .
1 2

ds Dt
(c) For dx( ) perpendicular to dx( ) , = 90o , we have,
1 2

D
= 2 ( n Dm ) = 2 Dnm .
Dt

Copyright 2010, Elsevier Inc


3-22
Lai et al, Introduction to Continuum Mechanics

_________________________________________________________________

3.47 Let e1 , e2 ,e3 and D1 , D2 , D3 be the principal directions and corresponding principal values of
a rate of deformation tensor D . Further, let dx( ) = ds e , dx( ) = ds e and dx( ) = ds e be
1 2 3
1 1 2 2 3 3
1 2 3
{
three material elements. Consider the material derivative ( D / Dt ) dx( ) dx( ) dx( ) and show }
1 D ( dV )
that = D1 + D2 + D3 , where dV = ds1ds2 ds3 .
dV Dt
-----------------------------------------------------------------------------------------
Ans. Since the principal directions are (or can always be chosen to be) mutually perpendicular,
therefore, dx( ) dx( ) dx( ) = ds ds ds = dV .
1 2 3
1 2 3
D ( dV ) D ( ds1ds2 ds3 ) D ( ds1 ) D ( ds2 ) D ( ds3 )
= = ds2 ds3 + ds1ds3 , + ds1ds2
Dt Dt Dt Dt Dt
1 D ( dV ) 1 D ( ds1 )
1 D ( ds2 ) 1 D ( ds3 )
= + + = D11 + D22 + D33 .
dV Dt ds1 Dt ds2 Dt ds3 Dt
_________________________________________________________________

3.48 Consider a material element dx = dsn (a) Show that ( D / Dt ) n = Dn + Wn ( n Dn ) n ,


where D is rate of deformation tensor and W is the spin tensor. (b) Show that if n is an
eigenvector of D , then,
Dn
= Wn = n
Dt
-----------------------------------------------------------------------------------------
D Dn Dds Dn 1 Dds Dn
Ans. (a) ( dsn ) = ds + n = ds +n = ds + (n Dn)n . [see
Dt Dt Dt Dt ds Dt Dt
D D
Eq.(3.13.12) ]. We also have, ( dsn ) = ( dx ) = ( v ) dx = ds ( v ) n , therefore,
Dt Dt
Dn Dn
( v ) n = + n ( n Dn ) = ( v ) n n ( n Dn ) = ( D + W ) n n ( n Dn ) .
Dt Dt
(b) If n is an eigenvector of D , then Dn = n , therefore,
Dn Dn
= ( D + W ) n n ( n Dn ) = n + Wn n = Wn . That is, = Wn .
Dt Dt
Since W is antisymmetric Wn = n , where is the dual vector for W . Thus
Dn
= Wn = n .
Dt
That is, the principal axes of D rotates with an angular velocity given by the dual vector of the
spin tensor.
_________________________________________________________________

3.49 Given the following velocity field: v1 = k ( x2 2 ) x3 , v2 = x1 x2 , v3 = kx1 x3 for an


2

incompressible fluid, determine the value of k , such that the equation of mass conservation is
satisfied.
-----------------------------------------------------------------------------------------
Copyright 2010, Elsevier Inc
3-23
Lai et al, Introduction to Continuum Mechanics

v1 v2 v3
Ans. + + = 0 0 x1 + kx1 = 0 k = 1
x1 x2 x3
_________________________________________________________________

3.50 Given the velocity field in cylindrical coordinates: vr = f (r , ), v = vz = 0 . For an


incompressible material, from the conservation of mass principle, obtain the most general form of
the function f (r , ) .
-----------------------------------------------------------------------------------------
Ans. The equation of continuity for an incompressible material is [see Eq.(3.15.11)]:
vr 1 v vr vz f f 1
+ + + =0 + =0 ( fr ) = 0, fr = g ( ) .
r r r z r r r r
Therefore, f = g ( ) / r , where g ( ) is an arbitrary function of .
_________________________________________________________________

3.51 An incompressible fluid undergoes a two-dimensional motion with vr = k cos / r . From


the consideration of the principle of conservation of mass, find v , subject to the condition that
v = 0 at = 0 .
-----------------------------------------------------------------------------------------
Ans.
k cos v 1 1 v (k cos ) v v 1 (k cos )
vr = r = ( k cos ) 3/2 , r = r + r = .
r r 2r r r 3/2 r r 2 r 3/2
The equation of continuity for an incompressible fluid is [see
v 1 v vr vz
Eq.(3.15.11)]: r + + + = 0 . Thus,
r r r z
v k cos k sin
= v = + f ( r ) . Since v = 0 at = 0 , Therefore,
2 r 2 r
k sin
f ( r ) = 0 . Thus, v = .
2 r
_________________________________________________________________

3.52 Are the following two velocity fields isochoric (i.e., no change of volume)?
xe +x e x e + x e
(i) v = 1 1 2 2 2 , r 2 = x12 + x22 and (ii) v = 2 1 2 1 2 , r 2 = x12 + x22
r r
-----------------------------------------------------------------------------------------
Ans. (i) With v1 = x1 / r 2 , v2 = x2 / r 2 , r 2 = x12 + x22 ,
v1 1 2 x1 r 1 2 x2 2 2 2 r r
= 2 3 = 2 41 . r = x1 + x2 2r = 2 x1 , 2r = 2 x2 .
x1 r r x1 r r x1 x2
v2 1 2 x2 r 1 2 x2 v1 v2 2 2 x2 2 x2 2 2
= 2 3 = 2 42 , + = 2 41 42 = 2 2 = 0.
x2 r r x2 r r x1 x2 r r r r r

(ii) v1 = x2 / r 2 , v2 = x1 / r 2 , r 2 = x12 + x22

Copyright 2010, Elsevier Inc


3-24
Lai et al, Introduction to Continuum Mechanics

v1 2 x2 r 2 x2 x1 2 2 2 r
= 3 = 4 r = x1 + x2 2r = 2 x1
x1 r x1 r x1
v2 2 x1 r 2 x1 x2 v1 v2 2 x2 x1 2 x1 x2
= 3 = 4 , + = 4 4 = 0.
x2 r x2 r x1 x2 r r
_________________________________________________________________

3.53 Given that an incompressible and inhomogeneous fluid has a density field given by = kx2 .
From the consideration of the principle of conservation of mass, find the permissible form of
velocity field for a two dimensional flow ( v3 = 0 ) .
-----------------------------------------------------------------------------------------
Ans. Since the fluid is incompressible, therefore,
D
=0 + v1 + v2 = 0 0 + v1 ( 0 ) + v2 k = 0 v2 = 0.
Dt t x1 x2
The conservation of mass equation of an incompressible fluid in two dimensional flow is
v1 v2 v
+ = 0 1 = 0 v1 = f ( x2 ) , v2 = 0.
x1 x2 x1
_________________________________________________________________

x1
3.54 Consider the velocity field: v = e1 . From the consideration of the principle of
1 + kt
conservation of mass, (a) Find the density if it depends only on time t , i.e., = (t ) , with
( 0 ) = o . (b) Find the density if it depends only on x1 , i.e., = ( x1 ) , with ( xo ) = * .
-----------------------------------------------------------------------------------------
Ans.(a) Equation of conservation of mass is
v v v x1
+ v1 + v2 + v3 + 1 + 2 + 3 = 0 . With v1 = , v2 = v3 = 0 ,
t x1 x2 x3 x1 x2 x3 1 + kt

d d
t
dt / k
+ =0 = ln = ln (1 + kt ) = (1 + kt ) .
dt 1 + kt
0
1 + kt o k o
o

x1
(b) with = ( x1 ) and v1 = , v2 = v3 = 0
1 + kt
v v v x1 d
+ v1 + v2 + v3 + 1 + 2 + 3 =0 + =0
t x1 x2 x3 x1 x2 x3 1 + kt dx1 1 + kt
x
d d 1
dx x x
x1 + = 0, = 1 ln = ln 1 = * o
dx1 *
x
x1 * xo x1
o

where o is the density at x1 = xo .


_________________________________________________________________

3.55 Given the velocity field: v = ( x1te1 + x2te 2 ) . From the consideration of the principle of
conservation of mass, determine how the fluid density varies with time, if in a spatial description,
it is a function of time only.
-----------------------------------------------------------------------------------------
Copyright 2010, Elsevier Inc
3-25
Lai et al, Introduction to Continuum Mechanics

Ans. Equation of conservation of mass is


v v v
+ v1 + v2 + v3 + 1 + 2 + 3 = 0 . With v1 = x1t , v2 = x2t , v3 = 0 ,
t x1 x2 x3 x1 x2 x3

d d
t
+ ( t + t ) = 0
2
= 2 tdt ln = t 2 = o e t .
dt
0
o
o

_________________________________________________________________

Wim Eik Ekm 1 u u


3.56 Show that = , where Eim = i + m is the strain tensor and
X k X m X i 2 X m X i
1 u u
Wim = i m is the rotation tensor.
2 X m X i
------------------------------------------------------------------------------------------
Ans.
Wim 1 ui um 1 2ui 2 um
= = =
X k X k 2 X m X i 2 X m X k X i X k

1 2ui 2 uk 2 uk 2 um
+ =
2 X m X k X m X i X m X i X i X k

1 ui uk uk um Eik Ekm
+ + =
2 X m X k X i X i X m X k xm xi
_________________________________________________________________

3.57 Check whether or not the following distribution of the state of strain satisfies the
compatibility conditions:
X1 + X 2 , X1 X2
X 3 , k = 104
[ E] = k X1 X2 + X3
X 2 X3 X1 + X 3
-----------------------------------------------------------------------------------------
Ans. Yes. We note that the given Eij are linear in X1 , X 2 and X 3 and the terms in the
compatibility conditions all involve second derivatives with respect to X i , therefore these
conditions are obviously satisfied by the given strain components.
_________________________________________________________________

3.58 Check whether or not the following distribution of the state of strain satisfies the
compatibility conditions:
X12 X 22 + X 32 X1 X 3

[E] = k X 22 + X 32 0 X1 , k = 104

X1 X 3 X1 X 22

-----------------------------------------------------------------------------------------
Ans.

Copyright 2010, Elsevier Inc


3-26
Lai et al, Introduction to Continuum Mechanics

2 E11 2 E22 2 E12


+ =2 0 + 0 = 0,OK
X 22 X12 X1 X 2
2 E22 2 E33 2 E23
+ =2 0 + 2k 0, not satisfied
X 32 X 22 X 2 X 3
The given strain components are not compatible.
_________________________________________________________________

3.59 Does the displacement field: u1 = sin X1 , u2 = X13 X 2 , u3 = cos X 3 correspond to a


compatible strain field?
-----------------------------------------------------------------------------------------
Ans. Yes. The displacement field obviously exists. In fact, the displacement field is given. There
is no need to check the compatibility conditions. Whenever a displacement field is given, there is
never any problem of compatibility of strain components.
_________________________________________________________________

3.60 Given the strain field: E12 = E21 = kX1 X 2 , k = 104 and all other Eij = 0 .
(a) Check the equations of compatibility for this strain field and (b) by attempting to integrate the
strain field, show that there does not exist a continuous displacement field for this strain field.
-----------------------------------------------------------------------------------------
2 E11 2 E22 2 E12
Ans. (a) + = 2 0 + 0 2k . This compatibility condition is not satisfied.
X 22 X12 X1 X 2
u1 u
(b) E11 = 0 = 0 u1 = u1 ( X 2 , X 3 ) . Also, E22 = 0 2 = 0 u2 = u2 ( X1 , X 3 ) .
X1 X 2
u1 u2 u ( X , X ) u ( X , X )
Now, 2 E12 = + 2kX1 X 2 = 1 2 3 + 2 1 3 = f ( X 2 , X 3 ) + g ( X1 , X 3 ) ,
X 2 X1 X 2 X1
That is,
2kX1 X 2 = f ( X 2 , X 3 ) + g ( X1 , X 3 ) . Clearly, there is no way this equation can be satisfied,
because the right side can not have terms of the form of X1 X 2 .
_________________________________________________________________

3.61 Given the following strain components:


1
E11 = f ( X 2 , X 3 ) , E22 = E33 = f ( X 2 , X 3 ) , E12 = E13 = E23 = 0 .

Show that for the strains to be compatible, f ( X 2 , X 3 ) must be linear in X 2 and X 3 .
-----------------------------------------------------------------------------------------
Ans
1 f ( X2, X3 ) 1 f ( X2, X3 )
2 2
2 E11 2 E22 2 E12 2 E11 2 E33 2 E13
+ = 2 = 0, + = 2 =0
X 22 X12 X1 X 2 X 22 X 32 X12 X1 X 3 X 32
,
1 f ( X2, X3 )
2
2 E11 E23 E31 E12
= + + = 0 , Thus,
X 2 X 3 X1 X1 X 2 X 3 X 2 X 3

Copyright 2010, Elsevier Inc


3-27
Lai et al, Introduction to Continuum Mechanics

2 f ( X 2 , X 3 ) 2 f ( X 2 , X 3 ) 2 f ( X 2 , X 3 )
= 0, = 0, = 0 . f ( X 2 , X 3 ) is a linear function of
X 22 X 32 X 2 X 3
X 2 and X 3 . We note also
2 E22 2 E33 2 f 2 f 2 E23
+ = + = 0 = 2 ,
X 32 X 22 X 32 X 22 X1 X 3
2 E22 2 f E31 E12 E23
= =0= + + ,
X 3X1 X 3X1 X 2 X 2 X 3 X1
2 E33 2 f E12 E23 E31
= =0= + + .
X1X 2 X1X 2 X 3 X 3 X1 X 2
Thus, if f ( X 2 , X 3 ) is a linear function of X 2 and X 3 , then all compatibility equations are
satisfied.
_________________________________________________________________

3.62 In cylindrical coordinates ( r , , z ) , consider a differential volume bounded by the three pairs
of faces: r = r and r = r + dr; = and = + d ; z = z and z = z + dz. The rate at which mass is
flowing into the volume across the face r = r is given by vr ( rd )( dz ) and similar expressions
for the other faces. By demanding that the net rate of inflow of mass must be equal to the rate of
increase of mass inside the differential volume, obtain the equation of conservation of mass in
cylindrical coordinates. Check your answer with Eq. (3.15.7 ).
-----------------------------------------------------------------------------------------
Ans. Mass flux across the face r = r into the differential volume dV is ( vr )( rd ) dz . That
across the face r = r + dr out of the volume is ( vr )r = r + dr ( r + dr ) d dz . Thus ,
the net mass flux into dV through the pair of faces r = r and r = r + dr is
( vr )r =r ( rd ) dz ( vr )r =r + dr ( r + dr ) d dz = ( vr )r =r ( vr )r =r + dr rd dz
( vr )r = r + dr drd dz .
( vr )
Now, ( vr )r = r ( vr )r = r + dr rd dz = dr ( rd dz ) and
r
( vr )r = r + dr drd dz = ( vr ) + d ( vr ) drd dz = ( vr ) drd dz , where we have dropped
the higher order term involving d ( vr ) drd dz which approaches zero in the limit compared to
the terms involving only three differentials. Thus, the net mass flux into dV through the pair of
faces r = r and r = r + dr is
vr
r ( vr ) drd dz . Similarly,
r
the net mass flux into dV through the pair of faces = and = + d is
v
d ( drdz ) ,

and the net mass flux into dV through the pair of faces z = z and z = z + dz is

Copyright 2010, Elsevier Inc


3-28
Lai et al, Introduction to Continuum Mechanics

vz
dz dr ( rd )
z
Thus, the total influx of mass through these three pairs of faces is:
vr ( vr ) 1 v vz
+ r + r + z dr ( rd ) dz
r

On the other hand, the rate of increase of mass inside dV is ( rd drdz ) = rd drdz .
t t
Therefore, the conservation of mass principle gives,
vr ( vr ) 1 v vz
+ + + dr ( rd ) dz = rd drdz , That is:
r r r z t
vr vr 1 v vz
+ + + + z = 0 , Or,
t r r r
v vr vr 1 v vz
+ vr + + vz + + + + = 0 . This is the same as
t r r z r r r z
Eq.(3.15.7).
_________________________________________________________________

3.63 Given the following deformation in rectangular Cartesian coordinates:


x1 = 3 X 3 , x2 = X1 , x3 = 2 X 2
Determine (a) the deformation gradient F , (b) the right Cauchy-Green tensor C and the right
stretch tensor U , (c) the left Cauchy-Green tensor B , (d) the rotation tensor R , (e) the
Lagrangean strain tensor E* (f) the Euler strain tensor e* , (g) ratio of deformed volume to initial
volume, (h) the deformed area (magnitude and its normal) for the area whose normal was in the
direction of e 2 and whose magnitude was unity for the undeformed area.
-----------------------------------------------------------------------------------------
0 0 3 0 1 0 0 0 3 1 0 0
Ans. (a) [ F ] = 1 0 0 , (b) [C] = [ F ] [ F ] = 0 0 2 1 0 0 = 0 4 0 ,
T

0 2 0 3 0 0 0 2 0 0 0 9
1 0 0
[ U ] = [C] = 0 2 0 . (The only positive definite root).
1/2

0 0 3
0 0 3 0 1 0 9 0 0
(c) [ B ] = [ F ][ F ] = 1 0 0 0 0 2 = 0 1 0 .
T

0 2 0 3 0 0 0 0 4
0 0 3 1 0 0 0 0 1
= 1 0 0 0 1 / 2 0 = 1 0 0 .

1
(d) [ R ] = [ F ][ U ]
0 2 0 0 0 1 / 3 0 1 0

Copyright 2010, Elsevier Inc


3-29
Lai et al, Introduction to Continuum Mechanics

0 0 0 0 0 0 4 / 9 0 0
1 1 1
(e) E = [C - I ] = 0 3 0 = 0 3 / 2 0 , (f) e* = I B = 0
* 1
0 0 .
2 2 2
0 0 8 0 0 4 0 0 3 / 8
V
(g) = det B = ( 9 )(1)( 4 ) = 6 ,
Vo
0 6 0
( ) 1
n o , dAo = 1, det F = 6, [ F ] = 0 0 3 , no = e2 ,
T 1
(h) dA = dAo ( det F ) F 1
6
2 0 0
0 0 2 0 0

( ) 1
[ dA ] = dAo ( det F ) F no = (1)( 6 ) 6 6 0 0 1 = 0 dA = 3e3 .
T
1

0 3 0 0 3
_________________________________________________________________

3.64 Do the previous problem for the following deformation:


x1 = 2 X 2 , x2 = 3 X 3 , x3 = X1 .
-----------------------------------------------------------------------------------------
0 2 0 0 0 1 0 2 0 1 0 0
Ans. (a) [ F ] = 0 0 3 . (b) [C] = [ F ] [ F ] = 2 0 0 0 0 3 = 0 4 0 .
T

1 0 0 0 3 0 1 0 0 0 0 9
1 0 0
[ U ] = [C] = 0 2 0 . (The only positive definite root).
1/2

0 0 3
0 2 0 0 0 1 4 0 0
(c) [ B ] = [ F ][ F ] = 0 0 3 2 0 0 = 0 9 0 .
T

1 0 0 0 3 0 0 0 1
0 2 0 1 0 0 0 1 0
(d) [ R ] = [ F ][ U ] = 0 0 3 0
1
1 / 2 0 = 0 0 1 .
1 0 0 0 0 1 / 3 1 0 0

0 0 0 0 0 0 3 / 8 0 0
1 1 1
(e) E = [C I ] = 0 3 0 = 0 3 / 2 0 , (f) e = I B = 0 4 / 9 0 .
* * 1
2 2 2
0 0 8 0 0 4 0 0 0
V
(g) = det B = ( 4 )( 9 )(1) = 6 .
Vo
0 0 6
( ) 1
= 3 0 0 , no = e 2
T 1
(h) dA = dAo ( det F ) F 1
n o , dAo = 1, det F = 6, [ F ]
6
0 2 0

Copyright 2010, Elsevier Inc


3-30
Lai et al, Introduction to Continuum Mechanics

0 3 0 0 3
( )
[ dA ] = dAo ( det F ) F 1
1
n o = (1)( 6 ) 0 0 2 1 = 0 dA = 3e1
T

6
6 0 0 0 0
_________________________________________________________________

3.65 Do Prob. 3.63 for the following deformation:


x1 = X1 , x2 = 3 X 3 , x3 = 2 X 2
-----------------------------------------------------------------------------------------
1 0 0
Ans. (a) [ F ] = 0 0 3 .
0 2 0
1 0 0 1 0 0 1 0 0
(b) [C] = [ F ] [ F ] = 0 0 2 0 0 3 = 0 4 0 , The only positive definite root
T

0 3 0 0 2 0 0 0 9
1 0 0
is [ U ] = [C] = 0 2 0 .
1/2

0 0 3
1 0 0 1 0 0 1 0 0
(c) [ B ] = [ F ][ F ] = 0 0 3 0 0 2 = 0 9 0 .
T

0 2 0 0 3 0 0 0 4
1 0 0 1 0 0 1 0 0
(d) [ R ] = [ F ][ U ] = 0 0 3 0
1
1 / 2 0 = 0 0 1 ,
0 2 0 0 0 1 / 3 0 1 0

0 0 0 0 0 0
1 1
(e) E = [C I ] = 0 3 / 2 0 , (f) e* = I B = 0 4 / 9 0 .
* 1
2 2
0 0 4 0 0 3 / 8

V
(g) = det B = (1)( 9 )( 4 ) = 6 .
Vo
6 0 0
( ) 1
n o , dAo = 1, det F = 6, [ F ] = 0 0 3 , n o = e 2
T 1
(h) dA = dAo ( det F ) F 1
6
0 2 0
6 0 0 0 0

( ) 1
[ dA ] = dAo ( det F ) F no = (1)( 6 ) 6 0 0 2 1 = 0 dA = 3e3
T
1

0 3 0 0 3
_________________________________________________________________

3.66 Do Prob. 3.63 for the following deformation:

Copyright 2010, Elsevier Inc


3-31
Lai et al, Introduction to Continuum Mechanics

x1 = 2 X 2 , x2 = X1 , x3 = 3 X 3
-----------------------------------------------------------------------------------------
0 2 0 0 1 0 0 2 0 1 0 0
Ans. (a) [ F ] = 1 0 0 . (b) [C] = [ F ] [ F ] = 2 0 0 1 0 0 = 0 4 0 ,
T

0 0 3 0 0 3 0 0 3 0 0 9
1 0 0
[ U ] = [C] = 0 2 0 . (The only positive definite root).
1/2

0 0 3
0 2 0 0 1 0 4 0 0
(c) [ B ] = [ F ][ F ] = 1 0 0 2 0 0 = 0 1 0 .
T

0 0 3 0 0 3 0 0 9
0 2 0 1 0 0 0 1 0
1
(d) [ R ] = [ F ][ U ] = 1 0 0 0 1 / 2 0 = 1 0 0 .
0 0 3 0 0 1 / 3 0 0 1

0 0 0 3 / 8 0 0
1 1
(e) E = [C I ] = 0 3 / 2 0 ,
*
(f) e = I B
* 1
= 0 0 0
2 2
0 0 4 0 0 4 / 9

V
(g) = det B = ( 4 )(1)( 9 ) = 6 .
Vo
0 6 0
( ) 1
n o , dAo = 1, det F = 6, [ F ] = 3 0 0 , n o = e 2
T 1
(h) dA = dAo ( det F ) F 1
6
0 0 2
0 3 0 0 3
1
[dA] = dAo (det F)[F ] [n o ] = (1)(6) 6 0 0 1 = 0 dA = 3e1
1 T
6
0 0 2 0 0
_________________________________________________________________

3.67 Given x1 = X1 + 3 X 2 , x2 = X 2 , x3 = X 3 .
Obtain (a) the deformation gradient F and the right Cauchy-Green tensor C , (b) The eigenvalues
and eigenvector of C , (c) the matrix of the stretch tensor U and U 1 with respect to the ei -basis
and (d) the rotation tensor R with respect to the ei -basis.
-----------------------------------------------------------------------------------------
1 3 0 1 0 0 1 3 0 1 3 0
Ans. (a) [ F ] = 0 1 0 , [C] = [ F ] [ F ] = 3 1 0 0 1 0 = 3 10 0 .
T

0 0 1 0 0 1 0 0 1 0 0 1
(b) the characteristic equation is

Copyright 2010, Elsevier Inc


3-32
Lai et al, Introduction to Continuum Mechanics

1 3 0
3 10 ( )
0 = 0 (1 ) 2 11 + 1 = 0,
0 0 1
11 121 4
1,2 = 1 = 10.908326, 2 =0.0916735, 3 = 1
2
For 1 = 10.908326 ,
(1 1 )1 + 3 2 = 0 2 = (1 1 )1 / 3 = 3.3027751 ,
1
n1 = ( e1 + 3.302775e2 ) = 0.289785e1 + 0.957093e2 .
3.450843
For 2 = 0.0916735 ,
(1 2 )1 + 3 2 = 0 2 = (1 2 )1 / 3 = 0.30277551 ,
1
n2 = ( e1 0.3027755e2 ) = 0.957093e1 0.289784e2 .
1.044832
For 3 = 1, n3 = e3 ,
(c) The matrices with respect to the principal axes are as follows
10.9083 0 0 3.30277 0 0
0.302774 0 .
[C]ni = 0 0.0916735 0 [ U ]n = 0
i
0 0 1 0 0 1
0.302774 0 0

U =
1 0 3.302772 0 .
ni
0 0 1
The matrices with respect to the ei -basis are given by the formula [ U ]{e } = [Q ] [ U ]{n } [Q ] :
T
i i

0.289785 0.957093 0 3.30277 0 0 0.289785 0.957093 0


0.302774 0 0.957093 0.289785 0
[ U ]e i
= 0.957093 0.289785 0 0
0 0 1 0 0 1 0 0 1

0.554704 0.832057 0
= 0.832057 3.05087 0 .
0 0 1
0.289785 0.957093 0 0.302774 0 0 0.289785 0.957093 0
U 1 = 0.957093 0.289785 0 0 3.302772 0 0.957093 0.289785 0
ei
0 0 1 0 0 1 0 0 1
3.050852 0.832052 0
= 0.832052 0.554701 0 .
0 0 1

Copyright 2010, Elsevier Inc


3-33
Lai et al, Introduction to Continuum Mechanics

1 3 0 3.050852 0.832052 0 0.55470 0.83205 0


(d) [ R ]e = [ F ] U = 0 1 0 0.832052 0.554701 0 = 0.83205 0.55470 0 .
1
i
0 0 1 0 0 1 0 0 1
_________________________________________________________________

3.68 Verify that with respect to rectangular Cartesian base vectors, the right stretch tensor U and
the rotation tensor R for the simple shear deformation , x1 = X1 + kX 2 , x2 = X 2 , x3 = X 3 ,

are given by: With f = (1 + k 2 / 4)1/2 ,


f kf / 2 0 f kf / 2 0


( 2
)
[ U ] = kf / 2 1 + k / 2 f 0 , [ R ] = kf / 2 f 0 .


0 0 1
0 0 1
-----------------------------------------------------------------------------------------
f kf / 2 0 f kf / 2 0

Ans. [ RU ] = kf / 2 f ( )
0 kf / 2 1 + k 2 / 2 f 0
0 0 1 0 0

1

f 2 + ( kf / 2 )( kf / 2 )
( )
f ( kf / 2 ) + ( kf / 2 ) 1 + k 2 / 2 f 0


( )
= ( kf / 2 ) f + f ( kf / 2 ) ( kf / 2 )( kf / 2 ) + f 1 + k / 2 f 0
2

0 0 1


2
( 2
)
f 1 + k / 4 kf 1 + k / 4 0
2
(2
) 1 k 0
=

0 ( )
f 1 + k / 4 0 = 0 1 0 = the given [ F ]
2 2


0 0 1 0 0 1

Since the decomposition of F is unique, therefore, the given R and U are the rotation and the
stretch tensor respectively.
_________________________________________________________________

3.69 Let dX( ) = dS1N( ) , dX( ) = dS2 N( ) be two material elements at a point P. Show that if
1 1 2 2

denotes the angle between their respective deformed elements dx( ) =ds m and dx( ) = ds n ,
1 2
1 2

C N(1) N (2)
, where N( ) = N(1)e , N( ) = N(2)e , 1 = 1 and 2 = 2 .
1 2 ds ds
then, cos =
12 dS1 dS2
-----------------------------------------------------------------------------------------
Ans. dx( ) dx( ) = FdX( ) FdX( ) = dX( ) F T FdX( ) = dX( ) CdX( ) ,
1 2 1 2 1 2 1 2

( 2)
ds1ds2 cos = dS1dS2 N( ) CN( ) = dS1dS2 ( N( )e ) C( N e ),
1 2 1

( 2)
C N( ) N
1
dS1dS2 (1) ( 2 )
cos = N N e Ce = .
ds1ds2 12

Copyright 2010, Elsevier Inc


3-34
Lai et al, Introduction to Continuum Mechanics

_________________________________________________________________

3.70 Given the following right Cauchy-Green deformation tensor at a point


9 0 0
[C] = 0 4 0

0 0 0.36
(a) Find the stretch for the material elements which were in the direction of e1 , e 2 ,and e3 . (b)
Find the stretch for the material element which was in the direction of e1 + e2 . (c) Find cos ,
where is the angle between dx( ) and dx( ) where dX( ) = dS e and dX( ) = dS e deform to
1 2 1 2
1 1 2 1

dx( ) = ds1m and dx( ) = ds2n .


1 2

-----------------------------------------------------------------------------------------

Ans. (a) For the elements which were in e1 , e 2 ,and e3 direction, the stretches are
C11 , C22 , C33 , that is, 3, 2 and 0.6 respectively.
9 0 0 1 9
1 1 1 13
(b) Let e1' = ( e1 + e2 ) C11 = [1 1 0] 0 4 0 1 = [1 1 0] 4 = .
'
2 2 2 2
0 0 0.36 0 0

That is, the stretch for dX = dSe1' is ( ds / dS ) = '


C11 = 13 / 2 .
(c) C12 = 0 cos = 0 = 90o . There is no change in angle. (note, {e1 , e 2 , e3} are principal
axes for C .
_________________________________________________________________

3.71 Given the following large shear deformation:


x1 = X1 + X 2 , x2 = X 2 , x3 = X 3 .
(a) Find the stretch tensor U (Hint: use the formula given in problem 3.68) and verify that
U 2 = C , the right Cauchy-Green deformation tensor. (b) What is the stretch for the element
which was in the direction e 2 ?
(c) Find the stretch for an element which was in the direction of e1 + e 2 .
(d) What is the angle between the deformed elements of dS1e1 and dS2e2 ?.
-----------------------------------------------------------------------------------------
Ans. (a) For x1 = X1 + kX 2 , x2 = X 2 , x3 = X 3 , from Prob. 3.68, we have
f kf / 2 0
1
k2
(1 + k / 2) f
2
[ U ] = kf / 2 2
0 where f = 1 +
. Thus, with k = 1 , f = 2 / 5
4
0 0 1
f f /2 0 1 1/ 2 0 1 1/ 2 0
2
[ U ] = f / 2 (3 / 2) f 0 = f 1 / 2 3 / 2 0 = 1 / 2 3 / 2 0 .
5
0 0 1 0 0 1 / f 0 0 5 / 2

Copyright 2010, Elsevier Inc


3-35
Lai et al, Introduction to Continuum Mechanics

1 1/ 2 0 1 1/ 2 0 1 1 0
4
[ U ][ U ] = 1 / 2 (3 / 2) 0 1 / 2 (3 / 2) 0 = 1 2 0 = [ C] .
5
0 0 5 / 2 0 0 5 / 2 0 0 1
(b) The stretch for the element which was in the direction e 2 is C22 = 2 .
(c) Let e1' = ( e1 + e2 ) / 2 ,
1 1 0 1 2
1 1 5
= [1 1 0] 1 2 0 1 = [1 1 0] 3 =

' ds
C11 = 5/ 2 .
2 2 2 dS
0 0 1 0 0
ds ds
dS1 dS2
( )
(d) 1 2 cos = C12 (1) 2 cos = 1 cos =
1
2
= 45o .

_________________________________________________________________

3.72 Given the following large shear deformation:


x1 = X1 + 2 X 2 , x2 = X 2 , x3 = X 3
(a) Find the stretch tensor U (Hint: use the formula given in problem 3.68) and verify that
U 2 = C , the right Cauchy-Green deformation tensor.
(b) What is the stretch for the element which was in the direction e 2 .
(c) Find the stretch for an element which was in the direction of e1 + e2 .
(d) What is the angle between the deformed elements of dS1e1 and dS2e 2 .
-----------------------------------------------------------------------------------------
Ans. For x1 = X1 + kX 2 , x2 = X 2 , x3 = X 3 , from Prob. 3.68, we have
f kf / 2 0
1
k2
( )
2
[ U ] = kf / 2 1 + k 2 / 2 f 0 where f = 1 +
. Thus, with k = 2 , f = 1 / 2
4
0 0 1
f kf / 2 0 1 1 0

[ U ] = kf / 2

( )
1 + k 2 / 2 f 0 =

1
1
2
3 0 .

0 0 1 0 0 2

f kf / 2 0 1 1 0
1
[ R ] = kf / 2 f 0 = 1 1 0 .
2
0 0 1 0 0 2

2 1 1 0 1 1 0 1 2 0
1
[ U ][ U ] = 1 3 0 1 3 0 = 2 5 0 = [C] .
2
0 0 2 0 0 2 0 0 1
(b) The stretch for the element which was in the direction e2 is C22 = 5 .
(c) Let e1 = ( e1 + e2 ) / 2 ,

Copyright 2010, Elsevier Inc


3-36
Lai et al, Introduction to Continuum Mechanics

1 2 0 1 3
1 1
= [1 1 0] 2 5 0 1 = [1 1 0] 7 = 5
ds
C11 = 5 = 2.236 .
2 2 dS
0 0 1 0 0
ds ds
1 2
dS dS
( )
(d) 1 2 cos = C12 (1) 5 cos = 2 cos =
2
5
.

_________________________________________________________________

A jn
3.73 Show that for any tensor A ( X1 , X 2 , X 3 ) ,

X m
det A = ( det A ) A 1 ( ) nj X m
-----------------------------------------------------------------------------------------
Ans

A11 A12 A13


A11 A12 A13
A11 A12 A13 X m X m X m A11 A12 A13
A A A22 A23
A = A21 A22 A23 = A21 A22 A23 + 21 + A21 A22 A23
X m X m X m X m
A31 A32 A33 A31 A32 A33 A31 A32 A33
A31 A32 A33
X m X m X m
.
A22 A23 c A A23
Let Aijc denote the cofactor of Aij , i.e., A11
c
= , A12 = 21 etc.
A32 A33 A31 A33
A A11 c A12 c A13 c A21 c A22 c
Then, = A11 + A12 + A13 + A21 + A22 + ...
X m X m X m X m X m X m
A Aij Acji
That is,
X m
=
X m
Aijc . On the other hand, A ( ) 1
ij
=
det A
Acji = det A A 1 ( ) ij
.

A Aij Anj
Thus,
X m ji X
( )
= det A A 1
m
jn X
m
. ( )
= det A A 1

_________________________________________________________________

3.74 Show that if TU = 0 , where the eigenvalues of U are all positive (nonzero), then T = 0 .
-----------------------------------------------------------------------------------------
Ans. Using the eigenvectors of U as basis, we have,
T11 T12 T13 1 0 0 1T11 2T12 3T13
[TU ] = [T][ U ] = T21 T22 T23 0 2 0 = 1T21 2T22 3T23
T31 T32 T33 0 0 3 1T31 2T32 3T33
Thus, TU = 0 gives, all Tij = 0 , that is, T = 0 .
_________________________________________________________________

2 2 2
r r r
3.75 Derive Eq. (3.29.21), that is, B = + +
ro ro o zo
-----------------------------------------------------------------------------------------
Copyright 2010, Elsevier Inc
3-37
Lai et al, Introduction to Continuum Mechanics

Ans. B = e Be = e FF T e . From Eq. (3.29.15). we have,


r o r o r o
F T e = er + e + ez , thus,
ro ro o zo
r o r o r o r r r
B = e F er + e + ez = e Feor + e Feo + e Feoz .
ro ro o zo ro ro o zo
r r r
Since, e Feor = , e Feo = , e Feoz = , [See Eq. (3.29.10)], therefore,
ro ro o zo
2 2 2
r r r
B = + + .
ro ro o zo
_________________________________________________________________

r z r z r z
3.76 Derive Eq. (3.29.23), i.e., Brz = + +
ro ro ro o ro o zo zo
-----------------------------------------------------------------------------------------
Ans. Brz = er Bez = er FF T ez , from Eq. (3.29.16), we have,
z o z o z o
FTez = er + e + e z , thus,
ro ro o zo
z o z o z o z z z
Brz = er F er + e + ez , = er Feor + er Feo + er Feoz .

or r
o o z o ro r
o o z o
r r r
From Eq. (3.29.9), er Feor = , er Feo = , e Feoz = , thus,
ro ro o zo
r z r z r z
Brz = + + .
ro ro ro o ro o zo zo
_________________________________________________________________

3.77 From ro = ro ( r , , z , t ) , o = o ( r , , z , t ) , zo = zo ( r , , z , t ) ,derive the components of B 1


with respect to the basis at x .
-----------------------------------------------------------------------------------------
Ans. From dX = F 1dx , where dx = drer + rd e + dzez and dX = dro eor + ro d o eo + dzo eoz , we
have, dro eor + ro d o eo + dzo eoz = F 1 ( drer + rd e + dzez )

( ) ( ) (
dro = dr eor F 1er + rd eor F 1e + dz eor F 1ez )

ro
r
r

r
( ) ( )
dr + o d + o dz = dr eor F 1er + rd eor F 1e + dz eor F 1ez
z
( )
1 ro r r
o
er F er = , eor F 1e = o , eor F 1e z = o .
r r z

Copyright 2010, Elsevier Inc


3-38
Lai et al, Introduction to Continuum Mechanics

Similarly,
ro o r r
eo F 1er = , eo F 1e = o o , eo F 1ez = o o .
r r z
z z z
eoz F 1er = o , eoz F 1e = o , eoz F 1ez = o .
r r z
Thus,
r r z r r z
F 1er = o eor + o o eo + o eoz , F 1e = o eor + o o eo + o eoz
r r r r r r
r r z
F 1ez = o eor + o o eo + o eoz .
z z z
Also, we have,

( ) r r
( )
T T
er F 1 eor = eor F 1er = o , e F 1 eor = eor F 1e = o ,
r r

( ) ( )

T r
ez F 1 eor = eor F 1 ez = o .
z
Thus,
r r r
( ) r r r
( )
T T
F 1 eor = o er + o e + o ez , F 1 eo = o o er + o o e + o o ez
r r z r r z

( )
T z z z
F 1 eoz = o er + o e + o ez .
r r z
The components of B 1 with respect to the basis at x are:
( ) ( ) (F e )
1 T
Brr1 = er B 1er = er FF T e r = e r F 1 1
r
2 2 2
ro o r r z
r
( ) ( ) zo
( )
T T T
= o er F 1 eor + er F 1 eo + e r F 1 eoz = o + o o + o .
r r r r r r

( ) ( ) (F e )
1 T
1
B = e B 1e = e FF T e = e F 1 1

2 2 2
ro o r r z
ro
( ) ( ) zo
( )
T T T
= e F 1 eor + e F 1 eo + e F 1 eoz = o + o o + o .
r r r r r r
2 2 2
1 ro r z
Bzz = + o o + o .
z z z

( ) ( ) (F e )
1 T
Br1 = er B 1e = er FF T e = e r F 1 1

ro o ro o ro o
ro
( ) ( ) zo
( ) ro ro zo zo
T T T
= e r F 1 eor + e r F 1 eo + e r F 1 eoz = + +
r r r r r r r r z

( ) ( ) (F e )
1 T
Brz1 = er B 1ez = er FF T ez = er F 1 1
z

r ro o ro o
ro
( ) e ( F ) e +
z
e (F )
r r zo zo
T T T
= er F 1 eor + o o r 1 o o
r
1
eoz = o o + r z + r z
z z z
r z
r r r r z z
Bz1 = o o + o o o o + o o .
r z r z r z

Copyright 2010, Elsevier Inc


3-39
Lai et al, Introduction to Continuum Mechanics

_________________________________________________________________

2 2 2
r r z
3.78 Derive Eq. (3.29.47), that is, Coo = + +
ro o ro o ro o
-----------------------------------------------------------------------------------------
r r z
Ans. Coo = eo Ceo = eo F T Feo . Now, Feo = er + e + e z [Eq.3.29.3] ,
ro o ro o ro o
therefore,
r r z r o T r o T
Coo = eo F T er + e + ez = e F e r + e F e
ro o ro o ro o ro o ro o
z o T
+ e F e z . Now, from Eqs. (3.29.14) (3.29.15) and (3.29.16),
ro o
r r z
eo F T er = , eo F T e = , eo F T ez = , thus.
ro o ro o ro o
2 2 2
r r z
Coo = + +
ro o ro o ro o
_________________________________________________________________

r r r r z z
3.79 Derive Eq. (3.29.49), Croo = + +
ro o ro ro o ro ro o ro
------------------------------------------------------------------------------------------
r r z
Ans. Croo = eor Ceo = eor F T Feo . Now, Feo = er + e + e z [Eq.(3.29.3)] ,
ro o ro o ro o
r r z r o T r o T
Croo = eor F T er + e + ez = er F er + e r F e
ro o ro o ro o ro o ro o
z o T
+ er F ez . From Eqs. (3.29.14), (3.29.15) and (3.29.16)
ro o
r o T r o T z
eor F T er = , e r F e = , er F ez = ,
ro ro ro
r r r r z z
Thus, Croo = + + .
ro o ro ro o ro ro o ro
_________________________________________________________________

3.80 Derive the components of C1 with respect to the bases at X .


-----------------------------------------------------------------------------------------
Ans.

( ) ( ) r r r
1 T
Cro1ro =eor F T F eor =eor F 1 F 1 eor = o eor F 1er + o eor F 1e + o eor F 1ez
r r z

Copyright 2010, Elsevier Inc


3-40
Lai et al, Introduction to Continuum Mechanics

ro ro ro ro ro ro
= + + . [See Eqs.(3.29.30), (3.29.31) and (3.29.32)].
r r r r z z
r r r
( ) ( )
1 T
Cro1o =eor F T F eo =eor F 1 F 1 eo == o o eor F 1er + o o eor F 1e + o o eor F 1ez
r r z
r r r r r r
= o o o + o o o + o o o .
r r r r z z
The other components can be similarly derived.
_________________________________________________________________

3.81 Derive components of B with respect to the basis {er ,e ,ez } at x for the pathline equations
given by r = r ( X , Y , Z , t ), = ( X , Y , Z , t ), z=z( X , Y , Z , t ) .
---------------------------------------------------------------------------------------------
Ans. From dx = drer + rd e + dzez and dX = dXeX + dYe Y + dZe Z and
r = r ( X , Y , Z , t ), = ( X , Y , Z , t ), z=z( X , Y , Z , t ) , we have,
dx = FdX dx = dre r + rd e + dzez = dXFeX + dYFeY + dZFe Z
r r r r r r
dX + dY + dZ er + dX + dY + dZ e
X Y Z X Y Z
z z z
+ dX + dY + dZ e z = dXFe X + dYFeY + dZFe Z
X Y Z
r r z r r z
Fe X = er + e + ez , FeY = er + e + ez ,
X X X Y Y Y
r r z
Fe Z = er + e + e z , and
Z Z Z
r r
e X F T er = er FeX = , eY F T er = er FeY = , etc.
X Y
r r r r r r
F T er = eX + eY + e Z , F T e = eX + eY + eZ ,
X Y Z X Y Z
z z z
FTe z = eX + eY + eZ .
X Y Z
The components of B are:
2 2 2
r r r r r r
Brr = e r FF T e r =e r Fe X + e r Fe Y + er Fe Z = + + .
X Y Z X Y Z
r r r
Br = e r FF T e = er FeX + er FeY + er Fe Z
X Y Z
r r r r r r
= + + .
X X Y Y Z Z
_________________________________________________________________

3.82 Derive the components of B 1 with respect to the basis {e r ,e ,e z } at x for the pathline
equations given by X = X (r , , z , t ), Y = Y (r , , z , t ), Z =Z (r , , z , t ) .
-----------------------------------------------------------------------------------------
Ans. From dx = drer + rd e + dzez and dX = dXeX + dYe Y + dZe Z and
Copyright 2010, Elsevier Inc
3-41
Lai et al, Introduction to Continuum Mechanics

X = X (r , , z , t ), Y = Y (r , , z , t ), Z =Z (r , , z , t ) , we have,
dX = F 1dx dXe X + dYeY + dZe Z = drF 1er + rd F 1e + dzF 1ez
X X X Y Y Y
dr + d + dz e X + dr + d + dz e Y
r z r z
Z Z Z
+ dr + d + dz e Z = drF 1e r + rd F 1e + dzF 1ez .
r z
Thus,
X Y Z X Y Z
F 1e r = eX + eY + e Z , F 1e = eX + eY + eZ
r r r r r r
X Y Z
F 1e z = eX + eY + eZ .
z z z
and

( ) X
( ) Y
T T
e r F 1 e X = eX F 1er = , er F 1 eY = eY F 1er = ,
r r

( ) Z
T
e r F 1 e Z = e Z F 1er = , etc. that is,
r

( ) X X X
( ) Y Y Y
T T
F 1 e X = er + e + e z , F 1 e Y = er + e + ez
r r z r r z

( ) Z Z Z
T
F 1 e Z = er + e + ez .
r r z
Thus,

( ) ( ) X
( ) Y
( )
1 T T T
Brr1 = e r FF T e r = e r F 1 F 1e r = e r F 1 eX + er F 1 eY
r r
2 2 2
Z
( ) X Y Z
T
+ e r F 1 eZ = + + .
r r r r

( ) X
( ) Y
( )
T T T
Br1 = e r F 1 F 1e = e r F 1 e X + e r F 1 e Y
r r
e (F )
Z 1
T X X Y Y Z Z

+ r eZ = + + .,
r r r r r r r
etc.
_________________________________________________________________

3.83 Verify that (a) the components of B with respect to {e r ,e ,e z } can be obtained from FF T

o o o
{ }
and (b) the component of C , with respect to e r ,e , e z can be obtained from F F , where [ F ]

T

is the matrix of the two points deformation gradient tensor given in Eq. (3.29.12).
-----------------------------------------------------------------------------------------
Ans.
(a) Eq. (3.29.12)

Copyright 2010, Elsevier Inc


3-42
Lai et al, Introduction to Continuum Mechanics

r r r r r z

ro ro o zo ro ro ro
r r z
FF T = r r

r

ro o zo ro o ro o ro o
ro
z z z r r z

ro ro o zo zo zo zo
2 2 2
r r r r r r r r r
Brr = + + , Br = + + etc.
ro ro o zo ro ro ro o ro o zo zo
(b)
r r z r r r

ro ro ro ro ro o zo
r z r r r
F T F = r

ro o ro o ro o ro ro o zo
r r z z z z

zo zo zo ro ro o zo
2 2 2
r r z r r r r z z
Cro ro = + + , Croo = + + .
ro ro ro ro ro o ro ro o ro ro o
_________________________________________________________________

3.84 Given r = ro , = o + kzo , z = zo . (a) Obtain the components of the Left Cauchy-Green
tensor B , with respect to the basis at the current configuration ( r , , z ) . (b) Obtain the
components of the right Cauchy-Green tensor C with respect to the basis at the reference
configuration.
-----------------------------------------------------------------------------------------
Ans. (a), Using Eqs (3.29.19) to (3.29.24). we obtain
2 2 2
r r r
Brr = + + =1,
ro ro o zo
2 2 2 2
r r r r
= + ( kr ) = 1 + ( kr ) ,
2 2
B = + +
ro ro o zo ro
2 2 2
z z z
Bzz = + + = 1,
ro ro o zo
r r r r r r
Br = + + = 0,
ro ro ro o ro o zo zo
r z r z r z
Brz = + + =0,

o o o o o o o o
r r r r z z
z r z r z r
Bz = + + = rk .
ro ro ro o ro o zo zo
Copyright 2010, Elsevier Inc
3-43
Lai et al, Introduction to Continuum Mechanics

.
1 0 0

Thus, [ B ] = 0 1 + ( rk ) rk .
2

0 rk 1

(b) Using Eqs.(3. 29.43) to (3. 29.51), we have,
2 2 2 2 2 2
r r z r r z
Cro ro = + + = 1, Coo = + + = 1,
ro ro ro ro o ro o ro o
2 2 2
r r z
= 1 + ( rk ) ,
2
Cz o z o = + +
zo z o z o
r r r r z z
Croo = + + = 0,
ro o ro ro o ro ro o ro
r r r r z z
Cro zo = + + = 0,
ro zo ro zo ro zo
r r r r z z
C zo o = + + = rk .
zo ro o zo ro o zo ro o
1 0 0

Thus, [C] = 0 1 rk .
2
0 rk 1 + ( rk )

_________________________________________________________________

3.85 Given r = ( 2aX + b ) , = Y / a, z = Z , where ( r , , z ) are cylindrical coordinates for the


1/2

cuurent configuration and ( X , Y , Z ) are rectangular coordinates for the reference configuration.
(a) Obtain the components of [ B ] with respect to the basis at the current configuration and (b)
calculate the change of volume.
-----------------------------------------------------------------------------------------
Ans. (a) Using Eqs.(3.29.59) to (3.29.64), we have,
2 2 2 2 2 2 2 2
r r r a r r r r
Brr = + + = , B = + + =
X Y Z r X Y Z a
2 2 2
z z z
Bzz = + + =1,
X Y Z
r r r r r r
Br = + + =0,
X X Y Y Z Z
r z r z r z
Brz = + + = 0,
X X Y Y Z Z
r z r z r z
B z = + + = 0.
X X Y Y Z Z
Copyright 2010, Elsevier Inc
3-44
Lai et al, Introduction to Continuum Mechanics

( a / r ) 2 0 0

Thus, [ B ] = ( r / a ) 0 .
2
0

0 0 1

(b) det B = 1 =1, thus, there is no change of volume.
_________________________________________________________________

3.86 Given r = r ( X ), = g (Y ), z = h( Z ) , where ( r , , z ) and ( X , Y , Z ) are cylindrical and


rectangular Cartesian coordinate with respect to the current and the reference configuration
respectively. Obtain the components of the right Cauchy-Green Tensor C with respect to the basis
at the reference configuration.
-----------------------------------------------------------------------------------------
Ans. Using Eqs.(3.29.68) etc. we have,
2 2 2
r r z
= ( r ( X ) ) , CYY = ( rg (Y ) ) , CZZ = ( h( Z ) )
2 2 2
CXX = + X + X
X
r r r r z z
CXY = Y + X Y + X Y = 0, CYZ = 0, CXZ = 0 .
X
( r ( X ) ) 2
0 0

[ C] = 0
( g (Y ) ) 2
0 , where r ( X ) dr / dX , etc.,



0 0 ( h( Z ) )
2

_________________________________________________________________

Copyright 2010, Elsevier Inc


3-45
Lai et al, Introduction to Continuum Mechanics

CHARTER 4

1 2 3
4.1 The state of stress at a certain point in a body is given by: [ T ] = 2 4 5 MPa. .
3 5 0
ei

On each of the coordinate planes (with normal in e1 , e2 , e3 directions), (a) what is the normal
stress and (b) what is the total shearing stress
------------------------------------------------------------------------------
Ans. (a) The normal stress on the e1 plane (i.e., the plane whose normal is in the direction e1 ) is
1 MPa. , on the e2 plane is 4 MPa. , and on the e3 plane is 0 MPa.
(b) The total shearing stress on the e1 plane is 22 + 32 = 13 =3.61 MPa. On the e 2 plane is
22 + 52 = 29 =5.39 MPa. , and on the e3 plane is 32 + 52 = 34 = 5.83 MPa .
_________________________________________________________________

2 1 3
4.2 The state of stress at a certain point in a body is given by: [ T] = 1 4 0 MPa.
3 0 1
ei

(a) Find the stress vector at a point on the plane whose normal is in the direction of
2e1 + 2e2 + e3 . (b) Determine the magnitude of the normal and shearing stresses on this plane.
-------------------------------------------------------------------------------
Ans. (a) The stress vector on the plane is t = Tn , where n = (2e1 + 2e2 + e3 ) / 3 . Thus
2 1 3 2 5
1 1
[t ] = 3 1 4 0 2 = 3 6 , t = (5e1 + 6e2 + 5e3 ) / 3 MPa.
3 0 1 1 5
(b) Normal stress Tn = n t = (1 / 9) ( 2e1 + 2e2 + e3 ) ( 5e1 + 6e2 + 5e3 ) = 3 MPa.
2
Magnitude of shearing stress Ts = t Tn2 = 86 / 9 9 = 0.745 MPa. Or,
Ts = t - Tnn = ( 5e1 + 6e2 + 5e3 ) / 3 3 (1 / 3) ( 2e1 + 2e2 + e3 ) = ( e1 + 2e3 ) / 3 Ts = 5 / 3 = 0.745
__________________________________________________________________

4.3 Do the previous problem for a plane passing through the point and parallel to the plane
x1 2 x2 + 3 x3 = 4 .
-------------------------------------------------------------------------------
Ans. (a) The normal to the plane is n = (e1 2e2 + 3e3 ) / 14 . [ t ] = [ T][n ]
2 1 3 1 13
1 1
[t ] = 1 4 0 2 = 9 , t = (1 / 14)(13e1 9e2 ) = 3.47e1 2.41e2 MPa.
14 14
3 0 1 3 0
13
(b) Normal stress Tn = n t = [1 2 3] 9 = 31 / 14 = 2.21 MPa.
1
14
0

Copyright 2010, Elsevier Inc


4-1
Lai et al, Introduction to Continuum Mechanics

t Tn2 = 250 / 14 ( 2.21) = 3.60 MPa. Or,


2 2
Magnitude of shearing stress Ts =
Ts = t Tn n = (3.47e1 2.41e2 ) (2.21 / 14)(e1 2e2 + 3e3 ) = 2.88e1 1.23e2 1.77e3 ,
Ts = 3.60 MPa.
_________________________________________________________________

4.4 The stress distribution in a certain body is given by


0 100 x1 100 x2
0 MPa.
[T] = 100 x1 0
100 x2 0 0

Find the stress vector acting on a plane which passes through the point 1 / 2, ( )
3 / 2, 3 and is

tangent to the circular cylindrical surface x12 + x22 = 1 at that point.


------------------------------------------------------------------------------
Ans. Let f = x12 + x22 , then the unit normal to the circle f = 1 at a point ( x1 , x2 ) is given by
f 2 x e + 2 x2e2
n=
f
= 11 (
= x1e1 + x2e2 . At the point 1 / 2, )
3 / 2, 3 , n =
1
2
(e1 + 3e 2 . )
4 x12 + 4 x22
0 50 50 3

and [ T ] = 50 0 0 , thus,

50 3 0 0
0 50 50 3 1 / 2 25 3

[t ] = 50 0 0 3 / 2 = 25 t = 25 3e1 + 25e 2 25 3e3 MPa.

50 3 0 0 0 25 3
_________________________________________________________________

4.5 Given T11 = 1 MPa., T22 = 1 MPa. , and all other Tij = 0 at a point in a continuum. (a)
Show that the only plane on which the stress vector is zero is the plane with normal in the
e3 direction. (b) Give three planes on which there is no normal stress acting.
------------------------------------------------------------------------------
1 0 0 n1 n1
Ans. (a) [ t ] = 0 1 0 n2 = n2 t = n1e1 n2e 2 . t = 0 n1 = n2 = 0 t = e3 .
0 0 0 n3 0
(b) Tn = n t = n12 n22 . Thus, the plane with n12 n22 = 0 has no normal stress. These include
n = e3 , n = (e1 + e2 ) / 2, n = (e1 e2 ) / 2 etc.
_________________________________________________________________

10 50 50
4.6 For the following state of stress [ T ] = 50 0 0 MPa. , find T11 and T13 where e1
50 0 0
is in the direction of e1 + 2e2 + 3e3 and e2 is in the direction of e1 + e2 e3 .

Copyright 2010, Elsevier Inc


4-2
Lai et al, Introduction to Continuum Mechanics

------------------------------------------------------------------------------
Ans. e1 = (e1 + 2e2 + 3e3 ) / 14, e2 = (e1 + e2 e3 ) / 3 , thus
10 50 50 1 40
1
T11 = [1 2 3] 50 0
' 0 2 = [1 2 3] 50 = 90 / 14 = 6.43 MPa.
1
14 14
50 0 0 3 50
e3' = e1' e2' = (5e1 + 4e2 e3 ) / 42 , therefore,
10 50 50 5
[1 2 3] 50 0 0 4 = 450 / 588 =18.6 MPa .
1
T13 =
'
588
50 0 0 1
_________________________________________________________________

x2 0
4.7 Consider the following stress distribution [ T ] = 0 0 where and are
0 0 0
constants. (a) Determine and sketch the distribution of the stress vector acting on the square in the
x1 = 0 plane with vertices located at ( 0,1,1) , ( 0, 1,1) , ( 0,1, 1) and ( 0, 1, 1) . (b) Find the total
resultant force and moment about the origin of the stress vectors acting on the square of part (a).
------------------------------------------------------------------------------
Ans. (a) The normal to the plane x1 = 0 is e1 , thus, t e1 = x2e1 + e2 . On the plane, there is a
constant shearing stress in the e 2 direction and a linear distribution of normal stress x2 , (see
figure).

(b)
1 1
FR = tdA = ( x2e1 + e2 )dx2 dx3 = 0e1 + 4 e2 .
1 1
1 1
M o = x tdA = ( x2e2 + x3e3 ) ( x2e1 + e2 )dx2 dx3
1 1
1
x3 4
( )
1 1
= dx2 dx3 = 0e1 + 0e2 ( 2 ) 2 e3 =
x3e1 + x2 x3e 2 x22e3 e3
1 1 3 1 3
_________________________________________________________________

Copyright 2010, Elsevier Inc


4-3
Lai et al, Introduction to Continuum Mechanics

4.8 Do the previous problem if the stress distribution is given by T11 = x22 and all other
Ti j = 0 .
------------------------------------------------------------------------------
Ans. (a) The normal to the plane x1 = 0 is e1 , thus, t e1 = x22e1 . On the plane, there is a parabolic
distribution of normal stress x22 , (see figure).
(0,-1,-1) (0,-1,1)

x3 x1

(0,1,-1) (0,1,1)
x2

4
( x2 e1 )dx2 dx3 =
1 1
(b) FR = tdA = 2
e1 .
1 1
3

( x2e2 + x3e3 ) ( x2 e1 )dx2 dx3


1 1
M o = x tdA = 2

1 1

( x2 x3e2 - x2e3 )dx2 dx3 = 0e2 + 0e3


1 1
= 2 3

1 1
_________________________________________________________________

4.9 Do problem 4.7 for the stress distribution: T11 = , T12 = T21 = x3 and all other Ti j = 0 .
------------------------------------------------------------------------------
Ans. (a) The normal to the plane x1 = 0 is e1 , thus, t e1 = e1 + x3e 2 . On the plane, there is a
constant normal stress of and a linear distribution of shearing stress x3e2 , (see figure).

(0,-1,-1) (0,-1,1)

x3 x1

(0,1,-1) (0,1,1)
x2

1 1
(b) FR = tdA = ( e1 + x3e2 )dx2 dx3 = 4 e1 .
1 1

Copyright 2010, Elsevier Inc


4-4
Lai et al, Introduction to Continuum Mechanics

1 1
M o = x tdA = ( x2e2 + x3e3 ) ( e1 + x3e2 )dx2 dx3
1 1

4
( x3 e1 + x3e2 - x2e3 )dx2 dx3 =
1 1
= 2
e1
1 1
3
_________________________________________________________________

4.10 Consider the following stress distribution for a circular cylindrical bar:
0 x3 x2
0
[T] = x3 0
x2 0 0
(a) What is the distribution of the stress vector on the surfaces defined by (i) the lateral
surface x22 + x32 = 4 , (ii) the end face x1 = 0 , and (iii) the end face x1 = l ? (b) Find the total
resultant force and moment on the end face x1 = l .
------------------------------------------------------------------------------

Ans. (a) The outward unit normal vector to the lateral surface x22 + x32 = 4 is given by
x e + x3e3
n1 = 2 2 . The outward unit normal vector to x1 = 0 is n 2 = e1 and that to x1 = l is
2
n3 = e1 . Thus,
0 x3 x2 0 0
0 x2 = 0 t n1 = 0 .
1
t n = x3 0
1 2
x2 0 0 x3 0
t n 2 = Te1 = ( x3e 2 + x2e3 ) = x3e2 x2e3
t n 3 = Te1 = ( x3e 2 + x2e3 ) = x3e2 + x2e3
(b) On the end face x1 = l , t n 3 = Te1 = x3e 2 + x2e3
FR = t n3 dA = ( x3e 2 + x2e3 ) dA = ( e2 ) x3dA + ( e3 ) x2 dA = 0 .
[note: the axes are axes of symmetry, the integrals are clearly zero].
M o = x tdA = ( x2e 2 + x3e3 ) ( x3e 2 + x2e3 ) dA

( )
2 2
= x22 + x32 dAe1 = r 2 2 rdre1 = 2 r 3 dre1 = 8 e1
0 0
_________________________________________________________________

4.11 An elliptical bar with lateral surface defined by x22 + 2 x32 = 1 has the following stress
0 2 x3 x2

distribution: [ T ] = 2 x3 0 0 . (a) Show that the stress vector at any point ( x1 , x2 , x3 ) on
x2 0 0
the lateral surface is zero. (b) Find the resultant force, and resultant moment, about the origin O ,
of the stress vector on the left end face x1 = 0 .

Copyright 2010, Elsevier Inc


4-5
Lai et al, Introduction to Continuum Mechanics


x2 dA = 4 and x32 dA =
2
Note: .
2 8 2
------------------------------------------------------------------------------
Ans. (a) The outward unit normal vector to the lateral surface x22 + 2 x32 = 1 is given by:
0 2 x3 x2 0 0
x2e 2 + 2 x3e3 = 0
1
n1 = , thus, [ t ] = [ T][n1 ] = 2 x3 0 0 x2
x2 + 4 x3 0 2 x3 x2 + 4 x3 0
2 2 2 2
x2 0
(b) On the left end face x1 = 0 , n = e1 , the stress vector is t = 2x3e 2 x2e3 ,
FR = tdA = ( 2 x3e 2 x2e3 )dA = ( 2e 2 ) x3dA ( e3 ) x2 dA = 0 .
[note: the axes are axes of symmetry, the integrals are clearly zero]]
M o = x tdA = ( x2e2 + x3e3 ) ( 2 x3e2 x2e3 )dA
2
( )
= x22 + 2 x32 e1dA = + e1 =
4 2 8 2 2 2
e1.

_________________________________________________________________

4.12 For any stress state T , we define the deviatoric stress S to be S = T (Tkk / 3) I , where
Tkk is the first invariant of the stress tensor T . (a) Show that the first invariant of the deviatoric
6 5 2
stress vanishes. (b) Given the stress tensor [ T ] = 100 5 3 4 kPa. , evaluate S . (c) Show
2 4 9
that the principal directions of the stress tensor coincide with those of the deviatoric stress tensor.
------------------------------------------------------------------------------
Ans. (a) From S = T (Tkk / 3) I , we have, trS = trT (Tkk / 3) trI = Tkk (Tkk / 3) ( 3) = 0 .
6 5 2 0 500 200
(b) [S ] = 100 5 3 4 (1800 / 3) [ I ] = 500 300 400 kPa.

2 4 9 200 400 300
(c) Let n be an eigenvector of T , then Tn = n . Now Sn = Tn (Tkk / 3) In = n (Tkk / 3) n ,
that is Sn = n where = (Tkk / 3) . Thus, n is also an eigenvector of S with eigenvalue
(Tkk / 3) .
_________________________________________________________________

4.13 An octahedral stress plane is one whose normal makes equal angles with each of the
principal axes of stress. (a) How many independent octahedral planes are there at each point? (b)
Show that the normal stress on an octahedral plane is given by one-third the first stress invariant.
(c) Show that the shearing stress on the octahedral plane is given by
1 2 1/2
Ts = (T1 T2 ) + (T2 T3 ) + (T3 T1 ) , where T1 , T2 , T3 are principal values of the stress
2 2
3
tensor.
------------------------------------------------------------------------------
Ans. (a) There are four independent octahedral planes. They are given by the following unit
normal vectors:

Copyright 2010, Elsevier Inc


4-6
Lai et al, Introduction to Continuum Mechanics

e1 + e2 + e3 e1 + e2 e3 e1 e2 + e3 e1 e2 e3
n1 = , n2 = , n3 = ,n 4 =
3 3 3 3
e1 + e2 + e3
We note that gives the same plane as n1 , etc.
3
(b) Using the principal directions as the orthonormal basis, the matrix of T is diagonal, i.e.,
T1 0 0
e e e
[ T] = 0 T2 0 . The normal to an octahedral plane is 1 2 3 , thus,
3
0 0 T3
T1 0 0 1
Tn = n Tn = [1 1 1] 0 T2 0 1 where in this equation, the row matrix and column

1
3
0 0 T3 1
matrix of n have the same elements, that is if the row matrix is [1 1 1] then the column matrix
1 T1 0 0 1
is 1 . Thus, Tn = [1 1 1] 0 T2 0 1 = (T1 + T2 + T3 ) .
1 1
3 3
1 0 0 T3 1
2 1
( 1
) (
(c) Ts2 = t n Tn2 = T12 + T22 + T33 T12 + T22 + T33 + 2T1T2 + 2T1T3 + 2T2T3
3 9
)
9
(
2 2 2 3 1
9 )
= T1 + T2 + T3 T1T2 T1T3 T2T3 = (T1 T2 ) + (T2 T3 ) + (T3 T1 )
2 2 2

1 2 1/2
That is, Ts = (T1 T2 ) + (T2 T3 ) + (T3 T1 )
2 2
3
_________________________________________________________________

4.14 (a) Let m and n be two unit vectors that define two planes M and N that pass through a
point P . For an arbitrary state of stress defined at the point P , show that the component of the
stress vector t m in the n -direction is equal to the component of the stress vector t n in the
m direction. (b) If m = e1and n = e2 , what does the results of (a) reduce to?
------------------------------------------------------------------------------
Ans. (a) The component of the stress vector t m in the n -direction is n t m = n Tm and the
component of the stress vector t n in the m direction is m t n = m Tn = n TT m . Since T is
symmetric, therefore, n TT m = n Tm , therefore, n t m = m t n .
(b) If m = e1 and n = e2 , then e1 t e2 = e2 t e1 T12 = T21 .
_________________________________________________________________

4.15 Let m be a unit vector that defines a plane M passing through a point P . Show that the
stress vector on any plane that contains the stress traction t m , lies in the M plane.
------------------------------------------------------------------------------
Ans. Referring to the figure below, where m is perpendicular to the plane M , and t m is the stress
vector for the plane. Let N be any plane which contains the vector t m and let n be the unit vector
perpendicular to the plane N . Then t n = Tn . We wish to show that t n is perpendicular to m .

Copyright 2010, Elsevier Inc


4-7
Lai et al, Introduction to Continuum Mechanics

Now, t n m = Tn m = n TT m = n Tm = n t m = 0, because t m is on the N plane.


Thus, t n m = 0, so that t n lies on the M plane.
_________________________________________________________________

4.16 Let t m and t n be stress vectors on planes defined by the unit vector m and n respectively
and pass through the point P . Show that if k is a unit vector that determines a plane that contains
t m and t n , then t k is perpendicular to m and n .
------------------------------------------------------------------------------
Ans. Since k is a unit vector that determines a plane that contains t m and t n , therefore,
t t
k = m n . Since t k = Tk , t m = Tm, and t n = Tn , therefore,
tm tn
tm tm tn
m t k = m Tk = k TT m = k Tm = k t m = = 0 , similarly,
tm tn
tn tm tn
n t k = n Tk = k TT n = k Tn = k t n =
=0.
tm tn
_________________________________________________________________

4.17 Given the function f ( x, y ) = 4 x 2 y 2 , find the maximum value of f subjected to the
constraint that x + y = 2 .
------------------------------------------------------------------------------
Ans. Let g ( x, y ) = 4 x 2 y 2 + ( x + y 2) , then we have the following three equations to solve
for x, y and :
g g
= 2 x + = 0 , = 2 y + = 0 and x + y = 2 .
x y
Thus, 2 x + = 0 = 2 x, 2 y + = 0 = 2 y , therefore, x = y
x + y = 2 2 x = 2 x = 1 = y . That is, f max occurs at x = y = 1 . That is,
f max = 4 (1) 2 (1) 2 = 2
_________________________________________________________________

4.18 True or false:


(i) Symmetry of stress tensor is not valid if the body has an angular acceleration.
(ii) On the plane of maximum normal stress, the shearing stress is always zero.
------------------------------------------------------------------------------
Ans. (i) False. (ii) True.
_________________________________________________________________

Copyright 2010, Elsevier Inc


4-8
Lai et al, Introduction to Continuum Mechanics

4.19 True or false:


(i) On the plane of maximum shearing stress, the normal stress is always zero.
(ii) A plane with its normal in the direction of e1 + 2e2 2e3 has a stress vector
t = 50e1 + 100e2 100e3 MPa. It is a principal plane.
------------------------------------------------------------------------------
Ans. (i) Not true in general. Maybe true in some special cases.
(ii) True. We note that t = 50e1 + 100e2 100e3 = 50 ( e1 + 2e2 2e3 ) . Therefore, t is normal to
the plane, so that there is no shearing stress on the plane. That is, it is a principal plane.
_________________________________________________________________

4.20 Why can the following two matrices not represent the same stress tensor?
100 200 40 40 100 60
200 0 0 MPa., 100 100 0 MPa. .


40 0 50 60 0 20
------------------------------------------------------------------------------
Ans. The first scalar invariant for the first matrix is 50 MPa. The first scalar invariant for the
second matrix is 160 MPa. They are not the same, therefore, they can not represent the same
stress tensor.
_________________________________________________________________

0 100 0
4.21 Given [ T] = 100 0 0 MPa. (a) Find the magnitude of shearing stress on the plane
0 0 0
whose normal is in the direction of e1 + e2 . (b) Find the maximum and minimum normal stresses
and the planes on which they act. (c) Find the maximum shearing stress and the plane on which it
acts.
------------------------------------------------------------------------------
0 100 0 1 1
1 1 100
Ans. (a) Let n = ( e1 + e2 ) . Then tn = 100 0 0 1 = 1 i.e.,
2 2 2
0 0 0 0 0
t n = 100n shearing stress Ts = 0 .
0 100 0
(b) The characteristic equation is 100 0 ( )
0 = 0 2 1002 = 0
0 0
1 = 100 MPa., 2 = 100MPa., 3 = 0 . The maximum normal stress is 100 MPa. and the
minimum normal stress is 100MPa .
For 1 = 100MPa., 1001 + 100 2 = 0, so that 1 = 2 , n1 = (e1 + e2 ) / 2 .
For 2 = 100MPa., 1001 + 100 2 = 0 1 = 2 , n 2 = (e1 e2 ) / 2 .
(Tn )max (Tn )min 100 ( 100 )
(c) (Ts )max = = = 100 MPa. The maximum shearing stress acts on
2 2
the planes n = (n1 n 2 ) / 2 , i.e., on the planes e1 and e2 .
_________________________________________________________________

Copyright 2010, Elsevier Inc


4-9
Lai et al, Introduction to Continuum Mechanics

4.22 Show the equation for the normal stress on the plane of maximum shearing stress is
(Tn )max + (Tn )min
Tn = .
2
------------------------------------------------------------------------------
Ans. Let {n1 , n 2 , n3 } be the principal axes of the stress tensor with principal values T1 > T2 > T3 ,
T1 0 0
then [ T] = 0 T2 0 . On the plane n = ( n1 n3 ) / 2 , the shearing stress is a maximum. On
0 0 T3
this plane, the normal stress is:
T1 0 0 1
T +T (Tn )max + (Tn )min
Tn = n Tn Tn = [1 0 1] 0 T2 0 0 = 1 3 =

1
2 2 2
0 0 T3 1
_________________________________________________________________

4.23 The stress components at a point are given by: T11 = 100MPa., T22 = 300MPa.,
T33 = 400MPa . T12 = T13 = T23 = 0 . (a) Find the maximum shearing stress and the planes on
which they act. (b) Find the normal stress on these planes. (c) Are there any plane/planes on
which the normal stress is 500 MPa. ?
------------------------------------------------------------------------------
Ans. (a) The maximum normal stress is clearly T33 = 400MPa ., acting on the e3 plane and the
minimum normal stress is clearly T11 = 100 MPa. , acting on the e1 plane. Thus, the maximum
400 100 1
shearing stress is (Ts )max = = 150 MPa. , acting on the plane n = ( e1 e3 ) .
2 2
100 0 0 1 100
(b) Tn = [1 0 1] 0 300 0 0 = [1 0 1] 0 = 250 MPa.

1 1
2 2
0 0 400 1 400
Tmax + Tmin 400 + 100
Note: We can also use the result of Prob. 4.22 to obtain Tn = = = 250 MPa.
2 2
(c) No, because Tmax = 400MPa
_________________________________________________________________

4.24 The principal values of a stress tensor T are T1 = 10MPa., T2 = 10 MPa. and
T11 0 0
T3 = 30MPa. If the matrix of the stress is given by: [ T] = 0 1 2 10 MPa. , find the
0 2 T33
values of T11 and T33 .
------------------------------------------------------------------------------
Ans.
I1 = 10 10 + 30 = 10(T11 + 1 + T33 ) 2 = (T11 + T33 ) T11 =2 T33 (i)
I 3 = (10 )( 10 )( 30 ) = (10 ) (T11T33 4T11 ) 3 = (T11T33 4T11 ) (ii)
3

(i) and (ii) 3 = ( 2 T33 )(T33 4 ) T332 6T33 + 5 = 0. Thus,

Copyright 2010, Elsevier Inc


4-10
Lai et al, Introduction to Continuum Mechanics

T33 = [6 36 20] / 2 = 3 2. Thus, T33 is either 5 or 1.


To determine which is the correct value, we check
I 2 = ( 10 )( 30 ) + (10 )( 10 ) + (10 )( 30 ) = 102 (T33 4 ) + T11T33 + T11
T33 + T11T33 + T11 3 = 0 . (iii)
Try T33 = 1 first, from (i), T11 = 1 , so that (iii) is clearly satisfied. Next try T33 = 5 , eq (i)
gives T11 = 2 5 = 3 , then left side of (iii) becomes 5 + (3)(5) 3 3 = 16 0 .
Thus, T33 = 1 and T11 = 1 .
_________________________________________________________________

300 0 0
4.25 If the state of stress at a point is: [ T] = 0 200 0 kPa. , find (a) the magnitude of

0 0 400
the shearing stress on the plane whose normal is in the direction of ( 2e1 + 2e2 + e3 ) and (b) the
maximum shearing stress.
------------------------------------------------------------------------------
1
Ans. (a) Let n = ( 2e1 + 2e2 + e3 ) , then
3
300 0 0 2 600
[t n ] = 3 0 200 0 2 = 3 400 t n = 3 ( 6e1 4e2 + 4e3 )
1 1 100

0 0 400 1 400
100 800
Tn = n t n = (12 8 + 4 ) = = 88.89kPa.
9 9
2 104 68 104 64
Ts2 = t n Tn2 = = 104 6.76 Ts = 260 kPa.
9 81
400 ( 200 )
(b) (Ts )max = = 300 kPa.
2
_________________________________________________________________

1 4 0
4.26 Given [ T] = 4 1 0 MPa. (a) Find the stress vector on the plane whose normal is in
0 0 1
the direction of e1 + e2 . (b) Find the normal stress on the same plane. (c) Find the magnitude of
the shearing stress on the same plane. (d) Find the maximum shearing stress and the planes on
which this maximum shearing stress acts.
------------------------------------------------------------------------------
1 4 0 1 5
1 1 1 5
Ans. (a) Let n = ( e1 + e2 ) , then [t n ] = 4 1 0 1 = 5 tn = ( e1 + e2 ) .
2 2 2 2
0 0 1 0 0
1
(b) Tn = n t n = ( 5 + 5) = 5 MPa. (c) Ts2 = tn 2 Tn2 = 25 25 = 0
2

Copyright 2010, Elsevier Inc


4-11
Lai et al, Introduction to Continuum Mechanics

(d) The characteristic equation is (1 ) (1 ) 42 = 0 1 = 5, 2 = 3, 3 = 1 . Thus,


2

(Tn )max = 5 MPa. and (Tn )min = 3 MPa.
( )
For (Tn )max = 5 MPa. (1 5 ) 1 + 4 2 = 0 1 = 2 n1 = 1 / 2 ( e1 + e 2 ) .

(
For (Tn )min = 3 MPa. (1+3) 1 + 4 2 = 0 1 = 2 n 2 = 1 / 2 ( e1 e 2 ) .)
5 ( 3)
Thus, (Ts )max = = 4 MPa. , acting on the plane whose normal is
2
( )
n = 1 / 2 ( n1 n 2 ) n = e1 and n = e 2 .
_________________________________________________________________

4.27 The stress state in which the only non-vanishing stress components are a single pair of
shearing stresses is called simple shear. Take T12 = T21 = and all other Ti j = 0 . (a) Find the
principal values and principal directions of this stress state. (b) Find the maximum shearing stress
and planes on which it acts.
------------------------------------------------------------------------------
0 0
Ans. (a) With [ T ] = 0 0 , the characteristic equation is
0 0 0

( )
2 2 = 0 1 = , 2 = , 3 = 0.

( )
For 1 = , ( 0 )1 + 2 = 0 1 = 2 n1 = 1 / 2 ( e1 + e 2 ) .

(
For 2 = ( 0+ )1 + 2 = 0 1 = 2 n 2 = 1 / 2 ( e1 e 2 ) .)
For 3 = 0 n3 = e3 .
( )
(b) (Ts )max = = , acting on the plane whose normal is
2
( )
n = 1 / 2 ( n1 n 2 ) n = e1 and n = e 2 .
_________________________________________________________________

4.28 The stress state in which only the three normal stress components do not vanish is called a
tri-axial state of stress. Take T11 = 1 , T22 = 2 , T33 = 3 with 1 > 2 > 3 and all other Ti j = 0 .
Find the maximum shearing stress and the plane on which it acts.
------------------------------------------------------------------------------
3 1
Ans. (Ts )max = 1 , n= ( e1 e3 ) .
2 2
_________________________________________________________________

4.29 Show that the symmetry of the stress tensor is not valid if there are body moments per unit
volume, as in the case of a polarized anisotropic dielectric solid.
------------------------------------------------------------------------------

Copyright 2010, Elsevier Inc


4-12
Lai et al, Introduction to Continuum Mechanics

Ans. Let M* = M1*e1 + M 2*e 2 + M 3*e3 be the body moments per unit volume. Then referring to the
figure shown below, the total moments of all the surface forces and the body force and body
moment about the axis which passes through the center point A and parallel to the x3 axis is :
( M c )3 = T21 ( x2 x3 ) ( x1 / 2 ) + (T21 + T21 ) ( x2 x3 ) ( x1 / 2 )
T12 ( x1x3 ) ( x2 / 2 ) (T12 + T12 ) ( x1x3 ) ( x2 / 2 ) + M 3* ( x1x2 x3 )

= (1 / 12 ) (density) ( x1x2 x3 ) ( x1 ) + ( x2 ) 3
2 2

where 3 is the x3 components of the angular acceleration of the element. We now let
x1 0, x2 0, x3 0 and drop all terms of small quantities of higher order than
( x1x2 x3 ) , we obtain,
T21 ( x1x2 x3 ) T12 ( x1x2 x3 ) + M 3* ( x1x2 x3 ) = 0 T12 T21 = M 3* ,
Similarly, one can show that T13 T31 = M 2* and T23 T32 = M1* .

_________________________________________________________________

x1 + x2 T12 ( x1 , x2 ) 0

4.30 Given the following stress distribution: [ T ] = T12 ( x1 , x2 ) x1 2 x2 0 , find T12 so
0 0 x2

that the stress distribution is in equilibrium with zero body force and so that the stress vector on
the plane x1 = 1 is given by t = (1 + x2 ) e1 + ( 5 x2 ) e 2 .
------------------------------------------------------------------------------
Ti j
Ans. The equations of equilibrium are + Bi = 0 . Now with Bi = 0 , we have,
x j
T11 T12 T13 T
+ + = 1 + 12 = 0 T12 = x2 + f ( x1 ) .
x1 x2 x3 x2
T21 T22 T23 T12
+ + = 2 = 0 T12 = 2 x1 + g ( x2 ) , thus, T12 = 2 x1 x2 + C .
x1 x2 x3 x1
T31 T32 T33
+ + = 0 0 = 0.
x1 x2 x3
To determine C, we have, the stress vector on the plane x1 = 1 is
t = Te1 = T11e1 + T21e 2 + T31e3 = ( x1 + x2 ) e1 + ( 2 x1 x2 + C ) e2 . Thus,
x1 =1

(1 + x2 ) e1 + ( 2 x2 + C ) e2 = (1 + x2 ) e1 + ( 5 x2 ) e2 C = 3 T12 = 2 x1 x2 + 3 .
_________________________________________________________________

Copyright 2010, Elsevier Inc


4-13
Lai et al, Introduction to Continuum Mechanics

x2 x3 0
4.31 Consider the following stress tensor: [ T] = x3 0 x2 . Find an expression
0 x2 T33
for T33 such that the stress tensor satisfies the equations of equilibrium in the presence of body
force vector B = ge3 , where g is a constant.
------------------------------------------------------------------------------
Ti j
Ans. The equations of equilibrium are + Bi = 0 . With B1 = B2 = 0, B3 = g , we have,
x j
T11 T12 T13 T T T
+ + + B1 = 0 + 0 + 0 + 0 = 0, 21 + 22 + 23 + B2 = 0 + 0 + 0 + 0 = 0,
x1 x2 x3 x1 x2 x3
T31 T32 T33 T T g
+ + + B3 = 1 + 33 g = 0 33 = +1
x1 x2 x3 x3 x 3
g
T33 = + 1 x3 + f ( x1 , x2 )

_________________________________________________________________

4.32 In the absence of body forces, the equilibrium stress distribution for a certain body is
T11 = Ax2 , T12 = T21 = x1 , T22 = Bx1 + Cx2 , T33 = (T11 + T22 ) / 2, all other Ti j = 0 . Also, the
boundary plane x1 x2 = 0 for the body is free of stress.
(a) Find the value of C and (b) determine the value of A and B .
------------------------------------------------------------------------------
Ti j
Ans. (a) The equations of equilibrium are + Bi = 0 . With Bi = 0, , we have,
x j
T11 T12 T13
+ + + B1 = 0 + 0 + 0 + 0 = 0,
x1 x2 x3
T21 T22 T23
+ + + B2 = 1 + C + 0 + 0 = 0 C = 1 ,
x1 x2 x3
T31 T32 T33
+ + + B3 = 0 + 0 + 0 + 0 = 0 .
x1 x2 x3
(b) The unit normal to the boundary plane x1 x2 = 0 is n = ( e1 e 2 ) / 2 . Thus, on this plane
(note x1 = x2 ), we have,
Ax1 x1 0 1 Ax1 x1 0
1 1
[t ] = x1 Bx1 x1 0 1 = x1 Bx1 + x1 = 0 , thus,
2 2
0 0 (T11 + T22 ) / 2 0 0 0
Ax1 x1 = 0 A = 1 and x1 Bx1 + x1 = 0 B = 2 .
_________________________________________________________________

4.33 In the absence of body forces, do the following stress components satisfy the equations of
equilibrium:

Copyright 2010, Elsevier Inc


4-14
Lai et al, Introduction to Continuum Mechanics

( ) ( ) (
T11 = x22 + x12 x22 , T22 = x12 + x22 x12 , T33 = x12 + x22 ,
)
T12 = T21 = 2 x1 x2 , T13 = T31 = 0, T23 = T32 = 0.
------------------------------------------------------------------------------
Ti j
Ans. The equations of equilibrium are + Bi = 0 . With Bi = 0, , we have,
x j
T11 T12 T13
+ + + B1 = 2 x1 2 x1 + 0 + 0 = 0,
x1 x2 x3
T21 T22 T23
+ + + B2 = 2 x2 + 2 x2 + 0 + 0 = 0
x1 x2 x3
T31 T32 T33
+ + + B3 = 0 + 0 + 0 + 0 = 0 . Yes, the equations of equilibrium are all satisfied.
x1 x2 x3
_________________________________________________________________

4.34 Repeat the previous problem for the stress distribution


x1 + x2 2 x1 x2 0
[ T] = 2 x1 x2 x1 3x2 0
0 0 x1
------------------------------------------------------------------------------
Ti j
Ans. The equations of equilibrium are + Bi = 0 . With Bi = 0, , we have,
x j
T11 T12 T13
+ + + B1 = 0 (1 1 + 0 ) = 0 0 = 0,
x1 x2 x3
T21 T22 T23
+ + + B2 = (2 3 + 0) 0
x1 x2 x3
No, the second equation of equilibrium is not satisfied.
_________________________________________________________________

4.35 Suppose that the stress distribution has the form (called a plane stress state)
T11 ( x1 , x2 ) T12 ( x1 , x2 ) 0
[T] = T12 ( x1 , x2 ) T22 ( x1 , x2 ) 0
0 0 0

(a) If the state of stress is in equilibrium, can the body forces be dependent on x3 ? (b) If we
2 2 2
introduce a function ( x1 , x2 ) such that T11 = , T22 = and T12 = , What should
x22 x12 x1x2
be the function ( x1 , x2 ) for the equilibrium equations to be satisfied in the absence of body
forces?
------------------------------------------------------------------------------
T T T T ( x , x ) T ( x , x )
Ans. (a) 11 + 12 + 13 + B1 = 11 1 2 + 12 1 2 + B1 = 0 .
x1 x2 x3 x1 x2

Copyright 2010, Elsevier Inc


4-15
Lai et al, Introduction to Continuum Mechanics

T21 T22 T23 T ( x , x ) T ( x , x )


+ + + B2 = 21 1 2 + 22 1 2 + B2 = 0 .
x1 x2 x3 x1 x2
Thus, B1 and B2 must be independent of x3 .
T11 T12 T13 2 2 2 2
(b) + + + B1 + 0 + 0 = 2 2 = 0 .
x1 x2 x3 x1 x22 x2 x1x2 x1
2
x x1 x2
T21 T22 T23 2 2 2 2
+ + + B2 = + 2 + 0 + 0 = + = 0.
x1 x2 x3 x1 x1x2 x2 x1 x2 x12 x2 x12
Thus, the equations of equilibrium are satisfied for any function ( x1 , x2 ) which is continuous up
to the third derivatives.
_________________________________________________________________

4.36 In cylindrical coordinates ( r , , z ) , consider a differential volume of material bounded by


the three pairs of faces : r = r and r = r + dr ; = and = + d ; z = z and z = z + dz. Derive the
r and equations of motion in cylindrical coordinates and compare the equations with those
given in Section 4.8.
------------------------------------------------------------------------------
Ans.

From the free body diagram above, we have,


Fr = Trr ( rd ) dz + (Trr + dTrr ) ( ( r + dr ) d ) dz Tr drdz cos ( d / 2 )
+ (Tr + dTr ) drdz cos ( d / 2 ) T drdz sin ( d / 2 ) (T + dT ) drdz sin ( d / 2 )
Trz ( rd ) dr + (Trz + dTrz )( rd ) dr + Br ( rd ) drdz = ( rd ) drdz ar .
2 3
d 1 d d d 1 d
Now, cos =1+ + ...and sin = + ... and keeping only terms
2 2! 2 2 2 3! 2
involving products of three differentials (i.e., terms involving product of 4 or more differentials
drop out in the limit when these differentials approach zero), we have,
Trr drd dz + ( dTrr ) rd dz + dTr drdz 2T drdz ( d / 2 )
+ ( dTrz )( rd ) dr + Br ( rd ) drdz = ( rd ) drdz ar
Dividing the equation by rd drdz , we get,
Trr Trr Tr T Trz
+ + + + Br = ar . This is Eq. (4.8.1)
r r rd r z
Next,

Copyright 2010, Elsevier Inc


4-16
Lai et al, Introduction to Continuum Mechanics

F = T r ( rd ) dz + (T r + dT r ) ( ( r + dr ) d ) dz + Tr drdz sin ( d / 2 )
+ (Tr + dTr ) drdz sin ( d / 2 ) T drdz cos ( d / 2 ) + (T + dT ) drdz cos ( d / 2 )
T z ( rd ) dr + (T z + dT z ) ( rd ) dr + B ( rd ) drdz = ( rd ) drdz a .
2 3
d 1 d d d 1 d
Again, cos =1+ + ...and sin = + ... and keeping only terms
2 2! 2 2 2 3! 2
involving products of three differentials, we have,
(T r ) drd dz + ( dT r ) rd dz + 2Tr drdz ( d / 2 ) + ( dT ) drdz + ( dT z ) rd dr
+ B ( rd ) drdz = ( rd ) drdz a
Dividing the equation by rd drdz , we get,
T r T r Tr 1 T T z
+ + + + + B = a , this is Eq. (4.8.2).
r r r r z
_________________________________________________________________

4.37 Verify that the following stress field satisfies the z-equation of equilibrium in the absence
of body forces:
z 3r 2 z Az z 3z3 r 3rz 2
Trr = A 3 5 , T = 3 , Tzz = A 3 + 5 , Trz = A 3 + 5 , Tr = Tz = 0
R R R R R
R R
R2 = r 2 + z 2
------------------------------------------------------------------------------
Ans. The z equation of equilibrium in cylindrical coordinate is:
Tzr 1 Tz Tzr Tzz R r R z
+ + + + Bz = 0 . Now = , = so that
r r r z r R z R
Tzr r 3rz 2 1 3r R 3z 2 15rz 2 R
= A + = A + 6
r r R3 r R5 R3 R 4 r R5 R r

1 3r 2 3 z 2 15r 2 z 2 1 3r 2 3z 2 15r 2 z 2 Tz
= A 3 5 + 5 = A + 5 , =0
R 7 R3 R5 R 7
R R R R
Trz 1 3z 2
= A 3 + 5 ,
r R R

Tzz 1 3z R 9 z 2 15 z 3 R 1 3z 2 9 z 2 15 z 4
= A 3 4 + 5 6 = A 3 5 + 5 7
z R R z R R z R R
R R
Tzr T T 3 3r 2 3 z 2 3z 2 3z 2 9 z 2 15r 2 z 2 15 z 4
Thus, + rz + zz = A 3 5 5 + 5 + 5 + 5 7
r r z R R7 R
R R R R R
3 3r 2 3 z 2 3z 2 3z 2 9 z 2 15r 2 z 2 15 z 4
= A 3 5 5 + 5 + 5 + 5 7
R R7 R
R R R R R


= A 3
(
3 3 r +z
2 2
) +
(
15 z 2 15 z r + z
2 2 2
) = A
3 3R 2 15 z 2 15 z 2 R 2
+ 5 =0.
R R5 R5 R7 R3 R5 R 7
R
_________________________________________________________________

Copyright 2010, Elsevier Inc


4-17
Lai et al, Introduction to Continuum Mechanics

4.38 Given the following stress field in cylindrical coordinates:


3Pzr 2 3Pz 3 3Pz 2 r
Trr = , Tzz = , Trz = , T = Tr = Tz = 0, R 2 = r 2 + z 2
2 R 5
2 R 5
2 R 5

Verify that the state of stress satisfies the equations of equilibrium in the absence of body forces.
------------------------------------------------------------------------------
Ans.
Trr Tr Trr T Trz 3Pzr 2 1 3Pzr 2 3Pz 2 r
+ + + = + +
r rd r z r 2 R 5 r 2 R5 z 2 R5
3Pz 2 3Pzr 2 1 1 3Pzr 2 3Pr 2 3Pz 2 r 1
=
2 R5 r
( )
r +
2
+ +
r R 5 r 2 R5 2 R5 z
z +

2 z R5

3Pzr 15 Pzr 2 R 3Pzr 3P zr 15 Pz 2 r R
= + + +
R5 2 R 6 r 2 R5 R5 2 R 6 z

=
15 Pzr 15Pzr 3 15Pz 3 r
+ + = +

15 Pzr 15 Pzr r + z
2 2
( =0.
)
2 R 5 2 R 7 2 R 7 2 R 5 2 R 7

T r 2Tr 1 T T z
+ + + =0+0+0+0=0
r r r z
Tzr 1 Tz Tzr Tzz 3Pz 2 r 1 3Pz 2 r 3Pz 3
+ + + + Bz = + +
r r r z r 2 R5 r 2 R5 z 2 R 5
3Pz 2 3Pz 2 r 1 3Pz 2 9 Pz 2 3Pz 3 1
= +
2 R5
2 r R5 2 R5 2 R5 2 z R 5


3Pz 2 15Pz 2 r R 3Pz 2 9 Pz 2 15Pz 3 R
= + +
2 R5 2 R 6 r 2 R5 2 R5 2 R 6 z

3Pz 2 15Pz 2 r 2 3Pz 2 9 Pz 2 15 Pz 4 15 Pz 2 2r + z
2 2
= + + = + =0
2 R 7
15 Pz
2 R5 2 R 7 2 R5 2 R 5 2 R 7 2 R5

_________________________________________________________________

4.39 For the stress field given in Example 4.9.1, determine the constants A and B if the inner
cylindrical wall is subjected to a uniform pressure pi and the outer cylindrical wall is subjected to
a uniform pressure po .
------------------------------------------------------------------------------
Ans. The given stress field is:
B B
Trr = A + 2 , T = A 2 , Tzz = constant and Tr = Trz = T z = 0 .
r r
On the outer wall, r = ro , and Trr = po , and on the inner wall, r = ri , and Trr = pi , therefore,
B B
we have, po = A + 2 (i) and pi = A + 2 (ii).
ro ri

po pi =
B

B

( p pi ) ri2 ro2 ,
B= o A=
(pr i i
2
po ro2 ).
ri2 ro2 (ro
2
ri2 ) (r o
2
ri2 )

Copyright 2010, Elsevier Inc


4-18
Lai et al, Introduction to Continuum Mechanics

_________________________________________________________________

4.40 Verify that Eq. (4.8.4) to (4.8.6) are satisfied by the stress field given in Example 4.9.2 in
the absence of body forces.
------------------------------------------------------------------------------
2B B
Ans. The given stress field is: Trr = A 3 , T = T = A + 3 , Tr = Tr = T = 0 .
r r
2
(
1 r Trr
+
)1 (Tr sin )
+
1 Tr T + T
-
r 2 r r sin r sin r
1 2 2B 2 B 2 B 2 B
= 2 r
r A + 0 + 0 A + 3 = A + 3 + 0 + 0 A + 3 = 0.
r r r r r r r r

(
1 r T r
3
+
) 1 (T sin )
+
1 T Tr T r T cot
+
r 3 r r sin r sin r
T cot T cot
= 0 + 0 + 0+ = 0.
r r
(
1 r T r
3
+
) 1 T sin
+
( )
1 T Tr T r + T cot
+ = 0 + 0 + 0+ 0 = 0 .
r3 r r sin r sin r
_________________________________________________________________

4.41 In Example 4.9.2, if the spherical shell is subjected to an inner pressure pi and an outer
pressure po , determine the constant A and B .
------------------------------------------------------------------------------
2B 2B 2B
Ans. From the example, we have, Trr = A 3 , thus, po = A 3 and pi = A 3
r ro ri

A=
po ro3 pi ri3
and B =
( po pi ) ro3ri3 .
(r 3
o ri3 ) (
2 ro3 ri3 )
_________________________________________________________________

4.42 The equilibrium configuration of a body is described by:


1 1
x1 = 16 X1 , x2 = X 2 , x3 = X 3 . If the Cauchy stress tensor is given by:
4 4
T11 = 1000 MPa.,and all other Ti j = 0 , (a) calculate the first Piola Kirchhoff stress tensor and the
corresponding pseudo stress vector for the plane whose undeformed plane is e1 plane and (b)
calculate the second Piola-Kirchhoff tensor and the corresponding pseudo stress vector for the
same plane.
------------------------------------------------------------------------------
1 1
Ans. From x1 = 16 X1 , x2 = X 2 , x3 = X 3 , we obtain the deformation gradient F and its
4 4
inverse as:

Copyright 2010, Elsevier Inc


4-19
Lai et al, Introduction to Continuum Mechanics

16 0 0 1 / 16 0 0
[F ] = 0 1 / 4 0 , F = 0 4 0 and detF = 1 .
1

0 0 1 / 4 0 0 4

( )
T
(a) The first Piola-Kirchhoff stress tensor is, from To = ( det F ) T F 1 :
1000 0 0 1 / 16 0 0 1000 / 16 0 0
( )
[To ] = ( det F ) [T] F 1
T
= (1) 0 0 0 0 4 0 = 0 0 0 MPa.

0 0 0 0 0 4 0 0 0
For a unit area in the deformed state in the e1 direction, its undeformed area is
16 0 0 1 16
1 0 0 = 0 dAo n o = 16e1 .
( dAono ) = F n = (1) 0 1 / 4
T
det F
0 0 1 / 4 0 0
That is, its undeformed plane is also e1 plane. The corresponding pseudo stress vector is given by
1000 / 16 0 0 1
t o = To no , where no = e1 . Thus t o = To no [ t o ] = 0 0 0 0 t o = (1000 / 16 ) e1
0 0 0 0
We note that the pseudo stress vector is in the same direction as the Cauchy stress vector and the
intensity of the pseudo stress vector is 1/16 of the Cauchy stress vector simply because the
undeformed area is 16 times the deformed area and both areas have the same normal direction.

( )
T
% = ( det F ) F 1T F 1
(b)The second Piola-Kirchhoff stress tensor is, from T
1 / 16 0 0 1000 0 0 1 / 16 0 0
( )
1 T

T = ( det F ) F [ T] F
% 1
= (1) 0 4 0 0 0 0 0 4 0
0 0 4 0 0 0 0 0 4
1 / 16 0 0 1000 / 16 0 0 1000 / 256 0 0
0 4 0 0 0 0 = 0 0 0 MPa.

0 0 4 0 0 0 0 0 0
% , where n = e . Thus,
The corresponding pseudo stress vector is given t% = Tn o o 1
% t% = (1000 / 256 ) e .
t% = Tn o 1
_________________________________________________________________

4.43 Can the following equations represent a physically acceptable deformation of a body?
Give reason.
1 1
x1 = X1 , x2 = X 3 , x3 = 4 X 2 .
2 2
------------------------------------------------------------------------------
1 / 2 0 0

Ans. [ F ] = 0 0 1 / 2 det F = 1 . The given equations are not acceptable as a
0 4 0
physically acceptable deformation because it gives a negative ratio of deformed volume to the
undeformed volume.

Copyright 2010, Elsevier Inc


4-20
Lai et al, Introduction to Continuum Mechanics

_________________________________________________________________

4.44 The deformation of a body is described by:


x1 = 4 X1 , x2 = (1 / 4 ) X 2 , x3 = (1 / 4 ) X 3 . (a) For a unit cube with sides along the coordinate
axes what is its deformed volume? What is the deformed area of the e1 face of the cube? (b) If
the Cauchy stress tensor is given by: T11 = 100 MPa.,and all other Ti j = 0 , calculate the first
Piola Kirchhoff stress tensor and the corresponding pseudo stress vector for the plane whose
undeformed plane is e1 plane. (c) Calculate the second Piola-Kirchhoff tensor and the
corresponding pseudo stress vector for the plane whose undeformed plane is e1 plane. Also,
calculate the pseudo differential force for the same plane.
------------------------------------------------------------------------------
Ans. From x1 = 4 X1 , x2 = (1 / 4 ) X 2 , x3 = (1 / 4 ) X 3 , we have (a)
4 0 0
[F ] = 0 1 / 4 0 det F = 1 / 4 , thus dV = ( det F ) dVo dV = (1 / 4 ) (1) = 1 / 4 .

0 0 1 / 4
1 / 4 0 0 1 1 / 4
( ) n o = (1)(1 / 4 ) 0 4 0 0 = (1 / 4 ) 0 dA = (1 / 16 ) e1

T
dA = dAo ( det F ) F 1

0 0 4 0 0
That is, the deformed volume is 1/4 of its original volume and the e1 face of unit area deformed
into an area 1/16 of it original area and remain in the same direction. These results are quite
obvious from the geometry of the deformation.

(b) The first PK stress tensor is:

100 0 0 1 / 4 0 0 100 / 16 0 0
( )
[To ] = ( det F ) [T] F 1 1
T
= 4 0 0 0 0 4 0 = 0 0 0 MPa.

0 0 0 0 0 4 0 0 0
The corresponding pseudo stress vector for e1 -plane in the deformed state, whose undeformed
plane is also e1 -plane, is given by t o = To no , where no = e1 , that is t o = (100 / 16 ) e1MPa. The
Cauchy stress vector on the e1 face in the deformed state is t = 100e1MPa. Clearly the Cauchy
stress vector has a larger magnitude because the area in the deformed state is 1/16 of the
undeformed area.

(c) The second PK stress tensor is:


1 / 4 0 0 100 / 16 0 0 100 / 64 0 0
T = F [ To ] = 0 4 0 0
% 1
0 0 = 0 0 0 MPa.

0 0 4 0 0 0 0 0 0
The corresponding pseudo stress vector for the e1 -plane in the deformed state, whose undeformed
% , where n = e . Thus, t% = (100 / 64 ) e MPa. The
plane is also e1 -plane, is given by t% = Tn o o 1 1

Copyright 2010, Elsevier Inc


4-21
Lai et al, Introduction to Continuum Mechanics

pseudo force df% is related to the force df (= t o dAo = 100 / 16e1 for dAo = 1) by the formula
1 / 4 0 0 100 / 16
100
df = F df , Thus, df = F [ df ] = 0 4 0 0 df% =
% -1 % -1
e1
64
0 0 4 0
______________________________________________________________

4.45 The deformation of a body is described by:


x1 = X1 + kX 2 , x2 = X 2 , x3 = X 3 . (a) For a unit cube with sides along the coordinate axes what is
its deformed volume? What is the deformed area of the e1 face of the cube? (b) If the Cauchy
stress tensor is given by: T12 = T21 = 100 MPa.,and all other Ti j = 0 , calculate the first Piola
Kirchhoff stress tensor and the corresponding pseudo stress vector for the plane whose
undeformed plane is e1 plane and compare it with the Cauchy stress vector in the deformed state.
(c) Calculate the second Piola-Kirchhoff tensor and the corresponding pseudo stress vector for the
plane whose undeformed plane is e1 plane. Also, calculate the pseudo differential force for the
same plane.
------------------------------------------------------------------------------
Ans. From x1 = X1 + kX 2 , x2 = X 2 , x3 = X 3 , we have (a)
1 k 0
[ F ] = 0 1 0 det F = 1 , thus dV = ( det F ) dVo dV = dVo = 1 .
0 0 1
1 0 0 1 1
( )n o = (1)(1) k 1 0 0 = k dA = e1 ke 2
T
dA = dAo ( det F ) F 1

0 0 1 0 0
That is, the deformed volume is the same as its original volume and the e1 face of unit area
deformed into an area 1 + k 2 of it original area and whose normal is in the direction of e1 ke 2 .
These results are quite obvious from the geometry of the deformation.

(b) The first PK stress tensor is:

0 100 0 1 0 0 100k 100 0


( )
[To ] = ( det F ) [T] F 1 T
= 100 0 0 k 1 0 = 100 0 0 MPa.

0 0 0 0 0 1 0 0 0
The corresponding pseudo stress vector for the (e1 ke 2 ) plane, whose undeformed plane is the
e1 plane, is given by t o = To no , where no = e1 . Thus, t o = 100 ( ke1 + e 2 ) MPa. The Cauchy
stress vector on the (e1 ke 2 ) face in the deformed configuration is
0 100 0 1
1 100 0 0 k t = 100 ke + e
[t ] = [ T][n ] = 2 ( 1 2)
1+ k 0 0 0 0 1 + k 2

The Cauchy stress vector has a smaller magnitude because the deformed area is 1 + k 2 times the
undeformed area.

Copyright 2010, Elsevier Inc


4-22
Lai et al, Introduction to Continuum Mechanics

(c) The second PK stress tensor is:


1 k 0 100k 100 0 200k 100 0
% = F 1 [ T ] = 0 1 0 100
T 0 0 = 100 0 0 MPa.
o
0 0 1 0 0 0 0 0 0
The corresponding pseudo stress vector for the e1 ke 2 plane, whose undeformed plane is the
e1 plane, is given by t% = Tn % , where n = e . Thus, t% = 100 ( 2ke + e ) MPa. The pseudo force
o o 1 1 2

df% is related to the force df ( = t o dAo = 100 ( ke1 + e 2 ) for dAo = 1) by the formula df% = F -1df ,
1 k 0 k
Thus, df% = F [ df ] = 100 0 1
-1
0 1 df% = 100 ( 2ke1 + e 2 ) .

0 0 1 0
_________________________________________________________________

4.46 The deformation of a body is described by:


x1 = 2 X1 , x2 = 2 X 2 , x3 = 2 X 3 . (a) For a unit cube with sides along the coordinate axes, what is
its deformed volume? What is the deformed area of the e1 face of the cube? (b) If the Cauchy
100 0 0
stress tensor is given by: 0 100 0 Mpa. , calculate the first Piola Kirchhoff stress tensor

0 0 100
and the corresponding pseudo stress vector for the plane whose undeformed plane is the e1 plane
and compare it with the Cauchy stress vector on its deformed plane, (c) calculate the second
Piola-Kirchhoff tensor and the corresponding pseudo stress vector for the plane whose
undeformed plane is the e1 plane. Also, calculate the pseudo differential force for the same plane.
------------------------------------------------------------------------------
Ans. From, x1 = 2 X1 , x2 = 2 X 2 , x3 = 2 X 3 we have (a)
2 0 0
[F ] = 0 2 0 det F = 8 , thus dV = ( det F ) dVo dV = 8dVo = 8 .
0 0 2
1 / 2 0 0 1 1 / 2
( ) n o = (1)( 8 ) 0 1 / 2 0 0 = ( 8 ) 0 dA = 4e1

T
dA = dAo ( det F ) F 1

0 0 1 / 2 0 0
(b) The first PK stress tensor is:
100 0 0 1 / 2 0 0 400 0 0
( )
1 T
[To ] = ( det F ) [T] F = (8) 0 100 0 0 1 / 2 0 = 0 400 0 MPa.


0 0 100 0 0 1 / 2 0 0 400
The corresponding pseudo stress vector for the e1 plane in the deformed state, whose undeformed
plane is also e1 plane, is t o = 400e1 MPa. The Cauchy stress vector on the e1 plane is
t = 100e1 MPa. The Cauchy stress vector has a smaller magnitude because the area is four times
larger.

(c) The second PK stress tensor is:

Copyright 2010, Elsevier Inc


4-23
Lai et al, Introduction to Continuum Mechanics

1 / 2 0 0 400 0 0 200 0 0

= F [ To ] = 0 1 / 2 0 0 400 0 = 0 200 0 MPa.
% 1
T
0 0 1 / 2 0 0 400 0 0 200
The corresponding pseudo stress vector for the e1 plane in the deformed state, whose undeformed
plane is also e1 plane, is t% = 200e1MPa. The pseudo force df% is related to the force
df ( = t o dAo = 400e1 for dAo = 1) by the formula df% = F -1df . Thus,
1 / 2 0 0 1
df% = F -1 [ df ] = 400 0 1 / 2 0 0 df% = 200e1 .

0 0 1 / 2 0
_________________________________________________________________

Copyright 2010, Elsevier Inc


4-24
Lai et al, Introduction to Continuum Mechanics

CHAPTER 5, PART A

5.1 Show that the null vector is the only isotropic vector. (Hint: Assume that a is an
isotropic vector, and use a simple change of basis to equate the primed and unprimed
components).
--------------------------------------------------------------------------------
Ans. For an isotropic a , by definition, [a ]e = [a ]e , where {ei } and {ei } are any two orthonormal
i i

bases. That is [a ]e = [Q ] [a]e [a ]e = [Q ]e [a ]e for all [Q ]e .


T T
i ei i i i i i

1 0 0 a1 1 0 0 a1
Method I. Choose [Q ] = 0 1 0 , then a2 = 0 1 0 a2 gives

0 0 1 a3 0 0 1 a3
a1 = a1 = 0, a2 = a2 = 0, a3 = a3 = 0 . In other words, the only isotropic vector is the null
vector.
Method II. The matrix equation [a ]e = [Q ]e [a ]e , with the same basis for each matrix, is
T
i i i

equivalent to the equation a = Q a . That is, a is an eigenvector for QT for any orthogonal
T

tensor Q . But clearly, there is no non-zero vector which is an eigenvector for all orthogonal
tensors.
________________________________________________________________________

5.2 Show that the most general isotropic second-order tensor is of the form of I , where
is a scalar and I is the identity tensor.
-------------------------------------------------------------------------------
Ans. For an isotropic T , by definition, [ T ]e = [ T]e , where {ei } and {ei } are any two orthonormal
i i

1 0 0
bases. Choose [Q ] = 0 1 0 , then [ T]e = [Q ]e [ T]e [Q ]e gives
T
i i i i
0 0 1
T11 T12 T13 1 0 0 T11 T12 T13 1 0 0 1 0 0 T11 T12 T13
T T23 = 0 1 0 T21 T22 T23 0 1 0 = 0 1 0 T21 T22 T23
21 T22
T31 T32 T33 0 0 1 T31 T32 T33 0 0 1 0 0 1 T31 T32 T33
T11 T12 T13
= T21 T22 T23 T12 = T12 = 0, T21 = T21 = 0, T13 = T13 = 0 T31 = T31 = 0.
T31 T32 T33
1 0 0
Next, the choice of [Q ] = 0 1 0 gives,
0 0 1

Copyright 2010, Elsevier Inc

5-1
Lai et al, Introduction to Continuum Mechanics

T11 0 0 1 0 0 T11 0 0 1 0 0 1 0 0 T11 0 0


0 T T22 T23 0 1 0 = 0 1 0 0 T22 T23

22 T23 = 0 1 0 0

0 T32 T33 0 0 1 0 T32 T33 0 0 1 0 0 1 0 T32 T33


T11 0 0
= 0 T22 T23 T23 = T23 = 0, T32 = T32
= 0.
0 T32 T33
0 1 0
Next, the choice of [Q ] = 1 0 0 gives,
0 0 1
T11 0 0 0 1 0 T11 0 0 0 1 0 T22 0 0
0 T 0 = 1 0 0 0 T22 0 1 0 0 = 0 T11 0 T11 = T22

22
0 0 T33 0 0 1 0 0 T33 0 0 1 0 0 T33
1 0 0 T11 0 0 T11 0 0

Finally, the choice of [Q ] = 0 0 1 gives 0 T11 0 = 0 T33 0 T11 = T33 .

0 1 0 0 0 T33 0 0 T11

Thus, T11 = T22 = T33 = and T12 = T21 = T13 = T31 = T23 = T32 = 0 [ T] = [ I ]
________________________________________________________________________

5.3 For an isotropic linearly elastic body, (a) verify the = ( , EY ) as given in Table 5.1.
(b) Obtain the value of as EY / 0
-------------------------------------------------------------------------------
( EY 2 )
Ans. From Table 5.1, = 2 2 + ( 3 EY ) EY = 0
3 EY

( 3 EY ) + ( 3 EY )2 + 8EY
= .
4
( 3 EY ) + ( 3 EY ) 1 + 8 ( EY / ) / ( 3 EY / )
2
(b) =
4
As EY / 0 , 1 + 8 ( EY / ) / ( 3 EY / ) 1 + 4 ( EY / ) / 9 , where we have used the
2

binomial theorem. Thus,


{(3 EY ) + (3 EY )[1 + 4( EY / ) / 9]} / 4 = (3 EY )( EY / ) / 9 = (3 EY / )( EY ) / 9
Thus, as EY / 0 , EY / 3 .
________________________________________________________________________

EY 2 (1 + )
5.4 From = , = and k = obtain = ( EY , ) and
(1 + )(1 2 ) (1 2 ) 3
k = k ( , )

Copyright 2010, Elsevier Inc

5-2
Lai et al, Introduction to Continuum Mechanics

-------------------------------------------------------------------------------
EY 2 EY
Ans. = = = .
(1 + )(1 2 ) (1 2 ) 2 (1 + )
2 3 k 2 (1 + )
= = k = .
(1 2 ) 1 + 3 (1 2 )
________________________________________________________________________

5.5 Show that for an incompressible material ( 1 / 2 ) that


(a) = EY / 3, , k , but k = 2 / 3
(b) T = 2 E + (Tkk / 3)I where Tkk is constitutively indeterminate.
------------------------------------------------------------------------------
Ans. (a) From Table 5.1, we have
EY EY E EY 2 2
= = = Y , = , + = k k = .
2(1 + ) 2(1 + 1 / 2) 3 (1 + )(1 2 ) 3 3
(b) In general, T = eI + 2 E . Now, from Eq.(5.4.2), we have
Tkk T T
e= . As 1 / 2 , , and e kk so that T = kk I + 2 E .
(2 + 3 ) 3 3
We note that because of incompressibility, Tkk will be constitutively indeterminate. It becomes
determinate when the boundary condition(s) is (are) taken into account.
________________________________________________________________________

5.6 Given Aijkl = ij kl and Bijkl = ik jl . (a) Obtain A11 jk and B11 jk . (b) Identity those
A11 jk that are different from B11 jk .
-------------------------------------------------------------------------------
Ans. (a) A11kl = 11 kl = kl , B11kl = 1k 1l .
(b)
A1111 = A1122 = A1133 = 1, all other A11kl = 0, B1111 = 1, all other B11kl = 0 .
A1122 B1122 , A1133 B1133 .

________________________________________________________________________

5.7 Show that for an anisotropic linear elastic material, the principal directions of stress and
strain are in general not coincident.
-------------------------------------------------------------------------------
Ans. We have, Tij = Cijkl Ekl . Let ei be the principal basis for E , then [ E ]e is diagonal. Thus,
i

T12 = C12 kl Ekl = C1211E11 + C1222 E22 + C1233 E33 . This equation shows that in general, T12 0 .
Similarly, in general T13 0 and T23 0 . Thus the matrix of T is not diagonal with respect to the
principal basis of E .
________________________________________________________________________

Copyright 2010, Elsevier Inc

5-3
Lai et al, Introduction to Continuum Mechanics

5.8 If the Lam Constants for a material are:


= 119.2GPa (17.3 106 psi), = 79.2GPa (11.5 106 psi )
Find Young's modulus, Poisson's ratio and the bulk modulus.
-------------------------------------------------------------------------------
Ans. From Table 5.1, we have,
( 3 + 2 ) 79.2 3 (119.2 ) + 2 ( 79.2 )
EY =
+
=
119.2 + 79.2
(
= 206 GPa 30 106 psi )
119.2
= = = 0.3 ,
2( + ) 2(119.2 + 79.2)
k = + 2 / 3 = 119.2 + 2 ( 79.2 ) / 3 = 172 GPa (25 106 psi ) .
________________________________________________________________________

5.9 Given Young's modulus EY = 103 GPa and Poisson's ratio = 0.34 . Find the Lam
constants and . Also find the bulk modulus.
-------------------------------------------------------------------------------
Ans.
EY 0.34 (103)
= =
(1 + )(1 2 ) (1.34 )( 0.32 ) (
= 81.7 GPa 11.8 106 psi )
=
EY
=
103
2 (1 + ) 2 1.34
(
= 38.4 GPa 5.56 106 psi )
(
k = + 2 / 3 = 81.7 + 2 ( 38.4 ) / 3 = 107.3 GPa 15.6 106 psi )
________________________________________________________________________

5.10 Given Young's modulus EY = 193 GPa ., shear modulus = 76 GPa . Find Poisson's
ratio , Lam constant and the bulk modulus k .
-------------------------------------------------------------------------------
2 2 ( 76 )( 0.27 )
Ans. =
EY
2
1 =
193
2 ( 76 )
1 = 0.27, =
1 2
=
1 0.54
(
= 89.1 GPa 12.9 106 psi )
(
k = + 2 / 3 = 89.1 + 2 ( 76 ) / 3 = 140 GPa 20.3 106 psi )
________________________________________________________________________

5.11 The components of strain at a point of structural steel are:


E11 = 36 106 , E22 = 40 106 , E33 = 25 106
E12 = 12 106 , E23 = 0, E13 = 30 106

( )
Find the stress components. = 119.2 GPa 17.3 106 psi , = 79.2 GPa 11.5 106 psi ( )
-------------------------------------------------------------------------------
Ans. From T = eI + 2 E , we have, with e = ( 36 + 40 + 25 ) 106 = 101 106

Copyright 2010, Elsevier Inc

5-4
Lai et al, Introduction to Continuum Mechanics

1 0 0 36 12 30 17.7 1.9 4.75


[T] = (119.2 )(101) 0 1 0 10 + 2 ( 79.2 ) 12 40 0 10 = 1.9 18.4 0 MPa.
6 6

0 0 1 30 0 25 4.75 0 16.0
________________________________________________________________________

5.12 Do the previous problem if the strain components are:


E11 = 100 106 , E22 = 200 106 , E33 = 100 106
E12 = 100 106 , E23 = 0, E13 = 0
-------------------------------------------------------------------------------
Ans. From T = eI + 2 E , we have, with e = (100 200 + 100 ) 106 = 0
100 100 0 15.8 15.8 0
[T] = 2 ( 79.2 ) 100 200 0 10 = 15.8 31.7 0 MPa .
6

0 0 100 0 0 15.8
________________________________________________________________________

5.13 An isotropic elastic body ( EY = 207GPa, = 79.2GPa ) has a uniform state of stress
100 40 60
given by: [ T ] = 40 200 0 MPa .

60 0 200
(a) What are the strain components?
(b) What is the total change of volume for a five centimeter cube of the material?
-------------------------------------------------------------------------------
1
Ans. (a) We would like to use the equation Eij = (1 + )Tij (Tkk ) ij , therefore, we first
EY
E 207
obtain = Y 1 = 1 = 0.306 , then obtain Tkk = T11 + T22 + T33 = 100MPa . Thus,
2 2 ( 79.2 )
100 40 60 1 0 0
[ E] = E 1.31 40 200 0 ( 0.306 ) (100) 0 1 0
1
Y
60 0 200 0 0 1
100 52.4 78.6 0.483 0.253 0.380
=
1 52.4 292 0 = 0.253 1.41 0 103
3
207 10
78.6 0 231 0.380 0 1.12
(b) Dilatation e = Ekk = ( 0.483 1.41 + 1.12 ) 103 = 0.193 103 .

( )( )
Total change of volume = V = (V )( e ) = 53 0.193 103 = 24.1 103 cm3 .
________________________________________________________________________

Copyright 2010, Elsevier Inc

5-5
Lai et al, Introduction to Continuum Mechanics

5.14 An isotropic elastic sphere ( EY = 207GPa, = 79.2GPa ) of 5 cm radius is under the


uniform stress field
6 2 0
[T] = 2 3 0 MPa
0 0 0
Find the change of volume for the sphere.
------------------------------------------------------------------------------
E 207 1
Ans. = Y 1 = 1 = 0.306 , Eij = (1 + )Tij (Tkk ) ij gives
2 2 ( 79.2 ) EY

6 2 0 1 0 0 3.35 1.26 0

0 105
1
[E] = 1.31 2 3 0 ( 0.306 ) (3) 0 1 0 = 1.26 2.34
EY
0 0 0 0 0 1 0 0 0.443
4 53
Thus, e = 0.567 105 V = ( 0.567 ) 105 = 2.96 103
3

________________________________________________________________________

5.15 Given a motion


x1 = X1 + k ( X1 + X 2 ) , x2 = X 2 + k ( X1 X 2 ) , show that for a function f (a, b) = ab
f ( x1 , x2 ) f ( X1 , X 2 )
(a) f ( x1 , x2 ) = f ( X1 , X 2 ) + O(k ) , = + O( k ) ,
x1 X1
where O(k ) 0 as k 0
-------------------------------------------------------------------------------
Ans. (a) f ( x1 , x2 ) = x1 x2 = X1 + k ( X1 + X 2 ) X 2 + k ( X1 X 2 )
= X1 X 2 + k { X1 ( X1 X 2 ) + X 2 ( X1 + X 2 )} + k 2 ( X1 + X 2 )( X1 X 2 ) .
That is, f ( x1 , x2 ) = X1 X 2 + O ( k ) , where O(k ) 0 as k 0 , i.e.
f ( x1 , x2 ) = f ( X1 , X 2 ) + O(k ) f ( x1 , x2 ) f ( X1 , X 2 ) as k 0 .
f
(b) f ( x1 , x2 ) = x1 x2 = x2 = X 2 + k ( X1 X 2 ) = X 2 + O ( k ) , and
x1
f f f
f ( X1 , X 2 ) = X1 X 2 = X 2 . Thus, as k 0 .
X1 x1 X1
________________________________________________________________________

5.16 Do the previous problem for f ( a, b) = a 2 + b 2


-------------------------------------------------------------------------------
Ans. (a) f ( x1 , x2 ) = x12 + x22 = X1 + k ( X1 + X 2 ) + X 2 + k ( X1 X 2 )
2 2

= X12 + X 22 + 2k { X1 ( X1 + X 2 ) + X 2 ( X1 X 2 )} + k 2 {( X + X )
1 2
2
+ ( X1 X 2 )
2
}.
Copyright 2010, Elsevier Inc

5-6
Lai et al, Introduction to Continuum Mechanics

That is, f ( x1 , x2 ) = X12 + X 22 + O ( k ) , where O(k ) 0 as k 0 , i.e.


f ( x1 , x2 ) = f ( X1 , X 2 ) + O(k ) f ( x1 , x2 ) f ( X1 X 2 ) as k 0 .
f
(b) f ( x1 , x2 ) = x12 + x22 = 2 x1 = 2 X1 + 2k ( X1 +X 2 ) =2 X1 + O ( k ) and
x1
f f f
f ( X1 , X 2 ) = X12 + X 22 = 2 X1 . Thus, as k 0 .
X1 x1 X1
________________________________________________________________________

5.17 Given the following displacement field in an isotropic linearly elastic solid:

( )
u1 = kX 3 X 2 , u2 = kX 3 X1 , u3 = k X12 X 22 , k = 104
(a) Find the stress components and (b) in the absence of body forces, is the state of stress a
possible equilibrium stress field?
--------------------------------------------------------------------------------
Ans.
0 kX 3 kX 2 0 2X3 ( 2 X1 + X 2 )
k

(a) [u ] = kX 3 0
kX1 [ E] = 2 X 3 0 ( X1 2 X 2 )
2
2kX1 2kX 2 0 ( 2 X 1 + X 2 ) ( X 1 2 X 2 ) 0

0 2X3 ( 2 X1 + X 2 )

Thus, Ekk = 0 [ T] = 2 [ E] = k 2 X 3 0 ( X1 2 X 2 )
( 2 X 1 + X 2 ) ( X 1 2 X 2 ) 0

Since the displacement components are small (of the order of k ), therefore, xi X i , so that
T11 = T22 = T33 = 0, T12 = T21 = 2 kx3 , T13 = T31 = k ( 2 x1 + x2 ) , T23 = T32 = k ( x1 2 x2 ) .
(b) Substituting the above stress components into the equations of equilibrium, we have,
T11 T12 T13 T T T
+ + = 0 0 + 0 + 0 = 0 , 21 + 22 + 23 = 0 0 + 0 + 0 = 0 and
x1 x2 x3 x1 x2 x3
T31 T32 T33
+ + = 0 2 k 2 k + 0 = 0 . Thus, all equations of equilibrium are satisfied.
x1 x2 x3
Since the stress field is obtained from a given displacement field, therefore, the state of stress is a
possible equilibrium stress field.
________________________________________________________________________

5.18 Given the following displacement field in an isotropic linearly elastic solid:
u1 = kX 2 X 3 , u2 = kX1 X 3 , u3 = kX1 X 2 , k = 104
(a) Find the stress components and (b) in the absence of body forces, is the state of stress a
possible equilibrium stress field?
-------------------------------------------------------------------------------
Ans.

Copyright 2010, Elsevier Inc

5-7
Lai et al, Introduction to Continuum Mechanics

0 kX 2
kX 3 0 X3 X2
(a) [u ] = kX 30 kX1 = [ E] , Thus, Ekk = 0 [ T] = 2 [ E ] = 2 k X 3 0 X1

kX 2
kX1 0 X 2 X1 0
Since the displacement components are small (of the order of k ), therefore, xi X i , so that
T11 = T22 = T33 = 0, T12 = T21 = 2 kx3 , T13 = T31 = 2 kx2 , T23 = T32 = 2 kx1 .
(b) Substituting the above stress components into the equations of equilibrium, we have,
T11 T12 T13 T T T
+ + = 0 0 + 0 + 0 = 0 , 21 + 22 + 23 = 0 0 + 0 + 0 = 0 and
x1 x2 x3 x1 x2 x3
T31 T32 T33
+ + = 0 0 + 0 + 0 = 0 . Thus, all equations of equilibrium are satisfied. Since the
x1 x2 x3
stress field is obtained from a given displacement field, therefore, the state of stress is a possible
equilibrium stress field.
________________________________________________________________________

5.19 Given the following displacement field in an isotropic linearly elastic solid:

( )
u1 = kX 2 X 3 , u2 = kX1 X 3 , u3 = k X1 X 2 + X 32 , k = 104
(a) Find the stress components and (b) in the absence of body forces, is the state of stress a
possible equilibrium stress field?
--------------------------------------------------------------------------------
Ans.
0 kX 3 kX 2
(a) [u ] = kX 3 0 kX1 = [ E] , Thus, Ekk = 2kX 3 [ T] = ( 2kX 3 ) I + 2 [ E ]
kX 2 kX1 2kX 3
Since the displacement components are small (of the order of k ), therefore, xi X i , so that
x3 x3 x2
[ T] = 2k x3 x3 x1

.
x2
x1 ( + 2 ) x3

(b) Substituting the above stress components into the equations of equilibrium, we have,
T11 T12 T13 T T T
+ + = 0 0 + 0 + 0 = 0 , 21 + 22 + 23 = 0 0 + 0 + 0 = 0 and
x1 x2 x3 x1 x2 x3
T31 T32 T33
+ + = 0 0 + 0 + ( + 2 ) 0 . Thus, the stress field is not an equilibrium stress
x1 x2 x3
field in the absence of body forces. The given state of stress is not a possible equilibrium stress
field.
________________________________________________________________________

Copyright 2010, Elsevier Inc

5-8
Lai et al, Introduction to Continuum Mechanics

5.20 Show that for any function f ( s ) , the displacement u1 = f ( s ) where


2u1 2u1
s = x1 cL t satisfies the wave equation = cL2
t 2 x12
-------------------------------------------------------------------------------
Ans.
u1 df s df 2u d 2 f s d2 f
= = ( cL ) 21 = ( cL ) 2 = cL2 2 ,
t ds t ds t ds t ds
u1 df s df 2u d 2 f s d 2 f
= = 21 = 2 = .
x1 ds x1 ds x1 ds x1 ds 2
2u1 d2 f 2u1
Thus, = cL2 = cL2
t2 ds 2 x12
________________________________________________________________________

5.21 Calculate the ratio of the phase velocities cL / cT for Poisson 's ratio equal
1 / 3, 0.49 and 0.499 .
-----------------------------------------------------------------------------------------------------------
2 2 (1 ) 1 2
Ans. From Table 5.1, we have = + 2 = = ,
1 2 1 2 + 2 2 (1 )
cL + 2 2(1 )
Thus, = = . Thus, for
cT 1 2
= 1 / 3, cL / cT = 2(2 / 3) / (1 2 / 3) = ( 4 / 3) / (1 / 3) = 2.
= 0.49, cL / cT = 2(0.51) / (1 0.98 ) = 1.02 / 0.02 = 7.14.
= 0.499, cL / cT = 2(0.501) / (1 0.998 ) = 1.002 / 0.002 = 22.4.
__________________________________________________________________

5.22 Assume a displacement which depends only x2 and t , i.e., ui = ui ( x2 , t ) , i = 1, 2,3 .


Obtain the differential equations which ui ( x2 , t ) must satisfy in order to be a possible motion
in the absence of body forces.
-------------------------------------------------------------------------------
Ans. From the Navier equations, we have,
e = u2 / x2 e / x1 = 0, e / x2 = 2u2 / x22 , e / x3 = 0. Thus,
o (u12 / t 2 ) = ( 2u1 / x22 ) (u12 / t 2 ) = cT2 ( 2u1 / x22 ),
o (u22 / t 2 ) = ( + )( 2u2 / x22 ) + ( 2u2 / x22 ),
o (u22 / t 2 ) = ( + 2 )( 2u2 / x22 ) (u22 / t 2 ) = cL2 2u2 / x22 ( )
o (u32 / t 2 ) = ( 2u3 / x22 ) (u32 / t 2 ) = cT2 ( 2u3 / x22 ).

Copyright 2010, Elsevier Inc

5-9
Lai et al, Introduction to Continuum Mechanics

_________________________________________________________________

5.23 Consider a linear elastic medium. Assume the following form for the displacement field:
u1 = sin ( x3 ct ) + sin ( x3 + ct ) , u2 = u3 = 0
(a) What is the nature of this elastic wave (longitudinal, transverse, direction of propagation?)
(b) Find the strains, stresses and determine under what condition(s), the equations of motion are
satisfied in the absence of body forces.
(c)Suppose that there is boundary at x3 = 0 that is traction free. Under what condition(s) will the
above motion satisfy this boundary condition for all time.
(d) Suppose that there is boundary at x3 = l that is also traction free. What further conditions will
be imposed on the above motion to satisfy this boundary condition for all time.
-------------------------------------------------------------------------------
Ans. (a) Transverse wave, propagating in the e3 direction.
(b)The only nonzero strain components are:
E13 = E31 = (1 / 2 ) ( u1 / x3 ) = ( / 2 ) cos ( x3 ct ) + cos ( x3 + ct ) .
The only nonzero stress components are:
T13 = T31 = ( u1 / x3 ) = ( ) cos ( x3 ct ) + cos ( x3 + ct ) ,

( )
x1 equation of motion is: 0 2u1 / t 2 = T13 / x3 0 2 c 2u1 = 2 u1 c 2 = / 0 .
The other two equations are 0=0.
(c) The boundary condition on x3 = 0 is:
T ( e3 ) = 0 T13 ( 0, t ) = 0 [ cos ct + cos ct ] = 0 = 1 .
(d) The boundary condition on x3 = l is,
T ( e3 ) = 0 T13 ( l, t ) = 0 cos ( l ct ) cos ( l + ct ) = 0,[note = 1].
2sin l sin ct = 0 sin l = 0 = n / l, n = 1, 2,3...
_________________________________________________________________

5.24 Do the previous problem (Prob. 5.23) if the boundary x3 = 0 is fixed (no motion) and
x3 = l is still traction free.
-------------------------------------------------------------------------------
Ans. (a) and (b) are the same as in the previous problem.
(c) The boundary condition on x3 = 0 is: u1 ( 0, t ) = 0 u1 = [ sin ct + sin ct ] = 0 = 1 .
(d) The boundary condition on x3 = l is:
T ( e3 ) = 0 T13 ( l, t ) = 0 cos ( l ct ) + cos ( l + ct ) = 0
2cos l cos ct = 0 cos l = 0 = n / ( 2l ) , n = 1,3,5...
_________________________________________________________________

5.25 Do Problem 5.23 if the boundary x3 = 0 and x3 = l are both rigidly fixed (no motion)
--------------------------------------------------------------------------------
Ans. (a) and (b) are the same as in the previous problem 5.23.

Copyright 2010, Elsevier Inc

5-10
Lai et al, Introduction to Continuum Mechanics

(c) The boundary condition on x3 = 0 is: u1 ( 0, t ) = 0 u1 = [ sin ct + sin ct ] = 0 = 1


(d) The boundary condition on x3 = l is:
u1 ( l, t ) = 0 u1 = sin ( l ct ) + sin ( l + ct ) = 0
sin l cos ct 0 = n / l, n = 1, 2,3...
__________________________________________________________________

5.26 Do Problem 5.23, if the assumed displacement field is of the form:


u3 = sin ( x3 ct ) + sin ( x3 + ct ) , u1 = u2 = 0
--------------------------------------------------------------------------------
Ans. (a) Longitudinal, propagating in the e3 direction.
(b)The only nonzero strain components are:
E33 = ( u3 / x3 ) = cos ( x3 ct ) + cos ( x3 + ct ) .
The nonzero stress components are:
T11 = T22 = ( u3 / x3 ) , T33 = ( u3 / x3 ) + 2 ( u3 / x3 ) = ( + 2 ) ( u3 / x3 ) ,
where ( u3 / x3 ) = ( ) cos ( x3 ct ) + cos ( x3 + ct ) .
x3 equation of motion is:

( )
0 2u3 / t 2 = T33 / x3 0 2 c 2u3 = 2 ( + 2 ) u3 c 2 = ( + 2 ) / 0 .
The other two equations are 0=0.
(c) The boundary condition on x3 = 0 is:
T ( e3 ) = 0 T33 ( 0, t ) = 0 ( + 2 )( ) cos ( ct ) + cos ( ct ) = 0 = 1 .
(d) The boundary condition on x3 = l is:
T ( e3 ) = 0 T33 ( l, t ) = 0 cos ( l ct ) cos ( l + ct ) = 0.[Note = 1],
2sin l sin ct = 0 sin l = 0 = n / l, n = 1, 2,3...
__________________________________________________________________

5.27 Do the previous problem, Problem 5.26, if the boundary x3 = 0 is fixed (no motion) and
x3 = l is traction free ( t = 0 ).
-------------------------------------------------------------------------------
Ans. (a) and (b) are the same as the previous problem, problem 5.26.
(c) The boundary condition on x3 = 0 is: u3 ( 0, t ) = 0 u3 = [ sin ct + sin ct ] = 0 = 1 .
(d) The boundary condition on x3 = l is, with = 1 ,
T ( e3 ) = 0 T33 ( l, t ) = 0 cos ( l ct ) + cos ( l + ct ) = 0
2cos l cos ct = 0 cos l = 0 = n / ( 2l ) , n = 1,3,5...
__________________________________________________________________

5.28 Do Problem 5.26, if the boundary x3 = 0 and boundary x3 = l are both rigidly fixed.
-------------------------------------------------------------------------------

Copyright 2010, Elsevier Inc

5-11
Lai et al, Introduction to Continuum Mechanics

Ans. (a) (b) and (c) are the same as in Prob. 5.27, with = 1 .
(d) The boundary condition on x3 = l is
u3 ( l, t ) = 0 u3 = sin ( l ct ) + sin ( l + ct ) = 0
sin l cos ct = 0 sin l = 0 = n / l, n = 1, 2,3...
__________________________________________________________________

5.29 Consider the displacement field: ui = ui ( x1 , x2 , x3 , t ) . In the absence of body forces,


(a) obtain the governing equation for ui for the case where the motion is equivoluminal and
(b) obtain the governing equation for the dilatation e for the case where the motion is irrotational
( ui / x j = u j / xi ) .
-------------------------------------------------------------------------------
Ans. From the Navier equations of motion, Eq. (5.6.4)
2u e 2ui
o 2i = o Bi + ( + ) + , we have
t xi x j x j
2ui 2ui
(a) with e = 0 and Bi =0 , o = .
t 2 x j x j
ui u j 2ui u j u j e
(b) For irrotational motion = = = = . Thus,
x j xi x j x j x j xi xi x j xi
2ui e e e 2 u 2e
o = ( + ) + = ( + 2 ) o 2 i = ( + 2 )
t 2 xi xi xi t xi xi xi


2e
=
( + 2 )
2e
.
t 2 o xi xi
__________________________________________________________________

5.30 (a) Write a displacement field for an infinite train of longitudinal waves propagating in
the direction of 3e1 + 4e 2 . (b) Write a displacement field for an infinite train of transverse
waves propagating in the direction of 3e1 + 4e 2 and polarized in the x1 x2 plane.
-------------------------------------------------------------------------------
Ans. Let e n = (1 / 5 )( 3e1 + 4e 2 ) , then x e n = (1 / 5 )( 3x1 + 4 x2 ) . Also, e t = (1 / 5 )( 4e1 3e 2 )
2
(a) Equation 5.10.8 of Example 5.10.3 gives u = sin x e n cL t e n . Thus,
l
3 2 3x 4 x 4 2 3x 4 x
u1 = sin 1 + 2 cL t , u2 = sin 1 + 2 cL t , u3 = 0
5 l 5 5 5 l 5 5
2
(b) Equation 5.10.10 of Example 5.10.3 gives u = sin x e n cT t e t . Thus,
l
4 2 3x 4 x 3 2 3x 4 x
u1 = sin 1 + 2 cT t , u2 = m sin 1 + 2 cT t , u3 = 0
5 l 5 5 5 l 5 5

Copyright 2010, Elsevier Inc

5-12
Lai et al, Introduction to Continuum Mechanics

__________________________________________________________________

5.31 Solve for 2 and 3 in terms of 1 from the following two algebra equations:
1
2 (cos 21 ) + 3n(sin 2 3 ) = 1 cos 21 (i) 2 sin 21 3 (cos 21 ) = 1 sin 21 , (ii)
n
-------------------------------------------------------------------------------
Ans.
cos 21 n(sin 2 3 )
1
= ( cos 21 ) + n 2 (sin 2 3 )sin 21
2
= 1
sin 21 (cos 21 ) n
n
Thus,
1
2 1 (cos 21 ) n(sin 2 3 ) 1 cos 21
= n
sin 21
3 sin 2 cos 21 1
1

1 (cos 21 )2 n 2 (sin 2 3 ) ( sin 21 )


=
n n sin 41
That is,
(cos 21 ) n (sin 2 3 ) ( sin 21 )
2 2
n sin 41
2 = 1 , 3 = 1
( cos 21 ) 2
+ n (sin 2 3 )sin 21
2
( cos 21 ) + n2 (sin 2 3 )sin 21 2

__________________________________________________________________

5.32 A transverse elastic wave of amplitude 1 incidents on a traction free plane boundary. If
the Poisson's ratio = 1 / 3 , determine the amplitudes and angles of reflection of the reflected
waves for the following two incident angles (a) 1 = 0 and (b) 1 = 15o .
--------------------------------------------------------------------------------
Ans. From Eq. (5.11.14), we have, for = 1 / 3
n = cT / cL = (1 2 ) / 2 (1 ) = (1 2 / 3) / 2 (1 1 / 3) = (1 / 3) / 2 ( 2 / 3) = 1 / 2 .
Thus, (1 / 2 ) sin 3 = sin 1 . Using this equation, and Equations
(cos 21 )2 n 2 (sin 2 3 ) ( sin 21 ) n sin 41
2 = 1 , 3 = 1
( cos 21 ) + n 2 (sin 2 3 )sin 21
2
( cos 21 ) + n2 (sin 2 3 )sin 21
2

we have,
(a) 1 = 0 2 = 0 and (1 / 2 ) sin 3 = sin 1 = 0 3 = 0. Also, the above equations give
2 = 1 , and 3 = 0 ,. That is, there is no reflected longitudinal wave. There is only a reflected
transverse wave of the same amplitude which completely cancels out the incident transverse
wave.

(b) 1 = 15o 2 = 15o and sin 3 = 2sin15o = 0.5176 3 = 31.17o ,

Copyright 2010, Elsevier Inc

5-13
Lai et al, Introduction to Continuum Mechanics

2 = 1
( ) = 0.742
(cos30o )2 (1 / 4 ) (sin 62.34o ) sin 30o

( cos30 ) + (1 / 4) (sin 62.34 ) (sin 30 )


2 1
o o o

sin 600 / 2
3 = 1 = 0.5031
( ) ( )
2
cos30o + (1 / 4 ) (sin 62.34o ) sin 30o
That, the reflected transverse wave has an amplitude 2 = 0.7421 , with a reflected angle of
2 = 1 = 15o . The reflected longitudinal wave has an amplitude 3 = 0.5031 ,with a reflected
angle of 3 = 31.17o .
__________________________________________________________________

5.33 Referring to Figure 5.11.1 (Section 5.11), consider a transverse elastic wave incident on
a traction-free plane surface ( x2 = 0 ) with an angle of incident 1 with the x2 axis and
polarized normal to x1 x2 , the plane of incidence. Show that the boundary condition at
x2 = 0 can be satisfied with only a reflected transverse wave that is similarly polarized. What
is the relation of the amplitudes, wavelengths, and direction of propagation of the incident
and reflected wave?
--------------------------------------------------------------------------------
Ans. Let the plane of incidence be x1 x2 plane with the angle of incidence of the transverse wave
be 1 . That is, e n1 = sin 1e1 cos 1e 2 . The waves are polarized normal to the plane of
incidence, therefore, u1 = u2 = 0 , and u3 = 1 sin 1 + 2 sin 2 , with
2 2
1 = ( x1 sin 1 x2 cos 1 cT t 1 ), 2 = ( x1 sin 2 + x2 cos 2 cT t 2 )
l1 l2
The nonzero stress components are:
T13 = T31 = u3 / x1 = 2 ( 1 / l1 ) cos 1 sin 1 + ( 2 / l 2 ) cos 2 sin 2 ,
T23 = T32 = u3 / x2 =2 ( 1 / l1 ) cos 1 cos 1 + ( 2 / l 2 ) cos 2 cos 2 .

( )
The x3 equation of motion o 2u3 / t 2 = T31 / x1 + T32 / x2 gives:

( ) ( ) ( )
o ( 2 cT ) 1 / l12 sin 1 + 2 / l 22 sin 2 = ( 2 ) 1 / l12 sin 1 + 2 / l 22 sin 2
2

2
( )
o cT2 = cT2 = / o .
The traction free boundary at x2 = 0 requires that T12 = T22 = T32 = 0 on the surface, thus,
( 1 / l1 ) cos 1 cos 1 + ( 2 / l 2 ) cos 2 cos 2 = 0 , where
x2 = 0
2 2
1 =
( x1 sin 1 cT t 1 ), 2 = ( x1 sin 2 cT t 2 ) .
l1 l2
Thus, the boundary condition is satisfied if 1 = 2 , 1 = 2 , l1 = l 2 , 1 = 2 .
__________________________________________________________________

Copyright 2010, Elsevier Inc

5-14
Lai et al, Introduction to Continuum Mechanics

5.34 Do the problem of Section 5.11.(Reflection of Plane Elastic Waves, Figure 5.11-1) for
the case where the boundary x2 = 0 is fixed.
-------------------------------------------------------------------------------
Ans. As in Section 5.11, we assume
u1 = ( cos 1 ) 1 sin 1 + ( cos 2 ) 2 sin 2 + ( sin 3 ) 3 sin 3
u2 = ( sin 1 ) 1 sin 1 ( sin 2 ) 2 sin 2 + ( cos 3 ) 3 sin 3 , u3 = 0
where
2 2
1 = ( x1 sin 1 x2 cos 1 cT t 1 ), 2 = ( x1 sin 2 + x2 cos 2 cT t 2 )
l1 l2
2
3 = ( x1 sin 3 + x2 cos 3 cL t 3 )
l3
The equations of motion are satisfied with ( cL ) = ( + 2 ) / 0 , ( cT ) = / 0
2 2

Now, at x2 = 0 ,
( cos 1 ) 1 sin 1 + ( cos 2 ) 2 sin 2 + ( sin 3 ) 3 sin 3 =0
x2 =0

( sin 1 ) 1 sin 1 ( sin 2 ) 2 sin 2 + ( cos 3 ) 3 sin 3 =0


x2 =0
Thus, at x2 = 0 , sin 1 = sin 2 = sin 3 , so that
2 2 2
1 = ( x1 sin 1 cT t 1 ) = ( x1 sin 2 cT t 2 ) = ( x1 sin 3 cL t 3 ),
l1 l2 l3
2 = 2 ( pl 2 ) ,3 = 3 ( ql 2 )
Thus, as in Section 5.11, we have, with n = cT / cL ,
l 2 = l1 , nl 3 = l1 , 1 = 2 , n sin 3 = sin 1 , 2 = 1 , n3 = 1 .
However, the relations between the amplitudes are different. In fact, from
( cos 1 ) 2 + ( sin 3 ) 3 = ( cos 1 ) 1 ,
( sin 1 ) 2 ( cos 3 ) 3 = ( sin 1 ) 1.
we can obtain,

2 =
( 3 )( 1 ) (
sin sin cos 3 )( cos 1 )
, 3 =
sin 21
.
( sin 1 ) ( sin 3 ) + ( cos1 ) ( cos 3 ) ( sin 1 ) ( sin 3 ) + ( cos 1 ) ( cos 3 )
________________________________________________________________

5.35 A longitudinal elastic wave is incident on a fixed boundary x2 = 0 with an incident


angle of 1 with the x2 axis (similar to Fig. 5.11.1 of Section 5.11). (a) Show that in general,
there are two reflected waves, one longitudinal and the other transverse (also polarized in the
incident plane x1 x2 ). (b)Find the amplitude ratio of reflected to incident elastic waves.
------------------------------------------------------------------------
Ans. (a) Let
u1 = ( sin 1 ) 1 sin 1 + ( sin 2 ) 2 sin 2 + ( cos 3 ) 3 sin 3
u2 = ( cos 1 ) 1 sin 1 + ( cos 2 ) 2 sin 2 ( sin 3 ) 3 sin 3 , u3 = 0, where

Copyright 2010, Elsevier Inc

5-15
Lai et al, Introduction to Continuum Mechanics

2 2
1 = ( x1 sin 1 x2 cos 1 cL t 1 ), 2 = ( x1 sin 2 + x2 cos 2 cL t 2 )
l1 l2
2
3 = ( x1 sin 3 + x2 cos 3 cT t 3 )
l3
The equations of motion are satisfied with ( cL ) = ( + 2 ) / 0 , ( cT ) = / 0 .
2 2

Now, at x2 = 0 ,
( sin 1 ) 1 sin 1 + ( sin 2 ) 2 sin 2 + ( cos 3 ) 3 sin 3 =0
x2 = 0

( cos 1 ) 1 sin 1 + ( cos 2 ) 2 sin 2 ( sin 3 ) 3 sin 3 =0


x2 = 0
Thus, at x2 = 0 , sin 1 = sin 2 = sin 3 , so that
1 = (2 / l1 )( x1 sin 1 cL t 1 ) = (2 / l 2 )( x1 sin 2 cL t 2 ) = (2 / l 3 )( x1 sin 3 cT t 3 ),
2 = 2 ( pl 2 ) ,3 = 3 ( ql 3 ) .
Thus, we have,
1 = (2 / l1 )( x1 sin 1 cL t 1 ) = (2 / l 2 )( x1 sin 2 cL t 2 ) = (2 / l 3 )( x1 sin 3 cT t 3 ),
2 = 2 ( pl 2 ) ,3 = 3 ( ql 2 ) .
Thus,
sin 1 sin 2 sin 3 cL cL cT 1 2 3
= = , = = , = =
l1 l2 l3 l1 l 2 l 3 l1 l 2 l 3
So that 1 = 2 , l 2 = l1 , l 3 = nl1 , sin 3 = n sin 1 , 2 = 1 , 3 = n1
where n = cT / cL . We note that unlike the problem in Sect. 5.11, here l 3 = nl1 , sin 3 = n sin 1 (
instead of nl 3 = l1 , n sin 3 = sin 1 ). With sin 1 = sin 2 = sin 3 , we have
( sin 1 ) 2 + ( cos 3 ) 3 = ( sin 1 ) 1 , ( cos 1 ) 2 ( sin 3 ) 3 = ( cos1 ) 1
Thus,
3 / 1 = sin 21 / cos (1 3 ) , 2 / 1 = cos (1 + 3 ) / cos (1 3 )
__________________________________________________________________

5.36 Do the previous problem (Prob. 5.35) for the case where x2 = 0 is a traction free
boundary
-------------------------------------------------------------------------------
Ans. Let
u1 = ( sin 1 ) 1 sin 1 + ( sin 2 ) 2 sin 2 + ( cos 3 ) 3 sin 3
u2 = ( cos 1 ) 1 sin 1 + ( cos 2 ) 2 sin 2 ( sin 3 ) 3 sin 3 , u3 = 0, where
2 2
1 = ( x1 sin 1 x2 cos 1 cL t 1 ), 2 = ( x1 sin 2 + x2 cos 2 cL t 2 )
l1 l2
2
3 = ( x1 sin 3 + x2 cos 3 cT t 3 )
l3
The equations of motion are satisfied with ( cL ) = ( + 2 ) / 0 , ( cT ) = / 0 .
2 2

At x2 = 0 , T21 = T22 = T23 = 0 . Thus,

Copyright 2010, Elsevier Inc

5-16
Lai et al, Introduction to Continuum Mechanics

u1 / x2 + u2 / x1 = 0 and ( +2 )( u2 / x2 ) + ( u1 / x1 ) = 0 .
That is,
(1 / l1 ) cos 1 ( sin 21 ) + ( 2 / l 2 ) cos 2 ( sin 2 2 ) + ( 3 / l 3 ) cos3 ( cos 2 3 ) = 0 (i)
and
(1 / l1 ) + 2 ( cos 1 ) cos 1 + ( 2 / l 2 ) + 2 ( cos 2 )2 cos 2
2

(ii)
+ ( 3 / l 3 ) ( 2 ) ( sin 3 cos 3 ) cos 3 = 0
In order for the above two equations to be satisfied for all x1 and t , we must have, at x2 = 0
cos 1 = cos 2 = cos 3 , which gives
sin 1 sin 2 sin 3 cL cL cT 1 2 m pl 2 3 m ql 3
= = , = = , = =
l1 l2 l3 l1 l 2 l 3 l 1 l2 l3
Thus, 1 = 2 , l 2 = l1 , l 3 = nl1 , sin 3 = n sin 1 , 2 m pl 2 = 1 , 3 m ql 3 = n1 , where
n = cT / cL .
(i) and (ii) now gives
(1 / l1 )( sin 21 ) + ( 2 / l 2 )( sin 21 ) + ( 3 / l 3 )( cos 2 3 ) = 0 (iii)
(1 / l1 )( + 2 cos 2 1 ) + ( 2 / l1 )( + 2 cos 2 1 ) ( 3 / l 3 )(2 )sin 3 cos 3 = 0

+ 2
+ 2 cos 2 1 = + 2 2 sin 2 1 = 2sin 2 1 = (iv)


( 2 )
n
(1 2n 2
)
sin 2 1 ( 3 /)(2 )sin 3 cos 3 = 1
n
(1 2n 2
sin 2 1 )
Since
+ 2
+ 2 cos 2 1 = + 2 2 sin 2 1 =

2sin 2 1 = 2 1 2n 2 sin 2 1 ,
n
( )
and l 3 = nl1 , therefore, (iii) and (iv) become
( n sin 21 ) 2 + ( cos 2 3 ) 3 = 1n sin 21 (v)

(1 2n 2
) (
sin 2 1 2 2n sin 3 cos 3 3 = 1 2n 2 sin 2 1 1 ) (vi)
(v) and (vi) give
3 2n sin 21 (1 2n 2 sin 2 1 )
= ,
1 n 2 sin 21 sin 2 3 + (1 2n 2 sin 2 1 ) cos 2 3
2 n 2 sin 21 sin 2 3 (1 2n 2 sin 2 1 ) cos 2 3
=
1 n 2 sin 21 sin 2 3 + (1 2n 2 sin 2 1 )cos 2 3
__________________________________________________________________

5.37 Verify that the thickness stretch vibration given by Eq.(5.12.3), i.e.,
u1 = ( A cos kx1 + B sin kx1 )(C cos cL kt + D sin cL kt )
does satisfy the longitudinal wave equation 2u1 / t 2 = ( cL ) 2u1 / x12
2
( )
Copyright 2010, Elsevier Inc

5-17
Lai et al, Introduction to Continuum Mechanics

---------------------------------------------------------------------------------
Ans.
u1 = ( A cos kx1 + B sin kx1 )(C cos cL kt + D sin cL kt ) ,
2u1 / x12 = k 2 ( A cos kx1 + B cos kx1 )(C cos cL kt + D sin cL kt )
2u1 / t 2 = (kcL ) 2 ( A cos kx1 + B cos kx1 )(C cos cL kt + D sin cL kt )
that is, 2u1 / x12 = k 2u1 and 2u1 / t 2 = ( cL k ) u1 . Thus, cL2 2u1 / x12 = 2u1 / t 2
2

__________________________________________________________________

5.38 (a) Find the thickness-stretch vibration of a plate, where the left face ( x1 = 0 ) is
subjected to a forced displacement u = ( cost )e1 and the right face x1 = l is free.
(b)Determine the values of that give resonance.
---------------------------------------------------------------------------------
Ans. Let (a) u1 = ( A cos kx1 + B sin kx1 )(C cos cL kt + D sin cL kt ) . Using the boundary
condition u(0, t ) = ( cost )e1 , we have , cos t = u1 (0, t ) = AC cos cL kt + AD sin cL kt
Thus, AC = , k = / cL , D = 0 u1 = ( cos kx1 + BC sin kx1 ) cos t .
At x1 = l , T11 = T12 = T13 = 0 . Now, T11 = ( + 2 )( u1 / x1 ) , thus ( u1 / x1 ) x =l = 0 , i.e.,
1

k ( sin k l + BC cos k l) cos t = 0 BC = tan k l ,


u1 = [cos( x1 / cL ) + tan(l / cL )sin( x1 / cL )]cos t .
(b) Resonance occurs at : l / cL = n / 2, n = 1,3,5...
__________________________________________________________________

5.39 (a) Find the thickness stretch vibration if the x1 = 0 face is being forced by a traction
t = ( cos t ) e1 and the right hand face x1 = l is fixed. (b) Find the resonance frequencies.
--------------------------------------------------------------------------------
Ans. (a) u1 = ( A cos kx1 + B sin kx1 )(C cos cL kt + D sin cL kt ) .
At
x1 = 0, n = e1 , t = Te1 = (T11e1 + T21e2 + T31e3 ) = cos te1 T11 = cos t , T21 = T31 = 0
Since T11 = ( + 2 )( u1 / x1 ) , therefore, the boundary condition at x1 = 0 is:
( + 2 )( u1 / x1 ) x1 =0 = cost , ( + 2 ) k ( B )(C cos cL kt + D sin cL kt ) = cost ,

D = 0, cL k = , BC = , u1 = ( AC cos kx1 sin kx1 ) cos t
k ( + 2 ) k ( + 2 )
At x1 = l ,

u1 = ( AC cos k l sin k l) cos t = 0 AC cos k l sin k l = 0
k ( + 2 ) k ( + 2 )
cL l x cL x
AC = tan k l u1 = tan cos 1 sin 1 cos t
k ( + 2 ) ( + 2 ) cL cL ( + 2 ) cL
(b) Resonance occurs at

Copyright 2010, Elsevier Inc

5-18
Lai et al, Introduction to Continuum Mechanics

= n cL / ( 2l ) , n = 1,3,5...
__________________________________________________________________

5.40 (a) Find the thickness-shear vibration if the left hand face x1 = 0 has a forced
displacement u = ( cost )e3 and the right-hand face x1 = l is fixed. (b) Find the resonance
frequencies.
--------------------------------------------------------------------------------
Ans. (a) Let u3 = ( A cos kx1 + B sin kx1 )(C cos cT kt + D sin cT kt ), u1 = u2 = 0
In the absence of body forces, the x3 Navier equation of motion (5.6.7) gives:
2u3 e 2 2 2 2u3 2u3
o = ( + ) + 2 + 2 + 2 u3 o =
t 2 x3 x
1 x21 x3 t 2 x12

leads to o ( cT k ) u3 = k 2u3 ( cT ) = / o .
2 2

The boundary condition at x1 = 0 ,


u3 ( 0, t ) = ( cost ) u3 = ( A)(C cos cT kt + D sin cT kt ) = cos t D = 0, AC = , k = / cT .
u3 = ( cos kx1 + BC sin kx1 ) cos t .
The boundary condition at x1 = l ,
u3 (l, t ) = 0 u3 = ( cos k l + BC sin k l) cos t = 0 BC = cot k l .
u3 = [ cos( x1 / cT ) cot( l / cT )sin( x1 / cT ) ] cos t .
(b) Resonance occurs at = n cT / l, n = 1, 2,3...
__________________________________________________________________

5.41 (a) Find the thickness-shear vibration if the left hand face x1 = 0 has a forced
displacement u = (coste 2 + sin te3 ) and the right-hand face x1 = l is fixed. (b) Find the
resonance frequencies.
-------------------------------------------------------------------------------
Ans.
(a) If the left hand face x1 = 0 has a forced displacement u = coste 2 and the right-hand face
x1 = l is fixed, it is clear from the result of the previous problem,
u2 = cos ( x1 / cT ) cot ( l / cT ) sin ( x1 / cT ) cos t , u1 = u3 = 0 .
If the left hand face x1 = 0 has a forced displacement u = sin te3 and the right-hand face
x1 = l is fixed, the displacement field is clearly given by
u3 = cos ( x1 / cT ) cot ( l / cT ) sin ( x1 / cT ) sin t , u1 = u2 = 0
Thus, the solution to the present problem can be obtained by superposition to be
u1 = 0 ,
u2 = cos ( x1 / cT ) cot ( l / cT ) sin ( x1 / cT ) cos t ,
u3 = cos ( x1 / cT ) cot ( l / cT ) sin ( x1 / cT ) sin t .
(b) Resonance occurs at = n cT / l, n = 1, 2,3...

Copyright 2010, Elsevier Inc

5-19
Lai et al, Introduction to Continuum Mechanics

__________________________________________________________________

5.42 A cast iron bar, 200 cm long and 4 cm in diameter, is pulled by equal and opposite axial
force P at its ends. (a) Find the maximum normal and shearing stresses if P=90,000N. (b)
Find the total elongation and lateral contraction. ( EY = 103GPa., = 0.3)
--------------------------------------------------------------------------------
Ans. A = (4 102 )2 / 4 = 12.6 104 m 2 .

( )
(a) (Tn )max = P / A = 90,000 / 12.6 104 = 71.4 106 N , (Ts )max = P / ( 2 A) = 35.7 106 N .
(b) l = ( P / A)(l / EY ) = (71.4 106 ) 2 / (103 109 ) = 1.39 103 m,
d = ( P / A)(d / EY ) = (0.3)(71.4 106 ) (4 102 ) / (103 109 ) = 0.832 105 m.
____________________________________________________________________

5.43 A composite bar, formed by welding two slender bars of equal length and equal cross-
sectional area, is loaded by an axial load P as shown in Figure below. If Young's moduli of
the two portions are EY(1) and EY(2) , find how the applied force is distributed between the two
halves.
-------------------------------------------------------------------------------
Ans. Taking the whole bar as a free body, let P1 be the compressive reactional force from the
right wall to the bar and P2 be the compressive reactional force from the left wall to the bar, then
the equation of static equilibrium requires

P = P1 P2 . (i)
There is no net elongation of the composite bar, therefore,
P1l P2 l
+ =0 (ii)
AEY(1) AEY(2)
Combining Eq. (i) and (ii), we obtain
P P
P1 = , P2 = . (iii)
1 + ( EY / EY )
(2) (1)
1 + ( EY / EY(2) )
(1)

_________________________________________________________________

5.44 A bar of cross-sectional area A is stretched by a tensile force P at each end. (a)
Determine the normal and shearing stresses on a plane with a normal vector which makes an
angle with the axis of the bar. (b) For what value of are the normal and shearing stresses
equal? (c) If the load carrying capacity of the bar is based on the shearing stress on the plane
defined by = o to be less than o what is the maximum allowable load P ?
-------------------------------------------------------------------------------

Copyright 2010, Elsevier Inc

5-20
Lai et al, Introduction to Continuum Mechanics

0 0
Ans. [ T ] = 0 0 0 , n = cos e1 + sin e 2 ,
0 0 0
(a) For the plane with a normal given by n = cos e1 + sin e 2 , we have,
0 0 cos cos
[t ] = [T][n ] = 0 0 0 sin = 0 t = cos e1 Tn = t n = cos 2 ,
0 0 0 0 0
2
( )
Ts2 = t Tn2 = 2 cos 2 2 cos 4 = 2 cos 2 1 cos 2 = 2 cos 2 sin 2
Ts = sin 2 / 2 .
sin 2
(b) = cos 2 cos sin = cos 2 cos ( sin cos ) = 0 .
2
Thus, (i) cos = 0 = / 2 Ts = Tn = 0, and (ii) sin = cos = / 4 Ts = Tn = / 2 .
sin 2 o 2 o 2 o
(c) o . Max allowable P A
2 sin 2 o sin 2 o
_________________________________________________________________

5.45 A cylindrical bar, whose lateral surface is constrained so that there can be no lateral
expansion, is then loaded with an axial compressive stress T11 = . (a) Find T22 and T33 in
terms of and the Poisson's ratio , (b) show that the effective Young's modulus
( EY )eff T11 / E11 is given by ( EY )eff = (1 ) / (1 2 2 ) . [note misprint in text].

-------------------------------------------------------------------------------
Ans. (a) E22 = 0 T22 (T33 + T11 ) = 0 , E33 = 0 T33 (T11 + T22 ) = 0 . Thus,
T22 T33 = and T33 T22 = . From these two equations, we have,
T22 = T33 = / (1 ) .
1 2 1 2
2 2
1
(b) E11 = T11 (T22 + T33 ) = + 2 = E 1 1 = E 1

EY 1
EY Y
Y
T11 E (1 ) EY (1 )
Thus, ( EY )eff ( EY )eff = = Y = .
E11 E11 1 2 2
(1 2 )(1 + )
__________________________________________________________________

5.46 Let the state of stress in a tension specimen be given by T11 = and all other Tij =0 . (a)
Find the components of the deviatoric stress defined by To = T (1 / 3) Tkk I . (b) Find the
principal scalar invariants of To
-------------------------------------------------------------------------------
Ans. (a) Tkk = T11 + T22 + T33 = T11o = T11 Tkk / 3 = / 3 = 2 / 3 .

Copyright 2010, Elsevier Inc

5-21
Lai et al, Introduction to Continuum Mechanics

o
T22 = T22 Tkk / 3 = / 3 = T33
o
, T12o = T13o = T23
o
= 0.
(b)
I1 = T11 + T22 + T33 = 2 / 3 / 3 / 3 = 0.
o o o

I 2 = T11oT22
o
+ T22T33 + T11oT33
o o o
= ( 2 / 3)( / 3) + ( / 3)( / 3) + ( 2 / 3)( / 3) = 2 / 3.
I 3 = T11oT22T33 = ( 2 / 3)( / 3)( / 3) = 2 3 / 27.
o o

_________________________________________________________________

5.47 A circular cylindrical bar of length l hangs vertically under gravity force from the
ceiling. Let x1 axis coincides with the axis of the bar and points downward and let the
point ( x1 , x2 , x3 ) = ( 0,0,0 ) be fixed at the ceiling. (a) Verify that the following stress field
satisfies the equations of equilibrium in the presence of the gravity force: T11 = g ( l x1 ) ,
all other Tij = 0 and (b) verify that the boundary conditions of zero surface traction on the
lateral face and the lower end face are satisfied and (c) obtained the resultant force of the
surface traction at the upper face.
-------------------------------------------------------------------------------
Ans. (a) The body force per unit volume is given by B = ge1 . Thus, with T11 = g ( l x1 ) , we
have,
T11 T12 T13
+ + + g = g + 0 + 0 + g = 0 and the other two equations are trivially
x1 x2 x3
satisfied.
(b) On the bottom end face x1 = l , n = e1 , t = Te1 = T11 x =l e1 = g ( l l ) e1 = 0 .
1

T11 0 0 0 0
On the lateral face, n = n2e 2 + n3e3 , [ t ] = 0 0 0 n2 = 0 t = 0 .
0 0 0 n3 0
(c) On the top face at x1 = 0 , n = e1 , t = Te1 = T11 x =0 e1 = g ( l 0 ) e1 = g le1
1

Let the area of the face be A , then the resultant force is tA = g lAe1 = We1 where
W = gAl is the weight of the bar and the minus sign indicates that the resultant force at the
ceiling is upward which balances the weight of the bar.
__________________________________________________________________

5.48 A circular steel shaft is subjected to twisting couples of 2700 Nm . The allowable tensile
stress is 0.124 GPa . If the allowable shearing stress is 0.6 times the allowable tensile stress,
what is the minimum allowable diameter?
-------------------------------------------------------------------------------
Ma
(
Ans. (Tn )max = (Ts )max = t = M t a / a 4 / 2 =
Ip
) 2M t
a3
. Thus

Copyright 2010, Elsevier Inc

5-22
Lai et al, Introduction to Continuum Mechanics

2 ( 2700 ) 2 ( 2700 )
( )
( 0.6 ) 0.124 109 a3 = 23.1 106 m3
a 3
(
( 0.6 ) 0.124 10
9
)
a 2.85 102 m = 2.85 cm d 5.7 cm.
_________________________________________________________________

5.49 In Figure 5P.2, a twisting torque M t is applied to the rigid disc A . Find the twisting
moments transmitted to the circular shafts on either side of the disc.

Figure 5P.2

-------------------------------------------------------------------------------
Ans. Let M1 and M 2 be the twisting moments transmitted to the left and the right shaft
respectively. Then equilibrium of the disc demands that
M1 + M 2 = M t (i)
In addition, the disc is rigid, therefore, the angle of twist of the left shaft at the disc relative to the
left wall must equal the angle of twist of the right shaft at the disc relative to the right wall, i.e.,
M1l1 M 2 l 2
= M1l1 = M 2 l 2 (ii)
I p I p
Thus,
l2 l1
M1 = Mt , M2 = Mt (iii)
l1 + l 2 l1 + l 2
for l1 = l 2 , M1 = M 2 = M t / 2.
_________________________________________________________________

5.50 What needs to be changed in the solution for torsion of a solid circular bar obtained in
Section 5.14 for it to be valid for torsion of a hollow circular bar with inner radius a and
outer radius b ?
-------------------------------------------------------------------------------
Ans. The hollow circular bar differs from the solid circular bar in that there is an inner lateral
surface which is also traction free. However, the normal to the inner lateral surface differs from
that to the outer surface only by a sign so that the zero surface traction in the inner surface is also
satisfied since that for the outer surface is satisfied. However, in calculating the resultant force
and resultant moment due to the surface traction on the end faces, the integrals are now to be
integrated over the circular ring area between by r = a and r = b rather than the whole solid

Copyright 2010, Elsevier Inc

5-23
Lai et al, Introduction to Continuum Mechanics

circular area of radius b . Thus, the only change that needs to be made is that the polar area second

moment I p is now given by I p =
2
b4 a4 . ( )
__________________________________________________________________

5.51 A circular bar of radius ro is under the action of an axial tensile load P and a twisting
couple of M t . (a) Determine the stress throughout the bar. (b) Find the maximum normal and
shearing stress
-------------------------------------------------------------------------------
P M
Ans. Superpose the solutions for tension and for torsion, we have, with = , t
A Ip
(a) T11 = , T12 = T21 = x3 , T13 = T31 = x2 , all otherTij = 0 .
(b) The characteristic equation is
x3 x2
x3
2 2
( )
0 = 0 2 3 + ( x2 ) + ( x3 ) = 0 2 2 r 2 = 0, where r 2 = x22 + x32
x2 0

2 + 4 2 r 2
Thus, 1,2 = , 3 = 0 . Thus
2
+ 2 + 4 2 r 2
1
(Tn )max = 2 + 4 2 r 2 .
, Ts =
2 2
__________________________________________________________________

5.52 Compare the twisting torque which can be transmitted by a shaft with an elliptical cross-
section having a major diameter equal to twice the minor diameter with a shaft of circular
cross-section having a diameter equal to the major diameter of the elliptical shaft. Both shafts
are of the same material. Also compare the unit twist (i.e., twist angle per unit length) under
the same twisting moment. Assume that the maximum twisting moment which can be
transmitted is controlled by the maximum shearing stress.
----------------------------------------------------------------------------------
Ans. (a) For an elliptical shaft with major diameter 2b and minor diameter 2a (i.e., b > a ),
2 ( M t )ell
(Ts )max = .
a 2b
2 ( M t )cir
For a circular shaft with radius b , (Ts )max = , thus
b3
2 ( M t )ell 2 ( M t )cir ( M t )ell a 2 a 2 1
(Ts )max = = = = = .
( M t )cir b 2a 4
,
a 2b b3

(b) 'ell
a2 + b2
= 'cir
2
=
(
'ell b a + b
=
2 2
) ( )
( 2 a ) 5a 2
a3b3 t M t , thus,
M , = =5 .
b
4
'cir 2a 3 2a 3

Copyright 2010, Elsevier Inc

5-24
Lai et al, Introduction to Continuum Mechanics

__________________________________________________________________

5.53 Repeat the previous problem except that the circular shaft has a diameter equal to the
minor diameter of the elliptical shaft.
--------------------------------------------------------------------------------
Ans. (a) For an elliptical shaft with major diameter 2b and minor diameter 2a (i.e., b > a ),
2 ( M t )ell
(Ts )max = .
a 2b
2 ( M t )cir
For a circular shaft with radius a , (Ts )max = , thus,
a3
2 ( M t )ell 2 ( M t )cir ( M t )ell b 2a
(Ts )max = = = = = 2.
( M t )cir a a
,
a b2
a 3

a2 + b2 2 'ell 5a 2 a 4 5
(b) 'ell = M , ' = 4
M
t = = .
a3b3 t cir
'cir 8a 6
a 2 16
__________________________________________________________________

5.54 Consider torsion of a cylindrical bar with an equilateral triangular cross-section as


(
shown in Fig. P.5.3. (a) Show that a warping function = C 3x22 x3 x33 generate an)
equilibrium stress field. (b) Determine the constant C , so as to satisfy the traction free
boundary condition on the lateral surface x2 = a . With C so obtained, verify that the other
two lateral surfaces are also traction free. (c) Evaluate the shear stress at the corners and
along the line x3 = 0 .(d) Along the line x3 = 0 where does the greatest shear stress occur?
-------------------------------------------------------------------------------.
2
2 2 2
Ans. (a) = 6Cx 3 , = 6 Cx 3 , thus + = 0 , so that equations of equilibrium are
x22 x32 x22 x32
satisfied.

x3

x2
(2a,0) (a,0)

(b) For the lateral surface x2 = a , n =e2 , t = Te 2 = T12e1 = x3 '+ ( / x2 ) e =0


x2 = a 1

x3 ' = [ 6Cx2 x3 ]x x3 ' = 6Cax3 C = '/ 6a .


2 =a

( ) (
On the lateral surface x3 = 1 / 3 ( x2 + 2a ) 3 x3 x2 = 2a n = (1 / 2 ) e 2 + 3e3 )
( ) ( )
t = Tn = (1 / 2 ) Te 2 + 3Te3 = (1 / 2 ) T12 + 3T13 e1 . Now, for = ( '/ 6a ) 3 x22 x3 x33( )
Copyright 2010, Elsevier Inc

5-25
Lai et al, Introduction to Continuum Mechanics

T12 = x3 '+ ( / x2 ) = x3 '+ ( '/ a )( x2 x3 ),


,
T13 = x2 '+ ( / x3 ) = x2 '+ ( '/ 2a)( x22 x32 ).
'
Therefore, T12 + 3T13 =
2a
2ax3 2 x2 x3 + 2 3ax2 + 3 x22 x32 . With x2 = 3 x3 2a ,
( )
'
( ) (
) ( )
2
T12 + 3T13 = 2ax3 2 3 x3 2a x3 + 2 3a 3 x3 2a + 3 3 x3 2a x32
2a
'
=
2a
( ) (
2ax3 + 2 3 x32 + 4ax3 + 6ax3 4 3a 2 + 3 3 x32 4 3ax3 + 4a 2 x32
) ( )
'
=
2a
(
2ax3 + 4ax3 + 6ax3 12ax3 4 3a 2 + 4 3a 2 + 2 3 x32 + 3 3 x32 3 x32 = 0.
) ( )
( )
That is, on x3 = 1 / 3 ( x2 + 2a ) , t = 0 . Clearly, for the lateral surface

( )
x3 = 1 / 3 ( x2 + 2a ) , t = 0 .
(c) at the corner ( 2a,0 ) , T12 = x3 '+ ( '/ a ) ( x2 x3 ) = 0 and

( )
( )
T13 = x2 '+ ( '/ 2a ) x22 x32 = ( 2a ) '+ ( '/ 2a ) 4a 2 = 0 .

At the corners ( a, 3a ) , T = ' ( x / a ) ( x a ) = ' ( 3 ) ( a a ) = 0 and


12 3 2

T = ' x + ( x x ) / 2a = ' a + ( a 3a ) / 2a = 0 .
2 2 2 2

13 2 2 3
That is, the shear stress at all three corners are zero. Along x3 = 0 ,
T12 = x3 '+ ( '/ a ) ( x2 x3 ) = 0 ,

( )
T13 = ' x2 + x22 x32 / 2a = ( '/ 2a ) 2ax2 + x22 .
( )
(d) dT13 / dx2 = ( '/ 2a )( 2a + 2 x2 ) = 0 x2 = a (T13 ) x = ' a / 2 .
2 = a

( )
But at ( x2 , x3 ) = ( a,0 ) , T13 = ' x2 + x22 x32 / 2a = ' a + a 2 / 2a = ( 3a / 2 ) ' .
( )
Thus, along x3 = 0 , the greatest shear stress occurs at ( x2 , x3 ) = ( a,0 ) with Ts = ( 3a / 2 ) ' .
__________________________________________________________________

5.55 Show from the compatibility equations that the Prandtl's stress function ( x2 , x3 ) for
2 2
torsion problem must satisfy the equation + = constant
x33 x22
-------------------------------------------------------------------------------

Ans. With T12 = , T13 = , and all other Tij = 0 , we have, the nonzero strain components
x3 x2
1 1
are: E12 = , E13 = , all other Eij = 0 . All equations of compatibility are
2 x3 2 x2
identically satisfied except the following two:

Copyright 2010, Elsevier Inc

5-26
Lai et al, Introduction to Continuum Mechanics

2 E22 E31 E12 E23 2 E33 E12 E23 E31


= + + , = + +
x3x1 x2 x2 x3 x1 x1x2 x3 x3 x1 x2
which leads to
2
2 2
3 2 2
2 + 3 = 0 , + = 0 3 + 2 = constant
x2 x2 x3 x3 x33 x22 x3 x2
________________________________________________________________________

5.56 Given that the Prandtl' stress function for a rectangular bar in torsion is given by
32 ' a 2 1 ( n 1) /2 cosh ( n x3 / 2a ) n x2
= 3 ( 1) 1 cos

3
n =1,3,5 n cosh ( n b / 2a ) 2a
The cross section is defined by a x2 a and b x3 b . Assume b > a , (a) Find the
maximum shearing stress. (b) Find the maximum normal stress and the plane it acts.
--------------------------------------------------------------------------------

Ans. We know that when a rectangular membrane, fixed on its side, is subjected to a uniform
pressure on one side of the membrane, the deformed surface has a maximum slope at the mid
point of the longer side. Thus, based on the membrane analogy discussed in Example 5.17.3, on
any plane x1 = constant , the maximum shearing stress occurs on the mid point of the longer side.
That is at the point x2 = a and x3 = 0 . From the given function ( x2 , x3 ) , we obtain the stress
components as
32 ' a
( n 1) /2 n sinh ( n x3 / 2a ) n x2
2
1
T12 = = ( 1) cos .
x3

3
n =1,3,5 n
3
2a cosh ( n b / 2a ) 2a

32 ' a 2 1 ( n 1) /2 n cosh ( n x3 / 2a ) n x2
T13 = = 3 ( 1) 1 sin .
x2 3 n =1,3,5 n 2a cosh ( n b / 2a ) 2a

(
At x2 = a, x3 = 0, note sin ( n / 2 ) = ( 1)
( n +3) /2
)
, n = 1,3,5..

16 ' a 1 1
T13 = 2 1 cosh n b / 2a , T12 = 0
2
n =1,3,5 n ( )
That is
16 ' a
1
(Ts )max = 2 2 1
1
.
n =1,3,5 n cosh ( n b / 2a )

1 2 16 ' a 1 1
Or, since = , therefore (Ts )max = 2 ' a
2 n =1,3,5 n 2 cosh ( n b / 2a )
2 8
n =1,3,5 n

Since at this point, the only nonzero stress components are T13 and T31 ( = T13 ) , therefore, the
characteristic equation is 3 + T132 = 0 so that the maximum normal stress is
(Tn )max = T13 = (Ts )max , which acts on plane whose normal is in the direction (1 / )
2 ( e1 e3 ) .

Copyright 2010, Elsevier Inc

5-27
Lai et al, Introduction to Continuum Mechanics

_________________________________________________________________

5.57 Obtain the relationship between the twisting moment M t and the twist angle per unit
1 1 4
length ' for a rectangular bar under torsion. Note: 1 + + + ... = .
34 54 96
-------------------------------------------------------------------------------
Ans. We have, [see Eq. (5.18.10)],
32 ' a 1 ( n 1) /2 cosh ( n x3 / 2a ) n x2
2
= 3 ( 1) 1 cos .

3
n =1,3,5 n cosh ( n b / 2a ) 2a
32 'a 2 cosh ( n x3 / 2a )
Thus, if let A = and F ( x3 ) = , then, we have,
3
cosh ( n b / 2a )
M t = 2 dA =

( n 1) /2 n x2
( n 1) /2 a n x2 b
F ( x3 )dx3 dx2
1 1
2 A 3 ( 1) ( 2b ) cos dx2 2 A 3 ( 1)
a
n =1,3,5 n a 2a n =1,3,5 n a
cos
2a b

M N.

n x2 2a n 4a
( 1)( n+3)/2 , n = 1,3,5 , therefore,
a
Now, a cos
2a
dx2 =
n
2sin =
2 n

n x 32 '
( n 1) /2
( 2b ) a cos 2 dx2 = A ( 2b ) 4 = 4 ( 2a )3 ( 2b ) 4
1 8a 1 1
M = 2 A 3 ( 1)
a

n =1,3,5 n 2a n =1,3,5 n n =1,3,5 n


.
b b cosh ( n x3 / 2a ) 2a 2sinh ( n b / 2a ) 4a n b
Next, b F ( x3 )dx3 = b cosh ( n b / 2a )
dx3 =
n
cosh ( n b / 2 a
=
)
n tanh
2a
, so that


n x2 b 64 ' ( 2a )4 1 n b
1 ( n 1) /2
N = 2 A 3 ( 1)
a
cos F ( x3 )dx3 dx2 = tanh
a
2a b
5 n =1,3,5 n 5 2a
n =1,3,5 n
Thus,
1 64 ' ( 2a ) 1
4
32 ' n b
Mt M N = 4
( 2a ) ( 2b ) 4
3
tanh
n =1,3,5 n 5 n =1,3,5 n 5 2a

4 64 ' ( 2a ) 1
4
32 ' n b
( ) ( ) 96
3
= 2 a 2b tanh .
4 5 n =1,3,5 n5 2a

Or,
' 192 a
n b
( 2a )3 ( 2b ) 1 5 5 tanh
1
Mt =
3 b n =1,3,5 n 2a
_________________________________________________________________

Copyright 2010, Elsevier Inc

5-28
Lai et al, Introduction to Continuum Mechanics

5.58 In pure bending of a bar, let M L = M 2e 2 + M 3e3 = M R , where e 2 and e3 are not along
the principal axes, show that the flexural stress T11 is given by
M 2 I 23 + M 3 I 22 M 2 I 33 + M 3 I 23
T11 = x2 +
(I ) (I )
x3
33 I 22 I 23 2
33 I 22 I 232
-------------------------------------------------------------------------------
Ans. Refer to Section 5.19, we had [see Eq.(5.19.4)(5.19.6) and (5.19.7)]
T11 = x2 + x3 , where M 2 = I 23 + I 22 , M 3 = I 33 I 23
Solving the above two equations for and , in terms of M 2 and M 3 , we obtain
M I + M 3 I 22 M I + M 3 I 23
= 2 23 and = 2 33
( ) ( )
.
I 33 I 22 I 23 2
I 33 I 22 I 23
2

M 2 I 23 + M 3 I 22 M 2 I 33 + M 3 I 23
Thus, T11 = x2 +
(I ) (I )
x3 .
33 I 22 I 232 33 I 22 I 232
_________________________________________________________________

5.59 From the strain components for pure bending


M x M 2 x3
E11 = 2 3 , E22 = E33 = , E12 = E13 = E23 = 0
I 22 EY I 22 EY
Obtain the displacement field
-------------------------------------------------------------------------------
M2
Ans. Integration of u1 / x1 = Ax3 , u2 / x2 = Ax3 , u3 / x3 = Ax3 , whereA gives
I 22 EY
u1 = Ax3 x1 + f1 ( x2 , x3 ) , u2 = Ax3 x2 + f 2 ( x1 , x3 ) , u3 = Ax32 / 2 + f3 ( x1 , x2 ) (i)
where f1 ( x2 , x3 ) , f 2 ( x1 , x3 ) and f3 ( x1 , x2 ) are integration functions. Substituting (i) into
u1 / x2 + u2 / x1 = 0, u1 / x3 + u3 / x1 = 0 and u2 / x3 + u3 / x2 = 0 , we obtain
f1 ( x2 , x3 ) / x2 = f 2 ( x1 , x3 ) / x1 = g1 ( x3 )
f1 ( x2 , x3 ) / x3 = f3 ( x1 , x2 ) / x1 = g 2 ( x2 ) (ii)
f 2 ( x1 , x3 ) / x3 = f3 ( x1 , x2 ) / x2 = g3 ( x1 )
where g ( x1 ) , g ( x2 ) , g ( x3 ) are integration functions. Integrations of (ii) give,
f1 = g1 ( x3 ) x2 + g 4 ( x3 ) and f1 = g 2 ( x2 ) x3 + g 6 ( x2 ) (iii)
f 2 = g1 ( x3 ) x1 + g5 ( x3 ), and f 2 = g3 ( x1 ) x3 + g8 ( x1 ) (iv)
f3 = g 2 ( x2 ) x1 + g 7 ( x2 ) and f3 = g3 ( x1 ) x2 + g9 ( x1 ) (v)
From (iii), g1 ( x3 ) = a1 x3 + b1 , g 2 ( x2 ) = a1 x2 + b2 , g 4 ( x3 ) = b2 x3 + c2 , g6 ( x2 ) = b1 x2 + c2 (vi)
From (iv) and (vi), g3 ( x1 ) = a1 x1 + b3 , g8 ( x1 ) = b1 x1 + c3 , g5 ( x3 ) = b3 x3 + c3 (vii)
From (v) (vi),(vii), a1 = 0, g9 ( x1 ) = b2 x1 + c4 , g 7 ( x2 ) = b3 x2 + c4 (viii)
Thus,
f1 = b1 x2 + b2 x3 + c2 , f 2 = b3 x3 b1 x1 + c3 , f3 = b2 x1 b3 x2 c4 (ix)

Copyright 2010, Elsevier Inc

5-29
Lai et al, Introduction to Continuum Mechanics

So that,
M2 M2
u1 = x3 x1 + b1 x2 + b2 x3 + c2 , u2 = x3 x2 + b3 x3 b1 x1 + c3 ,
I 22 EY I 22 EY
(x)
M2
u3 = b2 x1 b3 x2 c4 .
x32
2 I 22 EY
_________________________________________________________________

5.60 In pure bending of a bar, let M L = M 2e2 + M 3e3 = M R , where e2 and e3 are along the
principal axes, show that the neutral axis, (that is, the axis on the cross section where the
flexural stress T11 is zero) is, in general, not parallel to the couple vectors.
--------------------------------------------------------------------------------
M x M x
Ans. From Eq.(5.19.10), we have, T11 = 2 3 3 2 , thus the neutral axis is given by:
I 22 I 33
M 2 x3 M 3 x2 x I M
= 0 . That is, the neutral axis is given by 3 = 22 3 . Thus, only when
I 22 I 33 x2 I 33 M 2
I 22 = I 33 is the neutral axis parallel to the couple vector M L = M 2e2 + M 3e3 = M R .
__________________________________________________________________

5.61 For plane strain problem, derive the bi-harmonic equation for the Airy stress function
-------------------------------------------------------------------------------
Ans. We have [Eq.(5.20.7)}
1 2 1 2
E11 =
EY
(
1
2
) x 2
2
(1 + )
x12
, E22 =
EY
(
1
2
) x
2
2
(1 + ) ,
x22
2 1

1 2
E12 = (1 + ) , E13 = E23 = E33 = 0.
EY x1x2
4 4 2 E22 2 4
2 E11
( ) ( )
4
EY = 1 2 (1 + ) , EY = 1 (1 + ) ,
x22 x24 x22 x12 x12 x14 x12 x22
2 E12 2
2 EY = 2(1 + ) . Thus, the compatibility equation
x22 x12 x12 x22
2 E11 2 E22 2 E12
+ 2 = 0
x2 x22 x22 x12
2


( ) x + x + {2 (1 + ) 2 (1 + )} x x
4 4 4
1 2 4 4 2 2
= 0,
2 1 2 1

4 4 4 4 4 4
( x )
1 2 4 + 4 + 2 2 2 = 0 4 + 4 + 2 2 2 = 0
x2 x1 x x2 x1
2 x1 2 x1
__________________________________________________________________

Copyright 2010, Elsevier Inc

5-30
Lai et al, Introduction to Continuum Mechanics

5.62 For plane stress problem, derive the bi-harmonic equation for the Airy stress function
-------------------------------------------------------------------------------
Ans.
1 2

2
1 2 2 (1 + ) 2
From E11 = 2 2 , E22 = 2 2 , E12 = , [Eq. (5.22.3)]
EY x2 x1 EY x1 x2 EY x1x2
we get

2 E11 2 E22 4 4 4 2 E12 2 2


EY
x 2
+ = 4 + 4 2 2 2 , 2 EY = 2 2 2 2 2 2
2 x12 x1 x2 x1 x2 x1x2 x1 x2 x1 x2

2 E11 2 E22 2 E12


The compatibility equation + =2 then gives
x22 x12 x1x2
4 4 4 2 2 4 4 2
+ 2 = 2 2 =0
Thus, + +2
x14 x24 x12 x22 x12 x22 x12 x22 x14 x24 x12 x22
__________________________________________________________________

5.63 Consider the Airy stress function = 1 x12 + 2 x1 x2 + 3 x22 . (a) Verify that it satisfies
the bi-harmonic equation. (b) Determine the in-plane stresses T11 , T12 and T22 . (c) Determine
and sketch the tractions on the four rectangular boundaries x1 = 0, x1 = b, x2 = 0, x2 = c .(d) As
a plane strain solution, determine T13 , T23 , T33 and all the strain components . (e) As a plane
stress solution, determine T13 , T23 , T33 , and all the strain components .
-------------------------------------------------------------------------------
Ans. (a) / x14 = 0, 4 / x24 = 0, 4 / x12 x22 = 0 , thus = 1 x12 + 2 x1 x2 + 3 x22 satisfies
4

the bi-harmonic equation.


(b) T11 = 2 / x22 = 2 3, T12 = 2 / x1x2 = 2, T22 = 2 / x12 = 21
(c)
On x1 = 0, t = Te1 = (T11e1 + T21e2 ) = 2 3e1 + 2e2 , on x1 = b, t = Te1 = T11e1 + T21e2 = 2 3e1 2e2 ,
on x2 = 0, t = Te 2 = (T12e1 + T22e2 ) = 2e1 21e2 , on x1 = c, t = Te2 = T12e1 + T22e2 = 2e1 + 21e2 .

Copyright 2010, Elsevier Inc

5-31
Lai et al, Introduction to Continuum Mechanics

2 1

2 2
2 3 c 2 3
b
2 2

2 1
(d) As a plane strain solution,
T33 = (T11 + T22 ) = 2 ( 3 + 1 ) , T13 = T23 = 0, E13 = E23 = E33 = 0,
E11 = (1 / EY ) (1 2 )T11 (1 + ) T22 = 2 (1 / EY ) (1 2 ) 3 (1 + )1 ,

E22 = (1 / EY ) (1 2 )T22 (1 + ) T11 = 2 (1 / EY ) (1 2 )1 (1 + ) 3 ,

E12 = (1 / EY )(1 + ) T12 = (1 / EY )(1 + ) 2,
(e) As a plane stress solution,
T33 = T13 = T23 = 0, E13 = E23 = 0, E11 = (1 / EY )(T11 T22 ) = 2 (1 / EY ) ( 3 1 ) ,
E22 = (1 / EY )(T22 T11 ) = 2 (1 / EY ) (1 3 ) , E12 = (1 / EY )(1 + ) T12 = 2 (1 + ) / EY = 2 / 2 .
E33 = (1 / EY ) (T11 + T22 ) = 2 ( / EY ) ( 3 + 1 ) .

Note, for this problem, since T11 + T22 is a linear function of x1 and x2 , in fact, a constant,
therefore, all the compatibility equations are satisfied so that E33 is meaningful and u3 does exist.
_________________________________________________________________

5.64 Consider the Airy stress function = x12 x2 . (a) Verify that it satisfies the bi-harmonic
equation. (b) Determine the in-plane stresses T11 , T12 and T22 . (c) Determine and sketch the
tractions on the four rectangular boundaries x1 = 0, x1 = b, x2 = 0, x2 = c . (d) As a plane strain
solution, determine T13 , T23 , T33 and all the strain components. (e) As a plane stress solution,
determine T13 , T23 , T33 . and all the strain components .
-------------------------------------------------------------------------------
Ans. (a) / x14 = 0, 4 / x24 = 0, 4 / x12 x22 = 0 , thus = x12 x2 satisfies the bi-harmonic
4

equation.
(b) T11 = / x2 = 0, T12 = / x1x2 = 2 x1 , T22 = 2 / x12 = 2 x2
2 2 2

Copyright 2010, Elsevier Inc

5-32
Lai et al, Introduction to Continuum Mechanics

(c) On x1 = 0, t = Te1 = (T11e1 + T21e2 ) = 2 x1e2 = 0, On x1 = b, t = Te1 = T11e1 + T21e2 = 2 be2 .


On x2 = 0, t = Te2 = (T12e1 + T22e2 ) = 2 x1e1 2 x2e2 = 2 x1e1.
On x2 = c, t = Te 2 = T12e1 + T22e2 = 2 x1e1 + 2 x2e2 = 2 x1e1 + 2 ce2 .

x2
2 c

2 x1

c 2 b

o b x1
2 x1
(d) As a plane strain solution,
T33 = (T11 + T22 ) = 2 x2 , T13 = T23 = 0. E13 = E23 = E33 = 0,
E11 = (1 / EY ) (1 2 )T11 (1 + ) T22 = 2 (1 + ) / EY x2 ,

E22 = (1 / EY ) (1 2 )T22 (1 + ) T11 = 2 (1 2 ) / EY x2 ,

E12 = (1 / EY )(1 + ) T12 = 2 (1 + ) / EY x1.
(d) As a plane stress solution,
T33 = T13 = T23 = 0, E13 = E23 = 0, E33 = (1 / EY ) (T11 + T22 ) = 2 ( / EY ) x2 .
E11 = (1 / EY )(T11 T22 ) = ( 2 / EY ) x2 , E22 = (1 / EY )(T22 T11 ) = ( 2 / EY ) x2 .
E12 = (1 / EY )(1 + ) T12 = 2 (1 + ) / EY x1 = x1 / .
Note, for this problem, since T11 + T22 is a linear function of x1 and x2 , therefore, all the
compatibility equations are satisfied so that E33 is meaningful and u3 does exist.
_________________________________________________________________

5.65 ( )
Consider the Airy stress function = x14 x24 . (a) Verify that it satisfies the bi-
harmonic equation. (b) Determine the in-plane stresses T11 , T12 and T22 . (c) Determine and
sketch the tractions on the four rectangular boundaries x1 = 0, x1 = b, x2 = 0, x2 = c . (d) As a
plane strain solution, determine T13 , T23 , T33 and all the strain components . (e) As a plane
stress solution, determine T13 , T23 , T33 and all the strain components .
--------------------------------------------------------------------------------

Copyright 2010, Elsevier Inc

5-33
Lai et al, Introduction to Continuum Mechanics

(
Ans. (a) 4 / x14 = 24 , 4 / x24 = 24 , 4 / x12 x22 = 0 , thus = x14 x24 satisfies the )
bi-harmonic equation.
(b) T11 = 2
/ x22 = 12 x22 , T22 = 2 / x12 = 12 x12 , T12 = 2 / x1x2 = 0 .
(c) On x1 = 0, t = Te1 = (T11e1 + T21e2 ) = 12 x22e1 , On x1 = b, t = Te1 = T11e1 + T21e2 = 12 x22e1.
On x2 = 0, t = Te2 = (T12e1 + T22e2 ) = 12 x12e2 , On x2 = c, t = Te2 = T12e1 + T22e2 = 12 x12e2 .

x2
12 x1
2

12 x22 c 12 x22

o b
x1
12 x
2
1
(d) As a plane strain solution,
T33 = (T11 + T22 ) = 12 ( x12 x22 ), T 13 = T23 = 0, E13 = E23 = E33 = 0, E12 = (1 / EY )(1 + ) T12 = 0.

E11 = (1 / EY ) (1 2 )T11 (1 + ) T22 = 12 (1 / EY ) x22 (1 2 ) + (1 + ) x12 .



E22 = (1 / EY ) (1 )T22 (1 + ) T11 = 12 (1 / EY ) (1 ) x1 + x2 (1 + ) .
2 2 2 2

(d) As a plane stress solution,


T33 = T13 = T23 = 0, E13 = E23 = 0, E11 = (1 / EY )(T11 T22 ) = 12 (1 / EY ) x22 + x12 , ( )
( )
E22 = (1 / EY )(T22 T11 ) = 12 (1 / EY ) x12 + x22 , E12 = (1 / EY )(1 + ) T12 = 0.

.E33 = (1 / EY ) (T11 + T22 ) = 12 (1 / EY ) x22 x12 .( )


Since T11 + T22 is not a linear function of x1 and x2 , E33 is meaningless, because u3 does not exist.
__________________________________________________________________

5.66 Consider the Airy's stress function = x1 x22 + x1 x23 . (a) Verify that it satisfies the bi-
harmonic equation. (b) Determine the in-plane stresses T11 , T12 and T22 . (c) Determine the
condition necessary for the traction at x2 = c to vanish and (d) determine the tractions on the
remaining boundaries x1 = 0, x1 = b and x2 = 0 .
-------------------------------------------------------------------------------
Ans. (a) / x14 = 0, 4 / x24 = 0, 4 / x12 x22 = 0 , thus = x1 x22 + x1 x23 satisfies the bi-
4

harmonic equation.
(b) T11 = 2 / x22 = 2 x1 + 6 x1 x2 , T22 = 2 / x12 = 0, T12 = 2 / x1x2 = 2 x2 3x22 .

Copyright 2010, Elsevier Inc

5-34
Lai et al, Introduction to Continuum Mechanics

(c)On x2 = c, t = Te2 = T12e1 + T22e2 = (2 c 3c 2 )e1 , 2 c 3c 2 = 0 2 c = 3c 2 = 3c / 2


.
( )
(d) On x1 = 0, t = Te1 = (T11e1 + T21e2 ) = 2 x2 + 3 x22 e2 = 3x2 ( x2 c ) e2 .
On x1 = b, t = Te1 = T11e1 + T21e2 = 3b ( 2 x2 c ) e1 3 x2 ( x2 c ) e2 .
On x2 = 0, t = Te 2 = (T12e1 + T22e2 ) = 0.
__________________________________________________________________

5.67 Obtain the in-plane displacement components for the plane stress solution for the
cantilever beam from the following strain strain-displacement relations.
u1 Px1 x2 u Px1 x2 P h2
E11 = = , E22 = 2 = , E12 = x22 .
x1 EY I x2 EY I 4 I 4

-------------------------------------------------------------------------------
Ans.
u1 Px1 x2 Px x2
u2 Px1 x2 Px1 x22
= u1 = 1 2 + f1 ( x2 ) , = u2 = + f 2 ( x1 ) ,
x1 EY I 2 EY I x2 EY I 2 EY I
u1 u2 P h2 Px12 df Px22 df 2 P h2
+ = 2 x22 + 1 + = x22 ,
x2 x1 4 I 4 2 E I dx EY I dx1 2 I 4
Y 2
Px12 df 2 df Px22 P P h2
+ = 1 + x22 + .
2 EY I dx1 dx2 EY I 2 I 2 I 4
In the above equation, the left side is a function of x1 only, right side is a function of x2 only,
thus both sides must equal to the same constant, say c1 . That is,
Px12 df 2 df Px12 Px13
+ = c1 2 = c1 f 2 = c1 x1 + c2 .
2 EY I dx1 dx1 2 EY I 6 EY I
df1 Px22 P P h2 Px23 x23 P P h2
= x22 +
1 c f = + x2 c1 x2 + c3
2 I 2 I 3 2 I 2 I 4
1
dx2 EY I 4 6 EY I

Thus,
Px12 x2
Px23 P
Px23
2
h
u1 = + + x2 c1 x2 + c3 ,
2 EY I 6 EY I 6 I 2 I 2
Px1 x22
Px13
u2 =
+ c1 x1 + c2 .
2 EY I 6 EY I
_________________________________________________________________

m x1
5.68 (a) Let the Airy stress function be of the form = f ( x2 )cos . Show that the most
l
general form of

Copyright 2010, Elsevier Inc

5-35
Lai et al, Introduction to Continuum Mechanics

f ( x2 ) is f ( x2 ) = C1 cosh m x2 + C2 sinh m x2 + C3 x2 cosh m x2 + C4 x2 sinh m x2 . (b) Is the


m x1
answer the same if = f ( x2 )sin ?
l
-------------------------------------------------------------------------------
Ans. (a) The function ( x1 , x2 ) must satisfy the bi-harmonic equation. Now,
m x1 m x1
2 4
2 m 4 m
= f ( x2 ) cos , = f ( x2 )cos ,
x1
2
l l x1 l
4 l
m x1 m x1
2
4 m d f
2
4 d 4 f
= cos , = 4 cos ,
x1 x2
2 2
l dx2 2 l x2 4
dx2 l
Thus,
m x1 m m d f d f
4 2 2
4 4 4 4
= 4 + 2 2 2 + 4 = cos
4
f ( x2 ) 2 + =0.
x1 x1 x2 x2 l l l dx22 dx24
d4 f 2
2 d f m
Therefore, 4
2 m 2
+ m4 f = 0, where m .
dx2 dx2 l
The characteristic equation for the above ODE is D 4 2m2 D 2 + m4 = 0 . The roots of this
equation consists of two sets of double roots. They are: D = m , m , m , m . Thus,
f ( x2 ) = C1 cosh m x2 + C2 sinh m x2 + C3 x2 cosh m x2 + C4 x2 sinh m x2 .

(b) Yes, the same


__________________________________________________________________

5.69 Consider a rectangular bar defined by l x1 l, c x2 c, b x3 b , where


b / l is very small. At the boundaries x2 = c , the bar is acted on by equal and opposite
cosine normal stress Am cos m x1 , where m = m / l (per unit length in x3 direction). (a)
Obtain the in-plane stresses inside the bar. (b) Find the surface tractions at x1 = l . Under
what conditions can these surface tractions be removed without affecting T22 and T12 (except
near x1 = l )? How would T11 be affected by the removal. Hint: Assume
= f ( x2 ) cos m x1 , where m = m / l and use the results of the previous problem
-------------------------------------------------------------------------------
Ans. (a) Boundary conditions are
(T12 ) x = c = 0 ,
2
(T22 ) x = c = Am cos m x1
2

Let = f ( x2 ) cos m x1 , where m = m / l . Then (see previous problem) ,


f ( x2 ) = C1 cosh m x2 + C2 sinh m x2 + C3 x2 cosh m x2 + C4 x2 sinh m x2 .
The in-plane stresses are:

Copyright 2010, Elsevier Inc

5-36
Lai et al, Introduction to Continuum Mechanics

2 2
T22 =
x12
= ( m ) f ( x2 ) cos m x1 , T11 =
2

x22
( )
= d 2 f / dx22 cos m x1 ,


T12 = = m ( df / dx2 ) sin m x1 . Now, applying the boundary condition:
x1 x2
(T22 ) x2 = c = Am cos m x1 ( m )2 f ( c ) cos m x1 = Am cos m x1 f ( c ) = Am / ( m )2 .
From f ( c ) = Am / ( m ) , f ( +c ) = f ( c ) , so that C2 = C3 = 0 and
2

f ( c ) = C1 cosh m c + C4 c sinh m c = Am / m2 (i)


Applying the other boundary condition:
(T12 ) x = c = 0 ( df / dx2 ) x = c = 0
2 2

C1m sinh m c + C4 ( sinh m c + m c cosh m c ) = 0 (ii)


(i) and (ii) give
2 A ( c ) cosh m c + sinh m c 2 Am m sinh m c
C1 = 2m m , C4 = 2 .
m sinh 2m c + 2m c m sinh 2m c + 2m c
With
f ( x2 ) = C1 cosh m x2 + C4 x2 sinh m x2 , we have ,
T22 = ( m ) f ( x2 ) cos m x1 = ( m ) {C1 cosh m x2 + C4 m x2 sinh m x2 } cos m x1
2 2

{( m c ) cosh m c + sinh m c} cosh m x2 {m x2 sinh m x2 sinh m c} cos m x1


= 2 Am .
sinh 2m c + 2m c

T12 = m ( df / dx2 ) sin m x1 = C1m2 sinh m x2 + C4 m ( sinh m x2 + m x2 cosh m x2 ) sin m x1



{ ( m c ) cosh m c} sinh m x2 + sinh m c ( m x2 cosh m x2 )
= 2 Am sin m x1.
sinh 2m c + 2m c
2
T11 =
x22
( )
= d 2 f / dx22 cos m x1 =

( m c ) cosh m c cosh m x2 + sinh m c ( m x2 sinh m x2 + cosh m x2 )


2 Am cos m x1.
sinh 2m c + 2m c

(b) Surface tractions at x1 = l are:


T12 ( l, x2 ) = []sin m = 0 .
( m c ) cosh m c cosh m x2 + sinh m c ( m x2 sinh m x2 + cosh m x2 )
T11 ( l, x2 ) = 2 Am cos m
sinh 2m c + 2m c
At x1 = l , T11 is an even function of x2 , which gives rise to equal and opposite resultant force of
magnitude FR at the two ends. Removal of these resultants will have little effects on T12 and T22 ,

Copyright 2010, Elsevier Inc

5-37
Lai et al, Introduction to Continuum Mechanics

if l / c is very large. However, T11 will need to be modified by subtracting the normal stress ( FR
/Area) caused by the resultant forces.
_________________________________________________________________

5.70 Verify that the equations of equilibrium in polar coordinates are satisfied by
1 1 2 2 1
Trr = + 2 , T = , Tr = .
r r r 2
r 2 r r
--------------------------------------------------------------------------------
Ans.

1 (rTrr ) 1 1 2
1 1 3
2
1 2 T 1 2
= + = + , =
r r r r r r 2 r r 2 r 2 r 2 r 3 2 r r r 2

1 Tr 1 1 1 1 2 1 1 3 1 2
= = = 2 2
3
r r r r r r r r 2 r r r
2

Thus, [See Eq.(4.8.1),


1 (rTrr ) 1 Tr T
+
r r r r
1 2 1 3 1 2 1 3 1 2 1 2
= + =0
r r 2 r 2 r 2 r 3 2 r 2 r 2 r 3 2 r r 2

Similarly,
1 (r T r )
2
1 2 1 1 2 1 3
= r = r =
r2 r r 2 r r r r 2 r r r r 2

1 T 1 2 1 3
= = ,
r r r 2 r r 2
1 (r 2T r ) 1 T
Thus, [See Eq.(4.8.2)] + = 0.
r2 r r
__________________________________________________________________

T Tr cos sin T11 T12 cos sin


From the transformation law : rr =
T sin cos T21 T22 sin cos
5.71
T r
and

2

2
2 1 1 2
T11 = , T22 = and T12 = , obtain T rr = +
x22 x12 x1x2 r r r 2 2
-------------------------------------------------------------------------------
Ans.

Copyright 2010, Elsevier Inc

5-38
Lai et al, Introduction to Continuum Mechanics

Trr Tr cos sin T11 T12 cos sin


T =
r T sin cos T21 T22 sin cos
T cos 2 + 2T sin cos + T sin 2 (T22 T11 ) sin cos + T12 ( cos2 sin 2 )
=
11 12 22
,

(
(T22 T11 ) cos sin + T12 cos sin
2 2
) ( T11 sin 2 + T22 cos 2 2T12 sin cos )

r 2 = x12 + x22 r / x1 = x1 / r = cos , r / x2 = x2 / r = sin ,
x2 x sin x cos
= tan 1 / x1 = 2 2 2 = , / x2 = 2 1 2 = ,
x1 x1 + x2 r x1 + x2 r
r 1
= + = sin + cos ,
x2 r x2 x2 r r
2 1 1 cos
T11 = = sin + cos sin + sin + cos
x22 r r r r r r
2 1 2 1 2 1 2 1 cos
= sin 2
+ cos sin

+
sin + cos + cos sin
r 2 r r r r r r 2 r
2
r
2 cos 2 cos 2 2 2cos sin 2 2cos sin
= sin 2 + + + ,
r 2 r r r 2 2 r r r2

r sin
= + = cos ,
x1 r x1 x1 r r
2 sin sin sin
T22 = = cos cos cos
x12 r r r r r r
2 sin 2 sin 2 2 2sin cos 2 2sin cos
= 2 cos 2 + + 2 + .
r r 2 r
r r r r2
sin sin cos
T12 = = cos sin + cos
x2 x1 r r r r r r
2 sin cos 2 sin 2 cos 2 cos 2 2 sin 2 2 sin cos
= cos sin + + .
r 2 r2 2 r 2 r 2 r r r r r r

Thus,

Copyright 2010, Elsevier Inc

5-39
Lai et al, Introduction to Continuum Mechanics

Trr = T11 cos 2 + 2T12 sin cos + T22 sin 2 =


2 2 cos 2 cos 2 2 2cos sin 2 2cos sin 2
2 sin + + + cos
r r r r 2 2 r r r2
2 sin cos 2 sin 2 cos 2 cos 2 2
2 cos sin + 2 +
r r 2
2
r r 2 r r
2 sin cos
sin 2 2 sin cos

r r r r
2 sin 2 sin 2 2 2sin cos 2 2sin cos
+ sin 2 2 cos 2 + + 2 + .
r r r 2 r r r2
r
2 2 2 1
(
Trr = sin 2 cos 2 2 2 2 + 2 + cos 4 + sin 4 + 2sin 2 cos 2
r r r
)
r r

( ) r1 r2 (sin cos sin cos + cos sin sin cos )


2
+ cos 4 + sin 4 + 2sin 2 cos 2 2 2 2
3 3 3 3

2 2
( sin cos3 cos3 sin + sin 3 cos sin 3 cos ) r r
.

That is,
1 1 2
Trr = + .
r r r 2 2
__________________________________________________________________

5.72
Obtain the displacement field for the plane strain solution of the axis-symmetric stress
distribution from that for the plane stress solution obtained in Section 5.28.
-------------------------------------------------------------------------------
Ans. From Section 5. 29, we have, for plane stress solution, [See Eq.(5.29.15) and .(5.29.16) and
note EY = 2 (1 + ) ]
A
(1 + ) + 2 B (1 ) r ln r (1 + ) Br + 2C (1 ) r + H sin + G cos ,
1
ur =
2 (1 + ) r
2 Br
u = + H cos G sin + Fr .
(1 + )
To obtain the corresponding displacement field for the plane strain solution, we replace the
Poisson ratio with / (1 ) in the above equation [see Section. 5.26]. That is,
1 2
(1 + ) 1 + (1 ) 1
1
= =
1 1
, .
1 1
Thus, for plane strain:

Copyright 2010, Elsevier Inc

5-40
Lai et al, Introduction to Continuum Mechanics

(1 )
A 1 (1 2 ) 1 (1 2 )
ur = + 2B r ln r B r + 2C r + H sin + G cos
2 r (1 ) (1 ) (1 ) (1 )
1 A
= + 2 B (1 2 ) r ln r Br + 2C (1 2 ) r + H sin + G cos
2 r
(1 + ) A
=
EY r + 2 B (1 2 ) r ln r Br + 2C (1 2 ) r + H sin + G cos .
and
2 Br (1 ) 4 Br (1 )(1 + )
u = + H cos G sin + Fr = + H cos G sin + Fr .
EY

_________________________________________________________________

5.73 Let the Airy stress function be = f (r )sin n , find the differential equation for f (r ) .
Is this the same ODE for f (r ) if = f (r )cos n ?
---------------------------------------------------------------------------------
Ans.

2
2
= f (r )sin n = f 'sin n 2 = f ''sin n ; = nf cos n 2 = n 2 f sin n .
r r
Thus,
1 1 2 2 1 1 2 2 f' 1 2
+ 2 + = + + f (r )sin n = 2 n f + f '' sin n g ( r ) sin n
r r r 2 r 2 r r r 2 2 r 2 r r
,
d 1 d n2
2
f' 1
where g ( r ) 2 n 2 f + f '' = 2 + f .
r r dr r dr r 2
Now,
1 1 2
1 1 2 1
2 2
1 2 2
+ 2 + + + = + + g ( r ) sin n
r r r
2
r 2
r r r
2 2
r 2 r r r 2 2 r 2
d 2 1 d n2 d 2 1 d n 2 d 2 1 d n 2
= 2 + 2 g ( r ) sin n = 2 + 2 + f ( r ) sin n = 0.
dr r dr r dr r dr r 2 r dr r 2
dr
Therefore,
d 2
1 d n d 2 1 d n 2
2
2 + 2 + f (r ) = 0 .
dr r dr r 2 dr r dr r 2
The same equation will be obtained if = f (r )cos n
_________________________________________________________________

5.74 Obtain the four independent solutions for the following equation
d 2 1 d n 2 d 2 f 1 df n 2
2 + 2 + f =0
dr r dr r
dr r dr r

Copyright 2010, Elsevier Inc

5-41
Lai et al, Introduction to Continuum Mechanics

---------------------------------------------------------------------------------
Ans. Let f = r m .
d 2 f 1 df n 2
2 +
r dr (
2 f = m ( m 1) + m n 2 r m 2 = m 2 n 2 r m 2 )
dr r
d 2 1 d n 2 d 2 f 1 df n 2
2 + 2 2 +

r dr r dr r dr r ( )
2 f = m2 n 2 ( m 3)( m 2 ) + ( m 2 ) n 2 r m 4 = 0

dr
( )
m 2 n 2 ( m 2 ) n 2 = 0.
2

Thus, m1 = + n, m2 = n, m3 = 2 + n, m4 = 2 n .
For n 0 and n 1 , the four independent solutions for f are: r + n , r n , r + n + 2 and r n+ 2 .
For n = 0 , m1 = m2 = 0, m3 = m4 = 2 . Two independent solutions for f are given by C and r 2 .
Additional solutions are given by
d n
dn r
n 0
= r n ln r
n = 0
( ) d
dn

= ln r , and r n + 2
n 0
= r n + 2 ln r
n 0
= r 2 ln r .

The four independent solutions are: C , r 2 ,ln r and r 2 ln r .


d
dn

For n = 1 , m1 = m4 = 1 , in addition to r , r 1 , r 3 , we have, r n
n1
= r n ln r ( ) n =1
= r ln r

Thus, the four independent solutions are: r , r 1 , r 3and r ln r .


__________________________________________________________________

5.75
d n
Evaluate
dn
(
r cos n , ) d n
n =0 dn
(
r sin n ,

)
n =0

(
d n+2
dn r


)
cos n and
n =1
d n
dn
(
r cos n )
n =1
--------------------------------------------------------------------------------
Ans.
d n
(
dn r cos n


) ( )
= r n ln r cos n r n sin n
n =0 n =0
= ln r.

d n
(
dn r sin n

) ( )
= r n ln r sin n + r n cos n
n =0 n =0
= .

dn r

(
d n+ 2
)
cos n = r n + 2 ln r cos n r n + 2 sin n = r ln r cos r sin
n =1 n =1
d n

(
n =1
)
n n
dn r cos n = r ln r cos n r sin n n =1 = r ln r cos r sin
________________________________________________________________

Copyright 2010, Elsevier Inc

5-42
Lai et al, Introduction to Continuum Mechanics

5.76 In the Flamont Problem (Sect. 5.37), if the concentrated line load F , acting at the origin
on the surface of a 2D half-space (defined by / 2 / 2 ), is tangent to the surface
2 F sin
and in the direction of = 90o , show that: Trr = , T = Tr = 0
r

.
-------------------------------------------------------------------------------
Ans. The boundary conditions are: T = Tr = 0 at = / 2, r 0 . (i),
/2 /2
(Trr cos Tr sin ) rd = 0 (ii), (Trr sin + Tr cos ) rd = F . (iii)
/2 /2
From the stress field obtained in Sect. 5.37,
Trr = r 1 ( 2 B5 cos 2 B5 sin ) , T = 0, Tr = 0 , (iv)
we obtain, from Eqs.(ii) and (iv) :
/2 /2
( 2B cos B sin 2 ) d = 0 2B
5
2
5 5 cos 2 d = 0 B5 = 0 .
/2 /2
From Eqs.(iii) and (iv)
/2 /2

( )
B5 sin 2 2 B5 sin 2 d = F 2 B5 sin 2 d = F 2 B5
2
= F 2 B5 =
2F

/2 /2
Thus
sin 2 F sin
Trr = 2 B5 = , T = 0, Tr = 0 .
r
(v)
r
_________________________________________________________________

5.77 Verify that the displacement field for the Flamont Problem under a normal force P is
given by

ur =
P
EY
{(1 ) sin + 2ln r cos } , u =
P
EY
{(1 + ) sin + 2ln r sin (1 ) cos } ,
The 2D half space is defined by / 2 / 2 .
-------------------------------------------------------------------------------
Ans. From the given displacement field, we have,
2 P cos Trr
Err = ur / r = , i.e., Err = .
EY r EY
u

+ ur =
P
EY
{(1 + ) cos + 2ln r cos (1 ) cos + (1 ) sin }

P
EY
{(1 ) sin + 2ln r cos } = PE {(1 + ) cos (1 ) cos } = 2PEcos .
Y Y

Copyright 2010, Elsevier Inc

5-43
Lai et al, Introduction to Continuum Mechanics

1 u 2 P cos T
That is, E = + ur = , i.e., E = rr .
r EY r EY
Next,
1 u u P {(1 ) sin + (1 + ) sin } 2 P sin
2 Er = r u + = +
r r EY r EY r
2 P sin 2 P sin
= + = 0, i.e., 2 Er = 0 = Tr .
EY r EY r
_________________________________________________________________

1
5.78 Show that Eq. (5.38.6), i.e., u = (x + )
4(1 )
can also be written as:
2 (1 ) 2 (1 )
2 u = 4 (1 ) + ( x + ) where = , =

-------------------------------------------------------------------------------
2 (1 ) 2 (1 ) 2 (1 )
Ans. With = , = , we have, x = x .

1 2 (1 ) 1
u = (x + ) = + ( x + ) .
4(1 ) 2
That is,

2 u = 4 (1 ) + ( x + ) .
________________________________________________________________________

5.79 Show that with


1
ui = i ( xn n + ) , the Navier Equations become :
4(1 ) xi
2 n 2
xn (1 4 ) 2 i + + Bi = 0
2 (1 2 ) xi xi
-------------------------------------------------------------------------------
1 1 n
Ans. ui = i ( xn n + ) = i xn + i +
4(1 ) xi 4(1 ) xi xi

Copyright 2010, Elsevier Inc

5-44
Lai et al, Introduction to Continuum Mechanics

ui m 1 2n m 2
e= = xn +2 +
xi xm 4(1 ) xm xm xm xm xm
e m 1 2 n m 2
= xn +2 +
xi xi xm 4(1 ) xi xm xm xi xm xi xm xm

m (1 2 ) 1 2 n 2
= xn + 2i +
xi xm 2(1 ) 4(1 ) xi xm xm xi

e m 2 n 2
= xn + 2i +
1 2 xi 2(1 ) xi xm 4 (1 2 ) (1 ) xi xm xm xi

Also,
ui
2
2 j 2
= 2 i xn n + 2 + 2i +
x j x j 4(1 ) xi x j xi xi

2 j 2 2
= xn n + 2 i [1 4(1 )] +
2(1 ) x j xi 4(1 ) xi xi
Thus,
2ui e 2 2
+ = ( 2 2 ) xn n 2 (1 4 )(1 ) 2 i + ( 2 2 )
x j x j 1 2 xi 4(1 ) (1 2 ) xi xi
2 2
= xn n (1 4 ) 2 i +
2 (1 2 ) xi xi

i.e.,
ui 2
e 2 2
+ = xn n (1 4 ) 2 i + ,
x j x j 1 2 xi 2 (1 2 ) xi xi
so that the Navier Equations become:
2 n 2
xn (1 4 ) 2 i + + Bi = 0 .
2 (1 2 ) xi xi
_________________________________________________________________

5.80 Consider the potential function given in Eq. (5.38.32) [See Example 5.38.5], i.e.,
= ( R ) eR , = ( R ) ,
where
2 dd 2 d 2 2 d 2
2 = = + + = 0.
dR 2 R dR R 2
0 and
dR 2 R dR
Show that these functions generate the following displacements, dilatation and stresses as given
in Eq.(5.38.5) to (5.38.38):

Copyright 2010, Elsevier Inc

5-45
Lai et al, Introduction to Continuum Mechanics

d d
(a) Displacements: 2 u R = R + (3 + 4 ) + , u = u = 0
dR dR

(b) Dilation: e =
(1 2 ) d + 2

dR R
d d 2 d 3 1 d
(c) Stresses: TRR = ( 2 4 ) + ( 2 4 ) + 2 , T = T = ( 2 1) +
dR R dR dR R R dR
-------------------------------------------------------------------------------
Ans. With x = Re R , we have, x = R , thus 2 u = 4 (1 ) + ( x + )
d d
( R + ) eR = ( 3 + 4 ) eR + R + eR ,
d
2 u = 4 (1 ) e R +
dR dR dR
d d
i.e., 2 u R = ( 3 + 4 ) + R + ,
dR dR
(b) The non zero strain components are:
u d d 2 d d 2 d d 2 d 2
2 ERR = 2 R = ( 3 + 4 ) +R 2 + + 2 = ( 2 + 4 ) +R 2 + 2 .
R dR dR dR dR dR dR dR
d 2 d 2
2
d 2 2 d
2
But 2
+ 2 =0 =
dR R dR R dR 2 R 2 R dR
d d 2 d 2 2 d 2
2 ERR = ( 2 + 4 ) + R 2 + 2 = 4 (1 ) + + .
dR dR dR R R dR 2
2uR d 1 d
2 E = 2 E = = ( 3 + 4 ) + + .
R R dR R dR
Therefore,
d 2 d 2
d 1 d
2 e = 2 e = 4 (1 ) + + 2 + 2 ( 3 + 4 ) + 2 +2
dR R dR R dR R dR
d 2 d 2 2 d d 2
= ( 2 + 4 ) + ( 2 + 4 ) + 2 + = 2 (1 2 ) + .
dR R dR R dR dR R

e=
(1 2 ) d + 2 .
dR R
(c) the stresses are:
2 d 2 d 2 d 2
TRR = e + 2 ERR = 2 + 4 (1 ) + +
1 2 dR R dR R dR 2
d d 2
= ( 2 4 ) + ( 2 4 ) + 2 .
dR R dR
2 d 2 d 1 d
T = T = e + 2 E = 2 + + ( 3 + 4 ) + +
1 2 dR R R dR R dR
d 3 1 d
= (1 2 ) + .
dR R R dR
________________________________________________________________________

Copyright 2010, Elsevier Inc

5-46
Lai et al, Introduction to Continuum Mechanics

5.81 Consider the following potential functions for axis-symmetric problems:


= 0, = ( r , z ) = ( R, ) , 2 = 2 = 0 ,
where (r , , z ) and ( R, , ) are cylindrical and spherical coordinates respectively with z as the
axis of symmetry, the longitudinal angle and the angle between z axis and e R (the azimuthal
angle). Shows that these functions generate the following displacements, dilatation and stresses:

Cylindrical coordinates

(a) Displacements: 2 ur = , u = 0, 2 u z =
r z
(b) Dilation: e = 0
2 2 2 1
(c) Trr = e + 2 Err = 2 , T = e + 2 E =
1 2 r 1 2 r r
2
2
2
Tzz = e + 2 E zz = 2 , Er = 0, E z = 0, Trz = 2 Erz =
1 2 z r z
Spherical coordinates:
1
(d) Displacements: 2 u R = , u = 0, 2 u =
R R
(e) Dilation: e = 0

(f) Stresses:
2
2
2 1 2 1
TRR = e + 2 ERR = 2 , T = e + 2 E = 2 +
1 2 R 1 2 R 2 R R
2 1 cot
T = e + 2 E = + 2 , TR = 0, T = 0
1 2 R R R
1 2 1
TR = 2 ER = 2
R R R
-------------------------------------------------------------------------------
Ans. With x = rer + zez = ReR , = 0, = ( r , z ) = ( R, ) we have,[see Eqs.(2.34.4) and
(2.35.15)]
1
2 u = 4 (1 ) + ( x + ) 2 u = er + ez = eR + e
r z R R
That is, in cylindrical coordinates

2 ur = , u = 0, 2 u z =
r z
and in spherical coordinates;
1
2 uR = , u = 0, 2 u =
R R
(b) The non zero strain components are:

In cylindrical coordinates: [See Eqs.(3.7.20)]

Copyright 2010, Elsevier Inc

5-47
Lai et al, Introduction to Continuum Mechanics

ur 2 2 ur 1 u 2
2 Err = 2 = 2 , 2 E = = , 2 Ezz = 2 z = 2
r r r r r z z
u u 2
Er = 0, E z = 0, 2 Erz = r + z =
z r r z
2 1 2
2 e = 2 ( Err + E + E zz ) = 2 + + 2 = 2 = 0
r r r z
In spherical coordinates: [see Eqs.(3.7.21)]
u 2 2 u 2 u R 1 2 1
2 ERR = 2 R = 2 , 2 E = + = 2 +
R R R R R 2 R R
2 u R 2 u cot 1 cot
2 E = + = + 2 , ER = 0, E = 0
R R R R R
2 1 u R u u 1 1 2 1 1 2 1
2 ER = + = +
2 R R R 2 R R R 2 R R R 2

1 2 2 2 1 2 1
= 2 = 2
2 R R R R R R
2 1 2 1 1 cot
( )
2 e = 2 ERR + E + E = 2 + 2
R
+
R 2 R R R R
+ + 2
R

2
1 2 cot
2
+ + + 2 = 2 = 0 [see Eq.2.35.37)]
R 2
R
2 2 R R R

(c) the stresses are:

In cylindrical coordinates:
2 2 2 1
Trr = e + 2 Err = 2 , T = e + 2 E =
1 2 r 1 2 r r
2
2
2
Tzz = e + 2 E zz = 2 , Er = 0, E z = 0, Trz = 2 Erz =
1 2 z r z

In spherical coordinates:
2 2 2 1 2 1
TRR = e + 2 ERR = 2 , T = e + 2 E = 2 +
1 2 R 1 2 R 2 R R
2 1 cot
T = e + 2 E = + 2 , TR = 0, T = 0
1 2 R R R
1 2 1
TR = 2 ER = 2
R R R
These are the formulas given in Example 5.38.6.
__________________________________________________________________

Copyright 2010, Elsevier Inc

5-48
Lai et al, Introduction to Continuum Mechanics

5.82 For the potential functions given in Eq.(5.38.46), [see Example 5.38.7)], i.e., :
= ( R, ) ez , = 0 , where 2 = 0 ,
shows that these functions generate the following displacements ui , dilatation e and the stresses
Tij (in spherical coordinates) as given in Eq.(5.38.47) to (5.38.50)::
(a) Displacements:

2 u R = (3 4 ) R cos , 2 u = (3 4 ) sin + cos , u = 0 .
R
sin
(b) Dilation: 2 e = ( 2 4 ) cos .
R R
(c) Stresses
2 sin
2
TRR = 2(1 ) cos R cos ,
R 2 R
R
sin cos 2
T = ( 2 1) cos (2 2 ) ,
R R R 2

1
T = ( 2 1) cos ( 2 1) sin + ,
R sin R
1 2
TR = 2(1 ) cos cos sin (1 2 ) . TR = T = 0
R R R
-------------------------------------------------------------------------------
Ans (a) with x = R eR , eR ez = cos and
x = R eR e z = R cos , e z = cos eR sin e ,
2 u = 4 (1 ) + ( x + ) = 4 (1 ) e z + ( R cos )
1
(
2 u = 4 (1 ) cos e R sin e + ) R
( R cos ) eR +
R
( R cos ) e

( )
= 4 (1 ) cos e R sin e + cos eR + R cos
R
eR + cos

e sin e


= cos ( 3 4 ) R eR + ( 3 4 ) sin + cos

e .
R
(b) The strain components are:
u 2 2
2 ERR = 2 R = (3 4 ) R 2 cos = (2 4 ) R cos
R R R R R R 2
1 u u R 1 1
2 E = 2 + = (3 4 ) sin + cos (3 4 ) R cos
R R R R R
sin cos 2 1
= (2 4 ) + (3 4 ) cos + (3 4 ) R cos
R R R R
2 R

Copyright 2010, Elsevier Inc

5-49
Lai et al, Introduction to Continuum Mechanics

sin cos 2
= (2 4 ) + + cos .
R R 2 R
u u cot 1 cot
2 E = 2 R + = (3 4 ) R cos + (3 4 ) sin + cos
R R R R R

cot
= + cos .
R R
1 u R u u cos 2
(
2 2 ER ) = 2 + = (3 4 ) cos + (3 4 ) sin
R R R R R R R

sin cos 2
(3 4 ) + +
(3 4 )sin + cos
R R R R
cos 2
= 4(1 ) + 2cos + 2(1 2 )sin .
R R R
cos 2
i.e., 2 ER = 2(1 ) + cos + (1 2 )sin .
R R R
2 cot
(
2 ERR + E + E ) = (2 4 )
R
R 2 cos + +
R
cos
R R
sin cos 2
+(2 4 ) + + cos
R R 2 R
2 cos 2 1
= 4 + R 2 cos + (
+ cos 2 + (2 4 )sin 2
R sin
)
R R R 2

2 1 2 cot sin
= R cos 2 + 2 + 2 + 4 cos + (2 4 )
R R 2
R R R

2 sin sin
= R cos + 4 cos + (2 4 ) = ( 2 4 ) cos .
R R R R R R
2 1 2 2 cot
where we have used, the relation: + + + 2 = 0.
R 2 R 2 2 R R R
sin
Thus, 2 e = ( 2 4 ) cos .
R R
(c) The stresses are:
2 sin
with e = 2 cos , we have,
1 2 R R

Copyright 2010, Elsevier Inc

5-50
Lai et al, Introduction to Continuum Mechanics

2 sin 2
TRR = e + 2 ERR = 2 cos (2 4 ) R cos
1 2 R R R R 2
2 sin 2
= (2 + 2 ) cos + + R cos 2 .
R R R
Similarly,
2 sin cos 2
T = e + 2 E = ( 2 1) cos + (2 2 ) + .
1 2 R R R 2
2 1
T = e + 2 E = ( 2 1) cos + ( 2 1) sin + .
1 2 R sin R
cos 2
TR = 2 ER = 2(1 ) + cos + sin (1 2 ) . TR = T = 0
R R R
_________________________________________________________________

5.83 Show that (1 / R ) is a harmonic function (i.e., it satisfies the Laplace


Equation 2 (1 / R ) = 0 ), where R is the radial distance from the origin.
-------------------------------------------------------------------------------
R x1 R x2 R x3
Ans (a) R 2 = x12 + x22 + x32 , therefore, = , = , = so that
x1 R x2 R x3 R
1 1 R x1 2 1 1 3x1 x1 1 3x12
= = and = + = + .
x1 R R 2 x1 R3 x12 R R3 R 4 R R3 R5
2 1 1 3 x22 2 1 1 3 x32
Similarly, = + , = +
x22 R R3 R5 x32 R R3 R5
Thus,
1 1 1 3 x12 1 3 x22 1 3x32
2 2 2 2
= + + = + + + + +
x j x j R x12 x22 x32 R R3 R5 R3 R 5 R 3 R5
3 3( x12 + x22 + x32 ) 3 3
= 3
+ 5
= 3
+ = 0.
R R R R3
________________________________________________________________________

5.84 In Kelvin's problem, we used the potential function = ez where in cylindrical


A
coordinates: = , R 2 = r 2 + z 2 .Using the results in Example 5.38.6, obtain the stresses.
R
-------------------------------------------------------------------------------
1 z 2 3 z 2 1
Ans. = , = 3, = 5 3,
R z R z 2
R R
r 3r
2 2
1
2
r 3rz
= 3 2 = 5 3, = 3 = 5 ,
r R r R R zr z R R

Copyright 2010, Elsevier Inc

5-51
Lai et al, Introduction to Continuum Mechanics

2 z 3r z z
2
z 3r 2 z
Trr = 2 + z 2 = 2 3 + 5 3 = (1 2 ) 3 + 5 .
z r R R R R R
z z z r z
T = 2 + = 2 3 + 3 = (1 2 ) 3 .
z r r R r R R
r 3rz
2 2
r 3rz
Trz = (1 2 ) +z = (1 2 ) 3 + z 5 = (1 2 ) 3 + 5 .
r r z R R R R
2 3z 2 1 z 3z 3 z
Tzz = z 2 2(1 ) = z 5 3 + 2(1 ) 3 = 5 + (1 2 ) 3 .
z z R R
R R R
_________________________________________________________________

5.85 Show that for = C ln( R + z ), R 2 = r 2 + z 2 ,


2 z 1
=C 3 .
r2
R R( R + z )
-------------------------------------------------------------------------------
C r 1 C
Ans. = , = .
r ( R + z ) R r r R ( R + z )
2 r 1 1 r C r2 r2
= C + = + 1
r 2 R r ( R + z ) ( R + z ) r R R( R + z ) R ( R + z ) R
2

C r2 r 2 R ( R + z ) C R 2 r 2 + zR r 2
= 2+ = 2
R ( R + z ) R ( R + z ) R R ( R + z ) R ( R + z ) R ( R + z ) R
C z R 2 z 2 C Rz + z 2 R 2 z 1
= = = C .
R ( R + z ) R R 2 R ( R + z ) R2 R
3 R( R + z )
______________________________________________________________________________________

5.86 Given the following potential functions:


= ( / z )ez , = (1 ) , where = C ln( R + z ), R 2 = r 2 + z 2 .
From the results of Example 5.38.4, and Eqs (i), (ii) (iii) of Section 5.40, obtain
{ }
Trr = C 3r 2 z / R5 (1 2 ) / [ R( R + z )] , { }
T = C (1 2 ) z / R3 + 1 / [ R( R + z )] .

Tzz = C (3z 3 / R5 ) , Trz = C (3rz 2 ) / R5 .


--------------------------------------------------------------------------------
Ans.

3
1
2
Trr = z 2 + 2 +
r z r r r 2
3r 1 2 z 3r 2 z 1 2
2
1
= C z 5 3 + + 3 = C 5 ,
R R ( R + z ) R R( R + z )
R R R ( R + z )

Copyright 2010, Elsevier Inc

5-52
Lai et al, Introduction to Continuum Mechanics

2 z 2 1
T = 2 2 + + (1 2 )
z r r z r r
Cz z Cr C Cz C
= 2 3 + 3 + (1 2 ) = 3 (1 2 ) + (1 2 ) ,
R r R R( R + z R R( R + z
z 1
= C (1 2 ) 3 +
R R( R + z )
3 2 z 3 z 3 z 3z 3 3 3rz 2
Tzz = z 3 2 = C 3 5 + 3 = C 5 , Trz = z = C ,
z z R R r z 2 R5
R R
Tr = T z = 0 .
_________________________________________________________________

5.87 The stresses in Boussinesq problem in cylindrical coordinate are given by:
Fz 3r 2 z (1 2 ) Fz (1 2 ) z 1
Trr = , T = 3 + ,
2 R5 R ( R + z ) 2 R R( R + z )
Fz 3z 3 Fz 3rz 2
Tzz = , T = , Tr = T z = 0 .
2 R5 2 R5
rz

Obtain the stresses in rectangular Cartesian coordinates.


--------------------------------------------------------------------------------
Ans.
Txx Txy Txz cos sin 0 T 0 Trz cos sin 0

rr

Tyx Tyy Tyz = sin cos 0 0 T 0 sin cos 0
0 1 Tzr 0 Tzz 0 1
Tzx Tyz Tzz
0 0

Trr cos 2 + T sin 2 (Trr T ) sin cos Trz cos



(Trr T ) sin cos Trr sin 2 + T cos 2 Trz sin . Thus,

Trz cos Trz sin 1

Txx = Trr cos 2 + T sin 2
Fz 3r 2 z (1 2 ) 2 Fz (1 2 ) z 1 2
= 5 cos 3 + sin
2 R R ( R + z ) 2 R R( R + z )
Fz 3 x 2 z (1 2 ) (1 2 ) z sin 2 + (1 2 ) sin 2
= 5 cos 2
2 R R( R + z ) R3 R( R + z )
3 x 2 z (1 2 ) z (1 2 ) z
Fz
= 5 + ( R + z ) cos 2
2cos 2
+ 1
2
R R3 R( R + z ) R 2
F 3x 2 z (1 2 ) z (1 2 ) (1 2 ) z 2
= z 5 + + 2 ( R + z )cos 2cos
2
2 R R 3 R( R + z ) R( R + z ) R

Copyright 2010, Elsevier Inc

5-53
Lai et al, Introduction to Continuum Mechanics

Fz 3 x 2 z (1 2 ) z (1 2 ) (1 2 ) R( z R) + z 2 R 2 2
= 5 + + cos
2 R R3 R ( R + z ) R ( R + z ) R2
Fz 3 x 2 z (1 2 ) z (1 2 ) (1 2 ) R( z R) + z 2 R 2 2
= 5 + + cos
2 R R3 R ( R + z ) R ( R + z ) R2
Now,
x2 x2
r cos = x cos = x / r , r 2 + z 2 = R 2 , cos 2 = 2 =
R z 2 ( R + z )( R z )
Therefore,
F 3x 2 z (1 2 ) z (1 2 ) (1 2 ) R( z R) + z 2 R 2 x2
Txx = z 5 + +
2 R R3 R ( R + z ) R( R + z ) R2 ( R + z )( R z )
Fz 3 x 2 z (1 2 ) z (1 2 ) (1 2 ) 1 x 2 x 2
= 5 + + .
2 R R3 R ( R + z ) R ( R + z ) R ( R + z ) R 2
_________________________________________________________________

5.88 Obtain the variation of Tzz along the z axis for the case where the normal load on the
surface of an elastic half-space is uniform with intensity qo , and the loaded area is a circle of
radius ro with its center at the origin.
--------------------------------------------------------------------------------
Ans. Using Eq.(5.41.3), we have,
3 z 3 qo 2 r dr ro r dr
Tzz =
2 R' 5
= 3 z 3qo
r = o R '5
, where

Ro dR
R2 = r 2 + z 2 RdR = r dr Tzz = 3 z 3qo
z R 4
R
1 o qo z 3 qo z 3
Tzz = 3z 3qo = qo = qo .
3R '3 R '= z Ro3 (ro2 + z 2 )3/2
__________________________________________________________________

5.89 For the potential function = D1R 2 cos ez , where ( R, , ) are the spherical
coordinates with as the azimuthal angle. Find TRR and T R .
-------------------------------------------------------------------------------
2
Ans. = D1R 2 cos = 2 D1R 3 cos 2 = 6 D1R 4 cos , = D1R 2 sin .
R R
From Example 5.38.7,

Copyright 2010, Elsevier Inc

5-54
Lai et al, Introduction to Continuum Mechanics

2 2 sin
TRR = 2(1 ) cos + R cos +
R R 2 R
2 sin

( ) (
= D1 2(1 ) cos 2 R 3 cos + R cos 6 R 4 cos +
R
)
R 2 sin

( )
= D1 2 ( 5 ) cos 2 2 R 3 .

2(1 ) 2
TR = cos + cos + sin (1 2 )
R R R
2(1 )
= D1
R
( ) ( )
cos R 2 sin + cos 2 R 3 sin + sin (1 2 ) 2 R 3 cos

( )
= D1 2(1 ) R 3 cos sin + 2 R 3 sin cos 2 R 3 cos sin (1 2 )

= D1R 3 cos sin [ 2(1 ) + 2 2(1 2 )] = 2 D1R 3 (1 + ) cos sin .
__________________________________________________________________

5.90 (
For the potential function, % = % ( R, ) = C1[ R 3 3cos 2 1 / 2] + C2 R 1 , )
where ( R, , ) are the spherical coordinates with as the azimuthal angle, obtain TRR and T R .
--------------------------------------------------------------------------------
Ans. With
%
( )
% ( R, ) = ( C1 / 2 ) R 3 3cos 2 1 + C2 R 1
R
( )(
= ( C1 / 2 ) 3R 4 3cos 2 1 + ( C2 ) R 2 ) ( )
2%
(
2 = 6C1R 5 3cos 2 1 + 2C2 R 3
R
)
% %
and = 3C1R 3 sin cos = 9C1R 4 sin cos .
R
From Example 5.38.6, we have
2
(
TRR = 2 = 6C1R 5 3cos 2 1 + 2C2 R 3 .
R
)
1 1 1
( )
2 4 1
TR = = 9C1R sin cos 3C1R 3 sin cos = 12C1R 5 sin cos

R R R R R
.
TR = T = 0 .
_________________________________________________________________

CHARTER 5, PART B

5.91 Demonstrate that if only E2 and E3 are nonzero, then Eq.(5.46.4) becomes

Copyright 2010, Elsevier Inc

5-55
Lai et al, Introduction to Continuum Mechanics

C C23 E2
2U = [ E2E3 ] 22 .
C32 C33 E3
-------------------------------------------------------------------------------
Ans. Eq. .(5.46.4) gives
C12 E2 + C13 E3
C E + C E
22 2 23 3
C E + C33 E3
2U = [ 0 E2 E3 0 0 0] 23 2 = E2 ( C22 E2 + C23 E3 ) + E3 ( C23 E2 + C33 E3 ) .
C24 E2 + C34 E3
C25 E2 + C35 E3

C26 E2 + C36 E3
This is the same as
C C23 E2
[ E2E3 ] 22 = E2 ( C22 E2 + C23 E3 ) + E3 ( C23 E2 + C33 E3 ) .
C23 C33 E3
_________________________________________________________________

5.92 Demonstrate that if only E1 and E3 are nonzero, then Eq.(5.46.4) becomes
C C13 E1
2U = [ E1 E3 ] 11
C31 C33 E3
--------------------------------------------------------------------------------
Ans.
C11 C12 C13 C14 C15 C16 E1
C
12 C22 C23 C24 C25 C26 0
C C23 C33 C34 C35 C36 E3
2U = [ E1 0 E3 0 0 0] 13
C14 C24 C34 C44 C45 C46 0
C15 C25 C35 C45 C55 C56 0

C16 C26 C36 C46 C56 C66 0
= E1 ( C11E1 + C13 E3 ) + E3 ( C13 E1 + C33 E3 )
This is the same as
C11 C13 E1 1
[ E1 E3 ] C C E = 2 E1 ( E1C11 + C13 E3 ) + E3 ( C13 E1 + C33 E3 ) = 2U
13 33 3
__________________________________________________________________

5.93 Write stress strain laws for a monoclinic elastic solid in contracted notation, whose
plane of symmetry is the x1 x2 plane.
--------------------------------------------------------------------------------
Ans. All Cijkl = 0 where the indices ijkl contain an odd number of 3. Therefore,

Copyright 2010, Elsevier Inc

5-56
Lai et al, Introduction to Continuum Mechanics

C11 C12 C13 0 0 C16


C C26
12 C22 C23 0 0
C C23 C33 0 0 C36
[C ] = 13 .
0 0 0 C44 C45 0
0 0 0 C45 C55 0

C16 C26 C36 0 0 C66
________________________________________________________________________

5.94 Write stress strain laws for a monoclinic elastic solid in contracted notation, whose
plane of symmetry is the x1 x3 plane.
--------------------------------------------------------------------------------
Ans. All Cijkl = 0 where the indices ijkl contain an odd number of 2. Therefore,
C11 C12 C13 0 C15 0
C
12 C22 C23 0 C25 0
C C23 C33 0 C35 0
[C ] = 13
0 0 0 C44 0 C46
C15 C25 C35 0 C55 0

0 0 0 C46 0 C66
________________________________________________________________________

5.95 For transversely isotropic solid with e3 as the axis of transversely isotropy, show from
= Qmi Qnj Qrk Qsl Cmnrs that C1113
the transformation law Cijkl = 0 (See Sect.5.50)
-------------------------------------------------------------------------------
Ans. Since Q33 = 1, Q13 = Q23 = Q31 = Q32 = 0 , therefore,

= Qm1Qn1Qr1Qs 3Cmnrs = Qm1Qn1Qr1Q33Cmnr 3 = Qm1Qn1Qr1Cmnr 3 = Q11Qn1Qr1C1nr 3 + Q21Qn1Qr1C2 nr 3


C1113
= Q11Q11Qr1C11r 3 + Q11Q21Qr1C12 r 3 + Q21Q11Qr1C21r 3 + Q21Q21Qr1C22 r 3 .
Now, all Cijkl with odd number of either 1 or 2 are zero because e1 plane and e 2 -plane are planes
=0
of material symmetry. Thus, Qr1C11r 3 = Qr1C12 r 3 = Qr1C21r 3 = Qr1C22 r 3 = 0 . Thus, C1113
_________________________________________________________________________

5.96 Show that for a transversely isotropic elastic material with e3 as the axis of transverse
isotropy, C1133 = C2233 (see Sect.5.50) .
---------------------------------------------------------------------------------
Ans. e1 = cos e1 + sin e2 , e2 = sin e1 + cos e2 , e3 = e3
Q11 = cos , Q12 = sin , Q21 = sin , Q22 = cos , Q33 = 1, Q31 = Q13 = Q23 = Q32 = 0 . Thus,
= Qm1Qn 2Qr 3Qs 3Cmnrs = Qm1Qn 2Q33Q33Cmn33 = Qm1Qn 2Cmn33 = Q11Qn 2C1n33 + Q21Qn 2C2 n33
C1233
= Q11Q12C1133 + Q11Q21C1233 + Q21Q12C2133 + Q21Q22C2233 .

Copyright 2010, Elsevier Inc

5-57
Lai et al, Introduction to Continuum Mechanics

Now C1233 = C2133 = 0 because e1 plane (as well as e2 -plane) is a plane of material symmetry.
Thus,
= Q11Q12C1133 + Q21Q22C2233 = cos sin C1133 + sin cos C2233
C1233
= ( C1133 + C2233 ) cos sin .
= 0 , because e1 is also a plane of symmetry. Thus C1133 = C2233 .
Again, C1233
________________________________________________________________________

5.97 Show that for a transversely isotropic elastic material with e3 as the axis of transverse
isotropy(see Sect.5.50)
(sin ) 2 C1111 + (cos ) 2 (sin ) 2 C1122 + 2 (cos )2 (sin )2 C1212 (cos )2 C2222 = 0 .

--------------------------------------------------------------------------------
Ans. Since Q13 = Q31 = Q23 = Q32 = 0 and Cijkl = 0 when the indices ijkl contain an odd number
of either 1 or 2, therefore,
= Qm1Qn 2Qr 2Qs 2Cmnrs
C1222 = Q11Qn 2Qr 2Qs 2C1nrs + Q21Qn 2Qr 2Qs 2C2 nrs
= Q11Q12Qr 2Qs 2C11rs + Q11Q22Qr 2Qs 2C12 rs + Q21Q12Qr 2Qs 2C21rs + Q21Q22Qr 2Qs 2C22 rs
= Q11Q12Q12Q12C1111 + Q11Q12Q22Q22C1122 + Q11Q22Q12Q22C1212 + Q11Q22Q22Q12C1221
+Q21Q12Q12Q22C2112 + Q21Q12Q22Q12C2121 + Q21Q22Q12Q12C2211 + Q21Q22Q22Q22C2222 .
Thus,
= cos (sin ) C1111 sin (cos ) C1122 (cos )3 sin C1212 (cos )3 sin C1221
C1222 3 3

+ (sin )3 cos C2112 + (sin )3 cos C2121 + (sin )3 cos C2211 + (cos )3 sin C2222
= cos sin sin 2 C1111 + (cos 2 sin 2 )C1122 + 2(cos 2 sin 2 )C1212 cos 2 C2222 .

where we have used Cijkl = C jikl , Cijkl = C jilk and Cijkl = Cklij .
= 0 because e1 is also a plane of symmetry, therefore,
Now, C1222
sin C1111 + (cos 2 sin 2 )C1122 + 2(cos 2 sin 2 )C1212 cos 2 C2222 = 0 .
2

________________________________________________________________________

5.98 In Section 5.50, we obtained the reduction in the elastic coefficients for a transversely
isotropic elastic solid by demanding that each S plane is a plane of material symmetry. We
be the same for all . Use this
can also obtain the same reduction by demanding the Cijkl
procedure to obtain the result: C1133 = C2233 .
-------------------------------------------------------------------------------
Ans. Since Q31 = Q13 = Q32 = Q23 = 0, Q33 = 1 , therefore,
= Qm1Qn1Qr 3Qs 3Cmnrs = Qm1Qn1Q33Q33Cmn33 = Qm1Qn1Cmn33 .
C1133
Now, Cijkl = 0 when the indices contain an odd number of either 1 or 2, therefore,
= Q11Q11C1133 + Q21Q21C2233 = cos 2 C1133 + sin 2 C2233 .
C1133
= C1133 for all , therefore,
Now, C1133
C1133 = cos 2 C1133 + sin 2 C2233 C1133 sin 2 = sin 2 C2233 .

Copyright 2010, Elsevier Inc

5-58
Lai et al, Introduction to Continuum Mechanics

Thus, C1133 = C2233 .


________________________________________________________________________

5.99 Invert the compliance matrix for a transversely isotropic elastic solid to obtain the
relationship between Cij and the engineering constants. That is, verify Eq. (5.53.2) and
(5.53.3) by inverting the following matrix:
1 / E1 21 / E1 31 / E3
[ A] = 21 / E1 1 / E1 31 / E3

13 / E1 13 / E1 1 / E3
-------------------------------------------------------------------------------
1 / E1 21 / E1 31 / E3

Ans. [ A] = 21 / E1 1 / E1 31 / E3
13 / E1 13 / E1 1 / E3
1 1
= det [ A] = (1 2 2113 31 213 31 21 21 ) = (1 + 21 ) (1 21 213 31 ) .
E12 E3 E12 E3
Now, 31 / E3 = 13 / E1

(1 + 21 ) (1 21 2 312 ( E1 / E3 ) ) =
1 1
= (1 + 21 ) D
E12 E3 E12 E3

(
where D = 1 21 2 31
2
( E1 / E3 ) . )
31 / E3 E12 E3 1 31
2
( E1 / E3 )
1 1 / E1 1 E1
C11 = = ( 31 13 )
1 =

13 1/ E 1 / E3 D (1 + 21 ) E1E3 (1 + 21 ) D
C22 = C11

21 / E1 1 21
2
( E3 / E1 )
C33 =
1 1 / E1
21 / E1 1 / E3
=
1
E1E3
2
(
1 21 ( E3 / E1 ) =
E1
(1 + 21 ))
D

C12 =
1 21 / E1 31 / E3
=
1
( 21 + 31 13 ) =
E1 21 + 31
2
E1 / E3( )
13 / E1 1 / E3 E1E3 (1 + 21 ) D
1 21 / E1 31 / E3 1 E
C13 = =
31 / E3 E1E3
( 21 31 + 31 ) = 31 1
1 / E1 D
1 1 / E1 31 / E3 1 E
C23 = = ( 31 + 21 31 ) = 31 1
21 / E1 31 / E3 E1E3 D
________________________________________________________________________

5.100 Obtain Eq.(5.53.6) from Eq. (5.53.2) and (5.53.3).


-------------------------------------------------------------------------------
Ans. From

Copyright 2010, Elsevier Inc

5-59
Lai et al, Introduction to Continuum Mechanics

C11 =
E1
1 31

2
( E1 / E3 )
and C12 =
E1 21 + 31
2
(
E1 / E3 )
(1 + 21 ) D (1 + 21 ) D

C11 C12 =
E1 1 31

2
( E1 / E3 ) E1 21 + 31

2
(
E1 / E3 )
(1 + 21 ) D (1 + 21 ) D

)D{ }
E1 E1 E1
= 1 2 31
2
( E1 / E3 ) = {D} =
(1 + 21 21
(1 + 21 ) D (1 + 21 )
E1
Thus, [see Eq.5.53.5], 2G12 = .
(1 + 21 )
________________________________________________________________________

5.101 Invert the compliance matrix for an orthotropic elastic solid to obtain the relationship
between Cij and the engineering constants.
-------------------------------------------------------------------------------
1
1
21 31
E1 E2 E3
C11 C12 C13
12 1 32
Ans. Let [ A] =
1
= C12 C22 C23
E1 E2 E3
C13 C23 C33
23 1
13

E1 E2 E3

det [ A] =
[1 12 23 31 13 21 32 13 31 23 32 21 12 ] . Since
E1E2 E3
E1 21 E2 32 E3 13
12 23 31 = = 21 32 13 , therefore,
E2 E3 E1

=
[1 213 21 32 13 31 23 32 2112 ] .
E1E2 E3
Next
1 32

1 E2 E3 1 1 1 32 23 1
C11 = = = (1 32 23 ) , etc.,
23 1 E2 E3 E3 E2 E2 E3

E2 E3
21 31 21 31

1 E2 E3 1 1 E2 E3 1
C12 = = ( 21 + 31 23 ) , C13 = = ( 31 + 21 32 )
23 1 E2 E3 1 32 E2 E3

E2 E3 E2 E3

Copyright 2010, Elsevier Inc

5-60
Lai et al, Introduction to Continuum Mechanics

1 31

1 E1 E3 1
C23 = = ( 32 + 3112 )
12 32 E1E3

E1 E3
_________________________________________________________________

5.102 Obtain the restriction given in Eq.(5.54.8) for engineering constants for an orthotropic
elastic solid
--------------------------------------------------------------------------------
Ans.
1
21 31
E1 E2 E3 1
21
12 1 E E2
32 det 1 = 1 (1 21 12 ) 1 21 12 > 0 ,
E1 E2 E3 12 1 E1E2

1 E1 E2
13 23
E1 E2 E3
E E
But, 12 = 21 1 21 12 = 1 21 21 1 > 0 2 > 21 2
. Also,
E1 E2 E2 E1
12
2
E2 E1
1 21 12 = 1 >0 > 12
2
E1 E2
1 32
E
E3
Next, det
2 = 1 (1 ) 1 > 0 .
23 1 E2 E3
32 23 32 23

E2 E3
32 23 E3 2 E2
But, = 1 32 23 = 1 23
2
= 1 32 ,
E3 E2 E2 E3
E2 E
1 32 23 > 0 > 23
2
and 3 > 32
2
.
E3 E2
Also,
1 31
E
E3
det
1 = 1 (1 ) = 1 1 2 E3 = 1 1 2 E1
13 1 E1E3
31 13
E1E3
13
E1 E1E3
31
E3

E1 E3
E1 E
(1 3113 ) > 0 132 < and 31
2
< 3 .
E3 E1
________________________________________________________________________

5.103 Write down all the restrictions for the engineering constants for a monoclinic solid in
determinant form (no need to expand the determinant).

Copyright 2010, Elsevier Inc

5-61
Lai et al, Introduction to Continuum Mechanics

--------------------------------------------------------------------------------
E11 1 / E1 21 / E2 31 / E3 41 / G4 0 0 T11
E / E 1 / E2 32 / E3 42 / G4 0 0 T22
22 12 1
E / E 23 / E2 1 / E3 43 / G4 0 0 T33
Ans. 33 = 13 1
2 E23 14 / E1 24 / E2 34 / E3 1 / G4 0 0 T23
2 E31 0 0 0 0 1 / G5 65 / G6 T31

2 E12 0 0 0 0 56 / G5 1 / G6 T12
(i)
E1 > 0, E2 > 0, E3 > 0, G4 > 0, G5 > 0, G6 > 0
1 / E1 21 / E2 1 / E2 32 / E3 1 / E3 43 / G4
(ii ) > 0, > 0, >0
12 / E1 23 / E2 23 / E2 1 / E3 34 / E3 1 / G4
1 / G5 65 / G6 1 / E1 31 / E3 1 / E2 42 / G4 1 / E1 41 / G4
> 0, > 0, > 0, >0
56 / E2 1 / G6 13 / E1 1 / E3 24 / E2 1 / G4 14 / E1 1 / G4
(iii)
1 / E1 21 / E2 31 / E3 1 / E2 32 / E3 42 / G4
12 / E1 1 / E2 32 / E3 > 0, 23 / E2 1 / E3 43 / G4 > 0
13 / E1 23 / E2 1 / E3 24 / E2 34 / E3 1 / G4
1 / E1 31 / E3 41 / G4 1 / E1 21 / E2 41 / G4
13 / E 1 / E3 43 / G4 > 0, 12 / E1 1 / E2 42 / G4 > 0
14 / E1 34 / E3 1 / G4 14 / E1 24 / E2 1 / G4
(iv)
1 / E1 21 / E2 31 / E3 41 / G4
12 / E1 1 / E2 32 / E3 42 / G4
>0
13 / E1 23 / E2 1 / E3 43 / G4
14 / E1 24 / E2 34 / E3 1 / G4
_________________________________________________________________

CHAPRTR 5, PART C

5.104 Show that if a tensor is objective, then its inverse is also objective .
-------------------------------------------------------------------------------
Ans. Let T be an objective tensor, then in a change of frame: x* = c(t ) + Q(t )(x xo )
T* = Q(t )TQ(t )T . Taking the inverse of this equation, we get, since Q 1 = QT .

( )
1
T1* = Q(t )TQ(t )T = Q(t )T1Q(t )T . Thus, T1 is objective.
________________________________________________________________________

Copyright 2010, Elsevier Inc

5-62
Lai et al, Introduction to Continuum Mechanics

5.105 Show that the rate of deformation tensor D = [v + ( v ) ] / 2 is objective. [See


T

Example 5.56.2)].
-------------------------------------------------------------------------------
Ans. From Eq.(5.56.13), we have * v* = Q(t ) ( v ) QT (t ) + QQ & T . Thus,

( * v*)T = Q(t ) ( v ) Q T (t ) + Q Q ( )
T
T & . Now,

( d / dt ) (QQT ) = 0 Q& QT + QQ& T Q& QT = QQ& T . Thus,


* v * +( * v*)T = Q(t ) ( v ) Q T (t ) + QQ & T = Q (t ) ( v ) + ( v ) T Q T (t )
& T + Q(t ) ( v )T Q T (t ) QQ

D* = Q(t )DQT (t ) .
________________________________________________________________________

5.106 Show that in a change of frame, the spin tensor W = [v ( v ) ] / 2 transforms in


T

& Q T . [See Example 5.56.2)].


accordance with the equation W* = Q(t ) WQ T (t ) + Q
-------------------------------------------------------------------------------
Ans. From Eq.(5.56.13), we have * v* = Q(t ) ( v ) Q T (t ) + QQ & T . Thus

( * v*)T = Q(t ) ( v ) Q T (t ) + Q Q ( )
T
T & . Now,

( d / dt ) ( QQT ) = 0 = Q& QT + QQ& T QQ& T = Q& QT , ( * v*)T = Q(t ) ( v ) Q T (t ) Q


T & QT .

Thus, ( * v * ) ( * v*)T = Q(t )[( v ) ( v ) ]Q T (t ) + 2QQ


& T
T

& QT .
W* = Q(t ) WQT (t ) + Q
____________________________________________________________

5.107 Show that in a change of frame, the material derivative of an objective tensor
T transforms in accordance with the equation T &
& * = QTQ T &T,
& T (t ) + Q(t )TQ
(t ) + Q(t )TQ
where a super-dot indicates material derivative. Thus the material derivative of an objective
tensor T is not objective.
-------------------------------------------------------------------------------
Ans. Since T is objective, therefore, in a change of frame, T* = Q(t )TQ T (t ) . Taking the material
derivative of this equation and noting that t* = t , we have,
&
& * = QTQ
T T & T + QTQ
+ QTQ & T . Since T & T (t ) , therefore, DT is non-objective.
& * Q(t )TQ
Dt
________________________________________________________________________

5.108 The second Rivlin-Ericksen tensor is defined by:


A2 = A& + A ( v ) + ( v )T A , where A& D A , where A = 2D = v + ( v )T . Show
1 1 1 1 1 1
Dt
that A 2 is objective. [See Prob.Error! Reference source not found. and Error! Reference
source not found.].

Copyright 2010, Elsevier Inc

5-63
Lai et al, Introduction to Continuum Mechanics

-------------------------------------------------------------------------------
Ans. From Prob. 5.105, we had, D* = Q(t )DQ T (t ) A1* = Q(t ) A1Q T (t ) .
& *=Q
A & (t ) A QT (t ) + Q(t ) A
& QT (t ) + Q(t ) A Q& T (t ) . (i)
1 1 1 1

We also have, from Eq.(5.56.13), * v* = Q(t ) ( v ) Q (t ) + QQ


T & . T

Thus A1 * * v * + ( * v * ) A1 *
T

= QA1Q T [Q ( v ) QT + QQ
& T ] + [Q ( v ) QT + QQ
& T ]T QA QT
1

= [QA1 ( v ) Q T + QA1Q T QQ ( )
& T ] + [Q ( v )T A QT + QQ
1
& T QA QT ] .
1

& T + QQ
Since ( D / Dt )QQ T = 0 QQ & T = 0 QQ
& T = QQ
& T , therefore,

A1 * * v * + ( * v * ) A1 * =
T

[QA1 ( v ) Q T QA1Q T QQ ( )
& T ] + [Q ( v )T A Q T QQ
1
& T QA Q T ]
1

i.e.,
A1 * * v * + ( * v * ) A1 * = [QA1 ( v ) Q + Q ( v ) A1Q T ] QA1Q
& T QA
& QT
T T T
1 (ii)
(i) and (ii) give
A & * +A * * v * + ( * v * )T A *
1 1 1
& Q T + QA
= QA & Q T + QA Q& T + [QA ( v ) QT + Q ( v )T A QT ] QA Q& T QA
& QT
1 1 1 1 1 1 1
& Q T + QA ( v ) Q T + Q ( v )T A Q T = Q A T
1 + A1 ( v ) + ( v ) A1 Q .
= QA & T
1 1 1

Thus, A & + A ( v ) + ( v )T A is objective.


1 1 1
_________________________________________________________________

5.109 The Jaumann Derivative of a second order objective tensor T is : T & + TW WT , where
W is the spin tensor. Show that the Jaumann derivative of T is objective. [See Prob. 5.106
and Prob. 5.107]
--------------------------------------------------------------------------------
Ans. We have, since T is objective, therefore, in a change of frame, T* = QTQ T .
In Prob.5.106, we had W* = Q(t ) WQT (t ) + Q & QT and in Prob. 5.107, we had

T &
& * = QTQ T & T (t ) + Q(t )TQ
(t ) + Q(t )TQ & T . Also,

( D / Dt ) QQT = 0 QQ & T + QQ & T QQ& T = QQ &T


Thus,
T * W* = QTQ QWQ + QTQ T QQ
T T
(
& T = QTWQ T QT Q
& T, )
W * T* = QWQ T QTQ T + Q & Q T QTQ T = QWTQT + Q & TQT .
T * W * W * T* = Q ( TW WT ) Q T QT Q
& T Q& TQ T .
Thus,

Copyright 2010, Elsevier Inc

5-64
Lai et al, Introduction to Continuum Mechanics

&
& * +T * W * W * T* = QTQ
T T & T + Q ( TW WT ) QT QT Q
& T + QTQ
+ QTQ & T Q
& TQT
& T + Q ( TW WT ) QT .
= QTQ
& * +T * W * W * T* = Q T
That is, T (
& + TW WT QT . )
(
& + TW WT is objective.
Therefore, the Jaumann derivative of T , that is , T )
________________________________________________________________________

% is related to the first Piola-Kirchhoff stress


5.110 The second Piola Kirchhoff stress tensor T

% = F T , or to the Cauchy stress tensor T by
1
tensor T by the formula T
o o
1 1 T
% = (detF)F T(F ) Show that, in a change of frame, T %*=T % . [See Example 5.56.3 and
T
Example 5.57.1]
--------------------------------------------------------------------------------
Ans. In Example 5.56.3 and Example 5.57.1, we obtained that in a change of frame,
F* = Q(t )F and To * = QTo . Thus,
T% * = F *1 T * = (Q(t )F) 1 QT = F 1Q 1QT = F 1T . That is, T %*=T %.
o o o o
________________________________________________________________________

5.111 Starting from the constitutive assumption that T = H (F ) and T* = H (F*) , where T is
Cauchy stress and F is deformation gradient, show that in order that the assumption be
independent of observers, H (F ) must transform in accordance with the equation
QTQ T = H (QF) . (b) Choose Q = R T to obtain T = RH (U )R T , where R is the rotation
% = h(U ) , where
tensor associated with F and U is the right stretch tensor. (c) Show that T
h = (det U) U 1H (U)U 1 . Since C = U 2 , therefore, we may write T = f (C) .
-------------------------------------------------------------------------------
Ans. (a) In a change of frame, T* = QTQT and F* = QF , therefore,
T* = H (F*) QTQT = H (QF ) .
(b) From QTQ T = H (QF) , with Q = R T R T TR = H (R T F ) . But R T F = R T ( RU) = U where
U is the right stretch tensor. Therefore, R T TR = H (R T F ) T = RH (U) R T .
(c) F = RU R = FU 1 R T = U 1F T , thus,
T = RH (U)R T T = FU 1H (U)U 1F T F 1T(F T )1 = U 1H (U)U 1 .
Now since J = det F = det U , we can write JF 1T(F T ) 1 = (det U) U 1H (U)U 1 .
The left side of the above equation is the second Piola-Kirchhoff stress tensor T% and the right side
% = h(U) , or since U 2 = C , one can write
is a function of the right stretch tensor U . Thus, T
% = h(C) .
T
________________________________________________________________________

Copyright 2010, Elsevier Inc

5-65
Lai et al, Introduction to Continuum Mechanics

5.112 From r = ( 2 X + )
1/2
, =cY ,
z = Z , =1 / c , obtain the right Cauchy-Green
deformation tensor B .
-------------------------------------------------------------------------------
Ans., we have, with r = ( 2 X + )
1/2
, =cY , z = Z ,
r 1 1/2 1/2 r r
= ( 2 X + ) ( 2 ) = ( 2 X + ) = , = =0
X 2 r Y Z
z z z
= 0, = c, = 0; = 0, = 0, =1
X Y Z X Y Z
Thus, Using Eq.3.29.59 to 3.29.64,
2 2 2 2 2 2 2
r r r r r r
+ Y + Z = ( rc )
2
Brr = + + = , B =
X Y Z r X
2 2 2
z z z r r r r r r
Bzz = + + =1, Br = + + =0
X Y Z X X Y Y Z Z
r z r z r z
Brz = + Y Y + Z Z = 0 ,
X X
r z r z r z
B z = + Y Y + Z Z = 0
X X
________________________________________________________________________

5.113 From r = 1R, = + KZ ,


z = 3 Z , 12 3 = 1 , obtain the right Cauchy-Green
deformation tensor B .
--------------------------------------------------------------------------------
Ans. With r = 1R, = + KZ , z = 3 Z , 12 3 = 1 , we have,
r = 1R, = + KZ , z = 3 Z , 12 3 = 1,
r r r

= 1 , = 0, = 0, = 0, = 1, = K,
R Z R Z
z z z
= 0, = 0, = 3 .
R Z
Using Eq. (3.29.19) to Eq. (3.29.24) and noting that ro R, o , zo Z ,
2 2 2
r r r
Brr = + + = (1 ) 2 .
R R Z
r r r r r r
Br = + + = 0 = B r .
R R R R Z Z
2 2 2 2
r r r r
= + ( rK ) = ( 1 ) + ( rK ) .
2 2 2
B = + +
R R Z R
2 2 2
z z z
Bzz = + R + Z = (3 ) .
2
R

Copyright 2010, Elsevier Inc

5-66
Lai et al, Introduction to Continuum Mechanics

r z r z r z
Brz = + + = 0 = Bzr .
R R R R Z Z
z r z r z r z r
Bz = + + = = 3rK = B z .
R R R R Z Z Z Z
________________________________________________________________________

Copyright 2010, Elsevier Inc

5-67
Lai et al, Introduction to Continuum Mechanics

CHAPTER 6

6.1 In Figure P 6-1, the gate AB is rectangular with width b = 60 cm and length L= 4 m. The
gate is hinged at the upper edge A . Neglect the weight of the gate, find the reactional force at B .
Take the specific weight of water to be 9800 N / m3 and neglect frictions.

3m 30 o
hinge
A
so
B 4m
s
Figure P 6-1
-------------------------------------------------------------------------------
Ans. Take the gate AB as a free body. With so measured from the water surface along the inclined
plane to point A, s measured from point A along the length of the plate (AB) and = 30o as
shown in the figure, we have,
dF = pdA = g ( so + s ) sin (bds ), thus ,
L L2 L3
AM = 0 RB L = sdF = b g sin (o )
s s + s ds = b g sin so
2
+ . Therefore,
3
A 0
s sin L sin 3 4 ( 0.5 )
RB = bL g o + = ( 0.6 )( 4 )( 9800 ) + = 5.1 10 N .
4
2 3 2 3
_________________________________________________________________

6.2 The gate AB in Figure P 6-2 is 5 m long and 3 m wide. Neglect the weight of the gate,
compute the water level h for which the gate will start to fall. Take the specific weight of water
to be 9800 N / m3 .
20,000 N

20,000 N
A A
W

h h
F
5m
5m

o o
60 B 60 B

Figure P 6-2
--------------------------------------------------------------------------------
Ans. Consider the gate plus the triangular region of water above the gate as the free body
diagram. Then,
Horizontal force from water to gate: F = g ( h / 2)(bh) = gbh 2 / 2 acting at 1/3 from base.

Copyright 2010, Elsevier Inc


6-1
Lai et al, Introduction to Continuum Mechanics

Weight of water on gate: W= gb(1 / 2)[h(h tan 30o )] = gbh 2 / (2 3) .


M B = 0 W (1 / 3)(h / 3) + Fh / 3 = P ( AB)
9 P( AB ) 9 ( 20000 ) (5)
h3 = = = 15.31 h = 2.48m.
2 gb 2 ( 9800 )( 3)
__________________________________________________________________

6.3 The liquids in the U-tube shown in Figure P 6-3 is in equilibrium. Find h2 as a function of
1 , 2 , 3 , h1 and h3 . The liquids are immiscible.

3
1 h3
h1

h2
1 2 2

Figure P 6-3
--------------------------------------------------------------------------------
Ans. p1 = 1 gh1 , p2 = 3 gh3 + 2 gh2 ,
p1 = p2 1 gh1 = 3 gh3 + 2 gh2 h2 = ( 1h1 3h3 ) / 2 .
_________________________________________________________________

6.4 In Figure P 6-4, ,the weight WR is supported by the weight WL , via the liquids in the
container. The area under WR is twice that under WL . Find WR in terms of WL , 1 , 2 , AL , and h
( 2 < 1and assume no mixing ) .
WL WR
AL AR 4
1

h
2

2 3
1

Figure P 6-4
-------------------------------------------------------------------------------
Ans. p3 = p2 = p1 + 1 gh , p4 = p3 2 gh = p1 + ( 1 2 ) gh
WR = p4 AR = p1 AR + ( 1 2 ) ghAR , i.e.,
WR = 2 p1 AL + 2 ( 1 2 ) ghAL = 2WL + 2 ( 1 2 ) ghAL
_________________________________________________________________

Copyright 2010, Elsevier Inc


6-2
Lai et al, Introduction to Continuum Mechanics

6.5 Referring to Figure P 6-5, the radius and length of the cylinder are r and L respectively,
The specific weight of the liquid is .
(a) Find the buoyancy force on the cylinder and
(b) Find the resultant force on the cylindrical surface due to the water pressure. The centroid of a
semi-circular area is 4r / 3 from the diameter.

Figure P 6-5
-------------------------------------------------------------------------------
Ans. (a) Buoyancy force is the net upward force due to the water pressure on the left half of the
boundary of the cylinder which is submerged in the water. It is therefore equal to the weight of
the water displaced by this left half. That is, Buoyancy force = ( r 2 / 2) L .

( )
(b) Horizontal water force: Fx = (2r / 2) ( 2rL ) = 2r 2 L . The line of action of Fx is 2r / 3
above the ground. The line of action of Fy (the buoyancy force) passes through the centroid of
the semi-circular area, i.e., 4r / 3 left of the diameter.
_________________________________________________________________

6.6 A glass of water moves vertically upward with a constant acceleration a . Find the pressure
at a point whose depth from the surface of the water is h . Take the atmospheric pressure to
be pa .

-------------------------------------------------------------------------------
Ans. Let z axis be appointing vertically upward, then
dp dp
g = a = ( g + a) p = ( g + a) z + C .
dz dz
At the instant of interest, let the origin be at the free surface, then C = pa , the atmospheric
pressure. Thus, p pa = ( g + a ) z . At a point which is at z = h, p pa = ( g + a ) h .
__________________________________________________________________

6.7 A glass of water moves with a constant acceleration a in the direction shown in
Figure P 6-6. (a) Show that the free surface is a plane and find its angle of inclination and (b)
find the pressure at the point A . Take the atmospheric pressure to be pa .

Copyright 2010, Elsevier Inc


6-3
Lai et al, Introduction to Continuum Mechanics

x
a
g
ro h

Figure P 6-6
-------------------------------------------------------------------------------
Ans. (a) With respect to the coordinates shown, the governing equations are:
p p p
(i) = a cos , (ii) g = a sin , (iii) = 0 , thus
x y z
p df
(iii) p = p ( x, y ), (i) p = ( a cos ) x + f ( y ) = .
y dy
df
(ii) = ( g + a sin ) f = ( g + a sin ) y + C p = ( a cos ) x ( g + a sin ) y + C
dy
.
At the instant of interest, let the origin be at the center of the surface, then C = pa .and
p = ( a cos ) x ( g + a sin ) y + pa . On every point on the free surface, p = pa ,
therefore, ( a cos ) x ( g + a sin ) y = 0 . Thus, the free surface is a plane. The angle of
dy a cos
inclinations is given by tan = = .
dx g + a sin
(b) At the point A, x = ro , y = h . Thus, p = ( a cos ) ro + ( g + a sin ) h + pa
_________________________________________________________________

6.8 The slender U-tube shown in Figure P 6-7 moves horizontally to the right with an
acceleration a . Determine the relation between a, l and h .

h
a

Figure P 6-7
------------------------------------------------------------------------
a a h al
Ans. The slope of the free surface is given by . Thus = h = .
g g l g
_________________________________________________________________

Copyright 2010, Elsevier Inc


6-4
Lai et al, Introduction to Continuum Mechanics

6.9 A liquid in a container rotates with a constant angular velocity about a vertical axis.
Show that the free surface is a paraboloid given by z = r 2 2 / (2 g ) where the origin is on the
axis of rotation and z is measured upward from the lowest point of the free surface.
-------------------------------------------------------------------------------
Ans. Let z be pointing vertically upward with the origin at the lowest point of the free surface. We
have,
(i)
p
r
( )
= r 2 and (ii)
p
z
g = 0 p = gz + f (r )
p df
=
r dr
.

p df r 2 2 r 2 2
(i) = = r 2 f = + C , p = gz + +C.
r dr 2 2
r 2 2
At (r , z ) = (0,0) , p = pa , therefore, p = gz + + pa . The free surface is characterized
2
r 2 2
by p = pa , therefore, the equation of the surface is: z = .
2g
_________________________________________________________________

6.10 The slender U-tube rotates with an angular velocity about the vertical axis shown in
Figure P 6-8. Find the relation between h( h1 h2 ), , r1 and r2 .


h1
h2
r1 r2
o

Figure P 6-8

-------------------------------------------------------------------------------
Ans. The equation for the free surface is given by (see the previous problem) z = r 2 2 / (2 g ) ,
where the origin is on the axis of rotation and z is measured upward from the lowest point of the
free surface. Thus, we have,
r 2 2 r 2 2 2
z1 = 1
2g
and z2 = 2
2g
( )
z1 z2 = r12 -r22
2g
, but, z1 z2 = h1 h2

h1 h2 = ( r12 -r22 ) 2 / (2 g ) .
____________________________________________________________

6.11 For minor altitude differences, the atmosphere can be assumed to have constant
temperature. Find the pressure and density distribution for this case. The pressure p , density
and absolute temperature are related by the ideal gas law p = R .
--------------------------------------------------------------------------------
Ans.: Let gravity be in the negative x3 direction, then we have
p / x1 = 0, p / x2 = 0, p / x3 = g (i)

Copyright 2010, Elsevier Inc


6-5
Lai et al, Introduction to Continuum Mechanics

Thus, p depends only on x3 . Let po denote the pressure at x = 0 , then, we have

x3 + ln po p = po e (
dp dp g g g / R ) x3
= g = dx3 ln p = (ii)
dx3 p R R
If is the density at x = 0 , then = e (
g / R ) x3
o 3 . o
____________________________________________________________

6.12 In astrophysical applications, an atmosphere having the relation between the density and
the pressure p given by p / po = ( / o ) , where po and o are some reference pressure and
n

density, is known as a polytropic atmosphere. Find the distribution of pressure and density in a
polytropic atmosphere.
------------------------------------------------------------------------
Ans. Let z axis point upward, then dp / dz = g . From p / po = ( / o ) , we have,
n

= Cp1/ n , where C = o po1/ n . Thus, dp / dz = Cp1/ n g p 1/ n dp = Cgdz


p z
p 1/ n dp = Cgdz . Thus,
po zo
(A) for n 1 ,
p
[n / (n 1)] p(
n 1) / n
= Cgz z p( ) po( ) = [(n 1) / n]Cg ( z zo )
z n 1 / n n 1 / n
po o

p(
n 1) / n
( )
= po( ) [(n 1) / n] o po1/ n g ( z zo ) = po1/ n po {(n 1) / n}o g ( z zo )
n 1 / n

n / ( n 1)
1/ n 1 n 1
p = po ( ) po o g ( z zo ) .
n
(B) for n = 1 ,
p dp
po p = zo Cgdz ln( p / po ) = Cg ( z zo ) p = po exp Cg ( z zo )
z

p = po exp o po1/ n g ( z zo )

__________________________________________________________________

6.13 Given the following velocity field for a Newtonian liquid with viscosity

(
=0.982 mPa.s 2.05 105 lbs/ft 2 : )
v1 = c ( x1 + x2 ) , v2 = c ( x2 x1 ) , v3 = 0, c = 1 s 1
For a plane whose normal is in the e1 direction, (a) find the excess of the total normal
compressive stress over the pressure p, and (b) find the magnitude of the shearing stress.
------------------------------------------------------------------------------
v
Ans. (a) T11 = p + 2 D11 ( T11 ) p = 2 D11 , where D11 = 1 = c = 1 s 1 ,
x1
Thus, ( T11 ) p = 2 ( 0.982 )( 1) = 1.96 mPa.
v v v v
(b) T12 = 2 D12 = 1 + 2 = 2c = 1.96 mPa. T13 = 2 D13 = 1 + 3 = 0 .
x2 x1 x3 x1
Thus, the magnitude of shearing stress = 1.96 mPa .

Copyright 2010, Elsevier Inc


6-6
Lai et al, Introduction to Continuum Mechanics

__________________________________________________________________

6.14 For a steady parallel flow of an incompressible linearly viscous fluid, if we take the flow
direction to be e3 , (a) show that the velocity field is of the form
v1 = 0, v2 = 0 and v3 = v( x1 , x2 )
(b) If v( x1 , x2 ) = kx2 , find the normal and shear stresses on the plane whose normal is in the
direction of e2 + e3 in terms of viscosity and pressure p .
(c) On what planes are the total normal stresses given by p .
-------------------------------------------------------------------------------
v v v v
Ans. (a) From the equation of continuity 1 + 2 + 3 = 0 , we get, 3 = 0 , thus v3 is
x1 x2 x3 x3
independent of x3 i.e., v3 = v( x1 , x2 ) .
(b) with v1 = 0, v2 = 0 and v3 = kx2 , we have,
0 0 0 0 0 0 p 0 0
[v ] = 0 0 0 [ D] = 0 0 k / 2 [ T] = p [I ] + 2 [ D] = 0 p k

0 k 0 0 k / 2 0 0 k p
On the plane with n = (e 2 + e3 ) / 2 ,
p 0 0 0 0
[t ] = [T][n ] = 0 p k 1 = k p Tn = t n = k p .
1 1
2 2
0 k p 1 k p
1
Ts2 = t (Tn ) = ( k p ) + ( k p ) ( k p ) = 0 .
2 2 2 2 2
2
(c)
p 0 0 n1 pn1
[ t ] = 0 p k n2 = pn2 + kn3 , n12 + n22 + n32 = 1

0 k p n3 kn2 pn3

( ) ( )
Tn = t n = pn12 + pn22 + kn3 n2 + kn2 n3 pn32 = p + 2 kn3 n2 . Thus,
p + 2 kn3 n2 = p n2 = 0 and/or n3 = 0
That is, on any plane ( n1 ,0, n3 ) and ( n1 , n2 ,0 ) , where n12 + n22 + n32 = 1 , the normal component of
stress is p , these include the three coordinate planes (1,0,0 ) , ( 0,1,0 ) and ( 0,0,1) .
__________________________________________________________________

6.15 Given the following velocity field for a Newtonian incompressible fluid with a viscosity
= 0.96 mPa.s :

( )
v1 = k x12 x22 , v2 = 2kx1 x2 , v3 = 0, k = 1 s 1m 1 .
At the point (1,2,1)m and on the plane whose normal is in the direction of e1 ,
(a) find the excess of the total normal compressive stress over the pressure p and
(b) find the magnitude of the shearing stress.
------------------------------------------------------------------------------
Ans. (a)

Copyright 2010, Elsevier Inc


6-7
Lai et al, Introduction to Continuum Mechanics

2kx1 2kx2 0 p + 4 kx1 4 kx2 0


[v ] = 2kx2 2kx1 0 = [ D] , [ T] = 4 kx2
p 4 kx1 0 .
0 0 0 0 0 p
p + 4 8 0

At (1, 2,1) and for k = 1, [ T] = 8 p 4 0 .
0 0 p
On e1 -plane ( T11 ) p = 4 = 3.84 mPa.
(b) on the same plane, Ts = 8 = 7.68 mPa.
_________________________________________________________________

6.16 Do Problem 6.15 except that the plane has a normal in the direction 3e1 + 4e2 .
-------------------------------------------------------------------------------
Ans.
2kx1 2kx2 0 p + 4 kx1 4 kx2 0
[v ] = 2kx2 2kx1 0 = [ D] [ T] = 4 kx2 p 4 kx1 0 .

0 0 0 0 0 p
p + 4 8 0

At (1, 2,1) and for k = 1, [ T] = 8 p 4 0 ,
0 0 p
p + 4 8 0 3 / 5 3 p 20
[t ] = [T][n ] = 8 p 4 0 4 / 5 = 5 4 p 40 .
1

0 0 p 0 0
1 44 44
Tn = t n = ( 3 p 20 ) 3 + ( 4 p 40 ) 4 = p . (Tn ) p = .
25 5 5
(b) t =
2 1
25
( 3 p 20 ) 2
+ ( 4 p 40 ) 2
(
=
25
1
25 p 2 + 440 p + 2000 2 )
88 p 1936 2 64 8
(Tn )2 = p 2 + + . (Ts )2 = t 2 (Tn )2 = 2 Ts = .
5 25 25 5
__________________________________________________________________

6.17 Use the results of Sect. 2.34., chapter 2 and the constitutive equations for the Newtonian
viscous fluid, verify the Navier Stokes Equation in the r -direction for cylindrical coordinates,
i.e., Eq. (6.8.1).
-------------------------------------------------------------------------------
Ans. For a Newtonian fluid, the stress tensor in cylindrical coordinates is given by:

Copyright 2010, Elsevier Inc


6-8
Lai et al, Introduction to Continuum Mechanics

vr 1 vr v v v v
p + 2 + r + z
r r r r z r
1 v vr v 1 vz
[T] = T21 p + 2 + +
r r z r
v
T31 T32 p + 2 z
z
The Equations of motion in terms of the stress components in the r-direction is [see Eq.(4.8.1)]:
Trr 1 Tr Trr T Trz
+ + + + Br = ar
r r r z
We also have the equation of continuity [see Eq.(2.34.6) or Eq.(6.8.4)]:
vr 1 v vr vz
+ + + =0
r r r z
vr T p 2v
Now, Trr = p + 2 rr = + 2 2r
r r r r
1 vr v v 1 Tr 1 2 vr 1 v 1 2 v
Tr = + = 2 2 +
r r r r r 2
r r r
Trr T 1 vr 1 v vr
= 2
r r r r 2 r 2
v v T 2v 2 vz
Trz = r + z rz = 2r +
z r z z r z

Thus,
T 1 Tr Trr T Trz p 2v
(divT)r = rr + + + = + 2 2r
r r r z r r
1 2 vr 1 v 1 2 v 1 vr 1 v vr 2 vr 2 vz
+ 2 2 + + 2 + +
r r r r r r 2 r 2
2 r z
r z
2

p 2v 1 2 vr 2 vr 1 vr 2 v vr
= + 2r + 2 + 2 +
r r r 2 z r r r 2 r 2
2v 1 v 1 2 v vr 1 vr 2 vz
+ 2r 2 + + +
r r r r r 2 r r r z

But using the equation of continuity, we have,
2 vr 1 v 1 2 v vr 1 vr 2 vz vr 1 v vr vz
2 2 + + + = + + + =0
r r r r r 2 r r r z r r r r z
Thus,
(divT)r + Br = ar
p 2v 1 2 vr 2 vr 1 vr 2 v vr
+ 2r + 2 + 2 + + Br = ar
r r r 2 z r r r 2 r 2
__________________________________________________________________

Copyright 2010, Elsevier Inc


6-9
Lai et al, Introduction to Continuum Mechanics

6.18 Use the results of Sect. 2.35, chapter 2 and the constitutive equations for the Newtonian
viscous fluid, verify Navier-Stokes Equation in the r-direction in spherical coordinates, i.e., Eqs.
(6.8.5).
--------------------------------------------------------------------------------
Ans. For a Newtonian fluid, the stress tensor in spherical coordinates is given by:
v 1 vr v v 1 vr v v
p + 2 r + +
r r r r r sin r r

1 v vr 1 v v cot 1 v
[T] = T p + 2 + +
r r
r sin r r

1 v v v cot

T r T p + 2 + r +

r sin r r
The equation of motion in the r-direction is given by [see Eq.(4.8.4)]
(
1 r Trr
2
)+
1 (Tr sin )
+
1 Tr T + T
- + Br = ar .
r2 r r sin r sin r
We also have the equation of continuity [see Eq.(2.35.26) or Eq.(6.8.8)]
vr 1 v 2vr 1 v v cot
+ + + + =0.
r r r r sin r
Now,
(
1 r Trr
2
=
)
p 2 p
+ 2
2 vr 2 vr
2 + ,
r2 r r r r r r
1 (Tr sin )
r sin
1 v v 1 v 1 2 v 1 v 1 2 v
= 2 r 2 + cot + r
+ .
r r r r 2 r 2 r r
r
2

1 Tr 1 2 vr 1 v 1 v
2
= 2 2 +
r sin r sin 2 r 2 sin r sin r
T + T 2p 1 v 2v 1 v v cot
=+ 2 2 + 2r 2 2 + .
r r r r r sin r2
Thus.

Copyright 2010, Elsevier Inc


6-10
Lai et al, Introduction to Continuum Mechanics

(
1 r Trr
2
+
) 1 (Tr sin )
+
1 Tr

T + T
=
r 2 r r sin r sin r
p 2 p 2v 2 vr 1 vr v 1 v
+ 2 2r + + 2 2 + r r cot
r r r r r r r
1 2 v 1 v 1 2 v 1 2 vr 1 v 1 v
2
+ 2 r
2 + + 2 2 +
r r r r r sin 2 r 2 sin r sin r
2

2p 1 v 2v 1 v v cot
+ 2 2 + 2r 2 2 +
r r r r sin r2
p 2v 2 vr 2vr 1 2 vr 1 vr 1 2 vr
= + 2r + 2 + 2 + cot +
r r r r r 2 r 2 r 2 sin 2 2
r
2 v 2 v 2v cot
+ 2 2 2
r sin r r
2v 2 vr 2vr 1 2 v 1 v 1 v v 1 v 1 v
2
+ 2 + r
2 + + cot 2 cot 2 +
r r r r r r r 2 r r r r sin r sin r
Now, differentiate the equation of continuity with respect to r, we have,
vr 2vr 1 v 1 v v cot
+ + + + = 0 , that is,
r r r r r sin r
1 v 1 v
2
2 vr 2 vr 2vr 1 2 v 1 v
+ + + 2 +
r 2 r r r 2 r r r r sin r r sin
2

cot v v cot
=0
r r r2

Thus, 2
(
1 r Trr
2
+
) 1 (Tr sin )
+
1 Tr

T + T
r r r sin r sin r
p 2v 2 vr 2vr 1 2 vr 1 vr 1 2 vr
= + 2r + 2 + 2 + cot +
r r r r r 2 r 2 r 2 sin 2 2
r
2 v 2 v 2v cot
+ 2 2 2
r sin r r
1 2 2 vr
p
= +
r r r r
2 (
r v r )
+
1
r 2 sin
sin
vr
+
1
r 2 sin 2 2

2 v 2
+ 2 2 ( v sin )
r sin r sin
Finally, the Navier-Stokes equation in r direction is:

Copyright 2010, Elsevier Inc


6-11
Lai et al, Introduction to Continuum Mechanics

1 2 2 vr
=
p
r
+ r v r + ( 1
)
r 2 sin
sin
vr
+
1
r 2 sin 2 2

r r r
2

2 v 2
+ 2 2 ( v sin ) + Br = ar
r sin r sin
__________________________________________________________________

6.19 Show that for a steady flow, the streamline containing a point P coincides with the pathline
for a particle which passes through the point P at some time t .
-------------------------------------------------------------------------------
Ans. For a steady flow, the velocity at every point on a streamline does not change with time.
Therefore, any particle, which is at a point P on the streamline at a given time t, will move along
the streamline at all time. That is, its pathline coincides with the streamline containing the point
P. We can also demonstrate this mathematically as follows:
For steady flow, the velocity field is independent of time, that is, v = v (x) . Let x = x(t ) be the
pathline, then, the differential system for the pathline is:
= v {x ( t )} , subjected to the condition x ( to ) = xo
dx
(1)
dt
Let x = x( s ) be the parametric equation for the streamline passing though xo , then the differential
system for the streamline is:
= v {x ( s )} , subjected to the condition x ( so ) = xo
dx
(2)
ds
The two differential systems are identical. They determine the same curve.
_________________________________________________________________

6.20 Given the two dimensional velocity field


kx x
v1 = 1 2 , v2 = 0
1 + kx2t
(a) Find the streamline at time t , which passes through the spatial point (1 , 2 ) and,
(b) find the pathline equation x = x(t ) for a particle which is at ( X1 , X 2 ) at time to
-------------------------------------------------------------------------------
Ans. (a) Since the flow is in e1 direction only, therefore, both the streamline and the pathline are
straight line in the e1 direction. The streamline equation which passes through the spatial point
(1 , 2 ) is simply x2 = 2 .
(b) The pathline for a particle which is at ( X1 , X 2 ) at time to is simply x2 = X 2 . To find the time
history of the particle along the pathline, i.e., to find x = x(t ) with X = x(to ) ,we have,
x t
dx1 kx X 1
dx kX 2 x 1 + kX 2t
= 1 2 1= dt ln 1 = ln ,
dt 1 + kX 2t X
x1 t 1 + kX 2t X1 1 + kX 2to
1 o

1 + kX 2t
x1 = X1 and x2 = X 2 .
1 + kX 2to
_________________________________________________________________

6.21 Given the two dimensional flow: v1 = kx2 , v2 = 0

Copyright 2010, Elsevier Inc


6-12
Lai et al, Introduction to Continuum Mechanics

(a) Obtain the streamline passing through the point (1 , 2 ) . (b) Obtain the pathline for the
particle which is at ( X1 , X 2 ) at t = 0 , including the time history of the particle along the pathline
------------------------------------------------------------------------------
Ans. (a) The streamline is clearly x2 = 2 .
(b) The pathline for the particle which is at ( X1 , X 2 ) at time 0 is simply x2 = X 2 . To find the
time history of the particle along the pathline, i.e., to find x = x(t ) with X = x(0) , we have,
dx2 dx
= 0 x2 = X 2 , 1 = kX 2 x1 = X1 + kX 2t , 0 t .
dt dt
_________________________________________________________________

6.22 Do Prob. 6.21 for the following velocity field : v1 = x2 , v2 = x1 .


-------------------------------------------------------------------------------
dx1 dx2 dx1 x
Ans. (a) From = x2 , = x1 = 2 x1dx1 + x2 dx2 = 0 ,
ds ds dx2 x1
x12 + x22 = C x12 + x22 = 12 + 22 . The streamline is a circle.
(b) Since the flow is steady, clearly, the pathline is also a circle given by
x12 + x22 = X12 + X 22 . To find the time history of the particle along the pathline, i.e., to find
x = x(t ) with X = x(0) , we have,
dx1 dx2 d 2x dx d2x
= x2 , = x1 21 = 2 = 2 x1 21 + 2 x1 = 0 ,
dt dt dt dt dt
1 dx1
x1 = A sin t + B cos t , x2 = = A cos t B sin t .
dt
at t = 0, x1 = X1 , x2 = X 2 , thus, x1 = X 2 sin t + X1 cos t , x2 = X 2 cos t X1 sin t
_________________________________________________________________

Q
6.23 Given the following velocity field in polar coordinates ( r , ) : vr = , v = 0 .
2 r
(a) Obtain the streamline passing through the point ( ro , o ) ,
(b) Obtain the pathline for the particle which is at ( R, ) at t = 0 , including the time history of
the particle along the pathline
-------------------------------------------------------------------------------
Ans. Both the streamline and the path lines are radial lines with =constant.
(a) the streamline passing through the point r = ro , = o is = o .
(b) the pathline for the particle which is at ( R, ) at t = 0 is = . To find the time history of
the particle, we have,
d dr Q r t Q Q
= 0 = , = rdr = dt r 2 = R 2 + t .
dt dt 2 r R 0 2
_________________________________________________________________

6.24 Do Prob. 6.23 for the following velocity field in polar coordinates ( r , ) :
vr = 0, v = C / r .
------------------------------------------------------------------------

Copyright 2010, Elsevier Inc


6-13
Lai et al, Introduction to Continuum Mechanics

Ans. Both the streamline and the path lines are circles r = constant .
(a) the streamline passing through the point r = r , = o is r = ro
(b) the pathline for the particle which is at ( R, ) at t = 0 is r = R . To find the time history of
dr d C d C C
=0r = RR
the particle, we have, = = 2 = 2 t + .
dt dt R dt R R
_________________________________________________________________

6.25 From the Navier-stokes equations, obtain Eq. (6.11.2) for the velocity distribution of the
plane Couette flow.
-------------------------------------------------------------------------------
Ans. With x2 axis pointing vertically upward, we have
v1 = v ( x2 ) , v2 = 0, v3 = 0 and a1 = a2 = a3 = 0 , thus, with p = p ( x2 ) , the Navier-Stokes equation
in the x1 direction become
d 2v vo v
0= v = C1 x2 + C2 . At x2 = 0, v = 0, C2 = 0. At x2 = d , v = vo C1 = v = o x2 .
dx22 d d
_________________________________________________________________

6.26 For the plane Couette flow, if in addition to the movement of the upper plate, there is also
an applied negative pressure gradient p / x1 , obtain the velocity distribution. Also obtain the
volume flow rate per unit width.
-------------------------------------------------------------------------------
Ans. With x2 axis pointing vertically upward, we have
v1 = v ( x2 ) , v2 = 0, v3 = 0 and a1 = a2 = a3 = 0 , thus, the Navier-Stokes Equations become,
p d 2v p
0= + 2 =0
x1 dx2 x1 x1
p p p
0= = =0
x2 x1 x2 x2 x1
p p p
0= = =0
x3 x1 x3 x3 x1
p
Thus = constant
x1
d 2v x2
= v = 2 + C1 x2 + C2 , At x2 = 0, v = 0 C2 = 0. At x2 = d , v = vo
dx22 2
d2 v d
vo = + C1d C1 = o +
2 d 2
x 2 v d
v = 2 + o + x2 =
2 d 2 2
( vo
)
x2 d x2 + x2 .
2
d
The volume flow rate per unit width is given by

Copyright 2010, Elsevier Inc


6-14
Lai et al, Introduction to Continuum Mechanics

d d
x2 v d
Q = v ( x2 )dx2 = 2 + o + x2 dx2
0 2 d 2
0

d 3 v d d 2 d 3 vo d 2
= + o + = +
6 d 2 2 12 d 2
_________________________________________________________________

6.27 Obtain the steady uni-directional flow of an incompressible viscous fluid layer of uniform
depth d flowing down an inclined plane which makes an angle with the horizontal.
-------------------------------------------------------------------------------
Ans. With x2 axis normal to the flow and pointing away from the fluid and x1 axis in the flow
direction, we are looking for the velocity field in the following form: v1 = v ( x2 ) , v2 = 0, v3 = 0 ,
which clearly satisfies the continuity equation. Now the N-S equations give
p d 2v p
0= + g sin + 2 =0.
x1 dx2 x1 x1
p p p
0= g cos = =0
x2 x1 x2 x2 x1
p p p
0= =0 = = 0 . Thus
x3 x1 x3 x3 x1
p
=C.
x1
The constant C can be determined from the pressure condition on the free surface ( x2 = d ),
where pressure p = pa , the atmospheric pressure which is independent of x1 , thus
p / x1 = 0 C = 0 so that p / x1 = 0
for the whole flow field. Thus,
d 2v d 2v dv
0 = g sin + 2 2 = g sin = g sin x2 + C1
dx2 dx2 dx2
x22
v = g sin + C1 x2 + C2 . At x2 = 0, v = 0 (non slip condition) C2 = 0 .
2
At x2 = d , shear stress T12 = 0 dv /dx2 =0 C1 = gdsin .
x
v = gsin d 2 x2 .
2
__________________________________________________________________

6.28 A layer of water ( g = 62.4lb / ft 3 ) flows down an inclined plane ( = 30o ) with a
uniform thickness of 0.1 ft . Assuming the flow to be laminar, what is the pressure at any point
on the inclined plane. Take the atmospheric pressure to be zero.
------------------------------------------------------------------------------
Ans. With flow in the x1 direction, the N-S equation in the x2 direction (pointing away from the
inclined plane) gives, [note: p is independent of x1 and x3 , see Prob. 6.27]
p / x2 g cos = 0 p = ( g cos ) x2 + C .

Copyright 2010, Elsevier Inc


6-15
Lai et al, Introduction to Continuum Mechanics

At x2 = d , p = pa = 0 ( g cos ) d = C , p = ( g cos )( d x2 ) .

(
At x2 = 0 p = ( g cos )( d ) = 62.4cos30o ( 0.1) = 5.40 lb / ft 2 . )
We can also obtain the same result by using the fact that the piezometric head
p / ( g ) + z =constant for any points on the same plane perpendicular to the direction of the flow
p p
(see example 6.7.2), therefore a + za = b + zb pb = g ( za zb ) = gd cos .
g g
__________________________________________________________________

6.29 Two layers of liquids with viscosities 1 and 2 , densities 1 and 2 respectively, and with
equal depths b , flow steadily between two fixed horizontal parallel plates. Find the velocity
distribution for this steady uni-directional flow. Neglect body forces.
-------------------------------------------------------------------------------
Ans. We are looking for velocity fields in the two layers in the following form corresponding to
the uni-directional steady laminar flows:
(t ) (t ) (t )
For the top layer: v1 = v( ) ( x2 ) , v2 = v3 = 0 .
t

(b) (b) (b )
For the bottom layer: v = v( ) x , v = v = 0 . ( 2)
b
1 2 3
From the N-S equations for the top layer, we have
0 = p( ) / x1 + 1d 2 v( ) / dx22 ( / x1 )(p( ) / x1 ) = 0
t t t

0 = p( ) / x2 ( / x1 )(p( ) / x2 ) = ( / x2 )(p( ) / x1 ) = 0
t t t

0 = p( ) / x ( / x )(p( ) / x ) = ( / x )(p( ) / x ) = 0
t t t
3 1 3 3 1

Thus, p ( ) / x1 = 1 (a constant) . Now,


t

1d 2 v( t ) / dx22 = 1 1dv( t ) / dx2 = 1 x2 + A1 1v( t ) = (1 x22 / 2) + A1 x2 + B1.


Similarly from the N-S equations for the bottom layer, we have,
p( ) / x = (a constant) .
b
1 2

2 dv( b ) / dx2 = 2 x2 + A2 2 v( b ) = ( 2 x22 / 2) + A2 x2 + B2 .


The constants A1 , B1 , A2 , B2 will be determined from the boundary and the interface conditions:
At x = b (the top plate), v( ) = 0 , 0 = ( b 2 / 2) + A b + B
t
2 1 1 1 (1)
At x2 = b (the bottom plate), v( ) = 0 , 0 = ( 2b 2 / 2) A2b + B2
b
(2)
At x2 = 0 (the interface), there is no slip between the two layers of flow, i.e., v = v (t ) (b )
B1 / 1 = B2 / 2 (3)
rd
Also, according to Newtons 3 law, the action and reaction at the interface between the two fluid
must be equal and opposite, that is, both the shear stress and the normal stress must by continuous
at x = 0 . Since T
2
(t )
= dv( ) / dx
12
x2 =0
t
1 ( =A, T
(b)
2 ) = dv( ) / dx
x2 =0
t
1 =A
12
x2 =0
2 ( 2 ) x2 =0
2

Therefore, A1 = A2 , (4)
(t ) (b) (t ) (b)
= p( ) ( x1 ,0), T22 = p( ) ( x1 ,0),
t b
and T22 T22 = T22
x2 =0 x2 =0 x2 = 0 x2 = 0

p( ) ( x1 ,0) = p( ) ( x1 ,0) (p( ) / x1 )( x1 ,0) = (p( ) / x1 )( x1 ,0) 1 = 2 p / x1.


t b t b

Now, Eqs.(1)(2)(3)(4) determine the four constants A1 , B1 , A2 , B2 as a function of = p / x1 :

Copyright 2010, Elsevier Inc


6-16
Lai et al, Introduction to Continuum Mechanics

1 2 1 2 2
A1 = A2 = b 2 , B1 = b , B2 = b . Thus,
2 ( 1 + 2 ) 1 + 2 1 + 2
x22b 1 2 1
1v( t ) = 2 x2 b ,
2 2 1 + 2 1 + 2
x b
2
2
2 v( ) = 2 2 1 x2 b 2
b
.
2 2 1 + 2 1 + 2
__________________________________________________________________

6.30 For the Couette flow of Section 6.15, (a) obtain the shear stress at any point inside the fluid
(b) obtain the shear stress on the outer and inner cylinder (c) obtain the torque which must be
applied to the cylinders to maintain the flow.
-------------------------------------------------------------------------------
Ans. (a) Eq.(6.15.4) and (6.15.7), give v = Ar + B / r , where B = r12 r22 ( 1 2 ) / (r22 r12 ) .
d v 2 B 2 r12 r22 ( 1 2 ) 1
Thus, Tr = T r = 2 Dr = r = =
dr r r2 r22 r12 r2
(b) On the outer wall: r = r2 , the shear stress is Tr = T r = 2 r12 (2 1 ) / (r22 r12 ) ,
On the inner wall, r = r1 , the shear stress is Tr = T r = 2 r22 ( 2 1 ) / (r22 r12 ) .
(c) On the outer wall, per unit height, the torque is given by
2 r 2 ( ) 4 r12 r22 ( 2 1 )
r r
(
( M )r2 = (Tr )r =r2 ( 2 r2 ) (1) r2e = 1 2 2 2 1 2 r22 e =) r2 r2
e
2 1 2 1
The torque on the inner wall is equal and opposite to that on the outer wall.
__________________________________________________________________

6.31 Verify the equation 2 = / 2 for the oscillating plane problem of Section 6.16.
--------------------------------------------------------------------------------
Ans. With v = e x2 cos(t x2 + ) , v / t = e x2 sin(t x2 + ) ,
v / x2 = e x2 cos(t x2 + ) + e x2 sin(t x2 + ) and
2 v / x22 = 2 e x2 cos(t x2 + ) 2 e x2 sin(t x2 + )
2 e x2 sin(t x2 + ) 2 e x2 cos(t x2 + )
= 2 2 e x2 sin(t x2 + )
Thus v / t = 2 v / x22
e x2 sin(t x2 + ) = 2 2 e x2 sin(t x2 + )
= 2 2 2 = / (2 ).
__________________________________________________________________

6.32 Consider the flow of an incompressible viscous fluid through the annular space between
two concentric horizontal cylinders. The radii are a and b . (a) Find the flow field if there is no
variation of pressure in the axial direction and if the inner and the outer cylinders have axial

Copyright 2010, Elsevier Inc


6-17
Lai et al, Introduction to Continuum Mechanics

velocities va and vb respectively and (b) find the flow field if there is a pressure gradient in the
axial direction and both cylinders are fixed. Take body forces to be zero.
--------------------------------------------------------------------------------
Ans. (a) We look for the following form of velocity field in cylindrical coordinates:
vr = 0, v = 0, vz = v(r ) and p / z = 0 . The N-S equations give, in the absence of body forces
p 1 p d 2 v 1 dv
0= , 0= , 0 = 2 + .
r r dr r dr

The first two equations together with p / z = 0 give, p = constant .
d 2 v 1 dv 1 d dv dv dv C
0 = 2 + r = 0 r =C = v = C ln r + D
dr r dr r dr dr dr dr r

At r = a, v = va and at r = b, v = vb , thus,
va = C ln a + D and vb = C ln b + D va vb = C ln(a / b) .
v v v ln b vb ln a
C = a b and D = a . So that,
ln ( a / b ) ln ( b / a )
va vb v ln b vb ln a
v= ln r + a .
ln ( a / b ) ln ( b / a )
(b) 0 = p /r and 0 = (1 / r )p / p = p ( z ) ,
dp d 2 v 1 dv d 2 p dp
0= + 2 + 2 =0 = constant ,
dz dr r dr dz
dz
d 2 v 1 dv d dv d dv r
2 + = r = r =
dr r dr r dr dr dr dr

dv r 2
dv r C r2
r = +C = + v= + C ln r + D .
dr 2 dr 2 r 4
The boundary conditions are: v( a ) = v(b) = 0
a2 b2
0= + C ln a + D and 0 = + C ln b + D
4 4

C=
a b
2
( 2
)
=
1 dp a b
2
( 2
, D=
)
1 dp a ln b b ln a
2 2
( )
4 ln ( a / b ) 4 dz ln ( a / b ) 4 dz ln ( b / a )

v=
1 dp 2 a b
r +
2
(
2
)
ln r +
(
b 2 ln a a 2 ln b ) .
4 dz ln ( b / a ) ln ( b / a )

__________________________________________________________________

6.33 Show that for the velocity field : vx = v( y, z ), v y = vz = 0 ,

2v 2v 1 dp
the Navier-Stokes equations, with B =0, reduces to + = = = constant .
y z
2 2 dx
-------------------------------------------------------------------------------
Ans. With vx = v( y, z ), v y = vz = 0 , we have,

Copyright 2010, Elsevier Inc


6-18
Lai et al, Introduction to Continuum Mechanics

vx v v v
ax = + vx x + v y x + vz x = 0 + 0 + 0 + 0 = 0, and a y = az = 0 , thus, the N-S equations in
t x y z
the absence of body forces are:
p 2v 2v p p
0 = + 2x + 2x , 0 = , 0 = . Thus, p = p ( x) , and
x y z y z

2v 2 v dp 2v 2v d 2 p d2 p
2x + 2x = 2x + 2x = 2 0 = 2 ,
z dx x y z dx
y dx
dp 2v 2v 1 dp
= constant 2x + 2x = .
dx y z dx
__________________________________________________________________

6.34 Given the velocity field in the form of

( )
vx = v = A y 2 / a 2 + z 2 / b 2 + B , v y = vz = 0 . Find A and B for the steady laminar flow of a

Newtonian fluid in a pipe having an elliptical cross section given by y 2 / a 2 + z 2 / b 2 = 1 . Assume


no body forces and use the governing equation obtained in the previous problem.
-------------------------------------------------------------------------------
Ans. The governing equation is [see the previous problem] :
2 vx 2 vx 1 dp
y 2
+ 2 =
z dx
(
. Now, vx = v = A y 2 / a 2 + z 2 / b 2 + B )
2 vx 2 vx 2 2 a2 + b2 a2 + b2 a 2b 2
+ = A 2 + 2 = 2 A 2 2 = A =
( )
2 A
y 2 z 2 a a 2b 2 2 a 2 + b2
b a b
On the boundary y 2 / a 2 + z 2 / b 2 = 1 , no slip condition requires that vx = 0 , therefore,
A (1) + B = 0 B = A , thus,
y 2 z 2 a 2b 2 y 2 z 2
vx = A 2 + 2 1 = 2 + 2 1 .
a b 2 a 2 + b 2 ( ) a b
__________________________________________________________________

6.35 Given the velocity field in the form of


b b b
vx = A z + z + 3 y z 3 y + B , v y = vz = 0
2 3 3 3
Find A and B for the steady laminar flow of a Newtonian fluid in a pipe having an equilateral
triangular cross-section defined by the planes:
b b b
z+ = 0, z + 3 y = 0, z 3 y =0.
2 3 3 3
Assume no body forces and use the governing equation obtained in Prob. 6.33.
-------------------------------------------------------------------------------
2 vx 2 vx 1 dp
Ans. The governing equation is + 2 = . [See problem 6.33]
y 2 z dx
( )( )(
With vx = A z + b / (2 3) z + 3 y b / 3 z 3 y b / 3 + B , )

Copyright 2010, Elsevier Inc


6-19
Lai et al, Introduction to Continuum Mechanics

(
vx / y = A z + b / (2 3) ){ 3 ( z ) (
3y b / 3 3 z + 3y b / 3 )}
( )
= A z + b / (2 3) ( 6 y )

2 vx / y 2 = 6 A ( z + b / (2 3) ) .

Let f ( z ) = ( z + b / (2 3) ) , g ( y, z ) = ( z + ) (
3 y b / 3 , h( y , z ) = z 3 y b / 3 , )
then vx = Af ( z ) g ( y, z )h( y, z ) + B vx / z = Ag ( y , z )h( y, z ) + Af ( z ) {h( y , z ) + g ( y, z )}
( ) (
Now, h( y , z ) + g ( y, z ) = z + 3 y b / 3 + z 3 y b / 3 = 2 z b / 3 ) ( )
( )( ) (
f ( z ) {h( y, z ) + g ( y, z )} = 2 z b / 3 z + b / (2 3) = 2 z 2 zb / 3 b 2 / 3)

( 3 ) 3 y 2 = z 2 2bz /
2
g ( y , z ) h( y , z ) = z b / 3 + b 2 / 3 3 y 2 . Thus,
vx
= Ag ( y, z )h( y, z ) + Af ( z ) {h( y , z ) + g ( y , z )}
z
( ) ( ) (
= A z 2 2bz / 3 + b 2 / 3 3 y 2 + A 2 z 2 zb / 3 b 2 / 3 = A 3z 2 3bz / 3 3 y 2 )
vx
2
/ z = A ( 6 z 3b / 3 ) = 6 A ( z b / (2 3) ) . Thus,
2

2 vx v
= 6 A ( z + b / (2 3) ) + 6 A ( z b / (2 3) ) = 6 A ( b / 3 ) =
2
1 dp
+ x
,
y 2
z 2 dx
from which, A = / (2 3b) . The non-slip condition on the boundary requires B = 0 .
_________________________________________________________________

6.36 For the steady-state, time dependent parallel flow of water ( density = 103 Kg / m3 ,
viscosity, = 103 Ns / m2 ) near an oscillating plate, calculate the wave length for = 2 cps .
-------------------------------------------------------------------------------
Ans. v = ae x2 ( cos t x2 + ) , the wave length is given by 2 / , where

= . Here we have, = 103 Kg / m3 , = 4 rad / s , = 103 Ns / m 2 , thus
2

=

=
(10 ) ( 4 ) =10
3
3
2 m 1 wave length=
2
=
2
= 2.51 103 m
2 2 (10 ) 3 3
10
_________________________________________________________________

6.37 The space between two concentric spherical shells is filled with an incompressible
Newtonian fluid. The inner shell (radius ri ) is fixed; the outer shell (radius ro ) rotates with an
angular velocity about a diameter. Find the velocity distribution. Assume the flow to be
laminar without secondary flow.
-------------------------------------------------------------------------------

Copyright 2010, Elsevier Inc


6-20
Lai et al, Introduction to Continuum Mechanics

Ans. We look for solution in the form of vr = 0, v = 0, v = f ( r ) sin . This velocity field
clearly satisfies the continuity equation [see Section 6.8, Eq.(6.8.8)]:
vr 2vr 1 v v cot 1 v
+ + + + =0.
r r r r r sin
The N-S equations in spherical coordinates give[see Section 6.8]:
v2 1 p v2 cot 1 p
= (1), = (2),
r r r r
p 1 2 v 1 1
0=
1
r sin
+ 2 r + 2
r r r r sin
( v sin )

(3).

2 p 2 p 2 p
Eq.(1) = 0, eq.(2) = 0 and eq.(3) =0.
r
Thus, p / = constant . The constant must be zero, otherwise p will not be single-valued.
Eq. (3) now becomes, with p / = 0 and v = f ( r ) sin ,
sin d 2 df 2 f ( r ) d 2 df 2
2 d f df
0= 2 dr
r sin . That is, r 2 f = 0 r +2r 2f =0.
r dr r 2 dr dr dr 2 dr
(
The general solution of this equation is: f = Ar + B / r 2 . Thus, v = Ar + B / r 2 sin . )
The inner shell (radius ri ) is fixed; therefore, at r = ri , v = 0 (
0 = Ari + B / ri 2 ) (4)
The outer shell (radius ro ) rotates with an angular velocity , therefore, at

( ) (
r = ro , v = ( ro sin ) ( ro sin ) = Aro + B / ro2 sin ro = Aro + B / ro2 ) (5)
Equations (4) and (5) are two equations for the two unknowns A and B :
ro3 r 3r 3 B
A= , B = o i , and v = Ar + 2 sin .
(
ro3 ri3 ) ro3 ri3( ) r
_________________________________________________________________

6.38 Consider the following velocity field in cylindrical coordinates for an incompressible fluid:
vr = v(r ), v = 0, vz = 0
A
(a) Show that vr = where A is a constant so that the equation of conservation of mass is
r
satisfied. (b) If the rate of mass flow through the circular cylindrical surface of radius r and unit
length (in z direction) is Qm , determine the constant A in terms of Qm .
-------------------------------------------------------------------------------
Ans. (a) The equation of continuity is
1 1 v v 1 d A
( rvr ) + + z = 0 . Thus, ( rvr ) = 0 rvr = A vr = .
r r r z r dr r
Qm A Qm Qm
(b) vr ( 2 r )(1) = Qm vr = = A= .
2 r r 2 r 2
_________________________________________________________________

6.39 Given the following velocity field in cylindrical coordinates for an incompressible fluid:
vr = v(r , ), v = 0, vz = 0

Copyright 2010, Elsevier Inc


6-21
Lai et al, Introduction to Continuum Mechanics

(a) Show that vr = f ( ) / r , where f ( ) is any function of . (b) In the absence of body forces,
show that
d2 f f2 f k
+ 4 f + + k = 0 , p = 2 2 + 2 + C , where k and C are constants.
d 2 r 2r
-------------------------------------------------------------------------------
Ans. (a). The equation of continuity is
1 1 v v 1
( rvr ) + + z = 0 . Thus, ( rvr ) = 0 rvr = f ( ) vr = f ( ) / r
r r r z r r
(b).The N-S equation in the r direction gives
vr v v v v 1 p
+ vr r + r v + vz r = + Br
t r r z r
2 vr 1 2 vr 2 vr 1 vr 2 v vr f2 1 p 1 d 2 f
+ 2 + 2 + + = + (1)
r r 2 z 2 r r r 2 r 2 r3 r r 3 d 2
The N-S equation in the z direction gives p / z = 0 p is independent of z.
The N-S equation in the direction gives
1 p 2 vr 1 p 2 df p 2 df
= + 2 + =0 =
r r r r d
3 r 2 d
2 f p 4 f dg
p = 2 + g (r ) = 3 + . Thus, Eq.(1) gives:
r r r dr
f2 r 3 dg d 2 f d2 f f 2 r 3 dg
= + + 4 f + 4 f + = .
dr d 2 d 2 dr

The left side of the above equation is a function of , the right side is a function of r , therefore,
they must be equal to a constant, say, k , i.e.,
d2 f f 2 r 3 dg k
2 + 4 f + = = k g = 2 + C . Therefore,
d dr 2r
d2 f f2 2 f k
2 + 4 f + + k = 0, p = 2 + 2 + C .
d r 2r
_________________________________________________________________

6.40 Consider the steady two dimensional channel flow of an incompressible Newtonian fluid
under the action of an applied negative pressure gradient p / x1 , as well as the movement of the
top plate with velocity vo in its own plane.[See Prob. 6.26]. Determine the temperature
distribution for this flow due to viscous dissipation when both plates are maintained at the same
fixed temperature o . Assume constant physical properties.
-------------------------------------------------------------------------------
( )
Ans. From the result of Prob. 6.26, we have v1 = vo x2 / d + ( / 2 ) x2 d x22 , v2 = v3 = 0.
Let the temperature distribution be denoted by = ( x2 ) . From Eq. (6.18.3), we have,

c
D
Dt
= inc +
2
x j x j
, where inc = 2 D11
2
(
+ D22
2
+ D33
2
+ 2 D12
2
+ 2 D13
2
+ 2 D23
2
)
, represents

the heat generated through viscous forces. For this problem, only D12 is nonzero, thus,

Copyright 2010, Elsevier Inc


6-22
Lai et al, Introduction to Continuum Mechanics

2 2
1 v vo
2
inc = 4 D12
2
= ( 4 ) o + ( d 2 x2 ) = + ( d 2 x2 ) .
2 d 2 d 2
2 3
2 vo 2 vo
= + ( d 2 x )
2 = + ( d 2 x2 ) + C
2 x2 d 2 x2 3 d 2
4
3 vo
= + ( d 2 x2 ) + Cx2 + D .
12 2 d 2
4
3 vo d
At x2 = 0, = o o = + + D.
12 2 d 2
4
3 vo d
At x2 = d , = o o = + Cd + D.
12 2 d 2
Thus,
3 vo d
4
3 v d vo d
4 4
D = o + + and 0 = +
o
+ Cd
12 2 d 2 12 2
d 2 d 2

3 vo d vo d
4 4
C = + .
12 2 d d 2 d 2
_________________________________________________________________

6.41 Determine the temperature distribution in the plane Poiseuille flow where the bottom plate
is kept at a constant temperature 1 and the top plate 2 . Include the heat generated by viscous
dissipation.
-------------------------------------------------------------------------------
Ans. For the plane Poiseuille flow [see Eq.(6.12.9)],
( )
v1 = ( / 2 ) b 2 x22 , v2 = v3 = 0, p / x1 > 0
Let the temperature distribution be denoted by = ( x2 ) . From Eq. (6.18.3), we have,

c
D
Dt
= inc +
2
x j x j
(
, where inc = 2 D11
2
+ D22
2
+ D33
2
+ 2 D12
2
+ 2 D13
2
+ 2 D23
2
)
, represents

the heat generated through viscous forces. For this problem, only D12 is nonzero,
2
1 v1 1 1 2 2
D12 = = x2 inc = 4 D12
2
= 4 x2 = x . Thus,
2 x2 2 2 2
2
D 2 2 2 d 2 d 2 1 p 2
c = inc + 0= x2 + 2 = x
Dt x j x j dx2 dx22 x1 2
2 2
d 1 p x23 1 p x24
= + C = + Cx2 + D .
dx2 x1 3 x1 12

Copyright 2010, Elsevier Inc


6-23
Lai et al, Introduction to Continuum Mechanics

2
1 p b 4
At x2 = +b, = 2 2 = + Cb + D
x1 12
2
1 p b 4
At x2 = b, = 1 1 = Cb + D . Thus,
x1 12
2
1 + 2 1 p 4 2 1
D= + b . C= .
2 12 x1 2b
__________________________________________________________________

6.42 Determine the temperature distribution in the steady laminar flow between two coaxial
cylinders (Couette flow) if the temperatures at the inner and the outer cylinders are kept at the
same fixed temperature o .
-------------------------------------------------------------------------------
B
Ans. For Couette flow, we have, vr = 0, v = Ar + , vz = 0 , where
r
r 1r1
2 2 r r ( )
2 2
A = 2 22 , B = 1 2 2 1 2 2 . The only nonzero rate of deformation is
r2 r12
r2 r1
1 1 vr v 1 v v B
Dr = v + r = 2 r + r = 2 .
2 r r
2
4 B 2
( ) B
inc = 2 2 Dr2 = 4 2 = 4 . c
r r
D
Dt
= inc +
2
x j x j

4 B 2 1 d d d d 4 B 2 d 2 B 2 C
0= + r r = = +
r4 r dr dr dr dr r3 dr r3 r
B2
= + C ln r + D .
r2
B2
At r = ri , = o o = + C ln ri + D.
ri2
B2
At r = ro , = o o = + C ln ro + D.
ro2
Thus,

r 2 r 2 B 2 ri ro B 2 2
C = o 2 2i
r r / ln , D = o ln +
ri ri ro
2 2
ro ln {
ro ri
2
ln r}
i
r
/ ln o .
i o ro ri
_________________________________________________________________

6.43 Show that the dissipation function for a compressible fluid can be written as that given in
Eq. (6.17.10).
-------------------------------------------------------------------------------
Ans. From
= ( D11 + D22 + D33 ) + 2 D11
2 2 2
(
+ D22 + D33
2
+ 2 D12
2
+ 2 D13
2
+ 2 D23
2
)
, we get,

Copyright 2010, Elsevier Inc


6-24
Lai et al, Introduction to Continuum Mechanics

= ( + 2 ) D11
2
(+ D22
2
+ D33
2
)
+ 2 ( D11D22 + D11D33 + D22 D33 ) + 4 D12
2
+ D13
2
+ D23
2
( )
We now verify that this is the same as
2
= ( + 2 / 3) ( D11 + D22 + D33 ) + ( D11 D22 ) + ( D11 D33 ) + ( D22 D33 )
2 2 2 2
3
2
(
+4 D12 + D13
2
+ D23
2
)
Expanding the above equation, we have,
= ( + 2 / 3) D11
2
(
+ D22
2
+ D33
2
+ 2 D11D22 + 2 D11D33 + 2 D22 D33 )
(
+ ( 2 / 3) 2 D11
2
+ D22
2
+ D33
2
)
( 2 D11D22 + 2 D11D33 + 2 D22 D33 ) + 4 D12

2
+ D13
2
+ D23
2
(. )
i.e.,
= ( + 2 / 3) D11(
2
+ D22
2
+ D33
2

2
3
)
+ + ( 2 D11D22 + 2 D11D33 + 2 D22 D33 )

(
+ ( 4 / 3) D11
2
+ D22
2
+ D33
2
)
( 2 / 3) ( 2 D11D22 + 2 D11D33 + 2 D22 D33 ) + 4 D12
2
+ D13
2
+ D23
2
( )
(
= D11
2
+ D22
2
+ D33
2
)
( + 2 / 3 + 4 / 3) + 2 ( D11D22 + D11D33 + D22 D33 ) ( + 2 / 3 2 / 3)
2
(
+4 D12 + D13
2
+ D23
2
. )
That is
(
= D11
2
+ D22
2
+ D33
2
)
( + 2 ) + 2 ( D11D22 + D11D33 + D22 D33 ) + 4 D122 + D132 + D232 ( )
Thus, =
__________________________________________________________________

6.44 Given the velocity field of a linearly viscous fluid


v1 = kx1 , v2 = kx2 , v3 = 0
(a) Show that the velocity field is irrotational. (b) Find the stress tensor. (c) Find the acceleration
field. (d) Show that the velocity field satisfies the Navier-Stokes equations by finding the pressure
distribution directly from the equations. Neglect body forces. Take p = po at the origin. (e) Use
the Bernoulli equation to find the pressure distribution. (f) Find the rate of dissipation of
mechanical energy into heat. (g) If x2 = 0 is a fixed boundary, what condition is not satisfied by
the velocity field.
-------------------------------------------------------------------------------
k 0 0 k 0 0 0 0 0
Ans. (a) [v ] = 0 k 0 [ D] = 0 k 0 , [ W ] = 0 0 0

0 0 0 0 0 0 0 0 0
therefore, the flow is irrotational.
(b) T11 = p + 2 k , T22 = p 2 k , T33 = p, T12 = T13 = T23 = 0 .

0 kx1 k x1
2
a1 k 0
(c) a2 = 0 k
0 kx2 = k 2 x2 .
a3 0 0
0 0 0

(d)

Copyright 2010, Elsevier Inc


6-25
Lai et al, Introduction to Continuum Mechanics

2v 2v 2v
( )
k 2 x1 =
p
x1
+ 21 + 21 + 21 k 2 x1 =

p
x1
x1 x2 x3
2v 2v
(
k 2 x2 ) =
p
x2
2v
+ 22 + 22 + 22 k 2 x2 =

p
x
and
1 x x2 x3 2
p
0= p is independent of x3 .
x3
Thus,
p k 2 x12 p df ( x2 ) df ( x2 )
k 2 x1 = p= + f ( x2 ) = k 2 x2 =
x1 2 x2 dx2 dx2
k 2 x22 k 2
f =
2
+C p =
2
(x
2
1 )
+ x22 + C. At x1 = x2 = 0, p = po C = po

( )( ) (
That is, p = k 2 / 2 x12 + x22 + po p = ( / 2 ) v12 + v22 + po . )
(e) From the Bernoulli Equation, we have
p v2 p v 2 + v22 p v12 + v22 p
+ = constant + 1 = + = o +0
2 2 2
at origin

(
p = po k 2 x12 + x22 / 2. )
(f)
= ( D11 + D22 + D33 ) + 2 D11
22
+ D22
2
(+ D33
2
+ 2 D12
2
+ 2 D13
2
+ 2 D23
2
) (
= 2 k 2 + k 2 = 4 k 2 )
(h) if x2 = 0 is a fixed boundary, then v must be zero there. But v = k ( x1e1 x2e2 ) = kx1e1 0
at x2 = 0 , therefore the non slip boundary condition at x2 = 0 is not satisfied for a viscous fluid.
_________________________________________________________________

( )
6.45 Do Problem 6.44 for the following velocity field: v1 = k x12 x22 , v2 = 2kx1 x2 , v3 = 0 .

-------------------------------------------------------------------------------
Ans.
(a)
2kx1 2kx2 0 x1 x2 0 0 0 0
[v ] = 2kx2 2kx1 0 [ D] = 2k x2 x1 0 , [ W ] = 0 0 0

0 0 0 0 0 0 0 0 0
therefore, the flow is irrotational
(b) T11 = p + 4 kx1 , T22 = p 4 kx1 , T33 = p, T12 = T13 = T23 = 0
(c)

a1 2kx1 2kx2 0 k x1 x2
2
( ) (
2 2k 2 x x 2 + x 2
1 1 2
)
a = 2kx 2kx 0 2kx x = 2k 2 x x 2 + x 2
2 2 1 1 2


(
2 1 2 )
a3 0 0 0 0
0


(d)

Copyright 2010, Elsevier Inc


6-26
Lai et al, Introduction to Continuum Mechanics

2v 2v 2v
(
2k 2 x1 x12 + x22 =
p
)
+ 21 + 21 + 21 2k 2 x1 x12 + x22 =
x
x1 x2 x3
p
x1
( )
1
2v 2v
2k 2 x2
( x12 + x22 =
)
p
x2
2v
+ 22 + 22 + 22 2k 2 x2 x12 + x22 =

p
x2
( )
x1 x2 x3
p
0= p is independent of x3
x3
Thus,
x4
(
2k 2 x1 x12 + x22 =
p
)
p = k 2 1 + x12 x22 + f ( x2 )
x1
p
= 2 k 2 x12 x2 +

df
x2
2 dx2
and

(
2k 2 x2 x12 + x22 =

p
) (
2k 2 x2 x12 + x22 = 2 k 2 x12 x2 +
x2
df
) dx2
df k 2 x24
2 k 2 x23 = f = + C.
dx2 2
Since p = po at origin, therefore, C = po ,
k 2 k2
( ) (x )
2
p= x14 + 2 x12 x22 + x24 + po p = 2
1 + x22 + po .
2 2
Or, since

( )
( )
( )
2 2
v1 = k x12 x22 , v2 = 2kx1 x2 , v12 + v22 = k 2 x12 x22 + 4 x12 x22 = k 2 x12 + x22

(
p = v12 + v22 / 2 + po . )
(e) From the Bernoulli Equation, we have
p v2 p v 2 + v22 p v12 + v22 p
+ = constant + 1 = + = o +0
2 2 2
at origin

( )
2
p = po k 2 x12 + x22 / 2.
(f)
= ( D11 + D22 + D33 ) + 2 D11
2 22
+ D22 + 2 D12
2
( )
= 0 + 2 8k 2 x12 + 8k 2 x22 = 16 k 2 x12 + x22 (h) ( ) ( )
if x2 = 0 is a fixed boundary, then v must be zero there. But v = kx12e1 0 at x2 = 0 , therefore the
non slip boundary condition at x2 = 0 is not satisfied for a viscous fluid.
_________________________________________________________________

6.46 Obtain the vorticity vector for the plane Poiseuille flow.
-------------------------------------------------------------------------------
( )
Ans. With v1 = v ( x2 ) = ( / 2 ) b 2 x22 , where = p / x1 and v2 = v3 = 0 , the spin tensor is
0 v1 / x2 0
1
[ W ] = [ v ] 0
A
= v1 / x2 0
2
0 0 0

Copyright 2010, Elsevier Inc


6-27
Lai et al, Introduction to Continuum Mechanics

The vorticity tensor is 2W and the vorticity vector is twice the axial vector
1 v1 v1 x 1 p
= 2 = 2 (W32e1 + W13e2 + W21e3 ) = 2 e3 = e3 = 2 e3 = xe .
2 x2 x2 x1 2 3
_________________________________________________________________

6.47 Obtain the vorticity vector for the Hagen-Poiseuille flow.


-------------------------------------------------------------------------------
d2 p
Ans. With vz = r 2 , vr =v = 0, = , the spin tensor is
4 4 z

1 1 vr v v 1 vr vz
0 1 v
2 r r r 2 z r 0 0 z
2 r
1 v 1 vz
[ W ] = [v ] = W21
A
= 0 The
2 z r
0 0 0

W 1 vz
2 r
W32 0 0 0
31


vorticity tensor is 2W and the vorticity vector is twice the axial vector
v r 1 p
= 2 = 2 (Wz e r + Wrz e + W r ez ) = 2Wrz e = z e = e = re .
r 2 2 z
_________________________________________________________________

6.48 For a two-dimensional flow of an incompressible fluid, we can express the velocity
components in terms of a scalar function (known as the Lagrange stream function) by the

relations vx = , vy = . (a) Show that the equation of conservation of mass is
y x
automatically satisfied for any ( x, y ) which has continuous second partial derivatives.
(b) Show that for two-dimensional flow of an incompressible fluid, =constants are streamlines.

(c) If the velocity field is irrotational, then vi = where is known as the velocity potential.
xi
Show that the curves of constant velocity potential = constant and the streamline =constant
are orthogonal to each other. (d) Obtain the only nonzero vorticity component in terms of .
-------------------------------------------------------------------------------
v v y
Ans. (a) With vx = and v y = , we have, x + = = 0.
y x x y x y y x
(b) From ( x, y ) = C , we have,
dy / x dy vy
d = dx + dy = 0 = =
x y dx =constant / y dx =constant vx
Thus, ( x, y ) = C are streamlines.
(c) From
dy / x dy v
( x, y ) = C d = dx + dy = 0 = = x
x y dx =constant / y dx =constant vy

Copyright 2010, Elsevier Inc


6-28
Lai et al, Introduction to Continuum Mechanics

dy dy
Thus, = 1 .
dx =constant dx = constant
(d)
v y vx 2 2
(
= 2 = 2 Wzy e x + Wxz e y + Wyx ez = ) e z = 2 + 2 e z .
x y y x
_________________________________________________________________

a2
6.49 Show that = Vo y 1 2 represents a two-dimensional irrotational flow of an
x + y 2

inviscid fluid.
-------------------------------------------------------------------------------
a2
Ans. With = Vo y 1 2 2
, we have,
+
x y

2 xa 2 2 2a 2 ( 2 )( 2 x )2 a 2
= Vo y 2
2 = Vo y + 3
x
( )
x
( ) ( )
2
x2 + y 2 x2 + y 2 x2 + y 2


2
= Vo y 2a ( )1 2
3

= V o
2a 2 yo
4 x2

8a 2 yx 2
3
x
( ) ( ) ( ) ( )
2 2 2
x2 + y 2 x2 + y 2 x2 + y2 x2 + y 2


a2 2 y2a2
= Vo 1 2 2
+V
o 2 2
y x + y x + y2
(
)

2 2(2 y ) a 2 y 2 a 2 ( 2 )( 2 y )
2
2 ya 2
= Vo 2
+ Vo
y 2
( ) ( ) ( )
2 3
x2 + y 2 x2 + y 2 x2 + y 2


2 2 ya 2 4 ya 2 8 y3a 2
= Vo 2
+ Vo 3
. Thus,
y 2
( ) ( ) ( )
2
x2 + y 2 x2 + y 2 x2 + y 2


2 2
2a 2 yo 8a 2 yx 2 2 ya 2 4 ya 2 8 y 3a 2
+ = Vo 3
+ Vo + V
o 2 3
y 2 x 2
( ) ( ) ( ) ) ( ( )
2 2 2
x2 + y 2 x2 + y 2 x2 + y 2 x + y2 x2 + y 2

8a y ( x + y )
2 2 2
8a 2 y 8a 2 yx 2 8 y 3a 2 2
8a y
Vo 2
Vo + =V V =0
( ) ( ) ( ) ( ) ( )
3 3 o 2 o 3
x2 + y 2 x2 + y 2 x2 + y 2 2 2 2

2
x + y x + y

Therefore, the given stream function represents a two-dimensional irrotational flow of an
inviscid fluid.
__________________________________________________________________

Copyright 2010, Elsevier Inc


6-29
Lai et al, Introduction to Continuum Mechanics

6.50 Referring to Figure P 6-9, compute the maximum possible flow of water. Take the
atmospheric pressure to be 93.1 kPa. , the specific weight of water 9810 N / m3 , and the vapor
pressure 17.2 kPa. Assume the fluid to be inviscid. Find the length l for this rate
of discharge.
10cm dia


3m

5m

Figure P 6-9
------------------------------------------------------------------------------
Ans. The Bernoulli equation gives, with point 1 at the reservoir top and point 2 at the highest
point inside the tube, we have,
p1 / + v12 / 2 + g (0) = p2 / + v22 / 2 + g (3) . Thus, assuming v1 to be very small and negligible,
we have, with p1 = 93,100 Pa., p2 = 17, 200 Pa., = 1000 kg / m3 and g = 9.81m / s 2 .
v22 / 2 = ( p1 p2 ) / 3g = ( 93,100 17, 200 ) / 1000 3 ( 9.81) = 46.47

( )
v2 = 9.64 m / s. Qmax = v2 A = 9.64 ( 0.1) / 4 = 0.0757 m3 / s.
2

With point 3 at the exit, we have, p2 / + v22 / 2 + g (0) = p3 / + v32 / 2 + g (l)


now, v2 = v3 , p2 = pv (the vapor pressure), p3 = pa (atm.pressure)
l = ( pa pv ) / ( g ) = ( 93,100 17, 200 ) / 9810 = 7.74 m.
_________________________________________________________________

6.51 Water flows upward through a vertical pipeline which tapers from cross sectional area A1
to area A2 in a distance of h . If the pressure at the beginning and end of the constriction are
p1 and p2 respectively. Determine the flow rate Q in terms of , A1 , A2 , p1 , p2 and h . Assume
the fluid to be inviscid.
-------------------------------------------------------------------------------
Ans. Let the lower point be denoted as point 1, and the upper point denoted as point 2, we have
( )
p1 / + v12 / 2 + g (0) = p2 / + v22 / 2 + g (h) ( p1 p2 ) / g (h) = v22 v12 / 2
Let Q be the flow rate, then Q = A1v1 = A2 v2 and
1 Q Q 2 ( p1 p2 ) gh A22 A12
2 2
p1 p2
g ( h) =
Q =
2
2 A2 A1
A12 A22 ( )
2 ( p1 p2 ) g (h)
Q = A1 A2
(
A12 A22 )
_________________________________________________________________

Copyright 2010, Elsevier Inc


6-30
Lai et al, Introduction to Continuum Mechanics

6.52 Verify that the equation of conservation of mass is automatically satisfied if the velocity
components in cylindrical coordinates are given by
1 1
vr = , vz = , v = 0
r z r r
where the density is a constant and is any function of r and z having continuous second
partial derivatives.
-------------------------------------------------------------------------------
Ans. The equation of continuity is
1 1 v v 1 1
( rvr ) + + z = 0 . With vr = , vz = , v = 0 , we have,
r r r z r z r r
1 1 1 1 2 vz 1 1 2
( rvr ) = r = , = =
r r r r r z r r z z z r r r zr
Thus, the equation of continuity is automatically satisfied for any function ( x, y ) .
_________________________________________________________________

6.53 From the constitutive equation for a compressible fluid


Tij = p ij (2 / 3) ij + 2 Dij + k ij , = v j / x j , derive the equation

Dvi p v j 2 vi v j
= Bi + + +k
Dt xi 3 xi x j x j x j xi x j
------------------------------------------------------------------------------
Ans.
Tij p 2 1 vi v j
= ij ij + 2 + +k ij
x j x j 3 x j
x j 2 x j xi x j

p 2 2 vi
= + + +k
xi 3 xi
x j x j xi xi
That is,
Tij p 2 vi Dv Tij
= + + +k . Thus, i = Bi +
x j xi 3 xi x j x j xi Dt x j
Dvi p 2 vi
= Bi + + +k
Dt xi 3 xi x j x j xi
_________________________________________________________________

6.54 Show that for a one-dimensional, steady, adiabatic flow of an ideal gas, the ratio of
temperature 1 / 2 at sections 1 and 2 is given by
1
1 + ( 1) M12
1 2
=
2 1 + 1 1 M 2
( ) 2
2
where is the ratio of specific heat, M1 and M 2 are local Mach number at section 1 and section 2
respectively.
-------------------------------------------------------------------------------

Copyright 2010, Elsevier Inc


6-31
Lai et al, Introduction to Continuum Mechanics

p v2 p v12 p2 v22
Ans. + = constant , we have 1 + = + .
1 2 1 1 2 2 1 2
In terms of the Mach numbers M1 = v1 / c1 and M 2 = v2 / c2 , we have,
p1 c12 M12 p2 c22 M 22
+ = + .
1 1 2 2 1 2
p
For an ideal gas, p = R , and c 2 = = R , therefore,

1R1 R1M12 2 R2 R2 M 22
+ = +
1 1 2 2 1 2
2 M 22 1M12 1 1
1 2 = 1 + ( 1) 1M12 = 2 + ( 1) 2 M 22
1 1 2 2 2 2
1 + M 2 ( 1) / 2
2
1 1
1 1 + ( 1) M12 = 2 1 + ( 1) M 22 1 = .
2 2 2 1 + M12 ( 1) / 2
_________________________________________________________________

6.55 Show that for a compressible fluid in isothermal flow with no external work,
dM 2 dv
2
=2 , where M is the Mach number. (Assume perfect gas).
M v
-------------------------------------------------------------------------------
Ans. Since M 2 v 2 /c 2 and c 2 = R for ideal gas, therefore, M 2 v 2 / ( R)
For isothermal flow, =constant , therefore,
v2 2vdv dM 2 2vdv R 2dv
M2 dM 2 = = = .
R R M 2 R v 2 v
_________________________________________________________________

6.56 Show that for a perfect gas flowing through a duct of constant cross sectional area at
dp 1 dM 2
constant temperature = . [Use the results of the last problem].
p 2 M2
-------------------------------------------------------------------------------
Ans. We have, from Av = constant, ( d ) v + ( dv ) = 0 d / = dv / v
dp Rd d
Since = constant , therefore, p = R dp = Rd = =
p R
Thus, dp / p = dv / v . From the results of last problem, we have, dM 2 / M 2 = 2dv / v , therefore,
dp / p = (1 / 2)(dM 2 / M 2 ) .
_________________________________________________________________

6.57 For the flow of a compressible inviscid fluid around a thin body in a uniform stream of
speed Vm in the x1 direction, we let the velocity potential be = Vo ( x1 + 1 ) , where 1 is

Copyright 2010, Elsevier Inc


6-32
Lai et al, Introduction to Continuum Mechanics

assumed to be very small. Show that for steady flow the equation governing 1 is, with

(1 M ) x 21 21
2
M o = Vo / co , 2
o
1
+ + =0.
2
1 x22 x32
------------------------------------------------------------------------
v
Ans. For steady flow, the equation of continuity is vi + i = 0 , in terms of the potential
xi xi
2
function , we have, =0. (i).
xi xi xi xi
The equation of motion is:
v 1 p 1 p 1 p
vj i = = = c2 , note : c 2 = = local sound speed
x j xi xi xi
which becomes,
2 1 2
= c2 = 2 (ii)
x j x j xi xi xi c x j x j xi
2 2 2
(ii) into (i) c = 0 (iii). Now, with = Vo ( x1 + 1 ) , we have,
xi x j x j xi xi xi

2 21
= Vo i1 + 1 , = Vo i1 + 1 Vo j1 + 1 Vo
xi xi xi x j x j xi xi x j x j xi

21 21 21
Vo3 i1 + 1 j1 Vo i1 j1
3
= Vo3 .
xi x j xi x j xi x1x1

2 21 21 21
Thus, Eq.(iii) Vo3 + c 2
V o + 2 + 2 =0,
x12 x 2 x2 x3
1
21 21 21
(
1 M o2 ) +
x12 x22
+ 2 =0.
x3

_________________________________________________________________

6.58 For a one dimensional steady flow of a compressible fluid through a convergent channel,
obtain (a) the critical pressure and (b) the corresponding velocity. That is, verify equation
(6.30.7) and Eq. (6.30.8)
--------------------------------------------------------------------------------
Ans. (a) From Eq. (6.30.6),
1
2 +1 2
dm 2
p
p
= A2 p11 2 2 , (i)
dt 1 p1 p1

we have,

Copyright 2010, Elsevier Inc


6-33
Lai et al, Introduction to Continuum Mechanics

1 2 1
d ( dm / dt ) 1 dm

2 2 2 p2 + 1 p2
= p . (ii)
d ( p2 / p1 ) 2 dt 1 1 1 p1 p1

d ( dm / dt )
Thus, = 0 gives,
d ( p2 / p1 )
2 1 1 1
2 p2 + 1 p2 p2 2 p2 + 1
= 0, or =0. (iii)
p1 p1 p1 p1


That is,
1
2 p2 +1 p 2 1
= , therefore 2 = . (iv)
p1 p1 critical + 1
(b) Substituting this critical pressure into Eq. (6.30.4) for the velocity, we get
1
p2 2 p1 + 1 2 p1 1 p1 2
1
2 p1
v22 = = = = .
1 1 p1 crit 1 1 2 1 1 + 1 1 + 1
1 1

1
p p1 1 2 2 p p2
From 1 = 2 = 1 = 2 . But
p2 1 1 2 p2 1 1 1 2
1 1 1
1
2 p2 2 p p p p2
= = 2 1 = 2 . Now, at
1 p1 1 p1 1 p1 2

p2 + 1 1 p1 + 1 p2
= , we have, = . Thus,
p1 critical 2 1 2 2
p
v22 = 2 =speed of sound at section (2) .
2
__________________________________________________________________

Copyright 2010, Elsevier Inc


6-34
Lai et al, Introduction to Continuum Mechanics

CHAPTER 7

7.1 Verify the divergence theorem S v ndS = V div v dV for the vector field v = 2xe1 + ze2 ,
by considering the region bounded by x = 0, x = 2, . y = 0, y = 2, z = 0, z = 2 .
---------------------------------------------------------------------------------------
Ans. With v = 2xe1 + ze2 , we have
For the face x = 0, n = e1 , v n = 2x = 0, v ndS = 0 .
For the face x = 2 , n = +e1 , v n = +2x = 4, v ndS = 4 A = 4(4) = 16 .
2 2
For the face y = 0, n = e 2 , v n = z, v ndS = 0 z (2dz ) = 2 z / 2 0 = 4 .
2

2
For the face y = 2, n = +e2 , v n = z, v ndS = 0 z (2dz ) = +4 .
For the face z = 0, n = e3 , v n = 0, v ndS = 0 .
For the face z = 2 , n = +e3 , v n = 0, v ndS = 0 .
Thus, v ndS =16 4 + 4 = 16 and ( div v ) dV = 2dV = 2 ( 2 2 2 ) = 16.
S
So, v ndS = div vdV .
S V
________________________________________________________________________

7.2 Verify the divergence theorem S v ndS = V div v dV for the vector field, which in
cylindrical coordinates, is v = 2rer + zez , by considering the region bounded by r = 2 , z = 0 and
z = 4.
---------------------------------------------------------------------------------------
Ans. For the cylindrical surface r = 2 ,
n = e r , v n = 2r = 2(2) = 4, v ndS = 4 dS = 4 S = 4 ( 2 2 ) (4) = 64 .
For the end face z = 0 , n = e z , v n = z z =0 = 0, v ndS = 0 .

For the end face z = 4 , n = e z , v n = z z = 4 = 4, v ndS = 4S = 4 22 = 16 .


Therefore, S v ndS = 64 + 16 = 80 .
vr vr vz ( 2r ) 2r z
div v = + + = + + = 2 + 2 +1= 5.
r r z r r z
V div vdV =5 ( 2 ) ( 4 ) = 80 , thus, S v ndS = V divv dV .
2

________________________________________________________________________

7.3 Verify the divergence theorem S v ndS = V divv dV for the vector field, which in
spherical coordinates is v = 2rer , by considering the region bounded by the spherical surface
r = 2.
---------------------------------------------------------------------------------------

Copyright 2010, Elsevier Inc


7-1
Lai et al, Introduction to Continuum Mechanics

Ans. For the spherical surface at r = 2 ,


n = er , v n = 2r = 2(2) = 4, v ndS = 4dS =4S = 4 ( 4 2 ) = 64
2

On the other hand [see Eq.(2.35.26)],


2
(
1 d r vr )
1 d 2r
= 2
3
( ) 4 23
= 6, div v dV = 6 = 64 .
3
div v = 2
r dr r dr
________________________________________________________________________

7.4 Show that

S x n dS = 3V .
where x is the position vector and V is the volume enclosed by the boundary S .
---------------------------------------------------------------------------------------
x x x
Ans. x = x1e1 + x2e 2 + x3e3 , div x = 1 + 2 + 3 = 1 + 1 + 1 = 3 .
x1 x2 x3
Thus S x ndS = V div x dV = 3V .
________________________________________________________________________

7.5 (a) Consider the vector field v = a , where is a given scalar field and a is an arbitrary
constant vector (independent of position). Using the divergence theorem, prove that

V dV = S n dS .
(b) Show that for any closed surface S that
S ndS = 0 .
---------------------------------------------------------------------------------------
Ans. (a) With v = a , v n = a n v ndS = a ndS ,
ai
div v = div ( a ) = = ai = a .
xi xi
Thus, S v ndS = S div vdV a S ndS = a V dV . Since a is arbitrary, therefore,
S ndS = V dV .
(b) Take = 1 in the results of part (a), we have ndS = 0 .
S
________________________________________________________________________

7.6 A stress field T is in equilibrium with a body force B . Using the divergence theorem,
show that for any volume V and boundary surface S , that

S tdS + V BdV = 0 .
where t is the stress vector. That is, the total resultant force is equipollent to zero.
---------------------------------------------------------------------------------------

Ans. The stress vector t is related to the stress tensor T by t = Tn , therefore,


tdS = TndS = div TdV , thus, tdS + BdV = ( divT + B ) dV .
S S V S V V

Copyright 2010, Elsevier Inc


7-2
Lai et al, Introduction to Continuum Mechanics

But in equilibrium, ( divT + B ) = 0 , therefore, S tdS + V BdV = 0 .


________________________________________________________________________

1
Let u* define an infinitesimal strain field E* = u* + ( u* ) and let T** be the
T
7.7
2
symmetric stress tensor in static equilibrium with a body force B ** and a surface traction t ** .
Using the divergence theorem, verify the following identity (theory of virtual work).

S t ** u* dS + V ( B** ) u* dV = V Tij
** *
Eij dV .
---------------------------------------------------------------------------------------
Ans.
S t ** u* dS = S ( T** n ) u* dS = S n ( T** ) u* dS = V div ( T** ) u* dV .
T T

= div(T** u* )dV
V

(Tij**u*j ) Tij** u*j u*j


Now, div ( T** u* ) = = u*j + Tij** = divT** u* +Tij** , therefore,
xi xi xi xi

S t ** u* dS + V B** u* dV = ( divT** + B** ) u* +Tij u*j / xi dV = Tij**u*j / xi dV .


**

Now, since Tij** = T ji** , therefore,

1 u* u*j 1 u* 1 u* 1 u* 1 u*
Tij** Eij* = Tij** i + = Tij** i + Tij** j = Tij** i + T ji** j
2 x j xi 2 x j 2 xi 2 x j 2 xi

ui* u* u j *
= Tij** = T ji** i = Tij** .
x j x j xi
Thus,
S t ** u* dS + V ( B* ) u* dV = V Tij
** *
Eij dV
________________________________________________________________________

7.8 Using the equations of motion and the divergence theorem, verify the following rate of
work identity. Assume the stress tensor to be symmetric.
D v2
S t vdS + V B vdV = V Dt dV + V Tij Dij dV
2
---------------------------------------------------------------------------------------
Ans. t vdS = Tn vdS = n TT vdS = div(TT v ) dV = div(Tv )dV
S S S V V
Tij v j Tij v j v j
Now, div(Tv ) = = v j + Tij = divT v + Tij , therefore,
xi xi xi xi

S t vdS + V B vdV = V [( divT + B ) v + Tij v j / xi ]dV = V ( Dv / Dt ) vdV + Tij v j / xi .


Dv 1 D( v v) D v2 1 v v j v v j
Now, v = = and Tij Dij = Tij i + = Tij i = Tij .
Dt 2 Dt Dt 2 2 x j xi x j xi

Copyright 2010, Elsevier Inc


7-3
Lai et al, Introduction to Continuum Mechanics

D v2
Therefore, S t vdS + B vdV =
V
dV + V Tij Dij dV .
V

Dt 2

________________________________________________________________________

7.9 Consider the velocity and density fields1


v = x1e1 , = o e ( o )
t t

(a) Check the equation of mass conservation.


(b) Compute the mass and rate of increase of mass in the cylindrical control volume of cross-
section A and bounded by x1 = 0 and x1 = 3 .
(c) Compute the net mass inflow into the control volume of part (b). Does the net mass inflow
equal the rate of mass increase inside the control volume?
---------------------------------------------------------------------------------------
D
+ div v = o e ( o ) + o e ( o ) ( ) = 0 . That is, the conservation of mass
t t t t
Ans. (a).
Dt
equation is satisfied.
(b) Inside the control volume,
m = dV = e ( o ) Adx = 3 e ( o ) A, and dm / dt = 3 e ( o ) A . That is, the
3 t t t t t t
0 o 1 o o

mass inside the volume is decreasing at the rate of 3o e ( o ) A .


t t

(c). Rate of inflow from the face x1 = 0 is zero because at x1 = 0 , v1 = 0.


A = 3 e ( o ) A . There is no flow
t t
Rate of outflow from the face x = 3 is given by v
1 1 x =3 o
1

across the cylindrical surface because flow is only in the x1 direction. Thus, the rate of outflow
exactly equals the rate of decrease of mass inside the volume.

________________________________________________________________________

7.10 (a) Check that the motion


x1 = X1e ( o ) , x2 = X 2 , x3 = X 3
t t

corresponds to the velocity field v = x1e1 .


(b) For a density field = e ( o ) , verify that the mass contained in the material volume that
t t
o
was coincident with the control volume of Prob. 7.9 at time to , remains a constant at all times, as
it should (conservation of mass).
(c) Compute the total linear momentum for the material volume of part (b).
(d) Compute the force acting on the material volume
---------------------------------------------------------------------------------------
x x x
Ans. (a) v1 = 1 = X1e ( o ) = x1 , v2 = 2 = 0, v3 = 3 = 0 , i.e., v = x1e1 .
t t
t t t
(b) The particles which are at x1 = 0 at time to have the material coordinates X1 = 0 . These
particles remain at x1 = 0 at all time. The particles which are at x1 = 3 at time to have the material

1
It should be remarked that, for a real fluid, to achieve the given velocity and density fields in this and
some other problems may require body force distributions and/or a pressure density relationship that are not
realistic.

Copyright 2010, Elsevier Inc


7-4
Lai et al, Introduction to Continuum Mechanics

coordinates X1 = 3 . These particles move in such a way that x1 = 3e ( o ) . Thus, to find the mass
t t

inside this material volume as a function of time, we have


3e ( o )
t t

o e ( o ) Adx1 = o e ( o ) A 3e ( o ) = 3o A .
t t t t t t
M=
0
(c) Linear momentum in the material volume
3e ( o ) 3e ( o )
t t t t
( t to )
P= o e v1 Adx1e1 = o e ( t to ) x1 Adx1e1
0 0
( t to )
3e 9e 2 ( t to )
= o e ( o ) A
( t to )
e1 = Ao e ( o )e1.
t t 9 t t
1 11 ox dx e = e A
2 2
0
(d) Force acting on the material volume
= Ao 2 e ( o )e1 .
dP 9 t t
F=
dt 2
We see that both the linear momentum and the force increase exponentially with time. This is due
to the given data of density and velocity fields, which describe the space occupied by the fixed
material increases exponentially with time, 0 x1 3e (t to ) ,while the density decreases

exponentially o e
( t t )
o
to conserve the mass. We note also that at t = to , the materials occupy

9
Ao 2e1 .
the space between x1 = 0 and x1 = 3 , and F =
2
________________________________________________________________________

7.11 Do Problem 7.9 for the velocity field v = x1e1 and the density field = k o / x1 and for
the cylindrical control volume bounded by x1 = 1 and x1 = 3 .
---------------------------------------------------------------------------------------
D v
Ans. (a). + div v = v1 + 1 = x1 k 2o + k o ( ) = 0 . That is, the conservation of
Dt x1 x1 x1 x1

mass equation is satisfied.
3 dm
(b) Inside the control volume, m = dV = k o Adx1 = kAo ln 3, and =0.
1 x1 dt

(c). Rate of inflow from the face x1 = 1 is [ v1 A]x = k o x1 A = k o A .
1
x1 x1 =1

Rate of outflow from the face x1 = 3 is given by [ v1 A]x =3 = k o x1 A = ko A . There is
1
x1 x1 =3
no flow across the cylindrical surface because flow is only in the x1 direction. The net mass
inflow is 0 , which is equal to the rate of increase of mass inside the control volume.
________________________________________________________________________

7.12 The center of mass xc.m of a material volume is defined by the equation
mxc.m = x dV , where m = dV
Vm Vm

Copyright 2010, Elsevier Inc


7-5
Lai et al, Introduction to Continuum Mechanics

Demonstrate that the linear momentum principle may be written in the form
tdS + BdV = mac.m S V
where ac.m is the acceleration of the mass center.
---------------------------------------------------------------------------------------
D
Ans. We have from the principle of linear momentum: S tdS + V BdV = Dt V m
vdV
D D D D
Now, since ( dV ) = 0 , therefore, vdV = ( xdV ) = ( xdV ) = mxc.m = mv c.m .
Dt Dt Dt Dt
D
Therefore, tdS + BdV = mv c.m = mac.m .
S V
Dt
________________________________________________________________________

7.13 Consider the following velocity field and density field


x1
v= e1 , = o
1 + t 1+ t
(a) Compute the total linear momentum and rate of increase of linear momentum in a cylindrical
control volume of cross-sectional area A and bounded by the planes x1 = 1 and x1 = 3 .
(b) Compute the net rate of outflow of linear momentum from the control volume of (a)
(c) Compute the total force on the material in the control volume.
(d) Compute the total kinetic energy and rate of increase of kinetic energy for the control volume
of (a).
(e) Compute the net rate of outflow of kinetic energy from the control volume.
---------------------------------------------------------------------------------------
Ans. (a) Linear momentum is
x1 A 3 9 1 A 4 o A
3
P = vdV = o Adx1e1 = o 2 x1dx1e1 = o 2 e1 = e .
1
1 + t 1 + t (1 + t ) 1 2 2 (1 + t ) (1 + t ) 2 1

Rate of increase of linear momentum inside the control volume is


dP 8 A 2
= o 3 e1 .
dt (1 + t )
(b) Net rate of outflow of linear momentum in e1 direction
o A 9 2 2 8 2 o A
(
= Av12 ) x1 =3
(
Av12 ) x1 =1
= =
1 + t (1 + t )2 (1 + t )2 (1 + t )3
.

(c) Total force = Rate of inc. of P inside control volume + net outflux of P
8 A 2 8 2 o A
= o 3 e1 + e1 =0.
(1 + t ) (1 + t )3
(d) Total kinetic energy inside the control volume
1 x1 1 o A 2 13 o A 2
3 2 3
1
K .E. = v 2 dV = o
2 (1 + t )3 1
= 2
=
2 1 1 + t 1 + t
Adx x dx
3 (1 + t )3
1 1 1
2

d 13o A 3
K .E = .
dt (1 + t )4
(e) Net rate of outflow of kinetic energy from the control volume=

Copyright 2010, Elsevier Inc


7-6
Lai et al, Introduction to Continuum Mechanics

1 3 1 3 1 o A 27 3 3 13 3 o A
1
Av 1
Av = = .
2 x1 =3 2 x1 =1 2 1 + t (1 + t )3 (1 + t )3 (1 + t )4
________________________________________________________________________

7.14 Consider the velocity and density fields


v = x1e1 , = o e ( o )
t t

For an arbitrary time t , consider the material contained in the cylindrical control volume of cross-
sectional area A , bounded by x1 = 0 and x1 = 3 .
(a) Determine the linear momentum and rate of increase of linear momentum in this control
volume.
(b) Determine the outflux of linear momentum.
(c) Determine the net resultant force that is acting on the material contained in the control
volume.
---------------------------------------------------------------------------------------
Ans. (a) Linear momentum inside the control volume:
3 3
P = vdV = o e ( o ) v1 Adx1e1 = o e ( o ) x1 Adx1e1
t t t t

0 0
3
9
= o e ( o ) A x1dx1e1 = o e ( o ) A e1 = Ao e ( o )e1.
t t t t 9 t t

0
2 2
Rate of increase of linear momentum inside the control volume
= 2 A o e ( o )e1 .
dP 9 t t
dt 2
(b) Out flux of linear momentum from the control volume in the e1 direction:

( Av 2
1 ) Av 2
x1 =3
( )
= A 2 x 2
1 (
A 2 x 2
x1 = 0
1 ) x1 =3
( 1 )
= 9 A 2 e ( o ) .
t t
x1 = 0
o

(c) The total force = rate of increase of linear momentum inside the control volume + net
momentum outflux from the control volume. Thus,
F = 2 Ao e ( o )e1 + 9 Ao 2 e ( o )e1 = Ao 2 e ( o )e1
9 t t t t 9 t t
2 2
We see that the force exerted on the material within the control volume decreases exponentially
with time. This is due to the given data of density field and velocity field, which states that within
the fixed space defined by 0 x1 3 , the density decreases exponentially with time while speed at
9
each spatial point is independent of time. We also note that at t = to , F = Ao 2e1 , the same
2
results was obtained in Problem 7.10.
________________________________________________________________________

o
7.15 Do Problem 7.14 for the same velocity field, v = x1e1 but with = k and the
x1
cylindrical control volume bounded by x1 = 1 and x1 = 3 .
---------------------------------------------------------------------------------------
Ans. (a) Linear momentum inside the control volume:

Copyright 2010, Elsevier Inc


7-7
Lai et al, Introduction to Continuum Mechanics

3
P = vdV = ( k o / x1 ) v1 Adx1e1 = ( k o / x1 ) ( x1 ) Adx1e1 = o k A dx1e1 = 2 o k Ae1
3 3
1 1
1
Rate of increase of linear momentum inside the control volume
dP
= 0.
dt
(b) Out flux of linear momentum from the control volume in the e1 direction:

( Av )
2
1
x1 =3
(
Av12 ) x1 =1
(
= k ( o / x1 ) A 2 x12 ) x1 =3
(
k ( o / x1 ) A 2 x12 ) x1 =1
= 2k o A 2 .

(c) The total force = rate of increase of linear momentum inside the control volume + net
momentum outflux from the control volume F = 0 + 2k o A 2e1 = 2k o A 2e1
________________________________________________________________________

7.16 Consider the flow field v = k ( xe1 ye2 ) with =constant. For a control volume defined
by x = 0, x = 2, y = 0, y = 2, z = 0, z = 2 , determine the net resultant force and moment about the
origin (note misprint in text) that are acting on the material contained in this volume.
---------------------------------------------------------------------------------------
Ans. Since the flow is steady, the resultant force = net linear momentum outflux through the three
pairs of faces:
(i) through x = 0 and x = 2 ,
( ) ( 2
)
v1vdA x=2 v1vdA x=0 = k x ( xe1 ye2 ) dydz x=2 k x ( xe1 ye2 ) dydz x=0
2

2 2 2
= k2 1
( 4e 2 ye 2 ) dz dy = 2 k 2
1 ( 4e 2 ye 2 )dy i
y =0 z =0

y =0
= 2k 2 ( 8e1 4e 2 ) = k 2 (16e1 8e2 ) .
(ii) through y = 0 and y = 2 ,
( v2 vdA) y=2 ( v2 vdA) y=0 = k 2 ( y )( xe1 ye2 ) dxdz y=2 k 2 ( y )( xe1 ye2 ) dxdz y=0

( 2 xe1 + 4e2 ) dz dx = 2k x=0 ( 2 xe1 + 4e2 ) dx
2 2 2
= k2 2
x =0
z =0
= 2k 2 x 2e1 + 4 xe 2 = 2k 2 ( 4e1 + 8e 2 ) = k 2 ( 8e1 + 16e2 ) .
x =2
(iii) through z = 0 and z = 2 ,
( v3 vdA) z =2 ( v3 vdA)z =0 = 0, (v3 =0) .
Thus, the total net force
F = k 2 (16e1 8e 2 ) + k 2 ( 8e1 + 16e2 ) = k 2 ( 8e1 + 8e2 ) .

The flow is steady, the resultant moment about a point = net moment of momentum outflux about
the same point. Take the point to be the origin, then

Copyright 2010, Elsevier Inc


7-8
Lai et al, Introduction to Continuum Mechanics

(i) through x = 0 and x = 2 ,


( v1 (r v ) dA)x=2 ( v1 (r v ) dA)x=0 = k 2 x ( xe1 + ye2 + ze3 ) ( xe1 ye2 ) dydz x=2
= k 2 ( xyze1 + x 2 ze 2 - 2 x 2 ye3 ) dydz = k 2 ( 2 yze1 + 4 ze 2 - 8 ye3 ) dz dy
2 2
(i)
x=2 z =0
y =0
( 4 ye1 + 8e2 - 16e3 )dy = k 2 (8e1 + 16e2 - 32e3 ) .
2
= k2
y =0
through y = 0 and y = 2 ,
( v2 (r v ) dA) y=2 ( v2 (r v ) dA) y=0 = k 2 ( y ) ( xe1 + ye2 + ze3 ) ( xe1 ye2 ) dxdz y=2
= k 2 ( y 2 ze1 xyze 2 + 2 xy 2e3 ) dxdz = k 2 ( 4 ze1 2 xze 2 + 8 xe3 ) dz dx
2 2
y =2 x =0
=0
z

( 8e1 4 xe2 + 16 xe3 ) dx = k 2 ( 16e1 8e2 + 32e3 ) .


2
= k2
y =0
(iii) through z = 0 and z = 2 ,
( v3 (r v ) dA)z =2 ( v3 (r v ) dA) z =0 = 0
Thus, M o = k 2 ( 8e1 + 16e 2 32e3 ) + k 2 ( 16e1 8e2 + 32e3 ) = k ( 8e1 + 8e2 ) .
________________________________________________________________________

7.17 ( )
For Hagen-Poiseuille flow in a pipe, v = C ro2 r 2 e1 . Calculate the momentum flux
across a cross-section. For the same flow rate, if the velocity is assumed to be uniform, what is
the momentum flux across a cross section? Compare the two results.
---------------------------------------------------------------------------------------
Ans. Momentum flux across a cross section

(r ) ( 2 r ) dre1 = ( C 2 ro6 / 3) e1
ro 2
= v12 dAe1 = C 2 o
2
r2
o

Volume flow rate is Q = v1dA = C


o
ro
(r
o
2
) ( )
r 2 ( 2 r )dr = C ro4 / 2 .

The uniform flow which has the same flow rate Q is given by : v = (Q / ro2 )e1 = (Cro2 / 2)e1 .
The momentum flux across a cross section for this uniform flow is given by
Qv1e1 = ( C 2 ro6 / 4)e1 .
4
that of the uniform flow.
Thus, the momentum flux for the Hagen-Poiseuille flow is
3
________________________________________________________________________

7.18 Consider a steady flow of an incompressible viscous fluid of density , flowing up a


vertical pipe of radius R . At the lower section of the pipe, the flow is uniform with a speed vl and
a pressure pl .After flowing upward through a distance l , the flow becomes fully developed with
a parabolic velocity distribution at the upper section, where the pressure is pu . Obtain an
expression for the fluid pressure drop pl pu between the two sections in terms of , R and the
frictional force F f , exerted on the fluid column from the wall though viscosity.
---------------------------------------------------------------------------------------

Copyright 2010, Elsevier Inc


7-9
Lai et al, Introduction to Continuum Mechanics

Ans. Let the control volume encloses the fluid between the two sections. The linear momentum
theorem states that: For steady flow, Force on the fluid= Momentum outflux momentum
influx.
The force on the fluid in the control volume is given by: ( pl pu ) A g ( Al ) F f .

The momentum influx through the lower section = vl2 R 2 . ( )


vu ( 2 rdr ) , where vu = C ( R r 2 ) . The
2 2
The momentum outflux through the upper section
constant C can be obtain as follows

0
R
0
R
(
Q = vu 2 rdr = 2 C ( R 2 r 2 )rdr = CR 4 / 2 = vl R 2 C = 2vl / R 2 . )
2vl
Thus, vu = 2
(R2 r 2 ) .
R

( )
2
vu2 ( 2 rdr ) = 2 2vl / R 2 r 2 )2 ( rdr )
R R
o o ( R
2
Momentum outflux

(
= 8 vl2 / R 4 ) R
o
(
( R 2 r 2 ) 2 ( rdr ) = 8 vl2 / R 4 )( R / 6) = 4 v R
6 2
l
2
/ 3.

Thus, ( pl pu ) A g ( Al ) F f = 4 vl2 R 2 / 3 vl2 ( R 2 ) = vl2 ( R 2 ) / 3


( pl pu ) ( R 2 ) = vl2 ( R 2 ) / 3 + F f + g ( R 2 ) ( l ) . That is,
( pl pu ) = vl2 / 3 + F f / ( R 2 ) + g l .
________________________________________________________________________

7.19 A pile of chain on a table falls through a hole from the table under the action of gravity.
Derive the differential equation governing the hanging length x . [Assume the pile is large
compared with the hanging portion]
---------------------------------------------------------------------------------------
Ans. Using a control volume (Vc )2 [see Fig. 7.6-1 in Section7.6] enclosing the hanging down
portion x of the chain, we can obtain the same equation as that given in Eq. (iv) of Section 7.6,
i.e., with denoting m / l , mass per unit length:
gx T = xd 2 x / dt 2 (1)
where T is the tension on the chain at the hole. Next, using a control volume enclosing the pile
above the table, then, since the particles of the chain pile stay essentially at rest at any given
instant (except those near the hole), we can assume that the rate of change of momentum inside
the control volume is zero (quasi-static approximation). Further, we assume that the net force
acting at the pile is the tension T at the hole (the reaction of the supporting table exactly balances
the weight of the pile). Then, the momentum principle gives:
T = ( dx / dt )
2
(2).
Equations (1) and (2) give
2
dx d2x
+ gx = x (3)
dt 2
dt
We note that this equation is a good approximation when the length of the pile is large compared
with the hanging portion x . Eventually, when the pile reduces to essentially a flat straight
segment on the table, Eq. (vi) of Section 7.6 becomes a better approximation.
________________________________________________________________________

Copyright 2010, Elsevier Inc


7-10
Lai et al, Introduction to Continuum Mechanics

7.20 A water jet of 5 cm. diameter moves at 12 m / sec , impinges on a curved vane which
deflects it 60o from its original direction. Neglect the weight, obtain the force exerted by the
liquid on the vane. (see Fig. 7.6-2 of Example 7.6.2).
---------------------------------------------------------------------------------------
Ans. Referring to Fig. 7.6-2, we have, vo = 12 m / s, =60o , volume flow rate

( )
Q = vo ( d 2 / 4) = 12 (5 102 ) 2 / 4 = 235.6 104 m3 , Qvo = ( 998 ) (235.6 104 ) (12 ) = 282 N
Thus, force on the jet =
( )
Qvo 1 cos 60o e1 + Qvo sin 60o e 2 = 282 ( 0.5 ) e1 + 282 ( 0.866 ) e 2 = 141e1 + 244e 2 N .
Force on the vane from the jet is 141e1 244e 2 N .
________________________________________________________________________

7.21 A horizontal pipeline of 10 cm. diameter bends through 90o , and while bending, changes
its diameter to 5 cm. The pressure in the 10 cm. pipe is 140 kPa. Estimate the resultant force on
the bends when 0.005 m3 / sec .of water is flowing in the pipeline.
---------------------------------------------------------------------------------------
Ans. Let ( vu , pu , Au ) and ( vd , pd , Ad ) denote upstream and downstream (speed, pressure and
cross-sectional area) respectively and Q the volume discharge. We have, Q = 0.005 m3 / s,

( 2
) (
vu = 0.005 / ( 0.1) / 4 = 0.6366 m / s, vd = 0.005 / ( 0.05 ) / 4 = 2.546 m / s
2
)
Upstream pressure pu = 140,000 Pa . Down stream pressure can be obtained from Bernoulli
pu vu2 pd vd2
Equation: +
= + . Thus,
2 2
2 2
pd = pu +
2
( )
vu vd = 140,000 +
998
2
( )
0.63662 2.5462 = 137,000 .

Let e1 be the direction of the incoming flow and e 2 be the direction after the 90o bend, then, we
have,
Momentum outflux = Qvd = ( 998 )(.005 )( 2.546 ) e 2 = 12.7e 2 .
Momentum influx = Qvu = ( 998 )(.005 )( 0.6366 ) = 3.18e1 .
Momentum principle gives: pu Au e1 pd Ad e 2 + Fw = Qvd e 2 Qvu e1 .
Force on water Fw = ( Qvu + pu Au ) e1 + ( Qvd + pd Ad ) e 2

( ) ( )
= 3.177 + 140,000( ( 0.1) / 4) e1 + 12.7 + 137000 ( 0.05 ) / 4 e 2 = 1100e1 + 282e 2 N .
2 2

Thus, the force from water to the bend is Fw = 1100e1 282e 2 N .


________________________________________________________________________

7.22 Figure P7.1 shows a steady water jet of area A impinging onto the flat wall. Find the
force exerted on the wall. Neglect weight and viscosity of water.

Copyright 2010, Elsevier Inc


7-11
Lai et al, Introduction to Continuum Mechanics

vo

vo

vo
---------------------------------------------------------------------------------------
Ans. Let the control volume be coincident with the outline of the flow shown in the figure.

Force on the liquid FL = momentum outflux-momentum influx = 0 Qvo e1


Force on the wall = Qvo e1 = Avo2 e1 .
________________________________________________________________________

7.23 Frequently in open channel flow, a high speed flow jumps to a low speed flow with an
abrupt rise in the water surface. This is known as a hydraulic jump. Referring to Fig. p7.2, if the
flow rate is Q per unit width, show that when the jump occurs, the relation between y1 and y2 .is
given by
y1 1 8v 2
y2 = + y1 1 + 1
2 2 gy1
Assume the flow before and after the jump is uniform and the pressure distribution is hydrostatic.

v1
y2
y1

---------------------------------------------------------------------------------------

Ans. Use a control volume enclosing the water with an upstream section before the jump and a
downstream section after the jump. According to the momentum principle, the force on the fluid
per unit width is given by (neglect friction from the ground and air)
( )
Fx = gy12 / 2 gy22 / 2 = Qv2 Qv1 , thus , ( g / 2) y12 y22 = Q ( v2 v1 ) = v1 y1 ( v2 v1 ) .
Conservation of mass gives: v1 y1 = v2 y2 v2 v1 = v1 y1 / y2 v1 = v1 ( y1 y2 ) / y2 . Therefore,

( )
we have, ( gy2 / 2) y12 y22 = v12 y1 ( y1 y2 ) .
The above equation shows that y1 = y2 is a root for the equation. This solution corresponds to a
flow without a jump. To look for the jump solution, we eliminate the factor ( y1 y2 ) and obtain
gy2 ( y1 + y2 ) / 2 = v12 y1 y22 + y1 y2 (2v12 / g ) y1 = 0


1
2
(
y2 = (1 / 2 ) y1 + y12 + (8v12 y1 / g ) = y1 / 2 + y1 1 + 8v12 / gy1 )
________________________________________________________________________

Copyright 2010, Elsevier Inc


7-12
Lai et al, Introduction to Continuum Mechanics

7.24 If the curved vane of Example 7.6.2 moves with a velocity v < vo in the same direction as
the oncoming jet, find the resultant force exerted on the vane by the jet.
---------------------------------------------------------------------------------------
Ans. Fig. 7.6-2 of Example 7.6.2 is reproduced below.
vo
e2 B

e1

vo A

Let the control volume surrounding the jet moves with the vane, then the flow is steady with
respect to the moving control volume.
Momentum outflux relative to the control volume =
Q(vo v) ( cos e1 + sin e 2 ) = A(vo v)2 ( cos e1 + sin e2 )
Momentum influx relative to the control volume is
Q(vo v)e1 = A(vo v) 2 e1
Thus, since the control volume moves with a constant speed, there is no extra term to be added to
the momentum equation for the fixed control volume case. Thus, force acting on the jet is
Fjet = A(vo v) 2 ( cos e1 + sin e 2 ) A(vo v) 2 e1 = A(vo v) 2 ( cos 1) e1 + sin e 2
and the force on the vane is
Fvane = A(vo v) 2 (1 cos ) e1 sin e 2 .
________________________________________________________________________

7.25 For the half-arm sprinkler shown in Fig. P7.3, find the angular speed if
Q = 0.566m3 / sec. Neglect friction.

d=2.54 cm

1.83 m
---------------------------------------------------------------------------------------
Ans. Let the control volume Vc rotate with the arm. Then, relative to the control volume, the
outflux of moment of momentum about an axis passing through O and perpendicular to the plane
of the paper is Q ( Q / A ) ro e3 , where ro is the length of the arm. There is no influx of moment of
momentum about the same axis since the inflow is parallel to it. Since the control volume is
rotating with an angular velocity about the same axis, we need to add terms to the left hand
side of Eq. (7.9.8) , the moment of momentum principle. The terms that need to be added are
given in Eq. (7.9.9). With = e3 and x = xe1 , we have, x = xe2 , ( x ) = x 2e1 so
that x ( x ) = 0 . We also have, ao = 0 and & = 0 , therefore, the only non-zero term is
2 x ( v ) dm = 2 xe1 (e3 ( Q / A ) e1 ) Adx = 2 Q xdxe3 = Qro2e3 .
ro
0
Adding this term to Eq. (7.9.8), whose left hand side is zero (because frictional torque is
neglected) and whose right hand side is the net moment of momentum outflux, we have,

Copyright 2010, Elsevier Inc


7-13
Lai et al, Introduction to Continuum Mechanics

Qro2e3 = Q (Q / A)ro e3 = Q / ( Aro ) . Now, A = (2.54 102 )2 / 4 = 5.067 104 m 2 ,


therefore, = Q / ( Aro ) = 0.566 / [(5.067 104 )(1.83)] = 610.4 rad / s .
The minus sign means the rotation is clockwise looking from the top.
________________________________________________________________________

7.26 The tank car shown in fig. P7.4 contains water and compressed air which is regulated to
force a water jet out of the nozzle at a constant rate of Q m3 / sec. The diameter of the jet
is d cm. , the initial total mass of the tank car is M o . Neglecting frictional forces, find the velocity
of the car as a function of time.

---------------------------------------------------------------------------------------
Ans. Let the control volume Vc encloses the whole tank car and moves with the car. Then
relative to the control volume, the momentum outflux is
( )
Q 4Q / d 2 e1 = 4 Q 2 / ( d 2 )e1 .
There is no momentum influx. Since the control volume moves with the car which has an
acceleration ao e1 , therefore, the momentum principle in the e1 direction takes the form [see
Eq.(7.8.20}: (with al frictional/resistance force neglected):
( M o Qt ) (dv / dt ) = 4 Q 2 / ( d 2 ) . dv / dt = [4 Q 2 / ( d 2 )] / ( M o Qt ) .
Integrating, we have, v = [4Q / ( d 2 )]ln ( M o Qt ) + C .
If the initial velocity is zero then we have
v = [4Q / ( d 2 )] ln ( M o Qt ) + ln M o .
________________________________________________________________________

7.27 For the one dimensional problem discussed in Section 7.10,


(a) from the continuity equation 1v1 = 2 v2 and the momentum equation p1 p2 = 2 v22 1v12 ,
obtain
v2 1 p2
=1 1
v1 M1 p1
2

1 p1 p2 1 2
(b) From the energy equation + v12 = + v2 , obtain
1 1 2 1 2 2
1 v1
2
v2 1 v12 v22
p2
1+ =
+
2 a12 p1 v1 2 a12 v12
(c) From the results of (a) and (b), obtain
2
p2 p 2 1

p1 + 1
2
( )
1 + M12 2
p1 + 1 2


M12 = 0 .

---------------------------------------------------------------------------------------

Copyright 2010, Elsevier Inc


7-14
Lai et al, Introduction to Continuum Mechanics

Ans. (a)
Using 1v1 = 2 v2 , p1 p2 = 2 v22 1v12 p1 p2 = 1v1 ( v2 v1 )
p2 1v1 ( v2 v1 ) 1v12 ( v2 v1 ) 1v12 v2 1 p2 / p1 v
1 = = = 1 +1= 2
1v1 / p1
2
p1 p1 p1 v1 p1 v1 v1
1 p2 / p1 v2 1 p2 / p1 v2 v2 1 p2
+1= + 1 = . That is, = 1 1 .
v12 ( 1 / p1 ) v1 v12 / a12 v1 v1 M12 p1
(b)
p1 1 2 p2 1 2 1 p2 1 1 1 2
+ v1 = + v2 1+ 1 v12 = + v2
1 1 2 1 2 2 2 p1 2 p1 2 p1

1 v1
2
p2 1 1 v22 2
1 v1 p v 1 v12 v22
1+ = + , 1 + = 2 2 +
2 a12 p1 2 2 a1 2 2 a1 2 p1 v1 2 a12 v12 .
note p1 / 1 = a12 and 1v1 = 2 v2 .

(c)
v 1 p2 1 v12 p2 v2 1 v12 v22
Using 2 = 1 1 , the equation 1 + = +
v1 M12 p1 2 a12 p1 v1 2 a12 v12
2
1 p2 1 p2 1 2 1 p2
1+ M12 = 1 2 1
+ M 1 1 2
1
2
M1 p1
p1 2
M1 p1

p2 p2 1 2
2 2
1 p2 2 p2 1 p2
= + + + M1 1 +
M 2
1 1
p1 p1 p1 2 M12 p1 2M 4 p
1
1 1
2
1 p2 1 p2 1 M12 + 1 1 1 2 4
=
2 p 1 + + +
M1 + 2 M12 + 1 .
M1 1 2 p1 M1
2
2 M1
2

Thus,
1 p2 ( + 1)
2
1 p2 1 M12 + 1 1 1 2 4
1+ M12 = + + M1 + 2 M12 + 1 .
2 M12 p1 2 p1 M12 2 M 2
1
Rearranging,
( + 1) p
2
1 1 p2 1 M12 + 1
M12 = + 2 2
2 M12 p1
2 p1 M1
1 2 1 1 2 2 M12
+ M1 + 2 M 2
+ 1
2 2 M12
1
( 1)
1 p2 ( + 1)
2
p2 1 M12 + 1 1 1 2 M12
0= + + 2
1
M12 p1 2 p1 M12 2 M1 ( 1)
p2
That is,
2

(
2 M12 + 1 p
2 )
2 ( 1)
M12 = 0 .

p1 ( + 1) p1 ( + 1) 2
The above equation has two solutions:

Copyright 2010, Elsevier Inc


7-15
Lai et al, Introduction to Continuum Mechanics

(1) p2 = p1
1 1
(2) p2 = 2 1v12 ( 1) p1 , or p2 = 2 M12 ( 1) p1
+1 +1

Copyright 2010, Elsevier Inc


7-16
Lai et al, Introduction to Continuum Mechanics

CHAPTER 8
8.1 Show that for an incompressible Newtonian fluid in Couette flow, the pressure at the
outer cylinder ( r = Ro ) is always larger than that at the inner cylinder. That is, obtain

Trr ( Ro ) Trr ( Ri ) = o r 2 ( r ) dr
R
Ri
-------------------------------------------------------------------------------
B
Ans. In Couette flow, vr = vz = 0 and v = Ar + , [see Eq. (6.15.4) and (6.15.7)]. Thus,
r
dv v
Trr = T = Tzz = p , Trz = T z = 0 , and Tr = = function of r only .
dr r
T 1 Tr Trr T Trz
Thus, the r-equation of motion rr + + + = r 2 becomes:
r r r z
Trr Ro Trr
dr = r 2 ( r ) dr . Thus,.
Ro
= r 2 . Now,
r Ri r Ri
Ro
Trr ( Ro ) Trr ( Ri ) = r ( r ) dr . The right hand side of this last equation is always
2

Ri
positive.
____________________________________________________________

8.2 Show that the constitutive equation


= 1 + 2 + 3 , with n + n n / t = 2n D, n = 1, 2,3
is equivalent to
+ a1 / t + a2 2 / t 2 + a3 3 / t 3 = bo D + b1D / t + b2 2 D / t 2
where
a1 = ( 1 + 2 + 3 ) , a2 = ( 12 + 23 + 31 ) , a3 = 123
bo = 2 ( 1 + 2 + 3 ) , b1 = 2 1 ( 2 + 3 ) + 2 ( 1 + 3 ) + 3 ( 2 + 1 )
b2 = 2 ( 12 3 + 2 13 + 312 )
-------------------------------------------------------------------------------
Ans.
3 3 3 3 3 j 3 3 j
i = i j = i = i
i =1 t i =1 t j =1 i =1 j =1 t i =1 j =1 t
3
3 3 j 3 3 3 j
= i i + i = ( 2i D i ) + i
i =1 t i =1 j =1 t i =1 i =1 j =1 t
j i j i

3 3 3 3 j 3 3 3 j
= 2D i i + i = 2D i + i
i =1 i =1 i =1 j =1 t i =1 i =1 j =1 t
j i j i
That is,

Copyright 2010, Elsevier Inc


8-1
Lai et al, Introduction to Continuum Mechanics

3 3 2
i = 2D i + 1 + 1 3 + 2 1 + 2 3 + 3 1 + 3 2 (i)
i =1 t i =1 t t t t t t

Next, we have,
( 12 + 23 + 31 ) 2 / t 2 = ( 12 + 23 + 31 ) 2 1 / t 2 + ( 12 + 23 + 31 ) 2 2 / t 2
+ ( 12 + 2 3 + 31 ) 2 3 / t 2 = ( 2 1 + 31 ) 2 1 / t 2 + 2 3 2 1 / t 2 + ( 12 + 23 ) 2 2 / t 2
+ 31 2 2 / t 2 + ( 2 3 + 31 ) 2 3 / t 2 + 12 2 3 / t 2 .
i.e.,
( 12 + 23 + 31 ) 2 / t 2 = 21 ( 2 + 3 ) + 22 ( 1 + 3 ) + 23 ( 2 + 1 ) D / t
( 2 + 3 ) 1 / t ( 1 + 3 ) 2 / t ( 2 + 1 ) 3 / t + 2 3 2 1 / t 2 + 31 2 2 / t 2 (ii)
+ 12 2 3 / t 2 .
Finally, we have,
3 3 3 3
12 3 3 = 123 31 + 12 3 32 + 123 33
t t t t
D
2 2 D
2 2 2 D 2
= 2 3 21 2 21 + 13 2 2 2 22 + 12 23 2 23
t t t t t t

2D 2D 2D 2 1 2 2 2 3
= 22 3 1 + 213 2 + 212 3 23 13 12
t 2 t 2 t 2 t 2 t 2 t 2
3 2D 2D 2D 2 1 2 2 2 3
that is, 12 3 = 22 3 1 + 213 2 + 212 3 2 3 13 12 (iii)
t 3 t 2 t 2 t 2 t 2 t 2 t 2
Thus, (i) + (ii)+ (iii) gives
+ ( 1 + 2 + 3 ) / t + ( 12 + 2 3 + 31 ) 2 / t 2 + 123 3 / t 3
= 2D ( 1 + 2 + 3 ) + 2 1 ( 2 + 3 ) + 2 ( 1 + 3 ) + 3 ( 2 + 1 ) D / t
+2 ( 12 3 + 2 13 + 312 ) 2 D / t 2 .
That is,
2 3 D 2D
+ a1 + a2 2 + a3 3 = bo D + b1 + b2 2 .
t t t t t
where
a1 = ( 1 + 2 + 3 ) , a2 = ( 12 + 23 + 31 ) , a3 = 123
bo = 2 ( 1 + 2 + 3 ) , b1 = 2 1 ( 2 + 3 ) + 2 ( 1 + 3 ) + 3 ( 2 + 1 )
b2 = 2 ( 12 3 + 2 13 + 312 )
____________________________________________________________

8.3 Obtain the force-displacement relationship for the Kelvin-Voigt solid, which consists of a
dashpot (with damping coefficient ) and a spring (with spring constant G ) connected in
parallel. Also, obtain its relaxation function.
-------------------------------------------------------------------------------
Ans. Since the spring and the dashpot are connected in parallel, therefore, the total force is given
by: S = S sp + Sdash and the total displacement is given by = sp = dash . Now,

Copyright 2010, Elsevier Inc


8-2
Lai et al, Introduction to Continuum Mechanics

d d
S sp = G and Sdash = , therefore, S = G + . To find the relaxation function, we let
dt dt
= o H (t ) , where H (t ) is the Heaviside function. Then S = G o H (t ) + o (t ) . Thus, the
relaxation function is S / o = GH (t ) + (t ) .
____________________________________________________________

8.4 (a) Obtain the force-displacement relationship for a dashpot (damping coefficient o )
and a Kelvin-Voigt solid (damping coefficient and spring constant G, see the previous
problem) connected in series. (b) Obtain its relaxation function.
-------------------------------------------------------------------------------
Ans. (a) Let Skv and Sd be the force transmitted by the Kelvin-Voight element and the dashpot
respectively and let kv and d be the elongation of the Kelvin-Voight element and the dashpot
respectively. Then we have, the total force is given by S = Sd = Skv (i) and the total displacement
d d
is given by = d + kv (ii), where Sd = o d = S = Skv = G kv + kv (iii). From (ii) and
dt dt
d d d d kv S 1 S S G
(iii) we have, = + = + ( S G kv ) = + ( d ) (iv). Thus,
dt dt dt o o
d 2 + o dS G d G d d o d 2 ( + o ) dS d
= + , or, = o +S.
dt 2
o dt dt dt G dt 2 G dt dt
Thus, the force-displacement relationship is given by:

S+
( + o ) dS = d + o d 2 . (v)
o
G dt dt G dt 2
(b) Let = o H (t ) , where H (t ) is Heaviside function. Then Eq. (v) gives
dS G G d
+ S = o o (t ) + o o , (vi)
dt ( + o ) ( + o ) ( + o ) dt
where ( t ) is the Dirac function. The integration factor for this ODE is exp Gt / ( + o ) .
Gt Gt Gt
d +o oo G +o oo +o d
Thus, Se = e (t ) + e and
dt ( + o ) ( + o ) dt

Gt Gt Gt
+o G t d
e +o (t )dt + o o e +o
t
Se = o o dt
( + o ) t = ( + o ) dt
oo G t G
+ o o eG t /( +o ) (t ) G t /( +o )
t
=
( + o ) ( + o ) +
o
e
dt . That is,

Gt G t Gt
+o oo G oo +o G oo2G oo +o
Se = + e (t ) = + e (t ) .
( + o ) ( + o ) + o ( + o ) ( + o )
2


G t
S o2G +o o
Thus, the relaxation function is = e + (t ) .
o ( + o ) 2
( + o )
____________________________________________________________

Copyright 2010, Elsevier Inc


8-3
Lai et al, Introduction to Continuum Mechanics

8.5 A linear Maxwell fluid, defined by Eq. (8.1.2), is between two parallel plates which are
one unit apart. Staring from rest, at time t = 0 , the top plate is given a displacement u = vo t while
the bottom plate remains fixed. Neglect inertia effects, obtain the shear stress history.
------------------------------------------------------------------------
Ans. The velocity field for the fluid in this motion is given by (inertia neglected)
v1 = vo H (t ) x2 , v2 = v3 = 0 , where H ( t ) is the Heaviside Function. The only non-zero rate of
deformation component is D12 = vo H ( t ) / 2 . Thus, from the constitutive equation for the linear
dS12 d v
Maxwell fluid, we obtain, S12 + = vo H ( t ) . Thus, S12 et / = o et / H ( t ) . That is,
dt dt
vo t t / vo t t / vo t / t
S12 et / =

e H ( t ) dt =
0
e dt =

(
t /
)
e = vo e 1 . Thus, the shear
0

(
stress history is: S12 = vo 1 e t / . )
____________________________________________________________

Obtain Eq. (8.3.1) i.e., S = 2 (t t ' )D(t ' )dt ' , where (t ) = ( / ) e t / , by solving
t
8.6

dS
the linear non-homogeneous ordinary differential equation S + = 2 D .
dt
-------------------------------------------------------------------------
Ans. The integration factor for this ODE is exp [t / ] . That is the equation can be written as;
2 t /
d
dt
(
Se t / = )

e D . Thus, Set / = ( 2 / ) et / D ( t ) dt
t

S = e t / ( 2 / ) et / D ( t ) dt . That is,
t

( t t ' ) /
S = 2
t

e () t
( ) ()
D t ' dt ' 2 t t ' D t ' dt ' .

____________________________________________________________

(t t ' ) J ( t ' ) dt ' = t ,


t
8.7 Show that for the linear Maxwell fluid, defined by Eq. (8.1.2),
where ( t ) is the relaxation function and J ( t ) is the creep compliance function.
-------------------------------------------------------------------------------
Ans. Let S12 = So H ( t ) be applied to the top plate of a channel of unit depth in which is the linear
Maxwell fluid. [ H ( t ) is the unit step function, i.e., Heaviside function]. Neglecting inertia, the
velocity field is v ( x2 ) = vo x2 , where vo is the velocity of the top plate. Then from the
constitutive equation S + dS / dt = 2 D , we obtain So + So ( t ) = 2 D12 = 2 ( vo / 2 ) = duo / dt ,
duo So
where uo ( t ) is the displacement of the top plate. From = + S ( t ) , we obtain
dt o
t duo t So t So So
o dt dt = o dt + o So ( t ) uo = t + So = (t + ) . Thus, the creep compliance
function is: J (t ) = uo / So = (t + ) / .
Since the relaxation function is (t ) = ( / )et / , therefore,

Copyright 2010, Elsevier Inc


8-4
Lai et al, Introduction to Continuum Mechanics

( )
t t ' /
e ( ) (t + )dt
t t /
( t t ) J ( t ) dt = ( / )e
t t t
(t + ) / dt = (1 / )

e ( ) dt + (1 / ) e ( ) (t )dt .
t t t / t t t /
=

t ( t t ) / ( )
t t ' / t
Now e dt = [ e ]t = = and
t
( t t ) /
( t ) dt ' = ( de( t t)/ )(t )dt = e(t t)/ t e ( ) dt = t 2 .
t t /
t t t
e t =

( t t ' )J ( t ' ) dt ' = + ( t ) = + (t ) = t .


t 1 2
Thus,
____________________________________________________________

8.8 Obtain the storage modulus and loss modulus for the linear Maxwell fluid with a

H ( ) t /
continuous relaxation spectrum defined by Eq. (8.4.1), i.e., ( t ) = e d .
o

-------------------------------------------------------------------------------
Ans. Let the shear strain be: 12 = o ei t . For this strain history, the rate of deformation history is
d
given by 2 D12 = 12 = i o ei t . Thus, from the constitutive equation,
dt
S = 2 ( t t )D ( t ) dt , we have S12 = 2 ( t t )D12 ( t ) dt = i o ( t t ) ei t dt .
t t t


With ( t ) = H ( ) / e t /
d , we have,
o

H ( ) H ( ) e
t /
e ( ) ei t d dt = i o
t t /
et / ei t dt d
t t
S12 = i o '
o =o t =

e(
1+ i )t /
e(
t t / it t 1+ i )t /
Now, t= e e dt =
t =
dt =
(1 + i )
. Thus,

S12

H ( )
H ( )
= i ei t 1 + i ) d G*ei t , where G* = i (1 + i )d is the complex modulus.
=o (
o =o
Now,
H ( ) (1 i ) H ( )
G* = i d = i d
=o (1 + i ) =o (1 + i )(1 i )
(i + 2 ) H ( ) 2 H ( ) H ( )
= d = d + i d
=o (1 + 2 2 ) =o (1 + 2 2 ) =o (1 + 2 2 )

2 2 H ( ) H ( )
Thus, G = d, G = d
=o (1 + 2 2 ) =o (1 + 2 2 )

____________________________________________________________

Show that the viscosity of a linear Maxwell fluid, define by S = 2 ( t t )D ( t ) dt ,


t
8.9

is related to the relaxation function ( t ) and the memory function f ( s ) by the relation

= ( s ) ds = sf ( s ) ds .
o o

Copyright 2010, Elsevier Inc


8-5
Lai et al, Introduction to Continuum Mechanics

------------------------------------------------------------------------

( t t ) D12 ( t ) dt = 2 ( s )D12 ( t s ) ds = 2 ( s )D12 ( t s ) ds .
t 0
Ans. S12 = 2
t = s = s =0
For simple shearing flow, v1 = kx2 , v2 = v3 = 0 , 2D12 = k , so that

S12 = k ( s )ds = S12 / k = ( s )ds . Now, the memory function f ( s ) is related to the
s =0 s =0
relaxation function ( s ) by the relation d ( s ) / ds = f ( s ) . Thus,
d ( s )
= ( s ) ds = s ( s ) 0 s
ds = sf ( s ) ds .
0 0 ds 0
____________________________________________________________

8.10 Show that the relaxation function for the Jeffrey model [Eq. (8.2.7)] with a2 = 0 is given
by [note: Reference to Eq.(8.2.7) is missing in the problem statement in the text]
S12 bo b1 t / a1 b1
(t ) = = 1 e + (t ) , (t ) = Dirac Function .
o 2a1 bo a1 bo
------------------------------------------------------------------------
Ans. Let the shear strain 12 be given by 12 = o H (t ) . Then 2 D12 = d 12 / dt = o (t ) , where
(t ) is Dirac function. From the constitutive equation, we have,
S12 S 1 b b
S12 + a1 = bo o + b1 o 12 + S12 = o o + 1
t 2 2 t t a1 2 a1 a1 t
b b
2

t
( )
S12 et / a1 = et / a1 o o + 1
a1

a1 t
S b t b t d (t )
2 12 et / a1 = o et / a1 (t )dt + 1 et / a1 dt
o a1 a1 dt
bo b1 t / a1 t b t 1 b b b
= + e (t ) 1 (t )et / a1 dt = o + 1 et / a1 (t ) 12
a1 a1 a1 a1 a1 a1 a1
Thus, the relaxation function is:
S12 bo b1 t / a1 b1 b b b
(t ) = 1 e + ( t ) = o 1 1 e t / a1 + 1 ( t )
o 2a1 a1bo 2a1 2a1 a1bo bo
____________________________________________________________

8.11 Given the following velocity field: v1 = 0, v2 = v ( x1 ) , v3 = 0 . Obtain (a) the particle
pathline equations using the current time as the reference time, (b) the relative right Cauchy-
Green deformation tensor and (c) the Rivlin-Ericksen tensors using the equation
Ct = I + ( - t ) A1 + ( - t ) A 2 / 2 + .. (d) the Rivlin-Ericksen tensor A 2 using the recursive
2

equation, [ A 2 ] = [ DA1 / Dt ] + [ A1 ][v ] + [v ] [ A1 ] etc.


T

-------------------------------------------------------------------------------
Ans. (a) Let x = xiei be the position at time of the particle which is at x = xi ei at time t . Then
xi = xi ( x1 , x2 , x3 , ) gives the pathline equation. Thus,
dx1 dx2 dx3
= v1 = 0 (i) , = v2 = v ( x1 ) (ii) , = 0 (iii)
d d d

Copyright 2010, Elsevier Inc


8-6
Lai et al, Introduction to Continuum Mechanics

with the initial conditions: xi = xi ( x1 , x2 , x3 , t ) . Eq (i) gives x1 = f ( x1 , x2 , x3 ) = x1 , Eq. (iii)


dx
gives x3 = g ( x1 , x2 , x3 ) = x3 . Eq. (ii) becomes, 2 = v ( x1 ) x2 = v ( x1 ) + h ( x1 , x2 , x3 ) ,
d
x2 = v ( x1 ) t + h ( x1 , x2 , x3 ) h ( x1 , x2 , x3 ) = x2 v ( x1 ) t x2' = x2 + v ( x1 )( t ) .
Thus, x1 = x1 , x2 = x2 + v ( x1 )( t ) , x3 = x3 ,
1 0 0 1 0 0
(b) [ Ft ] = [xt ] = ( dv / dx1 )( t ) 1 0 = k ( t ) 1 0 , k dv / dx1

0 0 1 0 0 1

1 k ( t ) 0 1 0 0 1 + k 2 ( t ) k ( t ) 0
2

[Ct ] = [ Ft ]T [Ft ] = 0 1

0 k ( t ) 1 0 = k ( t ) 1 0
0
0 1 0 0 1 0 0 1

1 0 0 0 k 0 2k 2 0 0
( t )
2

= 0 1 0 + k 0 0 ( t ) + 0 0 0
0 2
0 0 1 0 0 0 0 0

0 k 0 2k 2 0 0

(c) [ A1 ] = k 0 0 ,
dv
[ A2 ] = 0 0 0 , k =
dx1
0 0 0 0 0 0

0 k 0
DA1
(d) [ A 2 ] = + [ A1 ][v ] + [v ] [ A1 ] , where [ A1 ] = k 0 0 . Thus,
T

Dt 0 0 0

DA1 A1 DA1 A1 ( A1 )ij


=
Dt t + [ A1 ][ v ] Dt = + vk = 0 + 0 = 0 .
ij t ij xk

0 k 0 0 0 0 k 0 0 k 2 0 0
2

[ A1 ][v ] = k 0 0 k 0 0 = 0 [v ]T [ A1 ] = 0 0 0 .
0 0 ,
0 0 0 0 0 0 0 0 0 0 0 0

2k 2
0 0

Thus, [ A 2 ] = [ A1 ][v ] + [v ] [ A1 ] = 0 0 0 .
T

0 0 0

____________________________________________________________

8.12 Given the following velocity field: v1 = kx1 , v2 = kx2 , v3 = 0 . Obtain (a) the particle
pathline equations using the current time as the reference time, (b) the relative right Cauchy-
Green deformation tensor and (c) the Rivlin-Ericksen tensors using the equation
Ct = I + ( - t ) A1 + ( - t ) A 2 / 2 + .. (d) the Rivlin-Ericksen tensor A 2 and A 3 using the recursive
2

equation, [ A 2 ] = [ DA1 / Dt ] + [ A1 ][v ] + [v ] [ A1 ] etc.


T

-------------------------------------------------------------------------------

Copyright 2010, Elsevier Inc


8-7
Lai et al, Introduction to Continuum Mechanics

Ans. (a) (a) Let x = xiei be the position at time of the particle which is at x = xi ei at time t .
Then xi = xi ( x1 , x2 , x3 , ) gives the pathline equation. Thus,
dx1 dx2 dx3
= v1 = kx1 (i) , = v2 = kx2 (ii) , = 0 (iii)
d d d
with the initial conditions: xi = xi ( x1 , x2 , x3 , t ) . Now,
dx1
= kx1 ln x1' = k + g ( x1 , x2 , x3 ) g ( x1 , x2 , x3 ) = ln x1 + kt
d
ln x = k + ln x + kt ln x ln x = k t x = x e ( ) .
k t
1 1 1 1 ( ) 1 1
dx
Similarly, 2 = kx2 x2 = x2 e ( ) and x3 = f ( x1 , x2 , x3 ) = x3
k t
d
Thus, x1 = x1e ( ) , x2 = x2 e ( ) , x3 = x3 ,
k t k t

e k ( t ) 0 0 e2 k ( t ) 0 0

(b) [ Ft ] = [xt ] = 0 e ( ) 0 , [Ct ] = [ Ft ] [ Ft ] = 0 e ( ) 0
k t T 2 k t

0 0 1 0 0 1

2 3
Since e ( ) = 1 m 2k ( t ) +
m2 k t 4k 8k
( t )2 m ( t )2 + ... , therefore,
2 3!
2k 0 0 4k 0 0 8k 3 0 0
2

( t)
2 ( t )3
[Ct ] = [ I ] + ( t ) 0 2k 0 + 0 4k 0 2 + 0 8k 0 3! + ...
2 3

0
0 0 0 0 0 0 0 0

2k 0 0 4k 2
0 0 8k 3
0 0

(c) [ A1 ] = 0 2k 0 , [ A 2 ] = 0 4k 2 0 , [ A3 ] = 0 8k 3 0
0
0 0 0 0 0 0 0 0

k 0 0 2 k 0 0
(d) with v1 = kx1 , v2 = kx2 , v3 = 0 , [v ] = 0 k 0 [ A1 ] = 0 2k 0

0 0 0 0 0 0

DA1 A1 DA1 A1 ( A1 )i j
=
Dt t + [ A1 ][ v ] Dt = + vk = 0 + 0 = 0
i j t i j xk

[ A 2 ] = [ A1 ][v ] + [v ]T [ A1 ]
0 4k 0
2
2k 0 0 k 0 0 k 0 0 2 k 0

0

= 0 2k 0 0 k 0 + 0 k 0 0 2k 0 = 0 4k 2 0
0
0 0 0 0 0 0 0 0 0 0 0 0 0 0

Next,

Copyright 2010, Elsevier Inc


8-8
Lai et al, Introduction to Continuum Mechanics

4k 2 0 0 k
0 0 4k
3
0 0

[ A 2 ][v ] = 0 4k 2 0 0
k 0 = 0 4k 3 0 .

0 0 0 0
0 0 0 0 0

8k 3
0 0

Thus, [ A3 ] = [ 0] + [ A 2 ][v ] + [v ] [ A 2 ] = 0
T
8k 3 0 .

0 0 0

____________________________________________________________

8.13 Given the following velocity field: v1 = kx1 , v2 = kx2 , v3 = 2kx3 . Obtain (a) the
particle pathline equations using the current time as the reference time, (b) the relative right
Cauchy-Green deformation tensor and (c) the Rivlin-Ericksen tensors using the equation
Ct = I + ( - t ) A1 + ( - t ) A 2 / 2 + .. (d) the Rivlin-Ericksen tensor A 2 and A3 using the recursive
2

equation, [ A 2 ] = [ DA1 / Dt ] + [ A1 ][v ] + [v ] [ A1 ] etc.


T

-------------------------------------------------------------------------------
Ans. (a) Let x = xiei be the position at time of the particle which is at x = xi ei at time t . Then
xi ( x1 , x2 , x3 , ) gives the pathline equation. Thus,
dx1 dx2 dx3
= v1 = kx1 (i) , = v2 = kx2 (ii) , = 2kx3 (iii)
d d d
with the initial conditions: xi = xi ( x1 , x2 , x3 , t ) . Now,
dx1
= kx1 ln x1 = k + g ( x1 , x2 , x3 ) g ( x1 , x2 , x3 ) = ln x1 kt
d
ln x = k + ln x kt ln x ln x = k t x = x e ( ) .
k t
1 1 1 1 ( ) 1 1
dx
Similarly, 2 = kx2 x2 = x2 e ( ) and x3 = x3e ( ) . Thus,
k t 2 k t
d
k ( t )
, x2 = x2 e ( ) , x3 = x3e ( )
k t 2 k t
x1 = x1e
(b)
e k ( t ) 0 0 e 2 k ( t ) 0 0

[Ft ] = xt' = 0 ek ( t ) 0 , [Ct ] = [ Ft ] [ Ft ] = 0

T

e ( )
2 k t
0

e ( ) e ( )
0 2 k t 0 4 k t
0 0

Since
4k 2 8k 3
e ( ) = 1 + 2k ( t ) +
2 k t
( t )2 + ( t )3 + ...,
2 3!
4 k ( t ) 16k 2 64k 3
e = 1 4k ( t ) + ( t )2 ( t )3 + ...,
2 3!
therefore, [Ct ] = [ I ]

Copyright 2010, Elsevier Inc


8-9
Lai et al, Introduction to Continuum Mechanics

0 4k 0 8 k 3 0 0
2
2k 0

0
( t )2 ( t )3
+ ( t ) 0 2k 0 + 0 4k 2
0 + 0 8k 3
0 + ...
0 2 3!
0 4k 0 0 16k 2 0 0 64k 3

2k 0 0 4k 2
0 0 8k 3
0 0

(c) [ A1 ] = 0 2k 0 , [ A 2 ] = 0 4k 2 0 , [ A 3 ] = 0 8k 3 0

0 0 4k 0 0 16k 2


0 0 64k 3

k 0 0 2 k 0 0

(d) with v1 = kx1 , v2 = kx2 , v3 = 2kx3 , [v ] = 0 k
0 [ A1 ] = 0 2k 0
0 0 2k 0 0 4k

DA1 A1 DA1 A1 ( A1 )ij


=
Dt t + [ A1 ][ v ] Dt = + vk = 0 + 0 = 0
ij t ij xk

[ A 2 ] = [ A1 ][v ] + [v ]T [ A1 ]
0 4k 0
2
2k 0 0 k 0 0 k 0 0 2k 0

0

= 0 2k 0 0 k 0 + 0 k 0 0 2k 0 = 0 4k 2 0 .
0
0 4k 0 0 2k 0 0 2k 0 0 4k 0 0 16k 2

Next,
4k 2 0 0 k 0 0 4k
3
0 0

[ A 2 ][v ] = 0 4k 2 0 0 k 0 = 0 4k 3 0 .

0 0 16k 2 0 0 2k 0 0 32k 3

8k 3
0 0

Thus, [ A3 ] = [ 0] + [ A 2 ][v ] + [v ] [ A 2 ] = 0 8k 3
T
0 Etc.

0 0 64k 3

____________________________________________________________

8.14 Given the following velocity field: v1 = kx2 , v2 = kx1 , v3 = 0 . Obtain (a) the particle
pathline equations using the current time as the reference time, (b) the relative right Cauchy-
Green deformation tensor and (c) the Rivlin-Ericksen tensors using the equation
Ct = I + ( - t ) A1 + ( - t ) A 2 / 2 + .. (d) the Rivlin-Ericksen tensor A 2 and A3 using the recursive
2

equation, [ A 2 ] = [ DA1 / Dt ] + [ A1 ][v ] + [v ] [ A1 ] etc.


T

-------------------------------------------------------------------------------
Ans. (a) Let x = xiei be the position at time of the particle which is at x = xi ei at time t . Then
xi = xi ( x1 , x2 , x3 , ) gives the pathline equation. Thus,
dx1 dx2 dx3
= v1 = kx2 (i) , = v2 = kx1 (ii) , = 0 (iii)
d d d

Copyright 2010, Elsevier Inc


8-10
Lai et al, Introduction to Continuum Mechanics

with the initial conditions: xi = xi ( x1 , x2 , x3 , t ) . Now,


dx1 d 2 x1 dx2 d 2 x1
(i) = kx2 = k = k 2
x1 k 2 x1 = 0
d d 2 d d 2

x1 = A sinh k + B cosh k x1 = A sinh kt + B cosh kt (iv)


1 dx1
(ii) x2 = = A cosh k + B sinh k x2 = A cosh kt + B sinh kt (v)
k d
(iv) and (v) gives A = x1 sinh kt + x2 cosh kt , B = x1 cosh kt x2 sinh kt
x1 = x1 ( cosh kt cosh k sinh kt sinh k ) + x2 ( cosh kt sinh k sinh kt cosh k )
x2 = x1 ( cosh kt sinh k sinh kt cosh k ) + x2 ( cosh kt cosh k sinh kt sinh k )
That is,
x1 = x1 cosh k ( t ) + x2 sinh k ( t ) , x2 = x1 sinh k ( t ) + x2 cosh k ( t ) , x3 = x3
cosh k ( t ) sinh k ( t ) 0
[Ft ] = [xt ] = sinh k ( t ) cosh k ( t ) 0 ,
0 0 1

(b)
cosh 2 x + sinh 2 x 2cosh x sinh x 0

[Ct ] = [ Ft ] [Ft ] = 2cosh x sinh x sinh 2 x + cosh 2 x 0 , x k ( t )
T


0 0 1

Since
x2 x3
cosh x = 1 + + O( x 4 ), sinh x = x + + O( x5 )
2 6
2 x3
cosh 2 x = 1 + x 2 + O( x 4 ), sinh 2 x = x 2 + O( x 4 ), sinh x cosh x = x + + O( x5 )
3
4 3
1 + 2 x + .. 2 x + 3 x + ... 0
2

0 2k 0
[Ct ] = 2 x + 3 x + ... 1 + 2 x + ... 0 = [I ] + 2k 0 0 ( t )

4 3 2

0 0 0
0 0 1

4k 2 0 0 0 8k 3 0
( t )2 ( t )3
+ 0 4k 2
0 + 8k 3
0 0 + ...
2 6
0 0 0
0 0 0

Thus,
(c)
0 2k 0 4k 2 0 0 0 8k 3 0

[ A1 ] = 2k 0 0 , [ A 2 ] = 0 4k 2 0 , [ A3 ] = 8k 3 0 0 etc.
0 0 0
0 0 0
0 0 0

Copyright 2010, Elsevier Inc


8-11
Lai et al, Introduction to Continuum Mechanics

0 k 0 0 2k 0
(d) with v1 = kx2 , v2 = kx1 , v3 = 0 , [v ] = k 0 0 [ A1 ] = 2k 0
0 .
0 0 0 0 0 0

DA1 A1 DA1 A1 ( A1 )ij


=
Dt t + [ A1 ][ v ] Dt = + vk = 0 + 0 = 0 .
ij t ij xk

[ A 2 ] = [ A1 ][v ] + [v ]T [ A1 ]
0 0 k 0 2k 0 4k 2 0
2
0 2k

0

0

[ A1 ][v ] = 2k 0 0 k 0 0 = 0 2k 2 0 [ A2 ] = 0 4k 2 0 .
0
0 0 0 0 0 0 0 0 0 0 0

Next,
4k 2 0 0 0 k 0 0 4k 3 0

[ A 2 ][v ] = 0 4k 2 0 k 0 0 = 4k 3 0 0 .

0 0 0 0 0 0 0 0 0

0 8k 0
3

Thus, [ A3 ] = [ 0] + [ A 2 ][v ] + [v ] [ A 2 ] = 8k 3 0 0 .
T


0 0 0

____________________________________________________________

8.15 Given the velocity field in cylindrical coordinates: vr = 0, v = 0, vz = v ( r ) , obtain the


second Rivlin-Ericksen tensors A N , N = 2,3,... using the recursive formula.
------------------------------------------------------------------------
0 0 0 0 0 k

Ans. [v ] = 0 0 0 , k =
dv
, [ A1 ] = [v ] + [v ] = 0 0 0
T
dr
k 0 0 k 0 0
Since ( A1 )ij =constant, independent of time and space, therefore
DA A
1
= 1 + ( A1 )ijk vk = [ 0] .
Dt ij t ij

Copyright 2010, Elsevier Inc


8-12
Lai et al, Introduction to Continuum Mechanics

[ A 2 ] = A1 ( v ) + ( v )T A1
0 0 k 0 0 0 0 0 k 0 0 k 2k 0 0
2

= 0 0 0 0 0 0 + 0 0 0 0 0 0 = 0 0 0 .
k 0 0 k 0 0 0 0 0 k 0 0 0 0 0

[ A3 ] = A 2 ( v ) + ( v )T A 2
2k 2 0 0 0 0 0 0 0 k 2k 2 0 0 0 0 0

= 0 0 0 0 0 0 + 0 0 0 0 0 0 = 0 0 0 .
0 0 0 k 0 0 0 0 0 0 0 0 0 0 0

Thus, A N = 0, N = 3, 4...
____________________________________________________________

8.16 Using the equations given in Appendix 8.1 for cylindrical coordinates, verify that the
rr component of the third order tensor T is given by:
1 Trr T r + Tr
( T )rr
=
r r
------------------------------------------------------------------------------
Ans. From the equations
T
( T )ijm hm = ij + Tqj qmi + Tiq qmj no sum on m, sum on q,
xm
and hr = 1, h = r , hz = 1; r = 1, r = 1 , all other ijk = 0
we have,
Trr T
( T )rr h = + Tqr q r + Trq q r = rr + T r r + Tr r , thus,

Trr 1 Trr T r + Tr
( T )rr r = + T r ( 1) + Tr ( 1) ( T )rr = .
r r
____________________________________________________________

8.17 Using the equations given in Appendix 8.1 for cylindrical coordinates, verify that the
r component of the third order tensor T is given by:
1 Tr Trr T
( T )r = +
r r
------------------------------------------------------------------------------
Ans. From the equations
T
( T )ijm hm = ij + Tqj qmi + Tiq qmj no sum on m, sum on q
xm
and hr = 1, h = r , hz = 1; r = 1, r = 1 , all other ijk = 0.
we have,

Copyright 2010, Elsevier Inc


8-13
Lai et al, Introduction to Continuum Mechanics

Tr T
( T )r h = + Tq q r + Trq q ( T )r h = r + T r + Trr r

T 1 Tr Trr T
( T )r r = r + T ( 1) + Trr (1) ( T )r = + .
r r
____________________________________________________________

8.18 Using the equations given in Appendix 8.1 for spherical coordinates, verify that the
rr component of the third order tensor T is given by:

( T )rr =
1 Trr

(T r + Tr )
r sin r
------------------------------------------------------------------------
Ans. From the equations
T
( T )ijm hm = ij + Tqj qmi + Tiq qmj no sum on m, sum on q
xm
and hr = 1, h = r , h = r sin ; r = 1, r = sin ,
r = sin , = cos , r = 1, = cos all other ijk = 0
we have,
Trr T
( T )rr h = + Tqr q r + Trq q r ( T )rr h = rr + T r r + Tr r

T
( T )rr ( r sin ) = rr + T r ( sin ) + Tr ( sin )

1 Trr T r + Tr
( T )rr = .
r sin r
____________________________________________________________

8.19 Using the equations given in Appendix 8.1 for spherical coordinates, verify that the
component of the third order tensor T is given by:

1 T
+
(
Tr + T r) (
+
)
T + T cot
r sin r r
-------------------------------------------------------------------------------
Ans. From the equations
T
( T )ijm hm = ij + Tqj qmi + Tiq qmj no sum on m, sum on q
xm
hr = 1, h = r , h = r sin ; r = 1, r = sin ,
r = sin , = cos , r = 1, = cos all other ijk = 0
we have,

Copyright 2010, Elsevier Inc


8-14
Lai et al, Introduction to Continuum Mechanics

T
( T ) h = + Tq q + T q q

T
( T ) h = + Tr r + T + T r r + T

T
( T ) h =

( ) (
+ Tr + T r r + T + T )
T
=

( ) ( )
+ Tr + T r ( sin ) + T + T cos

( T ) =
1 T
+
Tr + T r (
+
) (
T + T cot )
r sin r r
____________________________________________________________

8.20 Given the velocity field in cylindrical coordinates: vr = 0, v = v ( r ) , vz = 0 , obtain (a)


the first Rivlin-Ericksen tensor A1 (b) A1 (c) the second Rivlin-Ericksen tensors A 2 , using the
recursive formula..
------------------------------------------------------------------------
vr 1 vr vr
v 0
v( r )
0
r r z r

v 1 v v dv
Ans. [v ] = + vr

=
0 0

r r z dr

v 1 vz vz 0 0 0
z

r r z
0 k ( r ) 0
[ A1 ] = [v ] + [v ] = k ( r ) 0 0 , k =

T dv v( r )

0 dr r
0 0
DA A
1
= 1 + ( A1 )ijk vk
Dt ij t ij
( A1 ) v ( A1 ) v ( A1 ) v
rr r rz

= ( A1 )ijk vk = ( A1 ) r v ( A1 ) v ( A1 ) z v


( A1 ) zr v ( A1 ) z v ( A1 ) zz v
The components of the third order tensor ( A1 ) can be obtained from Appendix 8.1 as:
1 Arr A r + Ar 2k 1 Ar Arr A
( A1 )rr = = , ( A1 )r = + =0
r r r r r
1 Arz A z
( A1 )rz = =0
r r
1 A A A 2k 1 A A
( A1 ) r = r + rr = 0 , ( A1 ) = , ( A1 ) z = z + rz = 0
r r r r r

Copyright 2010, Elsevier Inc


8-15
Lai et al, Introduction to Continuum Mechanics

1 Azr Az 1 Az Azr 1 Azz


( A1 ) zr = = 0 , ( A1 ) z = + = 0 , ( A1 ) zz = =0
r r r r r
2k / r 0 0 2kv / r 0 0
DA
Thus, 1
= v 0 2k / r 0 = 0 2kv / r 0 ,
Dt ij 0 0 0 0 0 0
0 k ( r ) 0 0 v(r ) / r 0 kdv / dr 0 0
[ A1 ][v ] = k ( r ) 0 0 dv / dr

0
0 = 0 kv / r 0 ,
0 0 0 0 0 0 0 0 0

0 dv / dr 0 0 k ( r ) 0 kdv / dr 0 0
[v ] [ A1 ] = v(r ) / r 0 0 k ( r ) 0 0 = 0
T
kv / r 0 ,
0 0 0 0 0 0 0 0 0
k ( dv / dr v / r ) 0 0 2k 2 0 0
DA1
+ [ A1 ][v ] + [v ] [ A1 ] = 2
T
A2 = 0 0 0 = 0 0 0 .
Dt
0 0 0 0 0 0

____________________________________________________________

DA N
+ A N ( v ) + ( v ) A N .
T
8.21 Derive Eq. (8.11.3), i.e., A N +1 =
Dt
------------------------------------------------------------------------
Ans. We had {see Eq. (8.11.7)},
D N +1
DN
Dt N ( )
ds 2
= d x A N d x
Dt N +1 ( )
ds 2 =
Ddx
Dt
A N dx + dx
DA N
Dt
dx + dx A N
Ddx
Dt
. That is,

D N +1
Dt N +1 ( ds ) == (v ) dx A
2 DA N
N dx + dx
Dt
dx + dx A N ( v ) dx

DA N
= dx ( v ) A N dx + dx A N ( v ) dx + dx
T
dx
Dt
DA N
= dx ( v ) A N + A N ( v ) +
T
dx = dx A N +1dx.
Dt
DA N
+ A N ( v ) + ( v ) A N
T
Thus, A N +1 =
Dt
____________________________________________________________

8.22 Let S DT / Dt + TW WT , where T is an objective tensor and W is the spin tensor,


show that S is objective, i.e., S* = Q ( t ) SQ T ( t ) .
-------------------------------------------------------------------------
Ans. Since T is objective, therefore T* = Q ( t ) TQ T ( t ) and from Eq. (8.13.13),
W* = ( dQ / dt ) QT ( t ) + Q ( t ) WQT ( t ) , therefore,

Copyright 2010, Elsevier Inc


8-16
Lai et al, Introduction to Continuum Mechanics

DT*
S* + T* W* - W*T*
Dt
dQ DT T dQT dQ T
= TQ T + Q Q + QT + QTQT Q + QTQT QWQT
dt Dt dt dt
dQ T
Q QTQ T QWQT Q ( t ) TQT ( t ) .
dt
dQ T dQ T
Now, Q ( t ) Q T ( t ) = I Q ( t ) = Q ( t ) , therefore, the above equation becomes
dt dt
dQ DT T dQ T dQT
S* = TQT + Q Q + QT QT + QTWQ
T
dt Dt dt dt
dQ
TQT QWTQT ( t ) .
dt
That is,
DT
S* = Q ( t ) + TW WT Q T ( t ) = Q ( t ) SQ T ( t ) .
Dt
____________________________________________________________

8.23 Obtain the viscosity function and the two normal stress function for the nonlinear

viscoelastic fluid defined by S = f 2 ( s ) I Ct1 (t s ) ds
0
------------------------------------------------------------------------------
Ans. For v1 = kx2 , v2 = v3 = 0 , we have [see Section 8.9, Eq.(8.9.12)]
1 k ( t ) 0 1 + k 2 ( t )2 k ( t ) 0

[Ct ( )] = k ( t ) k ( t )2 + 1
2
0 , Ct1 ( ) = k ( t )

1 0
0 0 1
0 0 1

1 + k 2 s 2 ks 0 k 2 s 2 ks 0

Ct1 (t s ) = ks 1 0 . Thus, I Ct1 (t s ) = ks 0 0

0 0 1 0 0 0


S12
S12 = k sf 2 ( s )ds = sf 2 ( s )ds,.
0
k 0

S11 = k 2
s
2
f 2 ( s )ds, S22 = 0, S33 = 0 ,
0

1 = S11 S22 = k 2 s 2 f 2 ( s )ds, 2 = S22 S33 = 0.
0
____________________________________________________________

8.24 Derive the following transformation laws [Eqs.(8.13.8) and Eq. (8.13.12)] under a change
of frame.
Vt* = Q ( ) Vt QT ( ) and R*t = Q ( ) R t QT ( t )
-------------------------------------------------------------------------------

Copyright 2010, Elsevier Inc


8-17
Lai et al, Introduction to Continuum Mechanics

Ans. Since Ft = Vt R t and Ft* = Vt*R*t , therefore, from Ft* ( ) = Q ( ) Ft ( ) QT ( t ) , we get


Vt*R*t = Q ( ) Vt R t QT ( t ) = Q ( ) Vt QT ( ) Q ( ) R t QT ( t ) , where Q ( ) Vt QT ( ) is a

symmetry tensor and Q ( ) R t Q ( t ) is an orthogonal tensor. Therefore, the uniqueness of the
T

polar decomposition leads to
Vt* = Q ( ) Vt Q T ( ) and R*t = Q ( ) R t QT ( t ) .
____________________________________________________________

( DJ ( ) DF ( )
8.25 From T L and t = v , show that
D =t D =t
( o
T = T + TD + DT . [note misprint in the problem in text]
------------------------------------------------------------------------------
Ans. From J L ( ) = FtT ( ) T ( ) Ft ( ) , we have,
DJ L ( ) DFtT ( ) DT ( ) DFt ( )
= T ( ) Ft ( ) + FtT ( ) Ft ( ) + FtT ( ) T ( )
D D D D
DJ L ( ) DT
= ( v ) T ( t ) + + T ( t ) v [Note Ft ( t ) = FtT ( t ) = I ]
T
Thus,
D =t Dt
Now, v = D + W , therefore,
DJ L ( )

D
=
=t
DT
Dt
+ (D + W) T + T(D + W) =
T DT
Dt
+ DT + TD + W T T + TW ( )
DT
+ DT + TD + ( TW WT ) = T + DT + TD
o
=
Dt
____________________________________________________________

8.26 Consider J U ( ) = Ft1 ( ) T ( ) Ft1T ( ) . Show that (a) DJ U ( ) / D is objective


=t
DT
and (b) DJ U ( ) / D = T ( v ) ( v ) T = T ( TD + DT ) .
T o
=t D
-------------------------------------------------------------------------------

( ) ( )
T T
Ans. (a) Given J U ( ) = Ft1 ( ) T ( ) Ft1 ( ) , and J*U ( ) = F* t1 ( ) T* ( ) F* t1 ( ) .
In a change of frame (see Section 8.13. Eq.(8.13.6), Ft * ( ) = Q ( ) Ft ( ) Q ( t ) , so that
T

( ) ( ) = Q ( ) ( Ft1 ) ( ) QT ( t ) .
T T
F* t1 ( ) = Q ( t ) Ft1 ( ) QT ( ) and F* t1

Also, T* (t ) = Q(t )T(t )QT (t ) . Thus

( )
T
J*U ( ) = Q ( t ) Ft1 ( ) QT ( ) Q( )T( )QT ( )Q ( ) Ft1 ( ) QT ( t )
( )
T
= Q ( t ) Ft1 ( ) T( ) Ft1 ( ) QT ( t ) . That is,

(
J*U ( ) = Q ( t ) JU ( ) QT ( t ) and D N J*U ( ) / D N = Q ( t ) D N JU ( ) / D N QT ( t ) . Thus, )

Copyright 2010, Elsevier Inc


8-18
Lai et al, Introduction to Continuum Mechanics

D N J*U ( ) D N J ( )
= Q ( t ) QT ( t ) .
U

D
N
=t D N
=t

( )
T
(b) DJ U ( ) / D = DFt1 / D T ( t ) + [ DT / D ] =t + T ( t ) D Ft1 / D .
=t =t =t
Now, Ft ( ) Ft 1 ( ) = I [ DFt / D ] Ft 1 ( ) + Ft ( ) DFt 1 / D = 0

DFt ( ) / D = Ft ( t ) [ DFt / D ] Ft ( t ) = [ DFt / D ] = ( v ) Ft ( )
1 1 1
=t =t =t =t
= ( v ) . Thus,
DT DT
DJ U ( ) / D = T ( v ) ( v ) T = T(D W) (D + W)T .
T
=t Dt Dt
That is, the upper convected derivative of T can be written:

T DJ U ( ) = DT + TW WT ( TD + DT ) = T o
( TD + DT ) .
D =t Dt
____________________________________________________________

8.27 Given the velocity field of a plane Couette flow: v1 = 0, v2 = kx1 . (a) For a Newtonian
fluid, find the stress field [ T ] and the co-rotational stress rate T . (b) Consider a change of
o

frame (change of observer) described by:
x1* cos t sin t x1 cos t sin t
= , [Q ] =
x2* sin t cos t x2 sin t cos t
Find v* , * v* , D* and W* .

(c) Find the co-rotational stress rate for the starred frame
(d) Verify that the two stress rates are related by the objective tensorial relation.
------------------------------------------------------------------------------
Ans.
0 0 0 k / 2 0 k / 2
(a) [v ] = , [ D] = , [ W] =
0
. Thus, stress tensor is
k 0 k / 2 0 k / 2
p 0 0 k / 2 p k
[T] = 0
p
+ 2 =
0 k p
.
k / 2
p k 0 k / 2 0 k / 2 p k k 2 0
[TW ] [ WT] = k p k / 2

0 k / 2
=
0 k p 0

k 2

DT k 0 k 2 0
2
Co-rotational stress rate is: T =
o
+ = .
Dt 0 2
2

k 0 k
(b) From Eq. (5.56.12) of Chapter 5, we have,
[v * (x*)] = [Qv ] + ( dQ / dt ) x = [Qv ] + ( dQ / dt ) QT x * . Thus,
v1* cos t sin t 0 sin t cos t cos t sin t x1
*
= + cos t sin t sin t cos t * . Since,

v2* sin t cos t v2 x2

Copyright 2010, Elsevier Inc


8-19
Lai et al, Introduction to Continuum Mechanics

x1 cos t sin t x1
*

x sin t cos t * x1 = cos tx1 + sin tx2 v2 = k cos tx1 + sin tx2
2
=
x2
* *
( * *
)
Therefore,
v1* v2 sin t 0 1 x1
*
(
cos t sin tx* + sin 2 tx*
1 )2
x2*
= + 1 0 * = k +
v2* v2 cos t x2 (
cos 2
tx*
1 + sin t cos )
tx*
2

x1
*

from which, we get,
( sin 2t ) / 2 sin 2 t 0 1
( * v * ) = k + ,
cos t 2
( sin 2t ) / 2 1 0
( sin 2t ) / 2 ( cos 2t ) / 2 0 1 / 2 0 1
[ D *] = k cos 2t / 2 sin 2t / 2 , [ W *] = k 1 / 2 0 + 1 0 .
( ) ( )
(c) For the Newtonian fluid, the stress field in the starred-frame is:
p k ( sin 2t ) k cos 2t
[T *] = k cos 2t
) p + k ( sin 2t )
,
(
where the indeterminate pressure p is time independent. Thus,
DT * ( cos 2t ) ( sin 2t )
Dt = 2 k ( sin 2t ) ( cos 2t ) , and

k cos 2t p + k sin 2t k cos 2t p + k sin 2t
T* W* = k +
2 p + k ( sin 2t ) k cos 2t p + k sin 2t k cos 2t


k cos 2t p k sin 2t k cos 2t p k sin 2t
W*T* = k +
2 p k sin 2t k cos 2t p k sin 2t k cos 2t
Thus,
k cos 2t k sin 2t cos 2t sin 2t
T* W* W*T* = k + 2 k .
k sin 2t k cos 2t sin 2t cos 2t
o cos 2t sin 2t
Thus, T * = DT* / Dt + T* W* W*T* = k 2
sin 2t cos 2t
(d) [Q ] T [Q ]
o T

cos t sin t k 0 cos t sin t cos 2t sin 2t
2
= = k 2
2 sin t cos t
sin t cos t 0 k sin 2t cos 2t
Thus, we have
T* = [Q ] To [Q ]T .
o

____________________________________________________________

8.28 Given the velocity field: v1 = kx1 , v2 = kx2 , v3 = 0 . Obtain (a) the stress field for a
second-order fluid (b) the co-rotational derivative of the stress tensor
------------------------------------------------------------------------

Copyright 2010, Elsevier Inc


8-20
Lai et al, Introduction to Continuum Mechanics

k 0 0
Ans. (a) [v ] = 0 k 0 = [ D] , [ W ] = [ 0] ,
0 0 0

0 4k 0
2
2k 0 0 2k 0 0 2k 0

0

0 0 0 = 0
[ A1 ] = [ 2D] = 0 2k 0 [ A1 ] = 0
2
2k 2k 4k 2 0
0
0 0 0 0 0 0 0 0 0 0 0

[ A 2 ] = DA1 / Dt + A1 ( v ) + ( v )T A1 = A1 ( v ) + ( v )T A1
0 4k 0
2
2k 0 0 k 0 0 k 0 0 2 k 0

0

= 0 2k 0 0 k 0 + 0 k 0 0 2k 0 = 0 4k 2 0
0
0 0 0 0 0 0 0 0 0 0 0 0 0 0

The second-order fluid is defined by Eq.(8.18.6):
T = pI + 1A1 + 2 A12 + 3 A 2

2k 0 0 4k 2 0 0 4k 2 0 0

[T] = p [I ] + 1 0 2k 0 + 2 0 4k 2 0 + 3 0 4k 2 0
0
0 0 0 0 0 0 0 0

T11 = p 21k + 4( 2 + 3 )k 2 , T22 = p + 21k + 4( 2 + 3 )k 2 , T33 = p .

To obtain the pressure p, we first calculate the acceleration:


k 0 0 kx1 k x1
2

[a] = [v / t ] + [v ][ v ] = 0 k 0 kx2 = k 2 x2
0 0 0 0 0

Tij
Equations of motion = ai then give
x j
p p p
= k 2 x1 , = k 2 x2 , = 0, thus,
x1 x2 x2
p = k 2 ( x12 + x22 ) / 2 +C .
o
(b) The co-rotational derivative of T : T = DT / Dt + TW - WT . Since W = 0 ,
1 0 0
DT Tij Tij p p
( )
o

T =
ij Dt
ij 1 0 0 1

= x v1 + x v2 = x v1 x v2 0 1 0
2 1 2

1 0 0 1 0 0
p p
= k
x1
x1
x2
2 2
(
2 2
)
x2 0 1 0 = k k x1 + k x2 0 1 0 = k v22 v12 [ I ]

( )
0 0 1 0 0 1

____________________________________________________________

Copyright 2010, Elsevier Inc


8-21
Lai et al, Introduction to Continuum Mechanics

(
8.29 Show that the Lower Convected derivative of A1 is A 2 , i.e., A1 = A 2 .
-------------------------------------------------------------------------
Ans. From Eq.(8.19.22),
(
A1 = A1 + A1D + DA1 = ( DA1 / Dt + A1W WA1 ) + A1D + DA1
o

= DA1 / Dt + A1 ( W + D ) + ( D W ) A1 = DA1 / Dt + A1 ( v ) + ( v ) A1 = A 2 .
T

____________________________________________________________

8.30 The Reiner-Rivlin fluid is defined by the constitutive equation:


T = pI + S, S = 1 ( I 2 , I 3 ) D + 2 ( I 2 , I 3 ) D2
where Ii are the scalar invariants of D . Obtain the stress components for this fluid in a simple
shearing flow.
-------------------------------------------------------------------------------
Ans. In a simple shearing flow, v1 = kx2 , v2 = v3 = 0 ,

0 k / 2 0 k 2 / 4 0 0
k2
[ D] = k / 2 0 0 , D2 = 0 k 2 / 4 0 , I 2 = 4 , I3 = 0
0
0 0 0 0 0

0 k / 2 0 k 2 / 4 0 0

( ) (
[T] = p [I ] + 1 k 2 / 4,0 k / 2 0 0 + 2 k 2 / 4,0 0 k 2 / 4 0 .

)
0 0 0 0 0 0

____________________________________________________________

N
1 n
8.31 The exponential of a tensor A is defined as: exp [ A ] = I + A . If A is an objective
1 n!
tensor, is exp [ A ] also objective?
-------------------------------------------------------------------------------
Ans. Yes. Because

( )
2
A* = Q ( t ) AQ T ( t ) A* = Q ( t ) AQ T ( t ) Q ( t ) AQ T ( t ) = Q ( t ) A 2QT ( t )

( )
N
A* = Q ( t ) A N QT ( t )

( )
N
That is, A* is objective for all N . As a consequence, exp [ A ] is objective.
____________________________________________________________

8.32 Why is it that the following constitutive equation is not acceptable:


T = pI + S, S = ( v ) , where v is velocity and is a constant
-------------------------------------------------------------------------
Ans. Because v is not objective.
____________________________________________________________

Copyright 2010, Elsevier Inc


8-22
Lai et al, Introduction to Continuum Mechanics

8.33 Let da and dA denote the differential area vectors at time and time t respectively. For
an incompressible fluid, show that
D N da 2 / D N = dA D N Ct1 / D N dA dA M N dA
=t =t
where da is the magnitude of da and the tensors M N are known as the White-Metzner tensors.
--------------------------------------------------------------------------------
Ans. From Eq. (3.27.12), we have, [note here dA is the reference area and da is the area at the

( ) dA . For an incompressible fluid, ( det F ) = 1 , so that


T
running time ], da = ( det F ) F 1

( ) da da = ( F ) dA ( F ) dA = dA ( F )( F ) dA = dA ( F F ) dA .
T T T T 1
da = F 1 dA , 1 1 1 1 T

That is, da 2 = dA Ct1dA . Thus,


D N da 2 D N Ct1 D N Ct1
N
= d A N
d A dA M N d A , where M N = N
.
D =t D =t D =t
____________________________________________________________

8.34 (a) Verify that Oldroyd's lower convected derivatives of the identity tensor I are the
Rivlin-Ericksen tensor A N . (b) Verify that Oldroyd upper derivatives of the identity tensor are
the negative White-Metzner tensors [see Prob. 8.33 for the definition of White-Metzner tensor].
-------------------------------------------------------------------------------
Ans. (a) The Nth lower convected derivative of T is given by
D N J L / D N , where J L ( ) = FtT ( ) T ( ) Ft ( ) . For T = I ,
=t
DN JL D N Ct ( )
J L ( ) = FtT ( ) Ft ( ) = Ct ( ) . Thus, = = AN
D =t D
N N
=t
(b) The Nth upper convected derivative of T is given by

( )
T
D N JU / D N , where JU ( ) = Ft1 ( ) T ( ) Ft1 ( ) . For T = I ,
=t
D N JU D N Ct1 ( )
( )
T
JU ( ) = Ft1 ( ) Ft1 ( ) = Ct1 ( ) . Thus,
N
= = M N .
D =t D
N
=t
____________________________________________________________
( (
Obtain the equation T = DT / Dt + Tv + ( v ) T , where T is the lower convected
T
8.35
derivative of T.
-------------------------------------------------------------------------------
Ans. By definition, the lower convected derivative is DJ L ( ) / D , where
=t

J L ( ) = FtT ( ) T ( ) Ft ( ) . Thus, DJ L ( ) / D = DFtT / D T ( t ) Ft ( t )


=t =t
+ FtT ( t ) [ DT / D ] =t Ft ( t ) + FtT ( t ) T ( t ) DF T / D .
=t
Now, DFtT ( ) / D = DFtT / Dt = ( DFt / Dt ) = ( v ) [see Eq.(8.12.3)] and Ft ( t ) = I ,
T T
=t
therefore,

Copyright 2010, Elsevier Inc


8-23
Lai et al, Introduction to Continuum Mechanics

( DJ ( ) DT
+ T v + ( v ) T .
T
T= L =
D =t Dt
____________________________________________________________

8.36 Consider the following constitutive equation:


S + ( D*S / Dt ) = 2 D, where ( D*S / Dt ) S + ( DS + SD ) and S is co-rotational derivative
o o

of S . Obtain the shear stress function and the two normal stress functions for this fluid.
-------------------------------------------------------------------------------
Ans. With v1 = kx2 , v2 = v3 = 0 , the rate of deformation tensor and spin tensor are:
0 k / 2 0 0 k / 2 0
S
[ D] = k / 2 0 0 , [ W ] = k / 2 0 0 . Since the flow is steady, t = 0 .

0 0 0 0 0 0

The co-rotational derivative is, for symmetric S: S = SW WS = SW + ( SW ) . Now,


o T

S12 S11 0 S12 S22 S32


k
[SW ] = 2 S22 S21 0 , [SW ] = 2 S11 S21 S31 ,
T k

S32 S31 0 0 0 0
2S12 S11 S22 S32 2S12 S11 + S22 S32
o
S = k S S22 2S21
S31 , [SD] + [ DS ] = S11 + S22
k
2S21 S31 .
2 11 2
S32 S31 0 S32 S31 0

Thus, D*S / Dt S + ( DS + SD ) gives


o

2 ( 1) S12 (1 + ) S11 + ( 1) S22 ( 1) S32


D*S k
Dt = 2 (1 + ) S11 + ( 1) S22 2 (1 + ) S21 (1 + ) S31

( 1) S32 (1 + ) S31 0

D*S
Therefore, S + = 2 D
Dt
k
S11 + k ( 1) S12 = 0 (i) , S12 + (1 + ) S11 + ( 1) S22 = k (ii) ,
2
k
S13 + ( 1) S23 = 0 (iii), S22 + k (1 + ) S12 = 0 (iv),
2
k
S23 + (1 + ) S13 = 0 (v), S33 =0 (vi) .
2
Now, (iii) , (v) and (vi) give S13 = S23 = S33 = 0 . Eq. (i) gives S11 = k (1 ) S12 , Eq.(iv) gives
S22 = k (1 + ) S12 , thus, with

( )
A(k ) 1 + 1 2 ( k ) , we have,
2

S12 = k / A(k ) , S11 = k 2 (1 ) / A( k ) , S22 = k 2 (1 + ) / A(k )


The shear stress function is S12 = k / A(k ) . The normal stress functions are:
1 S11 S22 = 2 k 2 / A(k ), 2 = S22 S33 = k 2 (1 + ) / A( k ) .

Copyright 2010, Elsevier Inc


8-24
Lai et al, Introduction to Continuum Mechanics

____________________________________________________________

8.37 Obtain the apparent viscosity and the normal stress functions for the Oldroyd 3-constant
fluid [see (C) of Section 8.20].
-------------------------------------------------------------------------------
Ans. For the simple shearing flow,
0 k / 2 0 0 k / 2 0
[ D] = k / 2 0 0 , [ W ] = k / 2 0 0 .

0 0 0 0 0 0
2S12 S11 S22 S32

S = [ 0] + [SW ] [ WS ] = ( k / 2 ) S11 S22
o
2S21 S31 ,
S32 S31 0
2S12 S11 + S22 S23
[SD] + [ DS ] = ( k / 2 ) S11 + S22 2S12 S13 ,

S23 S13 0
4S12 2S22 2 S23
S = S ( SD + DS ) = ( k / 2 ) 2S22 0 ,
o
0
2S23 0 0
k 2 / 2 0 0 1 0 0
k2 0 1 0
D = [ 0] + [ DW ] [ WD] = 0
o
k 2 / 2 0 , DD =
4
0 0 0
0 0 0

k 2
0 0 22 k 2 k 0
= k
( )
o
= D 2D 2 = 0 0 0 , 2 D + D
D 2 0 0
0 0 0 0 0 0

S11 2k 1S12 S12 k 1S22 S13 k 1S23
S + 1S = S12 k 1S22 S22 S23

S13 k 1S23 S23 S33

(
S + 1S = 2 D + 2 D
)
S11 2k 1S12 S12 k 1S22 S13 k 1S23 22 k 2 k 0
S k S = k
12 1 22 S22 S23 0 0
S13 k 1S23 S23 S33 0 0 0

Thus,
S22 = S23 = S33 = S13 = 0, S12 = k , S11 2k 1S12 = 22 k 2 , so that, we have,
S12 = k , S11 = 2 k 2 ( 1 2 ) , all other Sij = 0 . The apparent viscosity is
( k ) = S12 / k = , 1 =T11 T22 = 2 k 2 ( 1 2 ) , 1 =T22 T33 = 0 .
____________________________________________________________

Copyright 2010, Elsevier Inc


8-25
Lai et al, Introduction to Continuum Mechanics

8.38 Obtain the apparent viscosity and the normal stress functions for the Oldroyd 4-constant
fluid [see (D) of Section 8.20]
-------------------------------------------------------------------------------
Ans. For the simple shearing flow
0 k / 2 0 0 k / 2 0 2S12 S11 S22 S23
k

[ D] = k / 2 0 0 , [ W ] = k / 2 0 0 , S = 2 S11 S22 2S12 S13 ,
o


0 0 0 0 0 0 S32 S31 0

4S12 2S22 2 S23


S = So ( SD + DS ) = ( k / 2 ) 2S 0 0 ,
22
2S23 0 0
k 2 / 2 0 0 1 0 0
k2
D = [ 0] + [ DW ] [ WD] = 0 0 1 0 ,
o
k2 / 2 0 , DD =
4
0 0 0 0 0 0

1 0 0 1 0 0 k 2 0 0
k2 k
2
0 1
0 = 0
o
D = D 2D =
2
0 1 0 0 0 ,
2 2
0 0 0 0 0 0 0 0 0

0 k / 2 0
o ( trS ) [ D] = o ( S11 + S22 + S33 ) k / 2 0 0 .

0 0 0

(
S + 1S + o ( trS ) D = 2 D + 2 D
)
S11 2k 1S12 S12 k 1S22 + o k ( S11 + S22 + S33 ) / 2 S13 k 1S23

S12 k 1S22 + o k ( S11 + S22 + S33 ) / 2 S22 S23
S13 k 1S23 S23 S33

T
22 k 2 k 0

= k 0 0 .
0 0 0

hus, S22 = S23 = S33 = S13 = 0, S11 2k 1S12 = 22 k 2 , S12 + o kS11 / 2 = k
From which, we get, with B (k ) (1 + 1o k 2 ) ,

( )
S11 =2 k 2 ( 1 2 ) / B (k ), S12 = k 1 + 2 o k 2 / B (k ) .

Thus, the apparent viscosity is: ( k ) = S12 / k = (1 + 2 o k 2 ) / B (k ) .


Normal stress functions are: 1 =T11 T22 = 2 k 2 (1 2 ) / B( k ), 2 =T22 T33 = 0 .
____________________________________________________________

Copyright 2010, Elsevier Inc


8-26
Lai et al, Introduction to Continuum Mechanics

1 0 0 0 1 0
8.39 Given [Q ] = 0 1 0 and [ N ] = 0 0 0 and

0 0 1 0 0 0
{ni } {ni }

( )
A1 = k N + N T and A 2 = 2k 2 N T N . (a) Verify that QA1QT = A1 and QA 2QT = A 2 . (b) From

( )
T = pI + f ( A1 , A 2 ) and Qf ( A1 , A 2 ) QT = f QA1QT , QA 2QT , show that QT ( k ) Q T = T ( k )
and (c) From the results of part (b), show that the viscometric functions have the properties:
S ( k ) = S ( k ) , 1 ( k ) = 1 ( k ) , 2 ( k ) = 2 ( k ) .
-------------------------------------------------------------------------------
0 k 0 0 0 0 0 1 0 0 0 0

( )
Ans. (a) [ A1 ] = k N + N = k 0 0 and A 2 = 2k 1 0 0 0 0 0 = 0 2k 2 0
T

2

0 0 0 0 0 0 0 0 0 0 0 0

1 0 0 0 k 0 1 0 0 0 k 0
QA1QT = 0 1 0 k 0 0 0 1 0 = k 0 0 = [ A1 ]

0 0 1 0 0 0 0 0 1 0 0 0

1 0 0 0 0 0 1 0 0 0 0 0

QA 2Q = 0 1 0 0 2k 2 0 0 1 0 = 0 2k 2 0 = [ A 2 ]
T

0 0 1 0 0 0 0 0 1 0 0 0

(b)
QT ( k ) QT = Q pI + f ( A1 ,A 2 ) QT = pI + Qf ( A1 ,A 2 ) QT

( )
= pI + f QA1QT ,QA 2QT = pI + f ( A1 ( k ) ,A 2 ( k ) ) .
Now, A1 ( k ) = A1 ( k ) and A 2 ( k ) = A 2 ( k ) , thus,.
QT ( k ) QT = pI + f ( A1 ( k ) ,A 2 ( k ) ) and QT ( k ) QT = pI + f ( A1 ( k ) ,A 2 ( k ) ) .
That is, QT ( k ) QT = T ( k ) and QT ( k ) QT = T ( k )
1 0 0 T11 T12 T13 1 0 0 T11 T12 T13
(c) [Q ] T ( k ) Q = 0 1 0 T21 T22 T23 0 1 0 = T21 T22
T
T23

0 0 1 T31 T32 T33 0 0 1 T31 T32 T33
QT ( k ) Q T = T ( k ) T11 ( k ) = T11 ( k ) , T22 ( k ) = T22 ( k ) , T33 ( k ) = T33 ( k )
T12 ( k )12 = T12 ( k ) , T13 ( k ) = T13 ( k ) , T23 ( k ) = T23 ( k ) . Thus,
1 ( k ) = 1 ( k ) , 2 ( k ) = 2 ( k ) , S ( k ) = S ( k ) . [Note, in viscometric flow, T13 = T23 = 0 ].
____________________________________________________________

8.40 For the velocity field given in example 8.21.2, i.e., vr = 0, v = 0, vz = v ( r ) , (a) obtain
the stress components in terms of the shear stress function S ( k ) and the normal stress functions
1 ( k ) and 2 ( k ) , where k = dv / dr , (b) obtain the following velocity distribution for the

Copyright 2010, Elsevier Inc


8-27
Lai et al, Introduction to Continuum Mechanics

Poiseuille flow under a pressure gradient of ( f ): v ( r ) = ( fr / 2 )dr , where is the inverse


R
r

shear stress function, and (c)obtain the relation ( Rf / 2 ) = 1 / ( R3 f 2 ) f 3Q / f .


( )
-------------------------------------------------------------------------------
Ans. (a) In example 8.21.2, we see that the velocity field vr = 0, v = 0, vz = v ( r ) describes a
viscometric flow with the nonzero Rivlin-Ericksen tensors given by
0 k ( r ) 0 0 0 0

[ A1 ] = k ( r ) 0 0 , [ A 2 ] = 0 2k 2 ( r ) 0 ,
0 0 0 0 0 0
ni ni
where n1 = e z , n 2 = er , n3 = e and k ( r ) = dv / dr (see Example 8.10.2, but, note the differences
in the order of bases). Thus the stress components with respect to the basis {ni } are given by (See
section 8.22):
S z r = (k ), S z z Sr r = 1 ( k ) , Sr r S = 2 ( k ) , S z = Sr = 0 .
(b) With Sij depending only on r , the equations of motion become:
S rr S rr S p p 1 p
+ = 0 (i ), = 0 (ii) , ( rSrz ) = 0 (iii)
r r r r r z
p p p
= 0 , Eq. (ii) gives = 0 and Eq. (iii) gives =0
z z
Eq. (i) gives
r z z
Thus, p / z = a constant f . Eq. (iii) becomes
1 fr C
( rSrz ) = f ( rSrz ) = fr Srz = + . Since Srz must be finite at r = 0 , thus,
r r r 2 r
C = 0 and Srz = fr / 2 . Now, Srz = (k ) where ( k ) is the shear stress function and k = dv / dr .
1
Thus, ( k ) = fr / 2 . Inverting this equation, we have, k = ( fr / 2 ) ( fr / 2 ) . Since
(k ) is an odd function of k , therefore, is also an odd function of k , so that
k = ( fr / 2 ) dv / dr = ( fr / 2 ) dv = ( fr / 2 ) dr .
Thus, v ( R ) v ( r ) = ( fr / 2 )dr . Since v ( R ) = 0 , therefore, v ( r ) = ( fr / 2 )dr .
R R
r r

(c) The volume discharge is given by Q = v ( r ) 2 rdr . Therefore,


R
0
R dv dv
Q = v ( r ) dr 2 = v ( r ) r 2 r 2 dr = r 2 dr = r 2 ( fr / 2 )dr
R R R R
0 0 0 dr 0 dr 0

Thus, Q / = r 2 ( fr / 2 )dr . Let fr / 2 s dr 2ds / f and r 2 = 4s 2 / f 2 , then ,


R
0

Q/ =
R
r =0
r ( fr / 2 ) dr =
2 R f /2
s =0
( 8s 2
)
/ f 3 ( s )ds f 3Q / =
R f /2
s =0
8s 2 ( s )ds .
Differentiating the last equation with respect to f , we obtain
3
(
1 f Q )
Rf Rf Rf
2
R Rf Rf
2
3 2 Rf
f
=
f 2 8 2 2 = 2 8 2 2 = R f 2 .

Rf
Thus,
1 f Q
3
( )
= .
2 R3 f 2 f

Copyright 2010, Elsevier Inc


8-28
Lai et al, Introduction to Continuum Mechanics

____________________________________________________________

Copyright 2010, Elsevier Inc


8-29

You might also like